Está en la página 1de 385

ALGEBRA CLASICA

BORIS AYUB NAHUM

Version Preliminar
2
Indice general

1. RELACIONES Y FUNCIONES 7
1.1. INTRODUCCION . . . . . . . . . . . . . . . . . . . . . . . . . . . . . . . . . . 7
1.1.1. EJERCICIOS . . . . . . . . . . . . . . . . . . . . . . . . . . . . . . . . . 10
1.2. UNA APLICACION A LOS SILOGISMOS . . . . . . . . . . . . . . . . . . . . 10
1.2.1. EJERCICIOS . . . . . . . . . . . . . . . . . . . . . . . . . . . . . . . . . 14
1.3. RELACIONES . . . . . . . . . . . . . . . . . . . . . . . . . . . . . . . . . . . . 14
1.3.1. EJERCICIOS . . . . . . . . . . . . . . . . . . . . . . . . . . . . . . . . . 19
1.4. FUNCIONES . . . . . . . . . . . . . . . . . . . . . . . . . . . . . . . . . . . . . 21
1.4.1. EJERCICIOS . . . . . . . . . . . . . . . . . . . . . . . . . . . . . . . . . 32
1.5. ALGO MAS SOBRE BIYECCIONES . . . . . . . . . . . . . . . . . . . . . . . 35
1.5.1. EJERCICIOS. . . . . . . . . . . . . . . . . . . . . . . . . . . . . . . . . 39
1.6. SUGERENCIAS O SOLUCIONES A LOS EJERCICIOS DEL CAPITULO. . . 40

2. NUMEROS ENTEROS, INDUCCION, SUMATORIAS Y PROGRESIO-


NES 47
2.1. NUMEROS ENTEROS E INDUCCION . . . . . . . . . . . . . . . . . . . . . . 47
2.1.1. EJERCICIOS ( DE INDUCCION) . . . . . . . . . . . . . . . . . . . . . 54
2.1.2. MAXIMO COMUN DIVISOR (M.C.D.) . . . . . . . . . . . . . . . . . . 56
2.1.3. EJERCICIOS . . . . . . . . . . . . . . . . . . . . . . . . . . . . . . . . . 59
2.2. SUMATORIAS . . . . . . . . . . . . . . . . . . . . . . . . . . . . . . . . . . . . 60
2.2.1. EJERCICIOS . . . . . . . . . . . . . . . . . . . . . . . . . . . . . . . . . 70
2.3. PROGRESIONES . . . . . . . . . . . . . . . . . . . . . . . . . . . . . . . . . . 73
2.3.1. EJERCICIOS . . . . . . . . . . . . . . . . . . . . . . . . . . . . . . . . . 81
2.4. SUMATORIAS DOBLES . . . . . . . . . . . . . . . . . . . . . . . . . . . . . . 83
2.4.1. EJERCICIOS . . . . . . . . . . . . . . . . . . . . . . . . . . . . . . . . . 85
2.5. SUGERENCIAS O SOLUCIONES A LOS EJERCICIOS DEL CAPITULO . . 86

3. COEFICIENTES BINOMIALES 99
3.0.1. EJERCICIOS . . . . . . . . . . . . . . . . . . . . . . . . . . . . . . . . . 110
3.1. SOLUCIONES O SUGERENCIAS A LOS EJERCICIOS DEL CAPITULO . . 112

4. ELEMENTOS DE COMBINATORIA 115


4.1. NOCIONES BASICAS . . . . . . . . . . . . . . . . . . . . . . . . . . . . . . . . 115
4.1.1. REGLA DE LA SUMA . . . . . . . . . . . . . . . . . . . . . . . . . . . 116
4.1.2. REGLA DEL PRODUCTO . . . . . . . . . . . . . . . . . . . . . . . . . 116
4.1.3. EJERCICIOS . . . . . . . . . . . . . . . . . . . . . . . . . . . . . . . . . 125
4.2. LA FORMULA DE INCLUSIONES Y EXCLUSIONES. . . . . . . . . . . . . . 127

3
4

4.3. DESPLAZAMIENTOS . . . . . . . . . . . . . . . . . . . . . . . . . . . . . . . . 132


4.3.1. EJERCICIOS . . . . . . . . . . . . . . . . . . . . . . . . . . . . . . . . . 134
4.4. PARTICIONES . . . . . . . . . . . . . . . . . . . . . . . . . . . . . . . . . . . . 135
4.4.1. EJERCICIOS . . . . . . . . . . . . . . . . . . . . . . . . . . . . . . . . . 137
4.5. SOLUCIONES O SUGERENCIAS A LOS EJERCICIOS DEL CAPITULO . . 139

5. ELEMENTOS DE PROBABILIDAD 145


5.1. GENERALIDADES . . . . . . . . . . . . . . . . . . . . . . . . . . . . . . . . . 145
5.1.1. EJERCICIOS . . . . . . . . . . . . . . . . . . . . . . . . . . . . . . . . . 152
5.2. PROBABILIDAD CONDICIONADA . . . . . . . . . . . . . . . . . . . . . . . 153
5.3. EVENTOS INDEPENDIENTES . . . . . . . . . . . . . . . . . . . . . . . . . . 161
5.4. EXPERIMENTOS INDEPENDIENTES . . . . . . . . . . . . . . . . . . . . . . 162
5.4.1. EJERCICIOS . . . . . . . . . . . . . . . . . . . . . . . . . . . . . . . . . 167
5.5. SOLUCIONES O SUGERENCIAS A LOS EJERCICIOS DEL CAPITULO . . 170

6. LOS NUMEROS REALES


1a. PARTE 173
6.1. AXIOMAS DE CAMPO . . . . . . . . . . . . . . . . . . . . . . . . . . . . . . . 173
6.2. AXIOMAS DE ORDEN . . . . . . . . . . . . . . . . . . . . . . . . . . . . . . . 178
6.2.1. EJERCICIOS . . . . . . . . . . . . . . . . . . . . . . . . . . . . . . . . . 181
6.3. NUMEROS NATURALES . . . . . . . . . . . . . . . . . . . . . . . . . . . . . . 182
6.4. NUMEROS ENTEROS Y RACIONALES . . . . . . . . . . . . . . . . . . . . . 184
6.4.1. EJERCICIOS . . . . . . . . . . . . . . . . . . . . . . . . . . . . . . . . . 187
6.5. SOLUCIONES O SUGERENCIAS A LOS EJERCICIOS DEL CAPITULO . . 188

7. ALGO MAS SOBRE RACIONALES 191


7.1. DECIMALES . . . . . . . . . . . . . . . . . . . . . . . . . . . . . . . . . . . . . 191
7.2. CAMBIOS DE BASE . . . . . . . . . . . . . . . . . . . . . . . . . . . . . . . . 196
7.3. SUCESIONES DE FAREY . . . . . . . . . . . . . . . . . . . . . . . . . . . . . 200
7.4. EJERCICIOS . . . . . . . . . . . . . . . . . . . . . . . . . . . . . . . . . . . . . 206
7.5. SOLUCIONES . . . . . . . . . . . . . . . . . . . . . . . . . . . . . . . . . . . . 207

8. DESIGUALDADES, VALOR ABSOLUTO, INECUACIONES 209


8.1. EJEMPLOS INTRODUCTORIOS . . . . . . . . . . . . . . . . . . . . . . . . . 209
8.2. ALGUNAS DESIGUALDADES IMPORTANTES . . . . . . . . . . . . . . . . 213
8.2.1. EJERCICIOS . . . . . . . . . . . . . . . . . . . . . . . . . . . . . . . . . 224
8.3. VALOR ABSOLUTO . . . . . . . . . . . . . . . . . . . . . . . . . . . . . . . . 226
8.4. INECUACIONES . . . . . . . . . . . . . . . . . . . . . . . . . . . . . . . . . . . 229
8.4.1. EJERCICIOS . . . . . . . . . . . . . . . . . . . . . . . . . . . . . . . . . 237
8.5. SOLUCIONES O SUGERENCIAS A LOS EJERCICIOS DEL CAPITULO . . 238

9. LOS NUMEROS COMPLEJOS 243


9.1. INTRODUCCION . . . . . . . . . . . . . . . . . . . . . . . . . . . . . . . . . . 243
9.2. EL CAMPO DE LOS NUMEROS COMPLEJOS . . . . . . . . . . . . . . . . . 244
9.3. FORMA CANONICA DE LOS COMPLEJOS . . . . . . . . . . . . . . . . . . 247
9.4. MODULO Y CONJUGADO . . . . . . . . . . . . . . . . . . . . . . . . . . . . 248
9.4.1. EJERCICIOS . . . . . . . . . . . . . . . . . . . . . . . . . . . . . . . . . 253
9.5. LOS COMPLEJOS COMO VECTORES . . . . . . . . . . . . . . . . . . . . . 254
5

9.6. FORMA TRIGONOMETRICA DE LOS COMPLEJOS . . . . . . . . . . . . . 257


9.7. APENDICE . . . . . . . . . . . . . . . . . . . . . . . . . . . . . . . . . . . . . . 267
9.7.1. EXTENSIONES AL CAMPO C DE ALGUNAS FUNCIONES REALES267
9.7.2. EJERCICIOS . . . . . . . . . . . . . . . . . . . . . . . . . . . . . . . . . 272
9.8. SOLUCIONES O SUGERENCIAS A LOS EJERCICIOS DEL CAPITULO . . 274

10.POLINOMIOS Y FUNCIONES RACIONALES 281


10.1. INTRODUCCION . . . . . . . . . . . . . . . . . . . . . . . . . . . . . . . . . . 281
10.2. PROPIEDADES DE LOS POLINOMIOS . . . . . . . . . . . . . . . . . . . . . 286
10.2.1. EJERCICIOS . . . . . . . . . . . . . . . . . . . . . . . . . . . . . . . . . 296
10.3. FUNCIONES RACIONALES . . . . . . . . . . . . . . . . . . . . . . . . . . . . 297
10.4. FRACCIONES PARCIALES . . . . . . . . . . . . . . . . . . . . . . . . . . . . 300
10.4.1. EJERCICIOS . . . . . . . . . . . . . . . . . . . . . . . . . . . . . . . . . 304
10.5. SOLUCIONES O SUGERENCIAS A LOS EJERCICIOS DEL CAPITULO . . 305

11.TEORIA DE ECUACIONES 309


11.1. GENERALIDADES . . . . . . . . . . . . . . . . . . . . . . . . . . . . . . . . . 309
11.2. ECUACIONES DE GRADO INFERIOR A 5 . . . . . . . . . . . . . . . . . . . 317
11.2.1. EJERCICIOS . . . . . . . . . . . . . . . . . . . . . . . . . . . . . . . . . 321
11.3. METODOS NUMERICOS ELEMENTALES . . . . . . . . . . . . . . . . . . . 323
11.3.1. EJERCICIOS . . . . . . . . . . . . . . . . . . . . . . . . . . . . . . . . . 336
11.4. SOLUCIONES O SUGERENCIAS A LOS EJERCICIOS DEL CAPITULO . . 338

12.LOS NUMEROS REALES


2a. PARTE 341
12.1. EL AXIOMA DEL SUPREMO . . . . . . . . . . . . . . . . . . . . . . . . . . . 341
12.2. AXIOMA DEL SUPREMO . . . . . . . . . . . . . . . . . . . . . . . . . . . . . 343
12.3. EXPONENTES RACIONALES . . . . . . . . . . . . . . . . . . . . . . . . . . . 347
12.4. LAS FUNCIONES ax Y lga x . . . . . . . . . . . . . . . . . . . . . . . . . . . . 350
12.4.1. EJERCICIOS . . . . . . . . . . . . . . . . . . . . . . . . . . . . . . . . . 353
12.5. SOLUCIONES O SUGERENCIAS A LOS EJERCICIOS DEL CAPITULO . . 354

13.PROBLEMAS COMPLEMENTARIOS 355


13.1. CAPITULO 1 . . . . . . . . . . . . . . . . . . . . . . . . . . . . . . . . . . . . . 355
13.2. CAPITULO 2 . . . . . . . . . . . . . . . . . . . . . . . . . . . . . . . . . . . . . 357
13.3. CAPITULO 3 . . . . . . . . . . . . . . . . . . . . . . . . . . . . . . . . . . . . . 359
13.4. CAPITULO 4 . . . . . . . . . . . . . . . . . . . . . . . . . . . . . . . . . . . . . 360
13.5. CAPITULO 5 . . . . . . . . . . . . . . . . . . . . . . . . . . . . . . . . . . . . . 362
13.6. CAPITULO 6 . . . . . . . . . . . . . . . . . . . . . . . . . . . . . . . . . . . . . 365
13.7. CAPITULO 7 . . . . . . . . . . . . . . . . . . . . . . . . . . . . . . . . . . . . . 365
13.8. CAPITULO 8 . . . . . . . . . . . . . . . . . . . . . . . . . . . . . . . . . . . . . 365
13.9. CAPITULO 9 . . . . . . . . . . . . . . . . . . . . . . . . . . . . . . . . . . . . . 367
13.10.CAPITULO 10 . . . . . . . . . . . . . . . . . . . . . . . . . . . . . . . . . . . . 368
13.11.CAPITULO 11 . . . . . . . . . . . . . . . . . . . . . . . . . . . . . . . . . . . . 370
13.12.CAPITULO 12 . . . . . . . . . . . . . . . . . . . . . . . . . . . . . . . . . . . . 370
13.13.BIBLIOGRAFIA . . . . . . . . . . . . . . . . . . . . . . . . . . . . . . . . . . . 385
6
Captulo 1

RELACIONES Y FUNCIONES

1.1. INTRODUCCION
Daremos por conocidos los elementos de algebra de conjuntos que normalmente se estudian
en el colegio. Sin embargo, para recordar y ponernos de acuerdo en la notacion, indicaremos
los conceptos y teoremas mas frecuentemente usados.

El smbolo {x : P }, indicara el conjunto de todos los elementos que satisfacen la propiedad


P.

Definicion 1.1

a) A B = {x : x A x B} (Union de conjuntos)

b) A B = {x : x A x B} (Interseccion de conjuntos)

c) A B = {x : x A x 6 B} (Diferencia)

d) A0 = U A (U conjunto Universal) (Complemento)

e) A B x(x A x B) (Subconjunto)

f) = {x : x 6= x} (Conjunto vaco)

g) SA = {X : X A} (Conjunto Potencia)

Las propiedades de uso mas frecuente vienen dadas en el siguiente teorema:

Teorema 1.1


a) A B = B A
(Conmutatividad)
b) A B = B A

7
8

c) A B A; A A B

d) (A B) C = A (B C)
(Asociatividad)
e) (A B) C = A (B C)


f) A (B C) = (A B) (A C)
(Distributividad)
g) A (B C) = (A B) (A C)

h) (A B)0 = A0 B 0

(Leyes de De Morgan)
i) (A B)0 = A0 B0

Una manera intuitiva de ilustrar los conjuntos y sus operaciones se consigue con los as lla-
mados, diagramas de Venn, en recuerdo de su creador John Venn, matematico y filosofo
britanico. Su forma general para dos conjuntos se muestra en el siguiente diagrama:

'$
A '$
B

1 2 3 4

&%
&%
U

Figura 1.1

El rectangulo representa el conjunto universal. Los conjuntos A y B corresponden a los crculos.


Se han enumerado los diferentes sectores que se forman. Notemos que ellos son disjuntos. La
siguiente lista describe los conjuntos como uniones de sectores:

A = 2 3; B = 3 4; C = 1 2 3 4

Es inmediato concluir que:

a) A0 = 1 4 b) B 0 = 1 2

Inversamente:

1 = A0 B 0 ; 2 = A B 0 ; 3 = A B; 4 = A0 B

En el siguiente diagrama se indican las posiciones relativas de dos conjuntos:


9

'$
A '$
B '$
B '$
A '$
B

A


&%
&% &% &%
&%
U U U

A B 6= AB AB =

Figura 1.2

El diagrama que continua muestra la posicion general de 3 conjuntos.

'$
A '$
B

2 '$
6
7
5
1 3 8
&%
&%
4

U
&%
C

Figura 1.3

En este caso, tenemos que los sectores son 8 y los conjuntos quedan descritos como se indica
a continuacion:
A = 2 3 5 6; A0 = 1 4 7 8
B = 5 6 7 8; B 0 = 1 2 3 4
C = 3 4 5 8; C 0 = 1 2 6 7
Usando estas descomposiciones podemos resolver otros casos. Por ejemplo:

(A B) C 0 = (2 3 5 6 7 8) (1 2 6 7) = 2 6 7

Tambien podemos proceder en forma inversa. Por ejemplo, 3 = A B 0 C.

En general, cualquier conjunto anotado por un numero puede representarse por intersecciones
de los conjuntos originales o de sus complementos.

Definicion 1.2
(a, b) = {{a}, {a, b}}

(a, b) se llama par ordenado de primer elemento a y segundo b.

Teorema 1.2
(a, b) = (c, d) si y solo si a=c y b=d

En palabras, dos pares ordenados son iguales si y solo si tienen las mismas componentes.

Hasta aqu las nociones que supondremos conocidas. En captulo posteriores sera necesario
agregar algunas mas.
10

1.1.1. EJERCICIOS
1. Representar los diferentes sectores de la Fig.1.3 en funcion de los conjuntos originales o
sus complementos.

2. Indique, usando la figura 1.3, los siguientes conjuntos:

(a) A (A0 B C)
(b) (A B) (A C) (B C)

3. Use diagramas de Venn para decidir cuales de las siguientes afirmaciones son validas:

(a) A (A B) = A
(b) (A B)0 = A0 B 0
(c) (A B) (A0 B) (A B 0 ) = U

1.2. UNA APLICACION A LOS SILOGISMOS


Se pueden usar los diagramas de Venn para representar y resolver silogismos.
Un silogismo es una forma de razonamiento que consta de dos premisas de las cuales, en
algunos casos, se puede inferir una conclusion,. Con mayor precision, un silogismo es una
argumentacion que a partir de dos premisas, donde la primera establece una relacion entre los
terminos P y M , y la segunda una relacion entre los terminos M y S se infiere una relacion
entre S y P , siendo posible que las premisas no permitan llegar a alguna conclusion.
Hay cuatro formas posibles para las premisas y la conclusion. Se designan con las vocales A,
E, I, O. Ellas son:

A: Todo x es y(Universal afirmativa)


Ejemplo: Todas las naranjas son frutos ctricos

E: Ningun x es y(Universal negativa)


Ejemplo: Ningun humano es cuadrupedo

I: Algunos x son y(Particular afirmativa)


Ejemplo: Algunas flores son rojas

O: Algunos x no son y(Particular negativa)


Ejemplo: Algunas uvas no tienen pepas

(Algunos debe entenderse por lo menos uno)

Interpretado en el lenguaje de conjuntos, la premisa universal afirmativa simplemente dice


que X es un subconjunto de Y , donde X representa el conjunto de todos los x e Y representa
el conjunto de todos los y . En forma similar, la universal negativa queda representada por
X Y = ; la particular afirmativa como X Y 6= y la particular negativa como X Y 0 6= .
Ellas corresponden a los siguientes diagramas:
11

'$ '$ '$'$ '$ '$ '$


Y  X Y Y X X Y
X q q q

&% &% &%&% &% &% &%
A E I O
Figura 1.4
El punto indica que hay por lo menos un elemento en esa region.

En cada premisa de un silogismos aparecen dos conjuntos, uno de los cuales se repite en la
siguiente premisa. Estos conjuntos seran representados por las letras S, M y P , donde M se re-
serva para el conjunto repetido. Mas precisamente, una premisa establece una relacion entre P
y M y la otra entre M y S y la conclusion, si es que existe, establece una relacion entre P y S.

Es frecuente que las premisas no permitan llegar a alguna conclusion. Por tal motivo, en este
texto, no la consideraremos como parte del silogismo.

Ejemplo 1.1
Sean las premisas:
Todos los chilenos hablan castellano.
Algunos chilenos viven en Australia.
Ellas son del tipo A e I respectivamente. En este caso M es el conjunto de todos los chilenos,
P es el conjunto de todos los humanos que hablan castellano y S es el conjunto de los humanos
que viven en Australia.

Cualquiera de los siguientes tres diagramas cumple con las condiciones de las premisas:
'$ '$
P  '$ '$
P 
'$
M P M
q  q
 S
Mq 
&%  &% S

&% &%
S
&%

Figura 1.5

La conclusion debera ser una premisa, de alguno de los tipos anteriormente senalado, entre
S y P , que se pueda obtener observando las posiciones relativas entre ellos o, mejor aun, las
posiciones que quedan excluidas. En este ejemplo, S y P no pueden ser disjuntos. Luego,
tenemos la siguiente premisa como conclusion Algunos P son S, que aplicado al ejem-
plo se convierte en Algunos habitantes de Australia hablan castellano. Tambien tenemos
la conclusion Algunos S son P . En palabras Algunos hispanoparlantes viven en Australia.

Este silogismo tiene la forma A, I I donde las premisas son de la forma A e I y la conclusion
es del tipo I.(La letra que sigue al guion indica la conclusion).
12

Ejemplo 1.2

Todos los humanos son mortales.

Socrates es humano.

En este caso P es el conjunto de todos los seres mortales, M el de todos los seres humanos y S,
el conjunto cuyo unico elemento es Socrates. Su estructura es A, A. La unica representacion
posible es:

'$

P
M

qk
S

&%

Figura 1.6

Del diagrama notamos que S es un subconjunto de P , que nos lleva a la conclusion Socrates
es mortal. En realidad, este esquema permite sacar dos conclusiones: Todo S es P y Algun
P es S, que corresponde Algun mortal es Socrates.

Si un silogismo tiene dos representaciones entre S y P esencialmente distintas no es posible


sacar alguna conclusion. Esto sucede cuando las dos premisas son particulares. Por ejemplo,
tomemos el caso I, I en la forma Algun P es M y Algun M es S. Esta estructura permite,
por lo menos, las dos representaciones siguientes:
'$
'$ '$
'$
P M P M
'$ q
'$ q
q
&%
&% &%
&%
&%
S
&%
S

Figura 1.7

Ambos esquemas satisfacen las premisas, pero en el primero S y P son disjuntos y no lo son
en el segundo. Luego, no hay conclusion posible.

Dejamos al lector el caso I, O en todas sus variantes, algunas de las cuales son: Algun
M es P y Algun M no es S o Algun M es S y Algun P no es M , etc.

Veamos un ejemplo del tipo E, E.

Ejemplo 1.3
Ningun humano es marciano y ningun humano es cuadrupedo. Tiene por lo menos dos
esquemas que lo representan. Como son esencialmente distintos, ya que en uno P y S son
disjuntos y en el otro no, es imposible sacar alguna conclusion. (Figura 1.8).
13

'$
'$'$
'$
q q
P M P M

'$
&%
&% &%
S &%
'$
S
&%
&%

Figura 1.8

En los silogismos, se llama figura al orden en que aparecen los conjuntos en las premisas.
Hay cuatro casos de figura, los cuales aparecen en la siguiente tabla.

Premisa 1 Premisa 2
Figura 1 MP SM
Figura 2 PM SM
Figura 3 MP MS
Figura 4 PM MS

En realidad, la figura queda determinada por la ubicacion del conjunto M . La primera


figura se caracteriza por M en el primer y cuarto lugar; la segunda, por M en el segundo y
cuarto lugar; la tercera, por M en el primer y tercer lugar y la cuarta, por M en el segundo
y tercer lugar.
Tomando en cuenta estas cuatro tipos de figuras y tambien considerando que hay cuatro
maneras de relacionar dos conjuntos, que son las indicadas por las letras A, E, I y O, es facil
concluir que existen exactamente 64 silogismos distintos. No todos ellos permiten sacar alguna
conclusion; solamente 19 de ellos son concluyentes. Estos son:
Figura 1: A A-A E A-E A I-I E I-O
Figura 2: E A-E A E-E E I-O A O-O
Figura 3: O A-O A A-I I A-I A I-I E A-O E I-O
Figura 4: A A-I A E-E I A-I E A-O E I-O

Nota
La letra que aparece despues del guion indica el tipo de la conclusion.

Ejemplo 1.4
En la segunda figura no aparece la combinacion A, I. Un silogismo con esa estructura dira
Todo P es M y algun S es M , que permite representaciones en que P y S son disjuntos
y otras en que no lo son. Pero esta misma combinacion, en la primera figura, afirma Todo
M es P y Algun S es M , que lleva a la conclusion Algun S es P
14

1.2.1. EJERCICIOS
4. Indique la figura y la conclusion, si es que existe.

(a) Ningun militar es bombero. Algunos bomberos son violinistas


(b) Ningun militar es bombero. Todo militar usa uniforme
(c) Ningun militar es bombero. Algunos escritores son bomberos

5. Note que la combinacion E I-O aparece en las cuatro figuras. Explique la causa. Tambien
explique porque la combinacion AA no aparece en la segunda figura.

1.3. RELACIONES
Antes de continuar, queremos hacer algunos comentarios sobre la definicion de produc-
to cartesiano y su utilizacion en las relaciones. Es muy poco lo que puede construirse en
matematica sin el concepto de relacion, el cual a su vez se presenta como un conjunto de
pares ordenados, es decir un subconjunto del producto cartesiano, como veremos un poco mas
adelante. Por que esta forma de presentar las relaciones tan alejadas de nuestra intuicion?
Informalmente entendemos por relacion un nexo que liga elementos de algun conjunto con los
de otro. Por ejemplo, entre los humanos y los libros puede establecerse entre otras la siguiente
relacion: la persona A se relaciona con el libro X si y solo si A leyo a X. Otra relacion,
pero ahora de los libros a las personas sera: el libro X se relaciona con la persona A si y
solo si el autor de X es A. En una biblioteca, el catalogo de ttulos de libros versus autor,
representa esta relacion para los libros que all se encuentran. Materialmente, este catalogo
esta compuesto de tarjetas, una por cada libro, ordenadas alfabeticamente segun los ttulos
donde se incluyen ademas, el nombre del autor y otros datos pertinentes. Lo que importa es
que la relacion ha sido materializada como un conjunto de tarjetas, en que cada una puede
asimilarse al par ordenado, ttulo de la obra versus el autor.

En otras ocasiones es practicamente imposible dar una formula que especifique la relacion
como se hizo en este ejemplo y la unica alternativa viable es listar, como pareja los elementos
que estan relacionados. Por ejemplo, se decide pintar los dgitos de las patentes de vehculos
de colores distintos segun el siguiente criterio: Los ceros se pintaran grices, los unos azules,
los dos amarillos, etc. En este caso se tiene una relacion entre los dgitos y los colores que
solamente se puede definir indicando en una lista el numero y su correspondiente color. Pero
este listado no es mas que la coleccion de parejas dgito y su color.

Consideraciones de esta naturaleza nos llevan a que, probablemente la mejor forma de definir
relacion en el contexto matematico es como subconjunto de parejas ordenadas. Se tiene la
ventaja de no agregar nuevos conceptos primitivos, usandose solo nociones ya establecidas de
la Teora de Conjuntos, obteniendose a cambio un concepto de relacion que no refleja la ley
que debe cumplirse para que dos elementos estan relacionados, sino la coleccion de parejas
que la satisface.

Con estas aclaraciones, esperamos que el lector no tenga inconvenientes en captar el contenido
intuitivo de las dos definiciones con que continua el texto.
15

Definicion 1.3
A B = {(a, b) : a A b B}
Se llama el producto cartesiano de A con B .

Ejemplo 1.5
Sea A = {1, 2} y B = {a, b, c}

Entonces A B = {(1, a), (1, b), (1, c), (2, a), (2, b), (2, c)}.

Ejemplo 1.6
Sea N = {1, 2, 3, . . . } los numeros naturales. Entonces:

N N = {(1, 1), (1, 2), (1, 3), . . .


(2, 1), (2, 2), (2, 3), . . . }

Ejemplo 1.7
Sea A = {1, 2, 3, 4} y B = {x, y, z}. Ubiquemos los elementos de A como puntos en una recta
horizontal y los de B en una vertical. Entonces el punto que se encuentra en la columna que
empieza en 1 y en la fila comienza con x representa la pareja (1, x), el que esta en la columna
del 3 y en la fila de la y es el par (3, y) y as sucesivamente.

Podemos hacer un dibujo que da una imagen geometrica conveniente del producto cartesiano
de dos conjuntos. (Figura 1.9).

4 r r r r

3 r r r r

2 r r r r

1 r r r r

1 2 3 4

Figura 1.9.

Este dibujo se llama la representacion grafica del producto cartesiano. Con esta imagen es
facil ver que si A tiene n elementos y B tiene m elementos entonces AB tiene nm elementos.

Esta representacion es especialmente util cuando A y B son subconjuntos de R. Si A = B = R


el grafico de R R es todo el plano.

Nota
Los numeros reales los estudiaremos mas adelante. Por el momento piense que R es el conjunto
de todos los numeros de la forma un entero seguido de infinitos decimales.
16

Definicion 1.4
Diremos que R es una relacion de A en B si y solo si R A B.

Ejemplo 1.8
Sea A = {1, 2, 3, 4} y B = {x, y, z}. Son relaciones de A en B:

a) R1 = {(1, x), (1, y), (3, z)}

b) R2 = {(1, x), (2, x), (3, x)}

c) A B

d)

Son relaciones de B en A:

e) R3 = {(x, 1), (y, 2), (z, 3)}

f) R4 = {(y, 1), (z, 1)}

Ejemplo 1.9
Sea SU la coleccion de todos los subconjuntos de U . Definamos una relacion de SU en SU
por xy ((x, y) R x y). Este ejemplo muestra que ser subconjunto cabe considerarlo
como una relacion.

Nota .
Es frecuente usar las siguientes notaciones:

R
a) Por R es una relacion de A en B, escribir R : A B o tambien A B.

b) Por (x, y) R, escribir xRy.

Ejemplo 1.10
Sea R el conjunto de los numeros reales. Definamos S : R R por xSy si y solo si y = x2 .
17

6
4

-
-2 -1 1 2

-1

-2

Figura 1.10

Este dibujo es la representacion grafica de la relacion; es decir, la curva C esta formada por
todos los puntos de R R que pertenecen a S. La expresion mas corriente para referirse a
este tipo de situaciones, es decir, que la ecuacion y = x2 tiene por grafico la curva C, o que
se ha hecho el grafico de la curva y = x2 .

Ejemplo 1.11
Sea S una relacion de R en R definida por xSy si y solo si 1 x 1 y 1 y 1.
El grafico de la relacion S es el cuadrado de lado 1 includos sus bordes.

Figura 1.11

Definicion 1.5
Sea R : A B.

a) Definimos el dominio de R y lo anotamos Dom R por

Dom R = {x A : y (y B (x, y) R}
18

En forma mas intuitiva, Dom R es el conjunto formado por los primeros elementos de
las parejas de R.

b) Definimos el recorrido de R y lo anotamos por Rec R al conjunto

Rec R = {y B : x (x A (x, y) R)}

Es decir, es el conjunto formado por los segundos elementos de las parejas de R.

Si (x, y) R se dice que y es una imagen del objeto x. Con este lenguaje se expresa el
Dom R como el conjunto de todos los objetos y Rec R como el de todas las imagenes.

Ejemplo 1.12
En el ejemplo 1.7 a) tenemos que Dom R1 = {1, 3} y Rec R1 = {x, y, z}. En b), Dom R2 =
{1, 2, 3} y Rec R2 = {x}.

Ejemplo 1.13
Sea Q el conjunto de los numeros racionales y R : Q Q definida por x R y si y solo si
1
y= .
x
Entonces Dom R = Q {0} y Rec R = Q {0},

Ejemplo 1.14
Sea S : N N definida por xSy y = x2 . Entonces Dom S = N y Rec S = {1, 4, 9, 16, . . .}.

Ejemplo 1.15
Sea Z el conjunto de los enteros y sea R : N Z definida por:
  
x 1x
xRy x par y = x impar y = .
2 2

Es obvio que el Dom R = N. Ademas Rec R = Z pues cualquiera sea el entero n, si es positivo,
es imagen del natural 2n y si es negativo o cero entonces es imagen del natural 1 2n.

Definicion 1.6
Sea R : A B. Definimos R1 : B A por (x, y) R1 si y solo si (y, x) R. R1 se
llama la relacion inversa de R. Notemos que su dominio es el recorrido de R y su recorrido el
dominio de R. Es un ejercicio facil probar que (R1 )1 = R.

Ejemplo 1.16
Sea R = {(1, a), (2, a), (2, b), (3, b)} entonces R1 = {(a, 1), (a, 2), (b, 2), (b, 3)}.

Ejemplo 1.17
Sea R : Z Z definida por la formula: (x, y) R si y solo si y = x2 . Sin mayor rigor
podemos escribir que:

R = {(1, 1), (2, 4), (3, 9), . . . , (1, 1), (2, 4), (3, 9), . . . }
Por lo tanto

R1 = {(1, 1), (4, 2), (9, 3), . . . , (1, 1), (4, 2), (9, 3), . . . }
19

Si queremos encontrar una formula para R1 similar a la que definio a R, se puede proceder
del modo siguiente:

Sabemos que (x, y) R1 si y solo si (y, x) R. Pero esta pertenencia es equivalente a x = y 2 ,



que, al despejar a y se transforma en y = x con x cuadrado perfecto, obteniendose como

formula para R1 la siguiente expresion: (x, y) R1 si y solo si y = x o y = x con x
cuadrado perfecto.

Se puede aplicar la misma idea aun si la relacion esta expresada en un lenguaje menos formal
como veremos a continuacion.

Ejemplo 1.18
En N definimos R por (x, y) R si y solo si x divide a y o x es mayor que y. Entonces
(x, y) R1 (y, x) R y divide a x o y es mayor que x. Luego, (x, y) R1 si y
solo si x es un multiplo de y o x es menor que y.

Volviendo a la situacion general, el grafico de R1 es facil de dibujar a partir del de R. Puesto


que los puntos (a, b) y (b, a) son simetricos con respecto a la recta bisectriz del primer y tercer
cuadrante, tenemos que el grafico de R1 es simplemente la figura simetrica con respecto
a dicha bisectriz del grafico de R. En las figuras que sigue, se han dibujado dos parejas de
curvas, en que cada una es la inversa de la otra. En una de ellas ambas curvas son funciones,
mientras que en la otra una no lo es. Tambien las figuras 1.15 y 1.16, que se encuentran mas
adelante, corresponden a una curva y su inversa.

Figra 1.12 Figra 1.13

1.3.1. EJERCICIOS
6. Sean A, B, C conjuntos no vacos. Demostrar que:

a) (A B) C = (A C) (B C)
b) (A B) C = (A C) (B C)
20

c) (A B) C = (A C) (B C)

7. Sean A y B conjuntos. Pruebe que:

(A B)0 = (A0 B) (A0 B 0 ) (A B 0 )

8. Haga una lista de todas las relaciones de {a, b, c} a {s}

9. Cuantas relaciones existen de un conjunto de m elementos a un conjunto de n elemen-


tos?

10. Sean S, T relaciones de X en Y . Pruebe que:

(a) (S T )1 = S 1 T 1
(b) (S T )1 = S 1 T 1

11. Las siguientes relaciones son de R en R. Indique la figura que representan.

(a) {(x, y) : y = x + 2}
(b) {(x, y) : y = x2 }
(c) {(x, y) : x2 + y 2 = 4}
(d) {(x, y) : y = 0}
(e) {(x, y) : y 2 = x}
(f) {(x, y) : 0 x 1, 0 < y < x}
(g) {(x, y) : 0 < x < 1, x + y = 1}
(h) {(x, y) : 0 < x < 1}

12. Usando el hecho que dos conjuntos son iguales si y solo si tienen los mismos elementos,
pruebe el teorema 1.2.
21

1.4. FUNCIONES
Sin formalismo, podemos decir que una funcion de un conjunto A en un conjunto B es
una regla o ley que a cada elemento de A le asocia un unico elemento de B. As, por ejemplo,
sean A y B los numeros enteros y la ley, que a cada numero se le asocia su cubo; entonces
el numero 2 esta asociada al 8, el 5 al 125, el 2 al 8, etc. En general podemos decir que el
numero x esta asociado al numero x3 . Podemos escribir cada elemento con su asociado como
una pareja ordenada. As esta funcion dara origen a todas las parejas del tipo (x, x3 ) con x
un numero entero.

Estas observaciones nos permiten reducir el concepto de funcion al concepto de relacion y


as hacer innecesario introducir nuevos conceptos primitivos. La definicion que viene indica
las condiciones que debe cumplir una relacion para que sea funcion.

Definicion 1.7
Sea F : A B una relacion. Diremos que F es una funcion de A en B si y solo si

1. Dom F = A

2. ((x, y) F (x, y 0 ) F ) y = y 0

La primera condicion dice que todo elemento del conjunto A es primer elemento de alguna
pareja de F y la segunda agrega que no puede ser elemento de mas de una pareja. En resumen,
tenemos que F es una funcion de A en B si y solo si F es una relacion de A en B y cada
elemento de A es primer elemento de una y solo una pareja de F .

Ejemplo 1.19
Sea A = {1, 2, 3, 4} y B = {x, y, z}. Son funciones de A en B:

a) F1 = {(1, x), (2, x), (3, y), (4, y)}

b) F2 = {(1, z), (2, z), (3, z), (4, z)}

No son funciones de A en B

c) R1 = {(1, z), (2, y), (3, z)} (no hay pareja que empiece con 4).

c) R2 = {(1, x), (1, y), (2, x), (3, z), (4, z)} (hay dos parejas que empiezan con 1).

Nota
Usaremos letras tales como F, G, H, . . . , f, g, h para indicar funciones.
Ademas si F : A B y x A, a su unica imagen la designaremos por F (x):

Ejemplo 1.20
Sea F : R R definida por (x, y) F si y solo si y = 2x + 3.
22

Notemos que cada x R tiene una y solo una imagen. En efecto:

Sean (a, b1 ), (a, b2 ) elementos de F . Entonces

b1 = 2a + 3
b2 = 2a + 3
Luego, b1 = b2 .

por lo tanto,F es una funcion. F 1 queda definida por


x3
(x, y) F 1 (y, x) F x = 2y + 3 y =
2
x3
La expresion y = , es la que consideraremos como la ecuacion que define la relacion
2
x3
inversa. Es conveniente reemplazar y por F 1 (x). Nos queda F 1 (x) = .
2
En este caso, es facil ver que F 1 tambien es funcion, pero en el ejemplo anterior ni F11 ni
F21 eran funciones.

Definicion 1.8
Sea F : A B una funcion.

a) F es 1 a 1 o inyectiva si y solo si

(x1 , x2 A x1 6= x2 F (x1 ) 6= F (x2 )).

b) F es sobre o epiyectiva si y solo si para todo y B existe x A tal que y = F (x).

c) F es biyectiva si y solo si es 1 a 1 y sobre.

La condicion a) dice que elementos distintos de A tienen imagenes distintas y la b) dice que
todos los elementos de B son imagenes de, por lo menos, algun elemento de A.

Ejemplo 1.21
Es facil comprobar que la funcion F : R R definida por F (x) = ax + b , con a 6= 0 es una
biyeccion ya que su grafico es una recta no paralela al eje x.

Ejemplo 1.22
Anteriormente vimos el ejemplo R : N Z definida por
  
x 1x
xRy x par y = x impar y = .
2 2
Es facil ver que R es una funcion. Probaremos que es una biyeccion.

Sea x1 6= x2 . Deberemos analizar por casos:


x1 x2
i) x1 par x2 par 6= R(x1 ) 6= R(x2 ).
2 2
23

x1 1 x2
ii) x1 par x2 impar 6= R(x1 ) 6= R(x2 )
2 2
1 x1 1 x2
iii) x1 impar x2 impar 6= R(x1 ) 6= R(x2 )
2 2

Esto prueba que R es 1 a 1.

Sea y Z. Si y > 0 tomemos x como 2y. Luego, R(x) = y.

Si y 0 tomemos x = 2y + 1. Luego, R(x) = y. Esto prueba que R es sobre.

Definicion 1.9
Por IA designaremos la funcion identidad del conjunto A. Ella queda definida por IA (x) = x.
Es inmediato que esta funcion es biyectiva.

Definicion 1.10
Sea F : A B y G : B C dos funciones. Definimos una relacion de A en C, que
anotaremos G F del modo siguiente:

(G F )(x) = G(F (x))

El siguiente esquema ilustra esta operacion

A F -B
Q
Q
Q
Q
GoF Q
Q
G
Q
Q
Q
Q
s?
Q
C
Figura 1.14

Teorema 1.3
Sea F : A B y G : B C funciones. Entonces G F es una funcion de A en C.

Demostracion
Sea x A. Entonces (x, F (x)) es la unica pareja de F con primer elemento x. Tambien, como
F (x) B se tiene que (F (x), G(F (x))) es la unica pareja de G con primer elemento F (x).
Entonces (x, G(F (x))) es la unica pareja de G F con primer elemento x. Notemos que su
imagen en esta nueva funcion es G(F (x)). Esto nos permite escribir que (GF )(x) = G(F (x)).

Ejemplo 1.23
Sea A = {1, 2, 3}; B = {x, y, z} y C = {a, b}.
24

Sea F = {(1, x), (2, x), (3, y)} y G = {(x, a), (y, b), (z, a)}.

Entonces:
(G F )(1) = G(F (1)) = G(x) = a

(G F )(2) = G(F (2)) = G(x) = a

(G F )(3) = G(F (3)) = G(y) = b


luego G F = {(1, a), (2, a), (3, b)}

Ejemplo 1.24
1
Sea F en Q {0} definida por F (x) = .
x
 
1
Entonces (F F )(x) = F = x.
x

Ejemplo 1.25
Sea F en A. Entonces (F IA )(x) = F (IA (x)) = F (x)

y (IA F )(x) = IA (F (x)) = F (x).

Nota
Naturalmente, dos funciones F y G son iguales cuando miradas como conjuntos son iguales,
es decir, tienen los mismos elementos; pero para que esto ocurra basta comprobar que tienen
el mismo dominio y que para todo x del dominio comun se cumple que F (x) = G(x). Por
esto, del ejemplo anterior, podemos escribir que IA F = F IA = F .

Ejemplo 1.26
Sea F en Q dada por F (x) = 2x + 1

y G en Q dada por G(x) = x2


Entonces

(F G)(x) = F (x2 ) = 2x2 + 1

(G F )(x) = G(2x + 1) = (2x + 1)2


Este ejemplo muestra que la composicion de funciones no es conmutativa.

Nota
Sea F : A B una funcion. En general, F 1 no tiene por que ser funcion. El siguiente
teorema nos muestra la condicion que debe cumplir F para que F 1 sea tambien una funcion.

Teorema 1.4
Sea F : A B una funcion. F es una biyeccion si y solo si F 1 es una funcion.

Demostracion
25

Supongamos que F es biyeccion. Debemos probar que Dom F 1 = B y que cada elemento de
B tiene solo una imagen por F 1 .

Sea y B. Como F es sobre existe x A tal que (x, y) F . Luego, (y, x) F 1 .Esto prueba
que el dominio de F 1 es todo B.

Supongamos que (y, x) , (y, x0 ) F 1 , entonces (x, y)(x0 , y) F . Por ser F 1 a 1 se tiene que
x = x0 . Esto prueba que cada elemento de B tiene solo una imagen por F 1 . Luego, F 1 es
una funcion.

Supongamos, ahora, que F 1 es una funcion. Debemos probar que F es biyeccion. Sean
x, x0 A y tales que x 6= x0 . Probaremos que F (x) 6= F (x0 ).

Tenemos que (x, F (x)), (x0 (F (x0 )) F .

Luego, (F (x), x), (F (x0 ), (x0 )) F 1 .

Si F (x) = F (x0 ) entonces x = x0 , porque F 1 es funcion. Pero esto contradice el hecho que
x 6= x0 , luego F (x) 6= F (x0 ). Esto prueba que F es 1 a 1. Probemos que F es sobre.

Sea y B, como F 1 es funcion existe F 1 (y). Sea x = F 1 (y).


Entonces (y, x) F 1 . Luego, (x, y) F , es decir que y = F (x).

Teorema 1.5
Sea F : A B una biyeccion. Entonces F F 1 = IB y F 1 F = IA .

La demostracion es muy sencilla. La dejaremos al lector.

Definicion 1.11
Sea R una relacion de X en Y . Para cada A X, sea

R (A) = {y : x (x A (x, y) R)} .

R (A) se llama el conjunto imagen de A por la relacion R y esta formado por todas las image-
nes de los elementos de A.

Claramente se ve que R es una funcion de SX en SY . Se llama la funcion rango. En particular,


si F : X Y es funcion entonces F (A) = {F (x) : x A}. Ademas, en este caso, si B Y
dejamos al lector comprobar que:

F 1 (B) = {x : x X F (x) B}.


Las principales propiedades de esta funcion vienen dadas en el siguiente teorema.

Teorema 1.6
Sea R una relacion de X en Y . Sean A1 y A2 subconjuntos de X. Entonces,

a) R (A1 A2 ) = R (A1 ) R (A2 )

b) A1 A2 R (A1 ) R (A2 )
26

c) R (A1 A2 ) R (A1 ) R (A2 )

Demostracion

a)

y R (A1 A2 ) x(x A1 A2 (x, y) R)


x((x A1 x A2 ) (x, y) R)
x(x A1 (x, y) R) x(x A2 (x, y) R)
y R (A1 ) y R (A2 ) y R (A1 ) R (A2 )

b) A1 A2 A2 = A1 A2 . Entonces por a)
R (A2 ) = R (A1 ) R (A2 ). Esto fuerza que
R (A1 ) R (A2 ).

c) Como A1 A2 A1 y A1 A2 A2 tenemos, por b), que R (A1 A2 ) R (A1 ) y


R (A1 A2 ) R (A2 ).
Luego, R (A1 A2 ) R (A1 ) R (A2 ).

Aparte de la composicion, se puede definir otras operaciones entre funciones, cuando estas sean
con valores en R. De hecho, aprovechamos las operaciones aritmeticas de R para generalizarlas
como operaciones con funciones. La siguiente definicion indica el procedimiento.

Definicion 1.12
1
Sean f y g funciones de A en R. Sea c R. Definimos las funciones f + g, f g, c f y ,
f
todas de A en R por las formulas siguientes:

a) (f + g)(x) = f (x) + g(x)

b) (f g)(x) = f (x) g(x)

c) (c f )(x) = c f (x) para c numero real.

1 1
d) (x) = siempre que f (x) 6= 0.
f f (x)

Ejemplo 1.27
Sean f y g de R en R dadas por las formulas f (x) = 5x 1 y g(x) = 2x2 + 3. Entonces
27

(f + g)(x) = f (x) + g(x) = 5x 1 + 2x2 + 3 = 2x2 + 5x + 2

(f g)(x) = (5x 1)(2x2 + 3) = 10x3 2x2 + 15x 3

(1 g)(x) = (1)g(x) = 2x2 3

(f g)(x) = (f + (1)g)x = 5x 1 2x2 3 = 2x2 + 5x 4

1 1 1
(x) = siempre que x 6= .
f 5x 1 5

2x2 + 3
   
g 1 1
(x) = g (x) = siempre que x 6= .
f f 5x 1 5
 
1
En estos dos ultimos casos el dominio de las funciones es R .
5
Para hacer notar la diferencia calculemos tambien f g y g f .

(f g)(x) = f (g(x)) = f (2x2 + 3) = 5(2x2 + 3) 1 = 10x2 + 14

(g f )(x) = g(f (x)) = g(5x 1) = 2(5x 1)2 + 3 = 50x2 20x + 5.

Ejemplo 1.28
Sean f y g de R R en R dadas por las formulas f (x, y) = xy y g(x, y) = x2 + y 2 . Entonces

(f + g)(x, y) = f (x, y) + g(x, y) = xy + x2 + y 2

(f g)(x, y) = xy(x2 + y 2 )

Notemos que la definicion nos permite sumar y multiplicar estas funciones aunque en su do-
minio no existan estas operaciones.

Nota
Una observacion sobre los graficos de las relaciones en R.
Mirando sus graficos es posible percatarse si se trata de una funcion, si es 1 a 1, cual es su
recorrido y construir el grafico de la relacion inversa. Hay que observar las siguientes pro-
piedades: Si toda recta paralela al eje Y corta el grafico en, a lo mas, un punto, entonces es
funcion; si toda recta paralela al eje X corta grafico en a lo mas un punto, entonces es 1 a
1. La proyeccion del grafico sobre el eje Y indica su recorrido. No hemos justificado mayor-
mente estas afirmaciones porque son consecuencias inmediatas de las definiciones; pero para
el grafico de la relacion inversa repetiremos algunas explicaciones.

Recordemos que (a, b) es un punto de la relacion si y solo si (b, a) es un punto de su inversa,


pero (a, b) y (b, a) son puntos simetricos con respecto a la recta que es diagonal del primer
y tercer cuadrante. Luego, el grafico de la relacion inversa se construye dibujando la imagen
simetrica del grafico de la relacion con respecto a la recta y = x.
28

Ejemplo 1.29
1
Consideremos la relacion de R en R dada por y = . Tiene el grafico que muestra la
x2 1
figura.

Figura 1.15

Toda recta paralela al eje Y corta su grafico en exactamente un punto, excepto las rectas que
pasan por -1 y por 1 que no lo cortan. Luego, es una funcion de dominio R {1, 1}. Hay
rectas paralelas al eje X que cortan el grafico en mas de un punto. Luego, no es una funcion
1 a 1. El recorrido de ella es R+ {x : x 1}. Por ultimo, el grafico de la inversa es el de
la figura siguiente:

Figura 1.16

Notemos que la relacion inversa no es funcion.


Es inmediato de la definicion de R1 , que una formula para la relacion inversa se consigue
intercambiando x con y en la que define R. En este ejemplo obtenemos x = y211 de donde,
despejando y: r
1+x
y=
x
29

Nota
Aprovecharemos la ocasion para agregar otros casos de cambios de variables.

Si en la ecuacion de una curva se cambia:


1. x por x, la nueva curva es la reflexion de la original en el eje y. Analogamente al
cambiar y por y.
2. x por x e y por y se obtiene la reflexion de la original en el origen.
3. x por y e y por x obtenemos una rotacion en 90 grados en torno al origen.
4. x por (x h) se produce una traslacion de monto h en la direccion del eje x.

Definicion 1.13
Sea R una relacion en A. Diremos que R es:
a) Refleja si y solo si para todo x A, (x, x) R. En otras palabras, si y solo si IA R.
b) Irrefleja si y solo si para todo x A, (x, x) 6 R.
c) Simetrica si y solo si (x, y) R implica que (y, x) R.
d) Antisimetrica si y solo si (x, y) R y (y, x) R implica que x = y.
e) Transitiva si y solo si (x, y) R y (y, z) R implica (y, z) R.
Ejemplo 1.30
Sea R en N dada por (x, y) R si y solo si x es un divisor de y. Entonces, se puede comprobar
facilmente, que R es refleja, antisimetrica y transitiva.
Ejemplo 1.31
Sea L el conjunto de todas las rectas de un plano y sea R la relacion de L en L dada por
(x, y) R si y solo si x e y son rectas paralelas. Entonces R es irrefleja, simetrica y transitiva.
La relacion S de ser rectas perpendiculares es irrefleja, simetrica pero no es transitiva.
Definicion 1.14
a) Una relacion se dice que es de equivalencia si y solo si es refleja, simetrica y transitiva.
b) Una relacion se dice que es un orden parcial si y solo si es refleja, antisimetrica y
transitiva; es un orden estricto si y solo si es irrefleja, antisimetrica y transitiva.
Notemos que la igualdad es una relacion de equivalencia mientras que menor es un orden
estricto y subconjunto es un orden parcial.
Ejemplo 1.32
En N N definamos la siguiente relacion E:
(a, b) E (c, d) si y solo si a + d = b + c.
Comprobemos que E es de equivalencia. Es inmediato que es refleja y simetrica. Veamos la
transitividad:
(a, b)E(c, d) (c, d) E(e, f ) a + d = b + c c + f = d + e.
Sumando sale que a + d + c + f = b + c + d + e, de donde a + f = b + e. Luego, (a, b) E(e, f ).
30

Definicion 1.15
Sea A un conjunto y P1 , P2 , P3 , . . . , Pi , . . . una coleccion, que puede ser finita o infinita, de
subconjuntos de A. Diremos que esta coleccion es una particion de A si y solo si los Pi cumplen
con las siguientes condiciones:

i) Ninguno de ellos es el vaco. En smbolos: Pi 6= para todo i.

ii) Son disjuntos dos a dos; es decir Pi Pj = cuando i 6= j.

iii) La union de todos ellos es A.

De las dos ultimas condiciones se desprende que todo elemento de A es miembro de un y solo
un elemento de la particion.

Ejemplo 1.33
En N N definamos una coleccion de subconjuntos del modo siguiente: Para cada i numero
entero sea Pi = {(a, b) N N : a + i = b}. Veamos algunos de estos conjuntos en forma
concreta
P0 = {(1, 1), (2, 2), (3, 3), . . . }

P1 = {(1, 2), (2, 3), (3, 4), . . . }

P1 = {(2, 1), (3, 2), (4, 3), . . . }

P2 = {(1, 3), (2, 4), (3, 5), . . . }

P2 = {(3, 1), (4, 2), (5, 3), . . . } , etc.


Es bastante facil comprobar que ningun Pi es vaco; que son disjuntos dos a dos y que cada
elemento de N N pertenece a alguno de ellos. En resumen, la coleccion {Pi }iZ es una
particion de N N.
En lo que sigue estableceremos que relaciones de equivalencia y particiones pueden en un
cierto sentido, ser identificadas.

Definicion 1.16
Sea R una relacion en un conjunto A. Sea x A. Definimos la clase de x y la anotamos Cx
del modo siguiente
Cx = {y A : (x, y) R}.
Dicho en palabras Cx es la coleccion de imagenes de x por la relacion R.

Teorema 1.7
Sea E una relacion de equivalencia en un conjunto A. Entonces

i) Para todo x A, x Cx .

ii) Para todo x e y A se cumple que Cx = Cy si y solo si (x, y) E.

iii) Si (x, y) 6 E entonces Cx Cy = .

Demostracion
31

i) Por ser E refleja tenemos que (x, x) E. Luego, x Cx .

ii) Sea (x, y) E. Si t Cx entonces (x, t) E. Ademas, como E es simetrica, tenemos


que (y, x) E. Por la transitividad, obtenemos que (y, t) E, de lo que se concluye que
t Cy . Esto prueba que Cx Cy . En forma totalmente similar obtenemos que Cy Cx .
Luego, Cx = Cy . Supongamos ahora que Cx = Cy . Como y Cy tenemos que tanto x
como y son elementos de Cx . Luego, (x, y) E.

iii) Sea (x, y) 6 E y supongamos que t Cx Cy . Entonces tanto (x, t) como (y, t) estan en
la relacion E. Por simetra y transitividad obtenemos que (x, y) E. Contradiciendo la
hipotesis. Luego, Cx Cy = .

Observacion.
Cuando E es una relacion de equivalencia la clase Cx del elemento x de A recibe el nombre
de la clase de equivalencia de x; ademas la coleccion de todas las clases se llama el conjunto
cuociente y se denota por A/E. Del ultimo teorema vemos que este conjunto cuociente es una
particion de A.

Ejemplo 1.34
En los enteros definamos la relacion E del modo siguiente: xEy si y solo si x y es multiplo
de 5. Es facil ver que E es de equivalencia. El conjunto cuociente Z/E tiene los siguientes 5
elementos:
C0 = {0, 5, 10, . . . 5, 10, 15 . . . }
C1 = {1, 6, 11, . . . 4, 9, 14 . . . }
C2 = {2, 7, 12, . . . 3, 8, 13 . . . }
C3 = {3, 8, 13, . . . 2, 7, 12 . . . }
C4 = {4, 9, 14, . . . 1 6, 11 . . . }
El siguiente teorema es la otra cara de la observacion anterior.

Teorema 1.8
Sea {P1 , P2 , P3 , . . . } una particion de un conjunto A. En A definamos la siguiente relacion
que anotaremos por E : (x, y) E si y solo si x e y son miembros de un mismo Pi . Entonces
E es de equivalencia y A/E es la particion {P1 , P2 , P3 , . . . }.

La demostracion, totalmente rutinaria, se la dejamos al lector.

De estos dos ultimos teoremas vemos que cada relacion de equivalencia determina una par-
ticion, a saber, su conjunto cuociente y cada particion, una relacion de equivalencia. Mas
aun, es sencillo probar que si una particion {Pi } sobre A determina la relacion E entonces
{Pi } = A/E y viceversa, es decir, si con E construimos A/E esta particion determina la
relacion E. Es por esto que podemos identificar relaciones de equivalencia y particiones.
32

1.4.1. EJERCICIOS
13. Sea A un conjunto con m elementos y B uno con n elementos. Que relacion debe haber
entre m y n para que, exista una funcion de A en B 1 a 1; exista una funcion de A en
B sobre; exista una biyeccion de A en B?.

14. Encuentre una biyeccion de P en N y otra de N en N P. Haga la composicion de ambas.


(P es el conjunto de los numeros naturales pares).

15. Sea f : X Y y g : Y X funciones tales que g f es la identidad en X. Pruebe


que f es 1 a 1 y g es sobre.

16. Para cada A U se define su funcion caracterstica.

fA : U {0, 1}

1 si x A
fA (x) =
0 si x 6 A

Pruebe que esta funcion tiene las siguientes propiedades:

a) fA0 (x) = 1 fA (x)


b) fAB (x) = fA (x) fB (x)
c) fAB (x) = fA (x) fAB (x)

17. Sea f : X Y una funcion y sean A, A1 , A2 X y B Y . Pruebe que:

a) f (A1 ) f (A2 ) f (A1 A2 )


b) f (A f 1 (B)) = f (A) B.

18. Dadas f, g : R R tal que f (x) = x2 + 2x 3 y g(x) = 3x 4, encontrar

f g, g f, g 1 f, f g 1 , f g g.

1
19. Sea f (x) = con x 6= 1. Calcule f f f .
1x
20. Sea f : R R definida por


x + 2 si x 2
f (x) =
2x si x > 2

Pruebe que f es 1 a 1 y sobre. Encuentre la ecuacion para f 1 (x).


33

21. Sea
2
x + 2 si x > 0
f (x) =
x+2 si x 0


2x + 5 si x > 3
g(x) =
x2 si x 3

Calcular g(f (x)) para los siguientes valores de x: 0, 1, 2, -1, -2

22. Sean f, g : R R definidas por f (x) = ax + b y g(x) = cx + d. Encuentre las


condiciones para los coeficientes a, b, c y d para que f g = g f .
23. Dada las funciones f (x) = 2x + 4 y g(x) = senx, exprese usando solo estas y las
operaciones con funciones las siguientes:
sen x2 , ax2 + 6x + c, 4x + 9, 4x2 + 12x + 9.

24. Para cada una de las siguientes relaciones indique si ella, o su inversa es funcion. Justi-
fique la respuesta.

a) xRy 0 x < 1 0 y < 1 x < y


b) xRy x = y x = y
c) xRy (x = y y 0) (x = y y 0)
d) xRy x3 = y
e) xRy x = y 2 .

25. Dada la funcion f (x) cuyo grafico es:

Figura 1.17

Dibuje los graficos de las


siguientes funciones f (x 2),
f (2 x), f (x/2).
34

26. Se dice que una funcion es creciente si y solo si para todo par de numeros de su dominio
a y b se cumple que si a < b entonces f (a) f (b).,
Pruebe que la funcion f (x) = x3 + 3x + 5 es creciente en todo su dominio. Pruebe que
1
g(x) = x + es creciente si x 1.
x
x
Que puede decir de la funcion h(x) = ?
1 + x2
27. Indique las propiedades que tienen las siguientes relaciones:

a) Menor o igual en los numeros reales.


b) Interseccion no vaca en los subconjuntos de U .
c) Semejanza de triangulos en un conjunto de triangulos.
d ) {(1, 2), (1, 1), (2, 2), (3, 3), (2, 1)} en {1, 2, 3}.
e) Ser hermano en un conjunto de personas.

28. Sea n un numero natural fijo. En Z definimos la siguiente relacion R por (a, b) R si y
solo si (a b) es multiplo de n. Pruebe que R es una relacion de equivalencia.

29. Sea R una relacion en A. Demuestre que es antisimetrica si y solo si R R1 IA .

30. Sea R una relacion de A en A.


Que puede decir de R1 si R es de equivalencia?

31. Sea A = {0, 1, 2, 3, 4, 5, 6, 7}. Se definen en A las siguientes relaciones:

a) xy si y solo si y x < 8
b) xy si y solo si y x es divisible por 2
c) xy si y solo si 4 < x y
d ) xy si y solo si y x = 1

Indique las propiedades que tienen estas relaciones.


35

1.5. ALGO MAS SOBRE BIYECCIONES


Sean A y B dos conjuntos con un numero finito de elementos. Es obvio que si existe una
biyeccion entre ellos entonces tienen la misma cantidad de elementos y viceversa. Si solo su-
pieramos que existe una funcion inyectiva de A en B, entonces solamente podramos afirmar
que el numero de elementos de A no es mayor al de B. Si ademas conocieramos que la funcion
no es sobre, entonces podremos asegurar que A tiene menos elementos que B.

Estas ideas pueden extenderse facilmente a conjuntos infinitos, pero en este caso no podremos
hablar del mismo numero de elementos porque no hay un numero que los cuente. La forma
con la cual nos referiremos a este concepto para conjuntos infinitos, la exponemos en la
siguiente definicion.

Definicion 1.17
Diremos que los conjuntos A y B tienen la misma cardinalidad si y solo si existe una biyeccion
entre ellos.

Este concepto lo abreviaremos diciendo que A y B son equinumerosos. Ademas, si sucede que
un conjunto es equinumeroso con N o N0 diremos que es numerable.

Ejemplo 1.35
Sea P el conjunto de los naturales pares (el 0 incluido) e I el de los impares. Es evidente que
f : P I dada por f (x) = x + 1 es una biyeccion entre ellos. Luego, son equinumerosos.
Intuitivamente nos parece que P e I deben tener la misma cantidad de elementos y la biyeccion
confirma nuestra intuicion.

Ejemplo 1.36
Sea P el conjunto de los naturales pares y sea f : P N0 dada por f (x) = x2 . Es evidente
que es una biyeccion, por lo tanto P y N0 son equinumerosos, resultado que esta muy en
desacuerdo con nuestra intuicion. Nos obliga a aceptar que el todo no es mayor que alguna de
sus partes lo que contradice el principio euclideano el todo es mayor que cualquiera de sus
partes.

Ejemplo 1.37
El siguiente diagrama muestra una biyeccion entre el segmento [0, 1) y la semirecta [0, ).

-1 1
HH
H
HH
Hx
HH
H
HH f(x)
H -
0
Figura 1.18

La formula para f (x) se deduce facilmente por semejanza de triangulos. Se obtiene


36

x
f (x) = .
1x
Otro resultado que no concuerda con nuestra intuicion. Un trazo finito tiene tantos puntos
como una semirecta infinita.

Ejemplo 1.38
Consideremos el siguiente esquema:

6 21
5 15 &
4 10 16 &
3 6 11 17 &
2 3 7 12 18 &
1 1 4 8 13 19 &
0 0 2 5 9 14 20 27

0 1 2 3 4 5 ... eje x

En este diagrama los numeros naturales se han ubicado siguiendo las diagonales en 135 gra-
dos. El define una biyeccion de N N en N. Dejamos al lector comprobar (ver ejercicio 15 en
13.1) que queda descrita por la funcion

(x + y)2 + 3x + y
f (x, y) = .
2
Ademas nos muestra que N y N N son equinumerosos. Mas aun, cada fila es una sucesion
numerable dada por la formula

(x + q)2 + 3x + q
fq (x) =
2
donde q indica el numero de la fila. En particular, la formula para la primera fila es:

x2 + 3x
f0 (x) =
2
que se puede deducir independientemente como veremos en el siguiente captulo. La cantidad
de filas que tiene el diagrama es numerable y cada una de ellas es disjunta de las restantes.
De estas observaciones se desprende el siguiente teorema: La union numerable de conjuntos
numerables sigue siendo numerable.

Estos resultados tan paradojales, alguno de los cuales conocidos por los antiguos griegos, lle-
varon a Aristoteles (siglo IV a.d.C) a dar el siguiente consejo: En matematicas solo podemos
hablar del infinito como algo potencial, es decir como una actividad que se puede realizar
indefinidamente. Por ejemplo, cuando decimos que los numeros naturales son infinitos, lo que
realmente afirmamos es que ellos son potencialmente infinitos en el orden, en otras palabras,
dado un numero natural por muy grande que sea siempre podemos encontrar un mayor. Lo
mismo podemos afirmar de los numeros racionales, siempre podemos encontrar uno mayor o
tambien uno menor de alguno dado. Pero debemos evitar hablar del infinito como algo actual,
es decir considerar, como objetos matematicos, conjuntos con infinitos elementos, porque esto
37

nos lleva a contradecir el principio euclideano.

A lo largo de la historia, este consejo aristotelico fue tema de discusion no solo de matematicos
sino tambien de filosofos y teologos, estos ultimos, en general estaban en desacuerdo porque
consideraban que se contradeca con la existencia de Dios (existe y es infinito). Tambien,
entre los filosofos, encontramos a Giordano Bruno (1548-1600) que rechazaba la limitacion
aristotelica y entre los matematicos mas destacados, debemos considerar a Bernardo Bolzano
(1781-1848). Mas aun, este ultimo introduce la siguiente definicion para conjuntos infinitos.

Definicion 1.18
Un conjunto es infinito si y solo si es equinumeroso con alguna de sus partes propias (Se apro-
vecha para definir conjunto infinito, la propiedad que Aristoteles consideraba contradictoria).

Pero a favor de Aristoteles encontramos a matematicos tan ilustres como Gauss (1777-1855)
y Cauchy (1789-1857) ya que consideraban que usando solo el infinito en su forma potencial,
se poda desarrollar el concepto de lmite y todas sus aplicaciones.
Posteriormente, George Ferdinand Cantor (1845-1918), descartando la objecion aristotelica,
desarrolla la teora de conjuntos, llegando a resultados muy interesantes uno de los cuales
expondremos de inmediato por sus aplicacion a las biyecciones.

Teorema 1.9 (Cantor-Schroder-Bernstein).

Sean A y B dos conjuntos tales que existen f : A B y g : B A, ambas inyectivas.


Entonces existe h : A B biyectiva.

Demostracion
Probaremos primero el caso particular en que B A. Tomaremos como g la funcion identidad.
Sea C0 = A B. Definimos Cn = f (Cn1 ). Sea C la union de los Cn , con n N . Definimos
h : A B como:

f (x) si xC
h(x) =
x en otro caso

Es obvio que h esta bien definida.


Probaremos que h es 1 a 1. Sea x1 , x2 A. El unico caso no trivial se produce cuando x1 C
y x2 no pertenece a C. En este caso tenemos que h(x1 ) = f (x1 ) C. Mientras que h(x2 ) = x2
no pertenece a C. Luego, h(x1 ) 6= h(x2 ).

Probaremos ahora que h es sobre. Sea y B. Se tiene que y no pertenece C0 . Si y C,


entonces y Cn para algun n 6= 0. Luego, y es imagen de, por lo menos un, elemento de
Cn1 . Si y no pertenece a C, entonces es imagen de s mismo.
Pasamos, ahora, al caso general.
Sea f : A B y g : B A inyectivas.
Sea B1 = g(B). Entonces B1 A por lo ya probado. h1 : A B1 biyectiva.
Pero g es una biyeccion de B sobre B1 . Luego, h = h1 g 1 es una biyeccion de A sobre B.

El siguiente ejemplo muestra como aprovechar este teorema para construir biyecciones.
38

Ejemplo 1.39
2x+1
Sea A = [0, 1] y B = (0, 1). Sea f (x) = 4 . Entonces C0 = A B = {0, 1}.

C1 = { 14 , 34 }
C2 = { 38 , 58 }
7 9
C3 = { 16 , 16 }
..
.
n
1 2 +1 n
Cn = { 22n+1 , 2n+1 }
..
.
1n n
C = Cn = {0, 41 , 83 , . . . , 22n+1 , . . .} {1, 34 , 58 , . . . , 22n+1
+1
, . . .}.

Luego,
(2x+1)

4 si xC
h(x) =

x en otro caso
Es una biyeccion de [0, 1] en (0, 1).

Para concluir este apendice, dos situaciones desarrolladas por Cantor que aseguran la no
existencia de biyecciones para ciertos casos.

Teorema 1.10
Los numeros reales del intervalo (0,1) no son numerables.

Demostracion
Los numeros reales de este intervalo los podemos considerar como numeros de la forma 0,
a1 a2 . . . con infinitos decimales. (En el caso que el desarrollo decimal sea finito, por ejemplo
1
2 = 0, 5, simplemente los siguientes decimales seran 0). El argumento es por contradiccion.
Supongamos que existe una funcion biyectiva f : N (0, 1).

Consideremos el siguiente esquema

f (1) = 0, a1 a2 a3 . . .
f (2) = 0, b1 b2 b3 . . .
..
.
f (n) = 0, l1 l2 l3 . . .
..
.

Escribamos el siguiente decimal, al que llamaremos q.

0, x1 x2 x3 . . .

donde x1 6= a1 , 0, 9. x2 6= b2 , 0, 9. x3 6= c3 , 0, 9 etc.
Este numero, que pertenece al intervalo (0, 1), no esta en la lista, ya que para todo todo n, q
es distinto de f (n). Luego, f no es sobre.
39

Este resultado nos obliga a aceptar la existencia de infinitos de diferentes tamanos.

El argumento anterior, conocido como Argumento diagonal de Cantor, ha resultado ser util
en otros contextos. Veamos un ejemplo.

Ejemplo 1.40
Consideremos el siguiente diagrama:

0 1 2 3 4 5

{} no no no no no no
{1} no si no no no no
{1,3,5} no si no si no si

Nos muestra que cada subconjunto de N lo podemos caracterizar por una sucesion de s y
no. En el diagrama se muestra las sucesiones de ,{1}, {1, 3, 5}. En este listado no pueden
estar todos los subconjuntos de N, pues si en la diagonal se cambia cada s por un no y cada
no por un s, representara un subconjunto de N que no esta en el listado. Luego, tenemos la
siguiente conclusion: Los subconjuntos de N no son numerables.

1.5.1. EJERCICIOS.
32. Con respecto al Ejemplo 1.37, de que pareja es imagen el numero 1000?

33. Determine una formula para la siguiente biyeccion de N N


4 24 23 22 21 20
3 15 14 13 12 19
2 8 7 6 11 18 Donde las flechas indican el orden de la numera-
1 3 2 5 10 17
0 0 1 4 9 16

0 1 2 3 4 eje x

cion.

34. Defina una biyeccion entre los intervalos (0,1) y [0,1] distinta a la del ejemplo 1.39.
40

1.6. SUGERENCIAS O SOLUCIONES A LOS EJERCICIOS


DEL CAPITULO.
1. 1 = U (A B C)0 ; 2 = A (B C)0 ; 3 = A C B 0 etc.

2. a) (2, 3, 5, 6, 8) b) (3, 5, 6, 8)

3. a) a) es verdadera.
b) b) y c) son falsas.

4. a) Hay violinistas que no son militares. (Figura 4)


b) Algunos uniformados no son bomberos. (Figura 3)
c) Hay escritores que no son militares. (Figura 2)

5. Se debe a que Ningun M es P y Algun S es M son equivalentes con Ningun P es


M y Algun M es S respectivamente. Con respecto al otro punto, solo se afirma que
tanto S como P son subconjuntos de M, pero no sabemos si tienen elementos en comun
o son disjuntos.

6. a) (x, y) (A B) C (x A x B) y C

(x A y C) (x B y C) (x, y) A C B C
b) Similar a (a)
c)
(x, y) (A B) C x A x 6 B y C

x A y C x 6 B y C

(x, y) A C (x, y) 6 B C (x, y) A C B C


7.
(x, y) (A B)0 (x, y) 6 (A B)

(x 6 A y B) (x 6 A y 6 B) (x A y 6 B)

(x A0 y B) (x A0 y B 0 ) (x A y B 0 )

(x, y) (A0 B) (A0 B 0 ) (A B 0 )

8. Son todos los subconjuntos de {(a, s), (b, s), (c, s)} que son, ademas de este mismo, los
siguientes:

{(a, s), (b, s)}, {(a, s), (c, s)}, {(b, s), (c, s)}, {(a, s), (b, s)}, {(c, s)},

9. Son todos los subconjuntos de un conjunto con m n elementos. En total son 2mn .
41

10.
(x, y) (S T )1 (y, x) S T

(x, y) S 1 (x, y) T 1 (x, y) S 1 T 1

El otro caso es analogo.


11. a) Es una recta.
b) Es una parabola.
c) Es una circunferencia con centro en el origen y radio 2.
d) El eje X.
e) Es un parabola.
f) Es el interior del triangulo de vertices (0, 0), (1, 0) y (1, 1).
g) Es el interior del trazo de extremos (1, 0) y (0, 1).
h) Es el interior de la franja encerrada por el eje Y y la recta paralela a el, que pasa
por (1, 0).
12. (a, b) = (c, d) es equivalente con {(a), (a, b)} = {(c), (c, d)}.
Los siguientes casos son los posibles:

i) {a} = {c} y {a, b} = {c, d}. Resulta de inmediato que a = c y b = d.


ii) {a} = {c} y {a, b} = {c}. Esta situacion obliga que a = c = b. Ademas, ya sea
{c, d} = {a} o {c, d} = {a, b} siempre resultara que c = d = a.
iii) {a} = {c, d} y {a, b} = {c, d}. De la primera igualdad se desprende que a = c = d
y de la segunda a = b = c.
iv) {a} = {c, d} y {a, b} = {c}. Es similar al anterior. Luego, en todos los casos se
tiene que a = c y b = d.

13.
f : A B uno a uno, si y solo si m n

f : A B sobre, si y solo si m n

f : A B biyectiva, si y solo si m = n
14.
Por ejemplo:

f : P N dada por f (x) = x/2 es una biyeccion

g : N N P dada por g(x) = 2x 1 es biyeccion

(gof ) : P N P. Queda dada por (gof )(x) = g(f (x)) = g(x/2) = x 1.


15. Sean x1 , x2 X tales que f (x1 ) = f (x2 ). Luego, g(f (x1 )) = g(f (x2 )). Es decir,
(gof )(x1 ) = (gof )(x2 ). Luego, x1 = x2 , lo que demuestra que f es uno a uno.
Sea x X. Sea y = f (x). Tenemos que g(y) = g(f (x)) = x. Luego, x es imagen de y
por g, lo que prueba que esta es una funcion sobre.
42

16. Probaremos solo c). Debemos verificar para los casos x (A B) y x


/ (A B).
Sea x A B. Entonces, x A x 6 B. Luego, fA (x) = 1 fAB (x) = 0, de donde
fAB (x) = 1 = fA (x) fAB (x).
Sea x 6 A B. En este caso fAB (x) = 0.
Pero, x 6 A B (x A x A B) (x 6 A).
Luego, x 6 A B (fA (x) = fAB (x) = 1) (fA (x) = fAB (x) = 0).
Luego, en ambos casos se tiene que fA (x) fAB (x) = 0.
Los casos a) y b) son mas sencillos.

17. a) Sea t (f (A1 ) f (A2 )), es decir t f (A1 ) t 6 f (A2 ). Luego, s A1 tal que
f (s) = t y x A2 , f (x) 6= t. Luego, s A1 , pero s A2 . Esto implica que s A1 A2 ,
de lo cual se desprende que t f (A1 A2 ).
b) Notemos que x A f 1 (B) si y solo si x A y f (x) B. Entonces y f (A
f 1 B) si y solo si x A tal que y = f (x) y f (x) B. Pero x A tal que y = f (x)
es equivalente con y f (A). Luego, y f (A f 1 B) si y solo si y f (A) y y B,
es decir y f (A) B.

18.
(f og)(x) = 9x2 18x + 5

(gof )(x) = 3x2 + 6x 13

x+4
g 1 (x) =
3

x2 + 2x + 1
(g 1 of )(x) =
3

x2 + 14x + 13
(f og 1 )(x) =
9

(f ogog)(x) = 81x2 270x + 221


!  
1 1 x1
19. f (f (f (x))) = f (f ( )) = f 1 =f =x
1x 1 1x x

20. Debemos probar primeramente, que si a, b R con a 6= b se tiene que f (a) 6= f (b).
Podemos suponer que a < b.

i) Si a < b 2.
f (a) = a + 2, f (b) = b + 2.
Luego, f (a) 6= f (b).
ii) Si 2 < a < b. f (a) = 2a, f (b) = 2b. Luego, f (a) 6= f (b).
iii) Si a < 2 b.
f (a) = a + 2, f (b) = 2b.
Luego, f (a) < 4 y f (b) 4.
43

Luego, f (a) 6= f (b).


Esto prueba que f es uno a uno.

Sea b R. Si b 4, tomemos a tal que a + 2 = b. Es decir a = b 2. Se tiene que a 2


y por lo tanto f (a) = b.

Si b > 4, tomemos a tal que 2a = b, es decir a = b/2.


Se tiene que a > 2 y por lo tanto f (a) = b.

Luego, f es sobre. Finalmente



x 2 si x 4
f 1 (x) =
x/2 si x > 4

21. Los respectivos valores son: 4, 9, 17, 1, 0.

22. Las condiciones son a = c y b = d.


1
23. Tenemos que x = (f (x) 4).
2
Luego,:
  2 
sin(x2 ) = g f x 24 , etc.

24. a) No es funcion. Cada x tiene infinitas imagenes. Su inversa tampoco lo es.


b) Depende de la interpretacion del o.
c) Es funcion. Su inversa no lo es.
d ) Ambas son funciones.
e) No es funcion. Su inversa si lo es.

25. Ver figura 1.19.

Grafico de f(x-2) Grafico de f(x/2) Grafico de f(2-x)

Figura 1.19

26. i) f (b) f (a) = b3 a3 + 3(b a).


Pero b3 > a3 . Luego, f (b) f (a) > 0.
44

ii) g no es creciente.
iii) Si 0 < x < y, entonces h crece.
Si x < y < 0, entonces h decrece.

27. a) Refleja, antisimetrica y transitiva. Ademas todo par de elementos es comparable.


En resumen, es un orden total.
b) Es una relacion de equivalencia.
c) Solamente es simetrica.
d ) Simetrica y transitiva.
e) Irefleja, simetrica y transitiva.

28. Es obvio que es refleja y simetrica. Sean a-b=pn y b-c=qn. Entonces, sumando obtene-
mos que a-c=(p+q)n. Luego, es transitiva.

29. Sea R R1 IA y sea (a, b) R con a 6= b. Como (b, a) R1 se tiene que (b, a)
/R
1
ya que R R IA .
Inversamente, supongamos que R es antisimetrica y que (a, b) y (b, a) R R1 . Esto
implica que (b, a) R. Luego, a = b de lo que se concluye que R R1 IA .

30. R1 es de equivalencia.

31. a) Es AxA.

b) Es una relacion de equivalencia compuesta de 2 clases.

c) Es antisimetrica. Tambien es transitiva, puesto que no se puede probar que no lo


sea.

d ) Es antisimetrica. No es refleja ni transitiva.

32. Determinaremos, primero, el pie de la diagonal donde se encuetra el numero 1000. Este
x2 + 3x
es un elemento de la primera fila, por lo tanto, es de la forma . Luego, se trata
2
x2 + 3x
de encontrar el menor x tal que 1000. Este resulta ser 44. El valor de f (44, 0)
2
es 1034. Esto nos indica que debemos retroceder 34 lugares a lo largo de esa diagonal,
(10,34). En resumen, la preimagen de 1000 es (10,34)

33. La fila de orden 0 corresponde a la sucesion x2 y la columna de orden 0, a la sucesion


y 2 + 2y. De estas observaciones se desprende que:

2
x +y si y < x
f (x, y) =
y 2 + 2y x si x y

45

34. Seguiremos las ideas que introduce el teorema 1.7. Definiremos una funcion f del inter-
valo [0, 1] en (0, 1) del modo siguiente. Para cada extremo del intervalo [0, 1], definimos
a f como una sucesion con rango en (0, 1) arbitrariamente. Por ejemplo para 0 definimos
1 1 1 1 1
la sucesion f (0) = , f ( ) = 2 , f ( 2 ) = 3 , . . .
2 2 2 2 2
1
Para 1 definimos f como otra sucesion disjunta de la anterior. Por ejemplo f (1) = ,
3
1 1
f ( ) = 2 , . . . . Para los restantes puntos del intervalo f es la identidad. Es inmediato
3 3
que es una biyeccion.
46
Captulo 2

NUMEROS ENTEROS,
INDUCCION, SUMATORIAS Y
PROGRESIONES

2.1. NUMEROS ENTEROS E INDUCCION


En este captulo no desarrollaremos los fundamentos del sistema numerico. Ese aspecto
sera tratado en los captulos sobre los numeros reales. Solo supondremos que el lector conoce
las propiedades fundamentales de la suma, resta y producto de enteros, como asimismo la
relacion de menor y de menor o igual entre ellos. Ademas dejaremos sin demostrar los prime-
ros tres teoremas que nos serviran de base para el desarrollo de los temas subsiguientes. Sus
demostraciones se encuentran en los captulos ya citados.

Recordemos que designamos por N0 al conjunto de los numeros naturales, incluido el 0 y por
N a dicho conjunto sin el 0, por Z al conjunto de los enteros y por Q al de los racionales.

Teorema 2.1
Todo conjunto de numeros naturales contiene un menor elemento. Este teorema recibe el
nombre de Principio de buen orden.

Teorema 2.2
Sean a y b numeros naturales con a 6= 0. Entonces existe un natural n tal que n a > b.
Se conoce como la Propiedad Arquimidiana.

Teorema 2.3
Sea S N0 y a S. Supongamos ademas, que S tiene la siguiente propiedad:

n(n S n + 1 S).
En palabras, todo elemento de S tiene su sucesor tambien en S.

Entonces, en estas condiciones, S contiene todos los numeros naturales mayores o iguales que
a. Notemos que si a = 1, entonces S = N.

47
48

Este teorema, conocido con el nombre de Principio de Induccion, permite construir un cier-
to tipo de definiciones y un cierto metodo de demostraciones conocidas como definiciones y
demostraciones inductivas, aplicables fundamentalmente, a situaciones en que estan involu-
crados los numeros naturales.

Antes de pasar a explicar estos metodos, desarrollaremos algunos conceptos para as tener
mas material al cual referirse en los ejemplos.

Teorema 2.4 (Teorema del Resto)


Sean a y b numeros naturales. Entonces existen dos unicos naturales q y r, tales que

b=qa+r con r < a.

Demostracion
Consideremos el conjunto S = {b, b a, b 2a, }.

Algunos de ellos son enteros positivos. Por el teorema 1, entre los no negativos, hay uno que
es el menor. Sea este, el numero b q a y designemoslo por r.

Entonces b = q a + r y r 0. Tenemos que probar que r < a. Si as no fuese, se tendra que


r a y entonces:
0 r a = b (q + 1)a < b q a = r
Luego, r no sera el menor elemento no negativo de S, ya que b (q + 1)a sera miembro de
S. En consecuencia, el suponer que r a nos llevo a una contradiccion. Por lo tanto, r < a.

Ahora demostraremos la unicidad.

Supongamos que existe otra pareja de naturales q 0 y r0 tales que

b = q0 a + r0 con 0 r0 < a.
Es inmediato que si q = q 0 se tiene que r = r0 . Por lo tanto, podemos suponer que q 0 > q.
Restando las dos expresiones para b y ordenando:

a(q 0 q) = r r0
El primer miembro de esta igualdad es mayor que a, pero el segundo es menor. Esta contra-
diccion se produjo por suponer que q 6= q 0 . Luego, q = q 0 y r = r0 .

El numero r se llama el resto de la division de b por a.

Ejemplo 2.1
Sea a = 37 y b = 15721. Por el algoritmo de la division se obtiene como cuociente q = 424, y
resto r = 33. Luego, 15721 = 424 37 + 33.
49

Definicion 2.1

a) Sean p y q dos enteros. Diremos que p divide a q o que p es un divisor de q, o que q es


un multiplo de p ssi existe un entero c tal que q = c p. En smbolos, escribiremos p|q
para denotar que p divide a q.

b) Sea p un numero natural distinto de 1. Diremos que p es un numero primo ssi sus unicos
divisores positivos son 1 y p.

c) Sean p y q dos naturales. Diremos que son primos relativos ssi no tienen divisores co-
munes excepto 1 y -1.

Nota
ssi es una abreviatura de si y solo si.

Teorema 2.5 (Propiedades de la divisibilidad)

1. a Z(a 6= 0 a|a)

2. a, b, k Z(a|b a|kb)

3. a, b, c Z(a|b b|c a|c)

4. a, b, c, m, n Z(a|b a|c a|(mb + nc))

5. a, b N(a|b b|a a = b)

6. a, b N(a|b a b)

Las demostraciones, todas muy elementales, las dejaremos de ejercicios.

El siguiente ejemplo muetra como generar reglas de divisibilidad. Escribiremos los numeros
naturales en la forma 10d + u donde u representa el dgito de las unidades y d la cantidad de
decenas. Por ejemplo, 346 se representara por 10 34 + 6.

Ejemplo 2.2 Probar que el natural n = 10 d + u es divisible por 19 ssi 19|(d + 2u).

19|(10 d + u) ssi 19|(20 d + 2u), ya que 19 y 2 son primos relativos. Si y solo si


19|(19d + d + 2u) ssi 19|(d + 2u).
En forma similar se prueba que 7|(10 d + u) ssi 7|(2 u d).

En los ejemplos que siguen veremos como el principio de induccion (Teorema 2.3) permite
construir cierto tipo de demostraciones y definiciones de uso muy frecuente, las cuales seran
llamadas demostraciones por induccion y definiciones inductivas.
50

Ejemplo 2.3
Probar que para cualquier natural n se tiene que 4n 1 es divisible por 3.

Demostracion
Sea S = {n N : 4n 1 es divisible por 3} es decir, S es el conjunto de todos los naturales
para los cuales la afirmacion es cierta. Entonces, probar que esta afirmacion es valida para
cualquier natural se convierte en probar que S = N.

El teorema 2.3 asegura que S = N si cumple con las siguientes hipotesis:

i) 1 S

ii) Si n S entonces su sucesor, es decir n + 1, tambien es elemento de S.

Como 41 1 es divisible por 3 tenemos que 1 S. Ahora debemos probar que el sucesor de
cada elemento de S tambien esta en ese conjunto. Para esto supongamos que n S, es decir,
supongamos que 4n 1 es divisible por 3. Nuestra meta es probar que n + 1 S; que equivale
a demostrar que 3 | (4n+1 1). Pero 4n+1 1 = 4(4n 1) + 3, que es divisible por 3, pues am-
bos sumandos lo son. En resumen: bajo el supuesto de que n S hemos probado que n+1 S.

Observemos que la clave del metodo consiste en probar las siguientes dos propiedades:

i) La afirmacion es valida para el numero 1.

ii) Si vale para el numero n entonces vale para su sucesor, n + 1.

Esta segunda propiedad es del tipo si p entonces q y para probarla se puede suponer la
validez de p, es decir, se puede suponer que la afirmacion es cierta para un numero n. Esta
suposicion se llama la hipotesis inductiva.

Ejemplo 2.4
1
Probar que 1 + 2 + 3 + + n = n(n + 1).
2
Lo que dice la formula es que la suma de los numeros naturales sucesivos hasta alguno es igual
a la mitad del producto del ultimo de ellos por su sucesor.

Para probarla, primero debemos establecer su validez para el caso particula n = 1. En es-
12
te caso, el primer miembro es la suma hasta 1 y el segundo es que obviamente son iguales.
2
Luego, la formula es valida para el numero 1.

Supongamosla valida para un numero n. Debemos probarla para su sucesor, es decir, debemos
probar que:
1
1 + 2 + 3 + + n + (n + 1) = (n + 1)(n + 2)
2
Por la hipotesis inductiva, como el primer miembro es (1 + 2 + 3 + + n) + (n + 1) valdra:
1
n(n + 1) + (n + 1)
2
51

Factorizando:
n  1
(n + 1) + 1 = (n + 1)(n + 2)
2 2
lo cual prueba que la formula es valida para n + 1, suponiendo su validez para n.

Ejemplo 2.5
Demostrar que n2 < 2n .

La formula es valida para en numero 1 pero no lo es para los numeros 2, 3 y 4. Vamos a probar
que si vale para n entonces, tambien es valida para n + 1 con la condicion que n sea mayor
que 4.
Es inmediato que vale para n = 5. suponiendo que vale para n, probaremos su validez para
n + 1.

(n + 1)2 = n2 + 2n + 1 < 2n + 2n + 1
ya que n2 < 2n cuando n > 4. Ademas, se prueba facilmente que 2n + 1 < 2n cuando n > 2.
Entonces, reemplazando se tiene que

(n + 1)2 < 2n + 2n = 2n+1 .


Con esto hemos establecido que la afirmacion n2 < 2n es valida para todo natural mayor que
4.

Ejemplo 2.6
Probar que el numero de puntos en que se cortan n rectas de un plano, sin que hallan dos
1
paralelas ni tres concurrentes es n(n 1).
2
Es obvio que la formula vale para una recta. Supongamos que vale para n rectas y considere-
mos n + 1 rectas en las condiciones dadas.
1
Por la hipotesis inductiva, las n primeras se cortaran en n(n1) puntos. La ultima cortara a
2
1
las n anteriores en n puntos mas. Luego, las n + 1 rectas se cortaran en n(n 1) + n =
2
1
(n + 1)n.
2

Hay una variante en las demostraciones por induccion que suele ser util. Consiste en
suponer la validez de la formula para todos los casos anteriores a n y demostrarla para n. La
ilustraremos en el siguiente ejemplo.

Ejemplo 2.7
Probar que todo entero mayor que 1 tiene un factor primo.

La proposicion es verdadera para el numero 2. Vamos a suponer que es valida para todos los
naturales menores que n y basandonos en esa suposicion, probaremos que vale para n.
Si n es primo, entonces la proposicion es cierta. Si no lo es, tiene algun factor que, obviamente,
52

es menor que n y el cual, por la hipotesis inductiva, tiene un factor primo. Por el teorema 2.5,
este factor primo es tambien factor de n, lo que concluye la demostracion.

Seguiremos con ejemplos de definiciones por induccion tambien llamadas definiciones recur-
sivas.

Ejemplo 2.8
Sea a un numero real y n un natural. Definimos an por

i) a1 = a

ii) an+1 = an a

Notemos que el metodo consiste en definir la expresion para el numero 1 y suponiendola de-
finida para el numero n se define para su sucesor, n + 1. Nuevamente el teorema 2.3 asegura
que la expresion ha quedado bien definida para todo natural.

Haciendo uso de esta definicion, dejamos al lector que pruebe por induccion las siguientes
propiedades de las potencias:

Sean a y b reales y m y n naturales. Entonces

1. am an = am+n

2. (am )n = amn

3. (ab)m = am bm .

Algunas veces es necesario definir la expresion para otros numeros aparte del 1, como veremos
en el siguiente ejemplo.

Ejemplo 2.9
Se define la sucesion de Fibonacci del modo siguiente:

a1 = 1, a2 = 1 an+1 = an + an1
Sus siguientes numeros son:

a3 = 3, a4 = 5, a5 = 8, a6 = 13, etc.
Posteriormente, expondremos una formula, conocida como Formula de Binet, que permi-
tira calcular su n-esimo termino.
Esta sucesion tiene propiedades muy interesantes. Probaremos una y otras las dejaremos de
ejercicios.
!n
1+ 5
Como ejemplo de estas propiedades demostraremos, usando induccion, que an < .
2
!1
1+ 5 1+2
Para 1, tenemos que a1 = 1 y > = 1, 5.
2 2
53

Luego, la afirmacion es valida para 1. En forma similar se prueba que vale para 2. Supongamos-
le valida para los naturales anteriores a n. Podemos suponer que n > 2. Por la definicion
!n1 !n2
1+ 5 1+ 5
an = an1 + an2 < +
2 2
!n2
1+ 5
Factorizando por
2
!n2 ! !n2 !2
1+ 5 1+ 5 1+ 5 1+ 5
+1 =
2 2 2 2

!n
1+ 5
=
2

La induccion solo permite probar formulas que se sospechan validas, pero no da un metodo
para encontrarlas.

Cuando se pide encontrar una formula, puede ser util proceder como muestra el siguiente
ejemplo.
Ejemplo 2.10
Encuentre una formula para el siguiente producto y pruebela inductivamente
    
1 1 1
1 2 1 2 1 .
2 3 (n + 1)2
Calculemos los primeros productos
1 3
1 2
=
2 4
  
1 1 4
1 2 1 2 =
2 3 6
   
1 1 1 5
1 2 1 2 1 2 =
2 3 4 8
n+2
lo que nos lleva a conjeturar que la formula es .
2(n + 1)
Trataremos a probarla por induccion. Supongamos que la formula vale para el producto de n
factores y consideremos el producto de n + 1 factores.
     
1 1 1 1
1 2 1 2 1 1
2 3 (n 1)2 (n + 2)2
 
n+2 1
= 1
2(n + 1) (n + 2)2

n+2 (n + 3)(n + 1) n+3


= =
2(n + 1) (n + 2)2 2(n + 2)
54

que es la formula para el numero n + 1.

2.1.1. EJERCICIOS ( DE INDUCCION)


1. Pruebe que:

a) 6 divide a 5n3 + 7n
b) 16 divide a 834n 2 972n + 1

2. Pruebe que:
a b es un factor de an bn

3. Pruebe las siguientes propiedades de los numeros de Fibonacci definidos en el ejemplo


2.8.

a) an+1 an1 = a2n (1)n


b) an+m = am an+1 + an1 an
c) an y an+1 son primos relativos

4. Si n > 2 pruebe que

3 1 1 1 1 1 1
+ 2 < 2 + 2 + + 2 < 2
2 n n 1 2 n n
5. Pruebe que un conjunto con n elementos tiene 2n subconjuntos.

6. Conjeture formulas para las siguientes expresiones y pruebelas por induccion:


2 n
a) (1 x)(1 + x)(1 + x2 )(1 + x2 ) (1 + x2 )
     
1 1 1 1
b) 1 1 1 1
2 3 4 n+1
c) 12 22 + 32 + (1)n1 n2
d ) n2 (n 1)2 + (n 2)2 + (1)n1

7. Sea p1 , p1 , , pn reales positivos tales que:

p0 = 1 y p2k+1 > pk pk+2 para todo k = 1, 2, , n.

Pruebe que


p1 > p2 > 3
p3 > > n
pn

8. Se define u1 = 0 y un+1 = (1 + x)un nx.


Pruebe que un = x1 [1 + nx (1 + x)n ].

9. Pruebe que n(n + 1)(n + 2) (n + p 1) es divisible por p.


55

8 12 16 1
10. Pruebe que + hasta completar n terminos es igual a +
35 57 79 3
1
(1)n1 .
2n + 3
11. Pruebe que el numero de partes en que queda dividido el plano por n rectas en posicion
general, es decir, sin que hayan dos paralelas o tres concurrentes, es (n2 + n + 2)/2.

12. Pruebe que el numero de partes en que se divide el espacio por n planos en posicion
general es (n + 1)(n2 n + 6)/6.

13. Pruebe que el numero de partes en que se divide el plano por n circunferencia en posicion
general es n2 n + 2,
56

2.1.2. MAXIMO COMUN DIVISOR (M.C.D.)


En esta seccion desarrollaremos algunas propiedades adicionales de los numeros enteros.

Definicion 2.2
Sean a y b numero enteros. Al mayor divisor comun que tengan lo llamaremos el maximo
comun divisor (m.c.d). Lo anotaremos por (a, b).

Observacion.
Como los divisores comunes a dos enteros son finitos, es obvio que el mayor de ellos existe
y ademas es positivo, ya que 1 siempre es un divisor comun. En el caso en que (a, b) = 1
diremos que son primos relativos. Por ejemplo, 6 y 35 son primos relativos aunque ni 6 ni 35
son numeros primos.

Teorema 2.6
Sean a y b enteros y d = (a, b), entonces existen dos enteros p y q tales que

d = pa + qb

Demostracion
Sea S el conjunto de todos los enteros de la forma xa + yb, con x e y enteros, es decir,
S = {xa + yb : x, y Z}. Es inmediato que a y b pertenecen a S y que, ademas, en S se
encuentran numeros positivos. Sea d el menor de los positivos de S. Vamos a probar que d
es un divisor de cada elemento de ese conjunto. Para esto, supongamos que m esta en S.
Entonces, por el teorema 2.4, podemos escribir que:

m = ud + r, con 0 r < d.
Pero como m y d son elementos de S, tenemos que existen enteros p, q, s, y t, tales que:

d = pa + qd y m = sa + tb
Reemplazando en la ecuacion anterior y despejando r

r = (s up)a + (t uq)b
lo que prueba que r esta en S. Pero, tomando en cuenta que r 0 y d > r y siendo este el
menor positivo de S, concluimos que r = 0. Luego, d es un divisor de m.
Por ultimo, debemos probar que d es el mayor divisor comun de a y b.
Sea c | a y c | b, entonces c | (pa + qb), es decir c | d, lo que implica que c d. por lo
tanto,d = (a, b).
Cuando a y b son numeros pequenos se puede encontrar su m.c.d. por simple inspeccion, como
lo muestra el siguiente ejemplo:

Ejemplo 2.11

1. (15, 6) = 3 = 1 15 + (2) 6

2. (7, 0) = 7 = 1 7 + 0
57

3. (18, 7) = 1 = (2)(18) + (5) 7

En estos ejemplos, recalcamos el hecho que el m.c.d se puede escribir como una combinacion
lineal de a y b.
Cuando a y b son numeros grandes, podemos hacer uso de un algoritmo conocido como
Algoritmo de Euclides. Su estructura queda justificada por el siguiente teorema.

Teorema 2.7
El m.c.d. de a y b es igual al de b y a + kb para cualquier entero k.

Demostracion
Sean d = (a, b) y d0 = (b, a + kb). Sabemos que existen enteros, x e y, tales que d0 =
xb + y(a + kb). Como d es un divisor de a y de b se tiene que d | d0 . Ademas, como d0 | b y
d0 | (a + kb) se concluye facilmente que d0 | a, lo que nos lleva a que d0 | d. Luego, d = d0 .

El siguiente ejemplo mostrara como funciona el Algoritmo de Euclides. El metodo consiste


en reemplazar al mayor de los numeros por uno que sea menor que el mas pequeno. Esto se
consigue usando el algoritmo de la division. Lo ilustraremos con un ejemplo.

Ejemplo 2.12
Encontrar el m.c.d. entre 469 027 y 18 867.

Sea a = 469 027 y b = 18 867. Por el algoritmo de la division tenemos las siguientes igualdades:

(1) a = 24b + 16 219; r1 = 16 219 = a 24b


Entonces, por el teorema 2.7, (a, b) = (b, r1 )

(2) b = r1 + 2 648; r2 = 2 648


Nuevamente, por el teorema 2.7 se cumple que (a, b) = (b, r1 ) = (r1 , r2 ) y as se continua
hasta obtener resto 0.

(3) r1 = 6r2 + 331; r3 = 331

(4) r2 = 8r3 + 0; r3 es el m.c.d.

Es importante notar, que usando las ecuaciones anteriores, podemos expresar r3 como una
combinacion lineal de a y b.
De 1. despejamos r1 y lo reemplazamos en 2. y en 3.
De 2. despejamos r2 y lo reemplazamos en 3.
Esta ultima operacion expresa r3 como una combinacion lineal de a y b. Se obtiene:

(1) r1 = a 24b

(2) b = a 24b + r2 de donde r2 = 25b a

(3) a 24b = 150b 6a + r3 de donde r3 = 7a 174b


Reemplazando por sus valores numericos:
331 = 7 469 027 174 18 867
58

Teorema 2.8
Sean a, b y c enteros, con a y b primos relativos. Entonces existen enteros x e y tales que
ax + by = c.

Este teorema se enuncia tambien: Si a y b son primos entre s, entonces la ecuacion ax+by = c
tiene solucion en los numeros enteros.

Demostracion
Por ser a y b primos entre s se tiene que (a, b) = 1. Por el teorema anterior existen p y q tales
que ap + bq = 1, y multiplicando por c se obtiene finalmente apc + bqc = c.

Ejemplo 2.13
Resolver la ecuacion 5x + 8y = 12.

Como 1 = 2 8 3 5 tenemos que 12 = 24 8 36 5.

Teorema 2.9
Si (a, b) = 1 y a|bc entonces a|c.

Demostracion
Existen enteros x e y tales que ax + by = 1. De donde acx + bcy = c. Pero a|(acx + bcy); luego
a|c.

Corolario
Si p primo y p|bc entonces p|b o p|c.

Demostracion
Si p no divide a b entonces (p, b) = 1 y por el teorema anterior p|c.

Terminamos esta seccion con algunas consideraciones sobre los numeros primos.
El teorema que viene a continuacion aparece en el texto conocido como Los Elementos, escrito
por Euclides al rededor del siglo III A.C.

Teorema 2.10
El numero de primos es infinito.

Demostracion
Supongamos que son finitos; y sean p1 , p2 , , pn todos los numeros primos.
Consideremos el numero N = p1 p2 pn + 1.

Ninguno de los pi (i = 1, 2, , n) divide a N ya que este es mayor en 1 a algun multiplo de pi .


Luego, N es primo o tiene un factor primo que no es ninguno de los numeros p1 , p2 , , pn .
Pero esto contradice la suposicion que estos eran todos los primos.

Teorema 2.11
Sea n un numero natural mayor que 1. Entonces es primo o es el producto de primos.
59

Demostracion
Por induccion. El teorema vale para n = 2. Supongamoslo valido para todos los numeros
menores o iguales a n.

Si n + 1 es primo el teorema es valido. Si n + 1 es compuesto entonces n + 1 = n1 n2 con


n1 n y n2 n. Como n1 es primo o es un producto de primos y lo mismo para n2 , tenemos
que n + 1 es un producto de primos.

Teorema 2.12
Todo entero mayor que 1 es primo o su descomposicion en factores primos es unica.

Demostracion
Por induccion. El teorema vale para n = 2. Supongamos que vale para los numeros 2, 3, 4, , n.

Vamos a probar que vale para n + 1.

Sean n + 1 = p1 p2 pr = q1 q2 qs dos factorizaciones en primos. Podemos suponer, sin


perdida de generalidad, que:

p1 p2 pr y q1 q2 qs .
Probaremos que r = s y que p1 = q1 , p2 = q2 pr = qs .

Como p1 |n + 1 tenemos que p1 |q1 q2 qs . Por ser todos numeros primos, tenemos que p1 = qi
para algun i. Tambien q1 |n + 1 y, por el mismo argumento, obtendremos que q1 = pj para
algun j, q1 = pj > p1 y p1 = qi q1 . Luego, p1 = q1 .
n+1
Como el numero = p2 p3 pr = q2 q3 qs es menor que n,
p1
por la hipotesis de induccion tiene solo una factorizacion en primos. Luego, r 1 = s 1
y p2 = q2 , p3 = q3 , , pr = qs .

2.1.3. EJERCICIOS
14. Factorice en primos los numeros 667 667, 45 658.767, 226 1.
15. Encuentre enteros x e y tales que 576x + 73y = (576; 73).
16. Pruebe que si p2 = 2q 2 entonces p y q son pares.
17. Si (b, c) = 1 encuentre los valores posibles para (b c, b + c).
18. Si p es primo y p|an entonces pn |an .
19. Si p y q son dos primos sucesivos e impares pruebe que p+q no es primo ni es el producto
de dos primos.
20. Pruebe que hay infinitos primos de la forma 4n 1. (Sugerencia: Si p1 , p2 , , pi son
todos los primos de esa forma considere el numero N = 4 p1 p2 pi 1).
21. Si p y q son mayores que 1 y no son multiplos de 2 ni de tres entonces p2 q 2 es multiplo
de 24.
60

2.2. SUMATORIAS
Hemos estado usando sin mayores comentarios expresiones del tipo a1 + a2 + + an .
En esta seccion precisaremos el significado de estas sumas y estableceremos algunas de sus
propiedades.

Definicion 2.3
Sea f : N {0} R. (Tales funciones se llaman sucesiones).
n
X
Definimos el smbolo f (k) inductivamente del modo siguiente:
k=1
1
X
f (k) = f (1)
k=1

n+1 n
!
X X
f (k) = f (k) + f (n + 1)
k=1 k=1

Nota
n
X
El smbolo f (k) se lee la sumatoria desde k = 1 hasta n de f (k).
k=1
En esta seccion se acostumbra usar expresiones del tipo ai , ui , etc. para para simbolizar
sucesiones, en vez de la tradicional notacion funcional tal como f (i).

Ejemplo 2.14
Sea f (k) = ak .

5 4
!
X X
ak = ak + a5
k=1 k=1

3
!
Entonces X
= ak + a4 + a5
k=1
..
.
l = a1 + a2 + a3 + a4 + a5 .
n
X
En general, ak tiene el significado intuitivo de a1 + a2 + + an .
k=1

Ejemplo 2.15
n
1 X 1 1 1
Si ak = (1)k1 entonces ak = 1 + + (1)n1
k 2 3 n
k=1

Ejemplo 2.16
La suma 1 + 4 + 7 + 11 + + 61 queda representada por
21
X
(3k 2)
k=1
61

Observemos que esta notacion no ocupa puntos suspensivos (smbolo que no hemos definido),
pero los seguiremos usando cuando nos parezca que queda mas claro el contexto.

Ejemplo 2.17
n
n(n + 1) 2
X  
Probar que: k3 = .
2
k=1

Haremos la demostracion por induccion.

Para n = 1 el primer miembro es

1
X
k3
k=1

que aplicando la definicion vale 1.

El segundo miembro para n = 1 tambien vale 1. Por lo tanto, la formula es verdadera para 1.
Debemos probar que vale para n + 1 suponiendola valida para n. Por la definicion tenemos
que

n+1
X n
X
k3 = k 3 + (n + 1)3
k=1 k=1

y por la hipotesis inductiva


 2
n(n + 1)
= + (n + 1)3
2
y despues de algun manipuleo algebraico
 2
(n + 1)(n + 2)
=
2

que corresponde al segundo miembro de la formula aplicada a (n + 1).

El siguiente teorema indica las propiedades de uso mas frecuente en las sumatorias.

Teorema 2.13
n
X n
X
1. ak = aj
k=1 j=1

El valor de la sumatoria no depende del smbolo que se use como ndice.


n
X n
X n
X
2. (ak + bk ) = ak + bk
k=1 k=1 k=1

Se conoce como la Propiedad aditiva.


62

n
X n
X
3. cak = c ak
k=1 k=1

Conocida como la Propiedad homogenea.


n
X
4. c=nc
k=1

Si todos los terminos de la suma son iguales a una constante c, entonces la suma vale
n c.
n
X
5. (ak ak1 ) = an a0
k=1

Es llamada la Propiedad telescopica.

Demostracion
Todas las demostraciones se hacen facilmente por induccion. Demostraremos solamente la 5,
dejando las restantes a cargo del lector.

Para 1 el primer miembro es a1 a0 al igual que el segundo. Luego, vale para n = 1. Probemos
que si es valida para algun n entonces tambien lo es para su sucesor n + 1. Por la definicion
de sumatoria
n+1
X n
X
ak ak1 = ak ak1 + an+1 an
k=1 k=1

y por la hipotesis inductiva

= an a0 + an+1 an = an+1 a0

que es el segundo miembro de la formula al reemplazar n por n + 1.

Definicion 2.4
Sean m y n naturales con m n. Definimos
n
X n
X m1
X
ak = ak ak
k=m k=1 k=1

El significado intuitivo de esta sumatoria es sumar desde el termino de orden m hasta el de


orden n.

Otra propiedad util la da el siguiente teorema. Para mayor claridad usaremos la notacion f (k)
en vez de ak .

Teorema 2.14
Sea f : N {0} R.

Sean n, m, h N con n m h. Entonces


63

n
X nh
X
f (k) = f (k + h)
k=m k=mh

Demostracion
Por induccion sobre n.

Si n = 1 entonces las sumatorias tendran sentido solo si m = h = 1 y para esos valores, la


igualdad se cumple.

Supongamos que se cumple para n con cualquier m y h tales que n m h. Entonces, para
n + 1, extrayendo el ultimo termino de la sumatoria, se tiene que

n+1h
X nh
X
f (k + h) = f (k + h) + f (n + 1 h + h)
k=mh k=mh

n
X
= f (k) + f (n + 1)
k=m

n+1
X
= f (k).
k=m

Nota
En forma analoga se prueba que

n
X n+h
X
f (k) = f (k h)
k=m k=m+h

ak
X
Bien mirado, la sumatoria no es mas que una funcion de los naturales en los reales
k=1
de variable n que podremos designar por S(n).
Xn
El Ejemplo 2.16 nos muestra que si S(n) = k 3 , su formula explcita es:
k=1
 2
n(n + 1)
S(n) =
2
El problema central de las sumatorias es encontrar la formula explcita en los casos en que
estas existan.
A continuacion, haremos un listado de algunas de ellas de uso frecuente. Omitiremos la variable
n:
n
X
(1) S0 = 1 = 1 + 1 + + 1 = n
k=1
64

n
X n(n + 1)
(2) S1 = k = 1 + 2 + 3 + + n = (Ejemplo 2.3)
2
k=1

n
X n(n + 1)(2n + 1)
(3) S2 = k 2 = 12 + 22 + 32 + + n2 = (Ejemplo 2.18)
6
k=1

n
# "
X
3 3 3 3 n(n + 1) 2 3
(4) S3 = k = 1 + 2 + 3 + + n = (Ejemplo 2.16)
2
k=1

n
X 1
(5) S1 = No tiene una formula elemental.
k
k=1

n
X 1 1
(6) S2 = =1
k(k + 1) n+1
k=1

n
X 1 1 1
(7) S3 = = (Ejemplo 2.25)
k(k + 1)(k + 2) 4 2(n + 1)(n + 2)
k=1

Continuaremos con ejemplos.

Ejemplo 2.18
n
X
Calcular (4k 3 2k + 3).
k=1

Por la propiedad aditiva tenemos que


n
X n
X n
X n
X
3 3
(4k 2k + 3) = 4k 2k + 3
k=1 k=1 k=1 k=1

Por la propiedad homogenea y por la cuarta del Teorema 2.13


n
X n
X
3
=4 k 2 k + 3n
k=1 k=1

Usando ejemplos anteriores


 2
n(n + 1) n(n + 1)
=4 2 + 3n
2 2

= (n3 + n2 + 2)n.

Ejemplo 2.19
n
X
Consideremos la sumatoria (k 3 (k 1)3 ).
k=1

Por la propiedad telescopica tenemos que


65

n
X
(k 3 (k 1)3 ) = n3 .
k=1

Desarrollando el primer miembro

n
X n
X n
X
3 k2 3 k+ 1 = n3
k=1 k=1 k=1

Despejando y sustituyendo

n
X n(n + 1)
3 k 2 = n3 + 3 n
2
k=1

Finalmente obtenemos

n
X n(n + 1)(2n + 1)
k2 =
6
k=1

Ejemplo 2.20
30
X
Calcular k(k 2).
k=3

Reemplazando k por j + 2

30
X 28
X
k(k 2) = (j + 2)j
k=3 j=1

28 28
X X 28 29 57
= j2 + 2 j= + 28 29
6
j=1 j=1

= 8526

Ejemplo 2.21
Calcular la suma de los n primeros terminos de:

1 11 + 2 12 + 3 13 +

Escribamos esta suma en smbolos. Por simple observacion encontraremos que la funcion f (k)
es k(10 + k). Entonces, lo que tenemos que calcular es

n
X
k(10 + k).
k=1

Aplicando resultados ya conocidos tenemos que:


66

n
X n
X n
X
k(10 + k) = 10 k+ k2
k=1 k=1 k=1

n(n + 1)(2n + 1)
= 5n(n + 1) +
6
n(n + 1)
= (2n + 31)
6

Ejemplo 2.22
2n
X n
X
k 2
Demostrar que (1) k = (4k 1).
k=1 k=1

Aunque el ejercicio se puede hacer por induccion procederemos de otra manera. Notemos que
en el primer miembro, si el sumando es par, es positivo y si es impar, es negativo.

Separemos el primer miembro en la suma de los pares menos la de los impares. Para mayor
claridad escribamos, sin mayor rigor, que

2n
X
(1)k k 2 = 22 + 42 + + (2n)2 (12 + 32 + + (2n 1)2 )
k=1

Entonces

2n
X n
X n
X
(1)k k 2 = (2k)2 (2k 1)2
k=1 k=1 k=1

que por la propiedad aditiva del teorema 2.13


n
X n
X
2 2
 
= 4k (4k 4k + 1) = (4k 1)
k=1 k=1

Ejemplo 2.23
n
X
Si, para todo n se cumple que ak = 2n2 + 3n encontrar ak .
k=1

Por la definicion tenemos que

n
X n1
X
an = ak ak
k=1 k=1

Luego,

an = 2n2 + 3n [2(n 1)2 + 3(n 1)] = 4n + 1


Entonces, cambiando de letra resulta que

ak = 4k + 1.
67

Ejemplo 2.24
n
X
A partir de la funcion f (k) = k 3 (k 1)3 calcule k5 .
k=1
Observemos que

f (k + 1) f (k) = (k + 1)3 k 3 k 3 (k 1)3

= 6k 5 + 2k 3 .
De donde
n
X n
X n
X
f (k + 1) f (k) = 6 k5 + 2 k3
k=1 k=1 k=1

Pero, por la propiedad telescopica, el primer miembro es f (n + 1) f (1), de donde se obtiene


n
X n
X
(n + 1)3 n3 = 6 k5 + 2 k3
k=1 k=1

Despejando
n
X 1 (n + 1)2 n2
k 5 = (n + 1)3 n3
6 2
k=1

1
= (n + 1)2 n2 (2n2 + 2n 1).
12

A continuacion desarrollaremos ejemplos en los que el ndice de la sumatoria aparece en


el denominador.

Ejemplo 2.25
n
X 1
Calcular .
(2k 1)(2k + 1)
k=1
Los problemas de este tipo conviene trabajarlos por el metodo de las fracciones parciales. No
es seguro que siempre obtendremos la formula, pero es un camino que conviene intentarlo.
Este metodo lo explicaremos a traves de ejemplos. El consiste en descomponer en sumas de
fracciones la fraccion dada. Veamos como es posible lograrlo para este caso.

La fraccion tiene un denominador de dos factores; por eso suponemos la siguiente descompo-
sicion

1 A B
= + (1)
(2k 1)(2k + 1) 2k 1 2k + 1
donde A y B deben ser numeros. Para determinarlos amplificaremos (1) por (2k 1)(2k + 1)
quedandonos
1 = A(2k + 1) + B(2k 1)
Dandole dos valores a k podemos escribir un sistema de dos ecuaciones. En este caso, tomando
para k los valores 0 y 1 tenemos que
68

1=AB

1 = 3A + B
1 1
De donde resulta que A = , B = . Finalmente:
2 2
1 1
1 2 2
=
(2k 1)(2k + 1) 2k 1 2k + 1
.

Volviendo al problema de la sumatoria tenemos que:


n n
!
1 1
X 1 X
2 2
=
(2k 1)(2k + 1) 2k 1 2k + 1
k=1 k=1

n  
1X 1 1
=
2 2k 1 2k + 1
k=1
En esta ultima sumatoria cada termino es una diferencia de dos elementos sucesivos can-
celandose casi todos como vemos al desarrollarla. (Propiedad telescopica).

n    
1X 1 1 1 1 1 1 1 1 1 1 1
= + + + +
2 2k 1 2k + 1 2 1 3 3 5 5 7 2n 1 2n + 1
k=1
 
1 1
= 1
2 2n + 1
n
=
2n + 1
Ejemplo 2.26
n
X 1
Calcular
k(k + 1)(k + 2)
k=1
En este caso debemos escribir la fraccion en la forma siguiente

1 A B C
= + +
k(k + 1)(k + 2) k k+1 k+2
Multiplicando por k(k + a)(k + 2)

1 = A(k + 1)(k + 2) + Bk(k + 2) + Ck(k + 1)


Tomando para k los valores 0, 1 2 obtenemos el sistema de ecuaciones

1 = 2A

1 = B

1 = 2C
69

con lo cual podemos escribir que

1 1 1 1
= +
k(k + 1)(k + 2) 2k k + 1 2(k + 2)
Ahora podemos poner la sumatoria propuesta en la forma
n n    
X 1 1 1 1X 1 1 1 1
+ =
2k k + 1 2(k + 2) 2 k k+1 k+1 k+2
k=1 k=1

Queda en la forma de 2 sumas telescopicas. Cancelando los terminos sucesivos se reduce a:


 
1 1 1

2 2 (n + 1)(n + 2)

Ejemplo 2.27
En este ejemplo generalizaremos el resultado anterior. Vamos a calcular
n
X 1
con p N.
k(k + 1) (k + p)
k=1

Siguiendo la idea del ejemplo anterior, comprobamos que:

 
1 1 1 1
=
k(k + 1) (k + p) p k(k + 1) (k + p 1) (k + 1)(k + 2) (k + p)

Entonces
n
X 1
k(k + 1) (k + p)
k=1

n  
1X 1 1
=
p k(k + 1) (k + p 1) (k + 1)(k + 2) (k + p)
k=1

de donde resulta, por la propiedad telescopica, que


 
1 1 1
=
p 1 2 3 p (n + 1)(n + 2) (n + p)

Nota
Veremos mas adelante que si s N entonces el producto 1 2 3 . . . s se escribira s!. Se lee
ese factorial. Con esta notacion, el resultado anterior se puede escribir.
 
1 1 n!

p p! (n + p)!

Ejemplo 2.28
n
X k 2 + 3k + 6
Calcular
k(k + 1)(k + 2)(k + 3)
k=1
70

Debemos expresar el numerador como suma de expresiones del tipo k + 3, (k + 3)(k + 2), etc.
El objeto de esto se apreciara en el desarrollo del problema.

k 2 = (k + 3)(k + 2) 5k 6

k = (k + 3) 3
De donde resulta que

k 2 + 3k + 6 = (k + 3)(k + 2) 2(k + 3) + 6

Con lo cual conseguimos la descomposicion

k 2 + 3k + 6 1 2 6
= +
k(k + 1)(k + 2)(k + 3) k(k + 1) k(k + 1)(k + 2) k(k + 1)(k + 2)(k + 3)

Por lo tanto
n n
X k 2 + 3k + 6 X 1
=
k(k + 1)(k + 2)(k + 3) k(k + 1)
k=1 k=1

n n
X 1 X 1
2 k=1 +6
k(k + 1)(k + 2) k(k + 1)(k + 2)(k + 3)
k=1

Usando el resultado anterior


   
1 1 1 1
= 1 2
n+1 2 2 (n + 1)(n + 2)
 
1 1 1
+6
3 6 (n + 19(n + 2)(n + 3)

5 1 1 2
= +
6 n + 1 (n + 1)(n + 2) (n + 1)(n + 2)(n + 3)

2.2.1. EJERCICIOS
22. Demuestre por induccion que:
n
X 1 n
a) =
4k 2 1 2n + 1
k=1
n
X
b) (k 2 + 1) k! = n(n + 1)!
k=1

23. Calcule la suma de los primeros n terminos de:

a) 12 + 32 + 52 +
b) 13 + 33 + 53 +
71

c) 1 3 + 2 4 + 3 5 +
n
X
24. A partir de f (r) = r2 (r 1)2 (2r 1) simplifique f (r + 1) f (r) y de aqu calcule r4 .
r=1

n
X 2n
X
2
25. Si ui = 2n + 3n. Calcule el valor de ui
i=1 i=n+1

26. Sume:

a) 2n terminos de 2 5 + 3 6 + 4 7 +
b) n terminos de n(n + 1) + (n + 1)(n + 2) + (n + 2)(n + 3) +
1 1 1
c) n terminos de + + +
n(n + 1) (n + 1)(n + 2) (n + 2)(n + 3)

27. Calcule:
n
X 1
a)
(2k 1)(2k + 1)(2k + 3)
k=1
n
X k+2
b)
k(k + 1)2k
k=1
n
X k
c)
1 + k2 + k4
k=1

28. Encuentre una formula para

n
X
k(k + 1) (k + p)
k=1

1
29. A partir de f (r) = . Calcule la suma de n terminos de
r2
3 5 7
+ 2 2 + 2 2 +
12 22 2 3 3 4
30. Encuentre la suma de los primeros n terminos de:

2 24 246
1+ + + +
3 35 357
2 4 (2r 2) 2 4 2r
(Sugerencia: Sea ur = vr = . Calcule vr vr1 )
3 5 (2r 1) 3 5 2r 1

31. Sume 2n terminos de 12 + 23 + 32 + 43 + 52 + 63 +

32. Encuentre la suma de n terminos de:

1 + (3 + 5 + 7) + (9 + 11 + 13 + 15 + 17) +
72

33. Pruebe que:

n  r  
X r+2 1 1 1
=1
r(r + 1) 2 n+1 2
r=1

34. Pruebe que:


n
X n
X
a) (ak + bn+1k ) = (an+1k + bk )
k=1 k=1
Xn n
X
b) ak bn+1k = an+1k bk
k=1 k=1

35. Pruebe que:


n n/2 n/2
X X X
ak = a2k + a2k+1 si n es par
k=0 k=0 k=0

Como es la expresion si n es impar?

36. Se define E(x), para x real, como el mayor entero menor o igual a x. Se llama la parte
entera de x o el piso de x.
Xn
Pruebe que E(k/2) = E(n2 /4).
k=1

37. Pruebe que E(x) + E(x + n1 ) + + E(x + n1


n = E(nx).

n
X n1
X
38. Pruebe que ak = nan k(ak+1 ak )
k=1 k=1

39. Suma por partes. Pruebe que

n
X n1
X
(ak+1 ak )bk = an bn a1 b1 ak+1 (bk+1 bk )
k=1 k=1
73

2.3. PROGRESIONES
En esta seccion estudiaremos tres tipos de sucesiones llamadas progresiones aritmeticas,
geometricas y armonicas que aparecen con cierta frecuencia en las aplicaciones.

Definicion 2.5
Diremos que la sucesion a1 , a2 , a3 , , ak , este en progresion aritmetica (se abrevia P.A.)
si y solo si existe d numero real, tal que

a1 = a, a2 = a + d, a3 = a + 2d , ak = a + (k 1)d

El numero a se llama el primer termino de la P.A. y d la diferencia.

Ejemplo 2.29

1.- 2, 4, 6, 8, es una P.A. con primer termino 2 y diferencia 2.


1 1 1
2.- 1, , 0, , 1, es una P.A. con primer termino 1 y diferencia .
2 2 2
3.- 7, 7, 7, es una P.A. con primer termino 7 y diferencia 0.

Teorema 2.15
n
La suma de los n primeros terminos de una P.A. es [2a + (n 1)d].
2
Demostracion
Por lo que ya sabemos, la suma

n
X n
X n
X
a + (a + d) + (a + 2d) + + (a + nd) = a + (k 1)d = a 1+d (k 1)
k=1 k=1 k=1

(n 1)nd
= na +
2
n
= [2a + (n 1)d]
2
Nota
n
Como an = a + (n 1)d tenemos que esta suma tambien la podemos escribir como (a1 + an ).
2
Ejemplo 2.30
La suma de los p primeros terminos de una P.A. es q y la suma de los q primeros terminos es
p. Encontrar el primer termino y la diferencia.

Podemos plantear las dos ecuaciones siguientes:


p
(2a1 + (p 1)d) = q
2
q
(2a1 + (q 1)d) = p
2
74

cuyas incognitas son a1 y d.

De ellas resultan las ecuaciones

2q
2a1 + (p 1)d =
p

2p
2a1 + (q 1)d =
q

Al restarlas nos permiten determinar que

p+q
d = 2
pq
Reemplazando este valor en una de ellas, encontramos que

p q 1 1
a1 = 1 + + .
q p p q

Ejemplo 2.31
Dados los numeros a y b se pide encontrar n + 2 numeros en P.A. a1 , a2 , a3 , . . . , an+2 tales que
a1 = a y an+2 = b.

Este problema se conoce tambien como Interpolar n medios aritmeticos entre a y b .

Sea d la diferencia. Entonces a1 = a, a2 = a + d , an+2 = a + (n + 1)d.

Pero an+2 = b. Luego, a + (n + 1)d = b, de donde encontramos que

ba
d=
n+1
y finalmente tenemos que

ba
ak = a + (k 1) .
n+1

Definicion 2.6
Diremos que la sucesion a1 , a2 , , ak , esta en progresion geometrica ( se abrevia P.G.) si
y solo si existen a y r numeros reales con r 6= 0 tal que a1 = a, a2 = a r, ak = a rk1 .

El numero a se llama el primer termino de la P.G. y r la razon.

Ejemplo 2.32

1. 1, 2, 4, 8, es una P.G. con primer termino 1 y razon 2.


1 1 1
2. 2, 1, , , es una P.G. con primer termino 2 y razon .
2 4 2
3. 1, 1, 1, 1, es una P.G. con primer termino 1 y razon 1.
75

Teorema 2.16
La suma de los n primeros terminos de una P.G. de primer termino a y razon r 6= 1 es:
rn 1
a
r1

Demostracion
n
X
Sea Sn la suma buscada; es decir: Sn = a rk1
k=1
Multiplicando por r
n
X
r Sn = a rk
k=1
y restando
n
X
(r 1)Sn = (ark ark1 )
k=1

Por la propiedad telescopica

(r 1)Sn = arn a
De donde

rn 1
Sn = a
r1
Nota
Si r = 1 la suma Sn es, obviamente, na.

Ejemplo 2.33
En una circunferencia de radio r se inscribe un cuadrado. En este se inscribe una circun-
ferencia, en esta un cuadrado y as sucesivamente.

Calcular la suma de las areas de las n primeras circunferencias.

Debemos calcular los radios.


Sea rk el radio de la k-esima circunferencia. Entonces el lado del
cuadrado inscrito a ella es 2rk . De donde resulta que el radio de la siguiente circunferencia

2rk rk
rk+1 = =
2 2
Luego, los radios son
r r r
r, , , ,
2 2 2 2

1 r2 r2 r2
que estan en P.G. de razon . Las areas son r2 , , , , que estan en P.G. de
2 2 4 8
1
razon . Luego, la suma de las areas de las n primeras circunferencias es
2
76

1
n 1
 
1
r2 21 2
= r 2
2 1
2n1
Nota
Si 1 < r < 1 se tiene que rn es cada vez mas cercano a cero a medida que n crece.
rn 1
Por lo tanto, en la formula Sn = a podemos considerar rn = 0 si n es muy grande
r1
y 1 < r < 1. As, si en el ejemplo anterior se nos pidiera sumar las areas de todas las
circunferencias obtendramos que es:

1
r2 = 1 = 2r2 ;
2 1
es decir 2 veces el area de la circunferencia inicial.

Ejemplo 2.34
Expresar en forma racional el decimal periodico

D = 0, 36 36 36 36

Observemos que

36 36 36
0, 36 36 36 36 = 2
+ 4 + 6 +
10 10 10
36 1
que es la suma de todos los terminos de una P.G. de primer termino y razon .
100 100
Por lo tanto

36 1 36 4
D= 1 = 99 = 11
100 1 100

La generalizacion de este ejemplo, que la dejamos al lector, permite expresar decimales pe-
riodicos como fracciones. Mas adelante desarrollaremos en profundidad este tema. (Captulo
7).

Ejemplo 2.35
Dados los numeros a y b se pide encontrar n + 2 numeros, a1 , a2 , , an+2 que estan en P.G.
y tales que a1 , = a y an+2 = b.

Este problema se conoce tambien como Interpolar n medios geometricos entre a y b.

Tenemos que a1 = a, a2 = a r, , an+2 = arn+1


q
n+1
b
Pero an+2 = b, donde b = arn+1 obteniendose para r = n+1
a y para ak = bk1 ank+2

Ejemplo 2.36
Calcular la suma Sn = r + 2r2 + 3r3 + + nrr .
77

Notemos que en esta suma los coeficientes numericos forman la P.A. 1, 2, 3, y las potencias
la P.G. r, r2 , r3 , . Calculando la expresion Sn r Sn resulta igual a r+r2 +r3 + +rn nrn+1
de donde sale que:

rn 1
(1 r)Sn = r nrr+1
r1
y finalmente, si r 6= 1

rn+1 rn+1 r
Sn = n
r1 (r 1)2

Ejemplo 2.37 Una aplicacion a las matematicas financieras.

Consideremos el caso de un prestamo de monto K que se cancelara en n cuotas iguales con


un interes i en cada periodo. Sea C el valor de la cuota. Deberemos calcular C en funcion de
los datos conocidos, a saber K, n e i.

Cada cuota tiene dos componentes; el pago de los intereses por el periodo y la amortizacion
de la deuda. Para la primera cuota tenemos que C = iK + A1 , donde iK es el interes corres-
pondiente al periodo y A1 a la amortizacion de la deuda en dicho periodo.

Para el segundo periodo, como la deuda se redujo en A1 , tenemos que


C = i(K A1 ) + A2 . Despejando A2 y reemplazando A1 por C iK, obtenemos:

A2 = (C iK)(1 + i)
Las expresiones para A1 y A2 nos sugieren que

Al = (C iK)(1 + i)l1
Probaremos este resultado por induccion.
Supongamos que la formula es valida para todos los casos anteriores a l. Para el periodo l
tenemos que

C = i[K (A1 + A2 + . . . + Al1 )] + Al (2.1)


Pero

A1 + A2 + . . . + Al1 = [C iK][1 + (1 + i) + (1 + i)2 + . . . + (1 + i)l2 ]


Notemos que el segundo parentesis es una progresion geometrica de primer termino 1 y razon
(1 + i). Por lo tanto,

(1 + i)l1 1
= [C iK]
i
Reemplazando en la ecuacion (2.1) y despejando Al , obtenemos que

Al = (C iK)(1 + i)l1
Es obvio que
78

K = A1 + A2 + . . . + An

= [C iK][1 + (1 + i) + (1 + i)2 + . . . + (1 + i)n1 ]

(1 + i)n 1
= [C iK]
i
De donde despejando C, se obtiene

iK
C=
1 (1 + i)n

Veamos algunos casos particulares

(1) Para n = 1, tenemos que C = iK + K donde el primer sumando corresponde al interes


y el segundo es la amortizacion.

(2) Para i = 0. Cuando estudie calculo, podra probar que

iK K
lm n
=
i0 1 (1 + i) n

que corresponde a lo que se espera si el interes es 0.

Ejemplo 2.38
Una institucion financiera ofrece: Lleve $3 000 000 y paguelo en 48 cuotas iguales de $94 990.
Calcule el interes mensual.

Tenemos que:

i 3 000 000
94 990 =
1 (1 + i)48
de donde, aproximando a 4 cifras decimales

i
0, 0317 =
1 (1 + i)48
La mejor forma de resolver esta ecuacion es por tanteo. Notemos que el denominador es un
numero menor que 1, por lo tanto, i sera un numero menor que 0,0317. Se obtiene i = 0, 0186,
que equivale a 1,86 % mensual.

Ejemplo 2.39 Formula de Binnet para las sucesiones tipo Fibonacci

Diremos que a1 , a2 , a3 , . . . es una sucesion de Fibonacci ssi n > 2 se cumple que an =


an1 + an2 . Los terminos a1 y a2 se llaman la base de la sucesion, y es claro que ellos la
determinan totalmente. Por ejemplo, a1 = 1 y a2 = 5 definen la sucesion 1, 5, 6, 11, 17, 28, . . ..
La mas conocida de estas sucesiones, que la llamaremos clasica queda definida por a1 = a2 = 1.
(Ver ejemplo 2.8) Ella fue la que originalmente estudio Leonardo de Pisa (1170-1250), que se
79

haca llamar Bigollo (bueno para nada), con el objeto de solucionar el conocido problema
de los conejos.
A partir de su definicion recursiva, es una tarea sencilla pero enormemente larga y aburrida,
calcular el valor de algun termino cuando es un numero grande. Considere el lector cuanto
tiempo le llevara calcular el termino de orden 100 para una de estas sucesiones. Nuestra meta
en esta seccion sera desarrollar una formula conocida como Formula de Binnet, que nos
permitira calcular rapidamente los terminos de las sucesiones. En realidad, la formula era ya
era conocida en tiempos de Binnet y se sospecha que su autor fue el matematico Augusto De
Morgan.

Teorema 2.17
Sean a1 , a2 , . . . y b1 , b2 , . . . dos sucesiones de Fibonacci que cumplen con a1 b2 a2 b1 6= 0.
Sea x1 , x2 , . . . otra sucesion de este tipo. Entonces, existen dos constantes p y q tales que
xn = pan + qbn .

Demostracion
Consideremos el sistema:

x1 = pa1 + qb1
x2 = pa2 + qb2
Es inmediato que xn = pan + qbn . Ademas, el sistema permite calcular p y q. Se obtiene:

b2 x1 b1 x2
p=
a1 b2 a2 b1
a1 x2 a2 x1
q=
a1 b2 a2 b1
Todo lo que nos resta es encontrar dos sucesiones de Fibonacci para las cuales sea facil calcular
sus terminos. Justamente hay P.G. que simultaneamente son de Fibonacci. Consideremos la
P.G. 1, r, r2 , . . . tales que r2 = 1 + r. En estas condiciones tenemos que

r + r2 = r(1 + r) = r3
Se prueba facilmente, usando induccion, que la progresion es de Fibonacci.

La ecuacion 1 + r = r2 tiene las soluciones:

1 
= 1+ 5
2
1 
= 1 5
2
Por lo tanto, las sucesiones 1, , 2 , 3 , . . . y 1, , 2 , 3 , . . . son de Fibonacci y podemos cal-
cular facilmente cualquiera de sus terminos. Ademas, y tienen las siguientes propiedades,
que nos ayudaran a expresar cualquier sucesion como una combinacion lineal de estas dos:

1. + 1 = 2 + 1 = 2
2. + = 1 = 5
3. = 1
80

Apliquemos estas sucesiones a la clasica. En este caso x1 = x2 = 1. Con estos valores obtene-
mos que

1
p=

1
q=

designando por fn el enesimo termino de la sucesion clasica:

1 n1 1 n1
fn =

reemplazando y por sus valores numericos:
1  n  n 
fn = 1+ 5 ) 1 5
2n 5
Nota n
1
Notemos que 52n
1
5 es un numero del intervalo (1, 1), por lo tanto fn es el
 n 
entero mas proximo a 2n15 1 + 5 .
Ahora es posible calcular f (100). Resulta ser un numero de 21 cifras.

Definicion 2.7
Diremos que la sucesion a1 , a2 , a3 , . . . esta en progresion armonica (se abrevia P.H.) si y solo
1 1 1
si la sucesion , , , esta en progresion aritmetica.
a1 a2 a3
Ejemplo 2.40
Son progresiones armonicas
1 1 1
(1) 1, , , ,
2 3 4
(2) 1, 1, 1, 1, 1,

Nota
Es bien sabido, aunque no lo demostraremos, que no hay una formula elemental para calcular
la suma de los n primeros terminos de una P.H.

Ejemplo 2.41
Dados a y b encontrar n+2 numeros a1 , a2 , , an+2 tales que estn en P.H. y a1 = a y an+2 = b.

Este problema se conoce tambien como interpolar n medios armonicos entre a y b.


1 1 1
a1 , a2 , , an+2 estan en P.H. si y solo si , , , estan en P.A. Del ejemplo 2.29
a1 a2 an+2
obtenemos que
1
1 1 1
= + (k 1) b a
ak a n+1

de donde despejando ak resulta que


81

(n + 1)ab
ak =
(k 1)a + (n + 2 k)b

El caso particular de interpolar un medio armonico entre a y b lo logramos con n = 1 y


resultan

a1 = a, a2 = 2ab/(a + b), a3 = b

2.3.1. EJERCICIOS
40. Encontrar tres numeros a, b y c sabiendo que estan en P.A., P.G. y P.H

41. Si los terminos p, q, r de una progresion aritmetica son a, b y c respectivamente, demos-


trar que:

(q r)a + (r p)b + (p q)c = 0

42. Cuantos terminos de la progresion 6, 10, 14, deben tomarse para sumar 1920?

43. La suma de tres numeros en progresion geometrica es 38 y su producto 1728. Hallar los
numeros.

44. Si los terminos de lugares p, q, r de una progresion geometrica son a, b, c respectvamente,


demuestre que:

aqr brp cpq = 1

45. Pruebe que una P.A. queda determinada si se conocen dos terminos cualquiera de ella.

46. Si b es el medio armonico entre a y c, demostrar que:

1 1 1 1
+ = + .
ba bc a c
47. La suma de los 10 primeros terminos de una P.G. es igual a 244 veces la suma de los 5
primeros. Cual es la razon?.

48. Si el termino de orden m de una P.H. es igual a n, y el termino de orden n es igual a


mn
m, entonces el termino de orden m + n es .
m+n
49. Si P es el producto de n terminos en P.G., S su suma y S 0 la suma de los recprocos de
dichos numeros, entonces p2 = (S/S 0 )n .

50. Si a, b, c estan en P.H. entonces a b/2, b/2, c b/2 estan en P.G.

51 Un capital C se coloca al i % de interes anual a n anos plazos. Calcular el interes ganado


si:

a) Los intereses se capitalizan.


b) Los intereses no se capitalizan.
82

52. Note que 9 1 + 2 = 11; 9 12 + 3 = 111; 9 123 + 4 = 1111; intuya una formula general
y demuestrela.

53. Pruebe que:

(1 + x2 + x4 + + x4n ) = (1 + x + x2 + + x2n ) (1 x + x2 x3 + + x2n )

54. Sume n terminos de


1 + 2x + 3x2 + 4x3 +

55. En una P.G. se tiene que la razon es la cuarta parte del primer termino y la suma de
los dos primeros es 24. Encontrar la progresion.

56. Sumar las siguientes expresiones:

8 2 1 3
a) + + + +
9 3 2 8
1 2 1 2
b) + 2 + 3 + 4 +
3 3 3 3
57. Si S1 , S2 , , Sp son las sumas de las series geometricas de primeros terminos 1, 2, . . . , p
1 1 1
y razones , , , respectvamente, probar que
2 3 p+1

p(p + 3)
S1 + S2 + + Sp =
2

58. En un triangulo equilatero de lado a se unen los puntos medios de sus lados formandose
un nuevo triangulo equilatero. En este segundo triangulo se repite el procedimiento y
as sucesivamente. Calcule la suma de los permetros de todos los triangulos equilateros
as obtenidos y la suma de sus areas.
83

2.4. SUMATORIAS DOBLES


En la ultima parte de este captulo haremos algunas observaciones sobre sumatorias de
sumatorias, tambien llamadas sumatorias dobles.

Definicion 2.8
Sea f (i, j) una funcion de N N en R. Definimos el smbolo

Xn X
m n
X Xm
f (i, j) = f (i, j)
i=1 j=1 i=1 j=1

que permite las dos interpretaciones siguientes:


n
X
A.- [f (i, 1) + f (i, 2) + + f (i, m)] o
i=1
m
X m
X m
X
B.- f (1, j) + f (2, j) + + f (n, j)
j=1 j=1 j=1

Obviamente, son dos desarrollos que al finalizarlos resultan identicos. Es costumbre reemplazar
f (i, j) por una letra con 2 subndices tales como aij , xij , etc.

Ejemplo 2.42
2 X
X 3
Sea aij = ij . Entonces el desarrollo de ij es:
i=1 j=1

2
X
A.- (i1 + i2 + i3 ) = 11 + 12 + 13 + 21 + 22 + 23
i=1

3
X 3
X
B.- 1j + 2j = 11 + 12 + 13 + 21 + 22 + 23
j=1 j=1

Teorema 2.18 Intercambio del orden de la suma


n X
X m m X
X n
aij = aij
i=1 j=1 j=1 i=1

Demostracion
Es inmediato al observar que los mismos sumandos aparecen en cada sumatoria doble.

El siguiente teorema se refiere al caso particular en que f (i, j) es de la forma ai bj ; es decir,


es el producto de una funcion que solo depende de i, por otra que solo depende de j.

Teorema 2.19
n X
X m n
X m
X
ai bj = ai bj
i=1 j=1 i=1 j=1
84

Demostracion

Xn m
X n
X m
X
ai bj = ai bj , puesto que ai no depende de j.
i=1 j=1 i=1 j=1
Xm
Como bj depende solo de j y no de i, sale afuera de la sumatoria como una constan-
j=1

m n
!
X X
te, por lo tanto, es igual a bj ai .
j=1 i=1

Ejemplo
2.43
n m n
! m

X X X X n(n + 1) m(m + 1)(2m + 1)
ij 2 = i j2 =
2 6
i=1 j=1 i=1 j=1


n
X k
X
Ejemplo 2.44 Consideremos ahora una sumatoria del tipo akj . La interpretare-
k=1 j=1
mos como

1
X 2
X n
X
a1j + a2j + . . . + anj
j=1 j=1 j=1

Veremos como se debe efectuar el cambio de orden de la suma, con un ejemplo:

4 X
X k 1
X 2
X 3
X 4
X
aij = a1j + a2j + a3j + a4j
k=1 j=1 j=1 j=1 j=1 j=1

= a11 + a21 + a22 + a31 + a32 + a33 + a41 + a42 + a43 + a44

Reordenando:

a11 + (a21 + a22 ) + (a31 + a32 + a33 ) + (a41 + a42 + a43 + a44 )

4 X
X k 1
X 2
X 3
X 4
X k
X 4
X
aij = a1j + a2j + a3j + a4j = akj
k=1 j=1 j=1 j=1 j=1 j=1 j=1 k=j

Generalizando este ejemplo, se obtiene:

Teorema 2.20

n X
X k k X
X n
akj = akj
k=1 j=1 j=1 k=j
85

2.4.1. EJERCICIOS
59. Calcule
10 X
X 20
2ji
i=1 j=1

60. Calcule

20 X
X 25 n X
X m
a) i2 j b) 2i+j
i=1 j=5 i=1 j=1

61. Pruebe que:


n X
X 1 n X
X n
aij = aij
i=1 j=1 j=1 ij

62. Pruebe que:


n X
k n
X 1 X n+1j
=
j j
h=1 j=1 j=1

63. Calcule:

n k
X
1 X 1
k(k 1) j
k=2 j=1
86

2.5. SUGERENCIAS O SOLUCIONES A LOS EJERCICIOS


DEL CAPITULO
1. (a) 5(n + 1)3 + 7(n + 1) = (5n3 + 7n) + [15n(n + 1) + 12].
Pero n(n + 1) es par. Luego, los dos parentesis son divisibles por 6.
(b) 834n+4 2 972n+2 + 1 (834n 2 972n + 1)
= 16 2966145 834n 16 588 2 972n

2.
an+1 bn+1 = an+1 an b + an b bn+1 = an (a b) + b(an bn )

3. (a) Por induccion en n.

an+2 an = (an+1 + an )an

= an+1 an + a2n = an+1 an + an+1 an1 + (1)n

= a2n+1 (1)n+1

(b) Por induccion en m.

an+m+1 = an+m + an+m1

= am an+1 + am1 an + am1 an+1 + am2 an

= am+1 an+1 + am an

(c) Si p|an+1 y p|an+2 , como an = an+2 an+1 se tiene que p|an y p|an+1 .
1 1 1
4. Sea Sn = + + + 2
12 22 n
1 3 1 1 1 3 n1 1
Sn+1 = Sn + (n+1) 2 > + + = +
2 n n2 (n + 1)2 2 n2 (n + 1)2
n1 1
Pero 2
< , como se comprueba al multiplicar en forma cruzada.
n n+1
3 1 1
Luego, Sn+1 > + .
2 n + 1 (n + 1)2
Con respecto a la otra desigualdad tenemos que

1 1 (n + 1)2 n
Sn+1 < 2 + = 2
n (n + 1)2 n(n + 1)2

(n + 1)2 n n2 + n + 1 n(n + 1) 1
Pero = > =
n(n + 1)2 n(n + 1)2 n(n + 1)2 n+1
1
Luego, Sn+1 < 2
n+1
87

5. Sea A un conjunto con n + 1 elementos. Sea x A y B = A {x}. El numero de subcon-


juntos de B es 2n . Los subconjuntos de A que contienen a x se consiguen agregandole
este elemento a cada uno de los subconjuntos de B. Luego, tambien son 2n .

6. (a) 1 xn+1
(b) 1/(n + 1)
(c) (1)n1 n(n + 1)/2
(d) n(n + 1)/2
1/(n1) 1/n (n+1)/n
7. p2n > pn1 pn+1 . Pero, por induccion pn1 > pn y reemplazando, sale que pn >
pn+1 .

8.
un+1 = (1 + x)un nx. Pero por induccion,

1
un+1 = (1 + x) [1 + nx (1 + x)n ] nx
x
1
= [1 + (n + 1)x (1 + x)n+1 ]
x
9. Notemos que si p|n, entonces p|(n + p).

10. El ultimo termino es

4(n + 1)
(1)n1
(2n + 1)(2n + 3)

11. Al agregar una nueva recta a las n rectas en posicion general, se producen n + 1 nuevas
regiones.

12. El plano n + 1 es cortado por los n planos en n rectas en posicion general. Ellas lo
dividen, segun el problema 11, (n2 + n + 2)/2 regiones y cada una de estas introduce
una nueva seccion del espacio. Luego, el numero de partes en que n + 1 planos dividen
al espacio es

(n + 1)(n2 n + 6) (n2 + n + 2) (n + 2)[(n + 1)2 (n + 1) + 6]


+ =
6 2 6
13. La circunferencia n + 1 queda cortada por las n anteriores en 2n arcos. Cada uno de
ellos introduce una nueva region en el plano. Luego, el numero de partes en que n + 1
circunferencias dividen al plano es:

n2 n + 2 + 2n = (n + 1)2 (n + 1) + 2

14. a) 23 29 1001
b) 3 7 11 19 101 103
88

c) 3 2731 8191.

15. 9 576 71 73 = 1

16. p2 = 2q 2 p2 par p par p2 = 4m q 2 = 2m q 2 = 4t q es par.

17. 1 o 2

18. p|an p|a pn (an )

19. p + q es par. Luego, p + q = 2t con t no primo, pues p < t < q.

20. Todo primo impar tiene la forma 4n + 1 o 4n 1. El producto de dos numeros de la


primera conserva ese formato. Como el numero N es de la forma 4n 1, su descompo-
sicion en primos debe tener algun factor de esa forma. Pero ninguno de ellos puede ser
p1 , p2 , , pl pues no dividen a N .

21. Estos numeros son de las formas 6m+1 o 6m1. Por lo tanto, si p = 6m1 y q = 6m1
se tiene que p2 q 2 = 12(m n)[3(m + n) 1] el cual siempre tiene el factor 24 para
todas las posibilidades de paridad de m y n.

n+1
X 1 n 1 n+1
22. a) = + =
4k 21 2
2n + 1 4(n + 1) 1 2n + 3
k=1
n+1
X
b) (k 2 + 1)k! = n(n + 1)! + ((n + 1)2 + 1)(n + 1)! = (n + 1)(n + 2)!
k=1

1
23. a) n(2n 1)(2n + 1)
3
b) n2 (2n2 1)
1
c) n(n + 1)(2n + 7)
6
24. f (r + 1) f (r) = 10r4 + 2r2 . Luego,

n
X n
X n
X
f (r + 1) f (r) = 10 r4 + 2 r2
r=1 r=1 r=1

n
X
4 n(n + 1)(2n + 1) X
Calculando f (n+1)f (1) = 10 r + de donde resulta que r4 =
3
r=1
1
n(n + 1)(2n + 1)(3n2 + 3n 1).
30
2n
X 2n
X n
X
25. ui = ui ui = 2(2n)2 + 3(2n) (2n2 + 3n) = 3n(2n + 1)
i=n+1 i=1 i=1

2n
X 4
26. a) (k + 1)(k + 4) = n(n + 2)(2n + 5)
3
k=1
89

n
X 1
b) (n + k 1)(n + k) = n(7n2 1)
3
k=1
2n1 n1
X 1 X 1
c) Se pide calcular A = .
k(k + 1) k(k + 1)
k=1 k=1
Pero, sabemos que
p
X 1 1
=1
k(k + 1) p+1
k=1
 
1 1 1
Luego, A = 1 1 =
2n n 2n
27. a) Por fracciones parciales se obtiene que

1 1 1 1 1 1 1
= +
(2k 1)(2k + 1)(2k + 3) 8 2k 1 4 2k + 1 8 2k + 3
Luego, la sumatoria se puede escribir como

n   n  !
1 X 1 1 X 1 1

8 2k 1 2k + 1 2k + 1 2k + 3
k=1 k=1

Aprovechando la propiedad telescopica se obtiene:


 
1 2n 2n n(n + 2)
=
8 2n + 1 3(2n + 1) 3(2n + 1)(2n + 3)
k+2 2 1
b) = .
k(k + 1) k k+1
Luego, la sumatoria que se pide se transforma en
n  
X 1 1
k1
k
2 k 2 (k + 1)
k=1

1
Aprovechando la propiedad telescopica obtenemos para ella el valor 1 .
2n (n + 1)
c) k 4 + k 2 + 1 = (k 2 + k + 1)(k 2 k + 1)
Entonces, por fracciones parciales se obtiene que
 
k 1 1 1
=
1 + k2 + k4 2 k2 k + 1 k2 + k + 1
1 1
Sea f (k) = 2 , entonces f (k + 1) = 2 como se comprueba facil-
k k+1 k +k+1
mente.
n
1X
Luego, la sumatoria pedida es (f (k) f (k + 1)) que vale
2
k=1
 
1 1 1
(f (1) f (n + 1)) = 1 2
2 2 n +n+1
90

28. Se prueba con facilidad que


n
X 1
k = n(n + 1)
2
1

n
X 1
k(k + 1) = n(n + 1)(n + 2)
3
1

n
X 1
k(k + 1)(k + 2) = n(n + 1)(n + 2)(n + 3)
4
1

Estos resultados sugieren que la formula es

n
X 1
k(k + 1) (k + p) = n(n + 1) (n + p + 1)
p+2
1

que se prueba por induccion.


2r + 1
29. f (r) f (r + 1) =
r2 (r
+ 1)2
Luego, la sumatoria pedida es

n
X 1
f (r) f (r + 1) = f (1) f (n + 1) = 1
(n + 1)2
1

n
X
30. La sumatoria a calcular es 1 + ur .
r=2
Pero, es facil ver que ur = vr vr1 si r > 2
n n
X 2 X
Luego, 1 + ur = 1 + + (vr vr1 ) = vn 1
3 r
r=2 3

31. La suma de los impares es:


n
X n(2n + 1)(2n 1)
(2k 1)2 =
3
k=1

La suma de los pares es:


n
X
(2k)3 = 2n2 (n + 1)2
k=1

32. Es facil ver que la cantidad de sumandos es n2 . por lo tanto,la suma es:
n 2
X
(2k 1) = n4
k=1
91

33. Por induccion.


34. Es una permutacion de los sumandos.
35. La suma esta descompuesta en dos; la de los terminos de ndice par y la de los ndices
impares.
Si n es impar

n (n1)/2 (n1)/2
X X X
ai = a2k + a2k+1
i=0 k=0 k=0

36. Dos casos:


Si n par.
Sea n = 2p. Entonces
n
X
E(k/2) = 0 + 1 + 1 + 2 + 2 + + (p 1) + (p 1) + p
k=1

p1
X
=p+2 k = (p 1)p = p2
k=1

Pero, E(n2 /4) = E(4p2 /4) = p2 .

El caso n impar, es similar.


37. Notemos que si x es 0 o un natural el teorema es verdadero.

Sea x = q + r con q = 0 o un natural y 0 < r < 1.

a) Si r < 1/n, entonces E(q + r + k/n) = E(q) y el teorema es valido.


b) Si 1/n r < 2/n, el unico sumando que aumenta en 1 es el ultimo y el enunciado
sigue siendo verdadero.
c) Si 2/n r < 3/n, los dos ultimos sumandos sumentan en una unidad y el enunciado
sigue siendo valido.
Etc.

38. Por induccion.

n+1
X n
X n1
X
ak = an+1 + ak = an+1 + nan k(ak+1 ak )
k=1 k=1 k=1

n
X
= an+1 + nan k(ak+1 ak ) + n(an+1 an )
k=1

n
X
= (n + 1)an+1 k(ak+1 ak )
k=1
92

39. Por induccion. Es similar al anterior.

40. La unica posibilidad son tres numeros iguales.

41. Sea m y d el primer termino y la diferencia de la P.A. Entonces

a = m + (p 1)d

b = m + (q 1)d

c = m + (r 1)d

Formando la expresion se prueba que vale 0.


n
X
42. Se tiene que 6 + (k 1)4 = 1920 que origina la ecuacion n2 + 2n 960 = 0 cuya
k=1
solucion positiva es 30.

43.
a
+ a + ar = 38
r

a3 = 1728

de lo que resulta a = 12 y r = 3/2 o 2/3.

44. Sea m el primer termino y t la razon. Entonces

a = mtp1 ; b = mtq1 ; c = mtr1 .

Formando la expresion con m y t resultan ambos con exponente 0.

45. Sean tp y tq los terminos conocidos de ordenes p y q respectivamente. Sea a el primer


termino de la P.A. y d su diferencia. Entonces:

tp = a + (p 1)d
tq = a + (q 1)d
Forman un sistema lineal de 2 ecuaciones con dos incognitas.
2ac
46. b = . Reemplazando este valor en el primer miembro, despues de algun manipuleo
a+c
elemental se obtiene la identidad.

47. r=3. Las soluciones 1 y 244 no sirven. Por que?

48. Sea x el primer termino de la P.A. correspondiente y a0 la diferencia. Entonces

1 1
x + (m 1)d = =
am n

1 1
x + (n 1)d = =
an m
93

1
Se obtiene que d = . Ademas
mn
1
= xx + (m + n 1)d
am+n

= x + (m 1)d + nd

1 n 1 1
= + = +
n mn am an

49.
rn 1
S=a
r1

1 ( 1r )n 1 rn 1
S0 = =
a 1r 1 arn1 (r 1)

P = an rn(n1)

Formando la expresion (S/S 0 )n se comprueba que vale P 2 .


ac
50. a, b, c en P.H. ssi 1/a, 1/b, 1/c en P.A. ssi b/2 = a+c .
2 ac
Por otra parte, a b/2, b/2, c b/2 en P.G. ssi b /4 = (a b/2)(c b/2) ssi b/2 = a+c .

51. a) Sea Ci el capital al termino del ano i. Entonces

C1 = C(1 + i)

C2 = C1 (1 + i) = C(1 + i)2

..
.

Cn = C(1 + i)n
Luego, la utilidad sera: C((1 + i)n 1)

b) En este caso la utilidad es nCi.

52. La formula es

9(10n + 2 10n1 + + n) + n + 1 = 10n+1 + 10n + + 1

Para probarla, sea Sn = 10n + 2 10n1 + + n


Entonces

10Sn = 10n+1 + 2 10n + + n 10

y restando
94

95n = 10n+1 + 10n + cdots + 10 n

Luego, 95n + n + 1 = 10n+1 + 10n + + 10 + 1.

53. Notemos que las sumas son P.G. de razones x, x y x2 .

1 x2n+1
1 + x + x2 + + x2n =
1x

1 + x2n+1
1 x + x2 + x2n =
1+x

1 x4n+2
Su produto es: , que corresponde a
1 x2

1 + x2 + x4 + + x4n

54. Sea Sn = 1 + 2x + 3x2 + + nxn1 . Entonces:

1 xn
(1 x)Sn = 1 + x + x2 + + xn1 nxn = nxn
1x

De donde:
1 xn nxn
Sn =
(1 x)2 1 x

55.
1
r = a
4

a + ar = 24

Sus soluciones son a = 8 y r = 2 o a = 12 y r = 3.

56. a) Es una P.G. con razon 3/4.


8 1 ( 34 )n 32
Sn = 3 que converge a .
9 1 4 9
b) Son dos P.G. ambas de razones 1/9
1 1 (1/9)n 2 1 (1/9)n 5
S2n = + , que converge a .
3 1 1/9 9 1 1/9 8

k
57. Sk = 1 =k+1
1 k+1
Luego,
95

S1 + S2 + + Sp = 2 + 3 + + (p + 1)

(p + 1)(p + 2) 2
= 1 + 2 + 3 + + (p + 1) 1 =
2
p(p + 3)
=
2

58. El permetro
 de cada triangulo
 es la mitad del anterior. Luego, la suma de los permetros
1 1
es 3a 1 + + + = 6a.
2 4

El area de un triangulo es
 1/4 de la anterior. Luego, la suma de las areas es
a2 1 1 a2
3 1+ + + = 3.
4 4 16 3

59.

20 20
X 1 X j 1
2ji = 2 = i 2(220 1)
2i 2
j=1 i=1

10 X
20 10
X
ji 20
X 1 (220 1)(210 1)
2 = 2(2 1) =
2i 2
i=1 j=1 i=1

20 X
25 20 25 20
X X X X 20 21 41
60. a) i2 j = i2 j = 315 i2 = 315 = 904050
6
i=1 j=5 i=1 j=5 i=1

n X
X m n
X m
X n
X
b) 2i+j = 2i 2j = 2(2m 1) 2i = 4(2m 1)(2n 1)
i=1 j=1 i=1 j=1 i=1

61. Por induccion en n.


96

n+1
XX i n X
X i n+1
X
aij = aij + an+1,j
i=1 j=1 i=1 j=1 j=1

n X
X n n+1
X
= aij + an+1,j
j=1 i=j j=1

n
X n
X n
X
= ai1 + ai2 + + ain
i=1 i=2 i=n

+an+1,1 + an+1,2 + + an+1,n+1

n+1
X n+1
X n+1
X n+1
X
= aij + ai2 + + a i, n + ai,n+1
i=1 i=2 i=n i=n+1

n+1
X n+1
X
= aij
j=1 i=j

62. Por induccion en n.

n+1 k n X
k n+1
XX 1 X 1 X 1
= +
j j j
k=1 j=1 k=1 j=1 j=1

n n
X n+1j X 1 1
= + +
j j n+1
j=1 j=1

n
X n+2j 1
= +
j n+1
j=1

n+1
X n+2j
=
j
j=1


1/k(k 1) si k 2
63. Sea f (k) =
0 si k = 1

Entonces

n k n k
X 1 X 1 XX 1
= f (k)
k(k 1) j j
k=2 j=1 k=1 j=1

Por el ejercicio 3
97

n X
n n n
X 1 X 1X
= f (k) = f (k)
j j
j=1 k=j j=1 k=j

Pero, es facil calcular que

1

n

1 si j = 1
X
n
f (k) =
k=j
1 1
si j 2



j1 n
Luego,

n  
X 1 1 1 1
= +1
j j1 n n
j=2

n
2 1X1
=2
n n j
j=2
98
Captulo 3

COEFICIENTES BINOMIALES
 
n
Los numeros , que definiremos a continuacion, llamados coeficientes binomiales o tam-
k
bien, numeros combinatorios, tienen propiedades muy utiles tanto para estadstica como en
combinatoria y computacion.

Se llaman coeficientes binomiales porque aparecen como los coeficientes en el desarrollo del
binomio (a + b)n , como veremos mas adelante.

Para su definicion, necesitaremos previamente el concepto de factorial.

Definicion 3.1
Sea n N0 . Definimos inductivamente n! (se lee n factorial) de la manera siguiente:

0! = 1 y (n + 1)! = n! (n + 1)

Ejemplo 3.1

1! = 0! 1 = 1
2! = 1! 2 = 2
3! = 2! 3 = 2 3 = 6
Intuitivamente, el significado de n! es el producto 1 2 3 . . . n

Definicion 3.2  
n
Sea n un numero natural y k un entero. Definimos el smbolo por:
k

n!
  si 0 k n
n
k!(n k)!
=
k

0 en los restantes casos

 
n
El smbolo , que tambien se escribe Cnk se llama, como ya dijimos, coeficiente binomial,
k
pero tambien es conocido como numero combinatorio por su utilizacion en la teora
combinatoria, tal como veremos en el siguiente captulo.
Ejemplo 3.2
Calculemos algunos coeficientes binomiales:

99
100
 
7 7! 1234567
= = = 35
3 3!4! 1231234
 
n n! n!
= = =1
n n!0! n!
 
n n! n!
= = =1
0 0!n! n!
 
n n! n (n 1)!
= = =n
1 1!(n 1)! (n 1)!

Nota  
n n(n 1) . . . (n k + 1)
Notemos que se simplifica de modo que se reduce a donde el
k k(k 1) . . . 1
denominador de la fraccion esta formado por el producto de los numeros sucesivos de 1 a k,
mientras el numerador esta formado por k numeros sucesivos, pero en sentido decreciente a
partir de n.
 
20 20 19 18
Por ejemplo: =
3 123

Teorema 3.1
Algunas propiedades de uso frecuente
   
n n
1. = Condicion de simetra.
k nk
     
n n n+1
2. + = Conocida como Formula de Pascal.
k k+1 k+1
       
n m n nk
3. = con o k m n. Se conoce como simplificacion del
m k k mk
producto.

Demostracion
Las demostraciones de 1. y de 2. las dejamos al lector.
   
n m n! m!
=
m k m!(n m)! k!(m k)!

Cancelando por m! y amplificando por (n k)!, obtenemos


   
n! (n k)! n nk
=
k!(n k)! (m k)!(n m)! k mk

El cuadro que viene a continuacion se conoce como Triangulo de Pascal. Solo lo desarrollaremos
para n de 1 a 8. En la columna de la izquierda estan los valores
  para n y en la fila superior,
n
los valores de k. Los numeros del interior corresponden a , con 0 k n. Los demas
k
valores de cada fila son ceros y no han sido escritos.
101

0 1 2 3 4 5 6 7 8
0 1
1 1 1
2 1 2 1
3 1 3 3 1
4 1 4 6 4 1
5 1 5 10 10 5 1
6 1 6 15 20 15 6 1
7 1 7 21 35 35 21 7 1
8 1 8 28 56 70 56 28 8 1
 
n
Los unos de la primera columna son los valores y los del final de cada fila corresponden
  0
n
a los valores de .
n
La formula de la suma (propiedad 2. del Teorema 3.1), interpretada en este cuadro, nos dice
que la suma de dos numeros sucesivos en una fila es igual al numero que se encuentra en la
fila siguiente, inmediatamente debajo del segundo sumando.
     
6 6 7
Por ejemplo: + = , lo que en el cuadro aparece 15 + 20 = 35.
2 3 3
Aprovechando esta propiedad, podemos escribir la novena fila. Ella resulta:

1, 9, 36, 84, 126, 126, 84, 36, 9, 1

La simetra que aparece en las filas es simplemente el reflejo de la propiedad 1. del Teorema 3.1.

Ejemplo 3.3        
8 8 8 10
Demostrar que +2 + = .
6 5 4 6
Partiendo del primer miembro tenemos:
             
8 8 8 8 8 8 8
+2 + = + + +
6 5 4 6 5 5 4
   
9 9
= +
6 5
 
10
=
6

Ejemplo 3.4  
n+1
r+1
Transformar la expresion   , usando factoriales, suponiendo que r n.
n
r1
Pasando a fraccion simple
102

 
n+1
r+1 (n + 1)! (r 1)!(n r + 1)!
  =
n (r + 1)!(n r)! n!
r1
Simplificando los factoriales de las fracciones respectivas, obtenemos finalmente

(n + 1)(n r + 1)
r(r + 1)

Ejemplo 3.5
   
n n n1
Probar que =
k nk k
   
n n! n (n 1)! n n1
= = =
k k!(n k)! n k k!(n k 1)! nk k

Teorema 3.2 Formulas de Sumas

n    
X r+k r+n+1
1. =
k n
k=0
n    
X k n+1
2. =
m m+1
k=0

Demostracion
La primera es inmediata por induccion. Dejamos los detalles al lector.
Demostraremos la segunda por induccion en n.
Cuando n=1, debemos considerar dos casos: El primero corresponde a m>1. En estas condi-
ciones ambos mienbros valen 0. Si m=1, ambos mienbros valen 1. por lo tanto,la afirmacion
es valida para n=1.
Supongamos el enunciado verdadero para n. Debemos probarlo para (n+1).
n+1
X  n          
k X k n+1 n+1 n+1 n+2
= + = + =
m m m m+1 m m+1
k=0 k=0

Nota
Observando el Triangulo de Pascal, la afirmacion 1 del teorema
  3.2, nos dice que la
 suma de
r r+n
los terminos que estan en la diagonal, que empieza en y termina en es igual
0 n
al numero que se encuentra
 inmediatamente bajo este ultimo. Por ejemplo, consideremos la
3
diagonal que empieza en . Ella esta formada por los numeros 1, 4, 10, 20, 35, 56, etc. La
0
103

suma de los dos primeros es 5, que es exactamente el numero que esta debajo del 4. La suma
de los tres primeros es 15, que es justamente el numero que esta bajo el 10, etc.

La afirmacion 2. nos dice que la suma de los numeros que estan en la columna n, sumados
desde la fila 0 hasta la fila n, es igual al numero que se encuentra en la siguiente fila y siguiente
columna.

Este teorema puede aprovecharse para calcular algunas sumatorias, como veremos en el
siguiente ejemplo:

Ejemplo 3.6
n
X
Calcular k(k + 1)(k 1)
k=0
   
k+1 (k + 1)k(k 1) k+1
Notemos que = de donde k(k + 1)(k 1) = 6
3 6 3
Luego,
n n   n+1
X  n+1
X 
X X k+1 k k
k(k + 1)(k 1) = 6 =6 =6
3 3 3
k=0 k=0 k=1 k=0

 
n+2
Por el Teorema de las Sumas parte 2., esta expresion es igual a 6
4

A continuacion, veremos algunos ejemplos en que aparecen desigualdades.

Ejemplo 3.7    
n n
Demostrar que > si n > 7.
4 3
Tenemos:
 
n n!
=
4 4!(n 4)!
Amplifiquemos la fraccion por (n 3) para as formar n3 .


   
n n!(n 3) n!(n 3) n n3
= = =
4 4!(n 4)!(n 3) 4!(n 3) 3 4
n
 n
 n3
Entonces, 4 sera mayor que 3 si y solo si es mayor que 1.
4
n n
 
Luego,: 4 > 3 si y solo si n 3 > 4, es decir si y solo si n > 7.

Ejemplo 3.8
Observando el triangulo de Pascal notamos que en cada fila los valores numericos aumentan
hasta llegar a un maximo que se encuentra en el coeficiente central o en los dos centrales depen-
diendo si n es impar o par, para despues disminuir
 nsucesivamente hasta llegar a 1. Probaremos
n
esta observacion analizando el cuociente k+1 / k . Pasando a factoriales y simplificando se
obtiene que
104

n

k+1 nk
n
 =
k
k+1

n
 n
 nk
Entonces el coeficiente k sera menor que su sucesor k+1 si y solo si > 1.
k+1
n1
Pero esto equivale a k < .
2
n1
Cuando n es impar, es un natural y los coeficientes creceran hasta que k tome el valor
2
n1 n
 n

. Este coeficiente es n1 . El siguiente es n+1 y, por la simetra, son iguales. Desde
2 2 2
all empezaran a disminuir sucesivamente hasta llegar a 1.
n1 n2 n
Cuando n es par no es entero, encontrandose entre los numeros enteros y . Por
2 n
 2 2
lo tanto los coeficientes creceran hasta n y despues disminuiran.
2

A continuacion estableceremos el teorema, conocido como el Teorema del Binomio, que


muestra como desarrollar (a+b)n . Antes de enunciarlo haremos algunas observaciones para su
mejor comprension. Desarrollaremos (a + b)1 , (a + b)2 , (a + b)3 y escribiendo sus coeficientes
en la forma de coeficientes binomiales.

Sean a, b numero reales, tenemos:


1 1
(a + b)1 = a + b =
 
0 a+ 1 b

2 2 2
(a + b)2 = a2 + 2ab + b2 = a2 + b2
  
0 1 ab + 2

3 3 3 3
(a + b)3 = a3 + 3a2 b + 3ab2 + b3 = a3 + a2 b + ab2 + b3
   
0 1 2 3

Observando la expresion de (a+b)3 , vemos que las potencias de a decrecen desde treshasta  ce-
ro y las de b crecen desde cero hasta tres. Los coeficientes de los productos son 30 , 31 , 32 , 33
respectivamente. Un analisis analogo se puede hacer para (a + b)1 y (a + b)2 .

Otra observacion que podemos hacer, es que el numero de terminos de cada desarrollo es
superior en uno al exponente del binomio.

Estas observaciones no sugieren el siguiente teorema.

Teorema 3.3
Sea n un numero natural y a, b reales. Entonces,
n  
X n nk k
n
(a + b) = a b
k
k=0

Demostracion
Por induccion en n.
105

1 1
(a + b)1 =
 
Para n = 1 0 a+ 1 b que es verdadero.

Supongamos que la igualdad es valida para n. Esto es,


n  
X n nk k
(a + b)n = a b
k
k=0

Pasando al caso n+1


n  
!
X n nk k
(a + b)n+1 = (a + b) a b
k
k=0

n   n  
X n n+1k k
X n
= a b + ank bk+1
k k
k=0 k=0

Para determinar la demostracion, deberemos usar algunas triquinuelas algebraicas. La primera


consiste en sacar fuera de la primera sumatoria el termino para k = 0 y de la segunda, el que
se obtiene con k = n. Hecho esto, la expresion anterior nos queda:
n   n1  
n+1
X n n+1k k X n nk k+1
=a + a b + a b + bn+1
k k
k=1 k=0

Ahora ajustamos el ndice de la segunda sumatoria para lograr que empiece en 1 y termine
en n y as reunir ambas sumatorias en una.
n   n  
X n X n
= an+1 + an+1k bk + an+1k bk + bn+1
k k1
k=1 k=1

n    
X n n
= an+1 + + an+1k bk + bn+1
k k1
k=1

n  
X n+1
= an+1 + an+1k bk + bn+1
k
k=1

Notando que la sumatoria para k = 0 es an+1 y para k = n + 1 es bn+1 podemos escribir que
n+1
X 
n + 1 n+1k k
= a b
k
k=0

lo que finalmente, termina la demostracion por induccion.

Corolario

n  
n
X
kn nk k
(a b) = (1) a b
k
k=0
106

Demostracion
n  
X n nk
(a b)n= [a + (b)]n
= a (b)k ; luego
k
k=0
n  
X n
(a b)n = (1)k ank bk .
k
k=0

Ejemplo 3.9
1 5
 
Desarrollar x + .
x

5  
1 5 5 5k 1 k
  X  
x+ = x
x k x
k=0
       
5 5 1 5 3 1 5 2 1 5 1 5 1
x5 x4
 
= 0 + 1 + x 2+ x 3+ x 4+
x 2 x 3 x 4 x 5 x5

10 5 1
= x5 + 5x3 + 10x + + 3 + 5.
x x x
Ejemplo 3.10
1 n
 
Si xr se encuentra en el desarrollo de x + , hallar su coeficiente.
x
Tenemos que
n  
1 n n nk 1 k
  X  
x+ = x
x k x
k=0

n  
X n
= xn2k
k
k=0

nr
El exponente de x, tomara el valor r, cuando k = . Notemos que el problema tiene
2 n
solucion solo si n r es un numero par o cero. En este caso, el coeficiente de xr es nr

.
2

Nota
n
En el desarrollo de (a + b)n el termino ank+1 bk1 se llama termino de orden k, y lo

k1
anotaremos por Tk .(k > 0).

Ejemplo 3.11
 16
1 2
Sea x . El decimo termino en el desarrollo de este binomio, esta dado por
x
   7  
16 1 16
T10 = (x2 )9 = x11
9 x 9

Si en el mismo desarrollo anterior queremos encontrar el termino del medio, dado que el
numero de terminos es 17, este termino es el noveno, luego
107

   8  
16 1 2 8 16 8
T9 = (x ) = x
8 x 8

Ejemplo 3.12
Dada la expresion:
 
50 1
(1 x) + 1 + x2
x
encontrar el coeficiente de x25 .

Desarrollando el primer factor obtenemos:

           
50 50 50 2 50 r 50 50 1 2
+ (x) + (x) + + (x) + + (x) +1+x .
0 1 2 r 50 x

Al multiplicar estos dos polinomios, los terminos que contienen x25 son:
     
50 26 1 50 25 50
(x) , (x) , (x)23 x2 .
26 x 25 23
De manera que el coeficiente del termino en x25 es:
     
50 50 50
.
26 25 23

Ejemplo 3.13
n   n  
X n X n
Calcular las sumatorias y (1)k .
k k
k=0 k=0
n  
X n x
Sabemos que: (1 + x)n = x para todo x real.
k
k=0
Dandole a x el valor 1, se obtiene:
n  
n
X n
2 =
k
k=0

lo cual nos dice que la suma de los coeficientes binomiales n0 + n1 + + nn es igual a 2n .


  

Mirados como elementos del Triangulo de Pascal: la suma de los coeficientes de la n-esima fila
es 2n .

Analogamente, partiendo de
n  
n
X n k
k
(1 x) = (1) x ,
k
k=0

y haciendo x = 1 obtenemos que


108

n  
k n
X
0= (1)
k
k=0

Pasando los negativos a la izquierda, se puede escribir:


           
n n n n n n
+ + + = + + +
0 2 4 1 3 5

que nos dice que la suma de los coeficientes de lugares pares es igual a la suma de los coeficientes
de lugares impares en cada fila del triangulo de Pascal.

Ejemplo 3.14 
n
X n
k
Calcular .
k+1
k=0
Tenemos
n

k n!
=
k+1 k!(n k)! (k + 1)

n!
=
(k + 1)! (n k)!
Amplificando por (n + 1) nos queda
n
  
k (n + 1)! 1 n+1
= =
k+1 (n + 1)(k + 1)!(n k)! n+1 k+1
Luego,:
n n
 n  
X
k 1 X n+1
=
k+1 n+1 k+1
k=0 k=0

Usando el ejemplo 3.13, nos queda


n n

X 1
k
= (2n+1 1).
k+1 n+1
k=0

Ejemplo 3.15
Usando la igualdad

(1 + x)2n = (1 + x)n (1 + x)n ,


demuestre que
 2  2  2
n n n (2n)!
+ + + = .
0 1 n (n!)2
El coeficiente de xn , en el desarrollo de (1 + x)2n , es:
109

 
2n (2n)!
= (1)
n (n!)2
Ahora calculemos el coeficiente de xn en el desarrollo de (1 + x)n (1 + x)n .

n  
! n  
X n k X n j
(1 + x)n (1 + x)n = x x
k j
k=0 j=0

n X
n   
X n n k+j
= x
k j
k=0 j=0

El coeficiente de xn lo encontramos tomando aquellos terminos de la sumatoria doble para


los cuales k + j = n. Ellos son: k = n; j = 0; k = n 1; j = 1;. . . ;k = 0; j = n.
Entonces e coeficiente de xn es
          2  2  2
n n n n n n n n n
+ + + = + + + (2)
n 0 n1 1 0 n 0 1 n
Como se partio de la identidad (1 + x)2n = (1 + x)n (1 + x)n se tiene que los coeficientes de
(1) y (2) son iguales, y por lo tanto:
 2  2  2
n n n (2n)!
+ + + =
0 1 n (n!)2
Para cerrar el captulo probaremos, en forma similar al ejemplo anterior, la identidad de
Vandermonde. Otra demostracion se vera en el siguiente captulo. Usaremos la notacion com-
binatoria.

Teorema 3.4
n
X
Cnm+r = m
Cnk Ckr
k=0

Demostracion
Consideremos la identidad:

(1 + x)m+r = (1 + x)m (1 + x)r

El coeficiente de xn del primer miembro es Cnm+r y debe ser igual al del segundo miembro.
Este coeficiente es:

C0m Cnr + C1m Cn1


r m
Ckr + + Cnm C0r

+ + Cnk

n
X
m
que se resume en Cnk Ckr .
k=0
110

3.0.1. EJERCICIOS
1. Simplifique:
n!
a) (r < n)
(n r)!
n! (n 1)!
b)
(n 1)!
2. Demuestre que:

(2n)!
= 2n 1 3 5 (2n 1)
n!
3. Demuestre que:

a) r nr = n n1
 
r1
b) nr > r1n
 
si n + 1 > 2r

4. Encuentre n > 0 sabiendo que:

a) n3 = n5
 

n

b) 2 = 55

5. Determine los numeros a, b, c y d de modo que


     
3 k k k
k =a +b +c +d
3 2 1
n
X
y usando el teorema 3.2 calcule k3
k=1

6. Encuentre el 4 termino en el desarrollo de:


(1 + 3x)n

7. Encuentre el termino independiente de x en el desarrollo de:


 7
2
(2x + 1) 1 +
x
8. Encuentre el mayor coeficiente numerico en el desarrollo de:

(1 + 5x)13

9. Encuentre el coeficiente de xn en el desarrollo de [1 + (n + 1)x] (1 + x)2n1 y a partir


de ese valor, pruebe que:
 2  2  2
n n n (n + 2)(2n 1)!
+2 + + (n + 1) =
0 1 n (n!)(n 1)!
111

10. Demuestre que:


        
n n n n n n (2n)!
+ + + =
0 1 1 2 n1 n (n + 1)!(n 1)!

11. Encuentre la suma y el producto entre (2 + 3)7 y (2 3)7 .

Pruebe que la parte entera de (2 + 3)7 es 10,083,

12. Demostrar que:


       
n n n n
+2 +3 + + n = n 2n1
1 2 3 n
p   
X m n
13. A partir de la identidad (1 + x)m (1 + x)n = (1 + x)m+n , pruebe que =
k pk
  k=0
m+n
.
p
14. Calcular:
n  
X n
a) a qk (a, q R)
k
k=0
n  
X n
b) (a + kd) (a, d R)
k
k=0

15. Pruebe que:

n
X k 1
=1
(k + 1)! (n + 1)!
k=1

16. Use el ejercicio 13 para probar que

s     
X m n m+n
= con s n p
k p+k m+p
k=0
112

3.1. SOLUCIONES O SUGERENCIAS A LOS EJERCICIOS


DEL CAPITULO

1. a) n(n 1) (n r + 1)
b) n 1

2. Por induccion.

(2(n + 1))! (2n)!(2n + 1)(2n + 2)


= = 2n+1 {1 3 (2n + 1)}
(n + 1)! n!(n + 1)
 
n
 r n! n(n 1)! n1
3. a) r r = = =n
r!(n r)! (r 1)!(n r)! r1
n

r  n!(r 1)!(n r + 1)! (n r + 1)
b) n = = .
r1
r!(n r)!n! r
Entonces nr > r1 n
 
si y solo si n r + 1 si n r + 1 > r, es decir, si y solo si
n + 1 > 2r.
n! n!
4. a) = si y solo si 20 = (n 3)(n 4) que tiene como solucion
3!(n 3)! 5!(n 5)!
n = 8.
1
b) Origina la ecuacion n(n 1) = 55 que tiene como solucion n = 11.
2
(Otras soluciones son negativas).

5. Es facil ver que


     
k 2 k k
k= yk =2 +
1 2 1

A partir de 6 k3 sale que k 3 = 6 k3 + 6 k2 + k


   
1 , de donde obtenemos a = b = 6, c = 1,
d = 0. Por lo tanto

n n n   n   n  
X
3
X
3
X k X k X k
k = k =6 +6 +
3 2 1
k=1 k=0 0 0 0

que por el teorema 2


     
n+1 n+1 n+1
=6 +6 +
4 3 2

que resulta igual a

 2
n(n + 1)
.
2

6. C3n 27x3 .
113

7 7 7
2k+1 2k

2 X X
7. (2x + 1) 1 + = Ck7 k1 + Ck7
x x xk
k=0 k=0
Luego, el termino independiente de x es

C17 22 + C07 20 = 29

8. Los coeficientes numericos son de la forma ak = Ck13 5k .


Estudiemos el cuociente

ak+1 C 13 5k+1 13 k
= k+113 k
=5
ak Ck 5 k+1

13 k
Cuando este cuociente es mayor o igual que 1 se tiene que ak+1 5 ak 1 si y
k+1
a11 a12
solo si k 10, 7. Luego, el valor > 1 pero < 1. Luego, el mayor coeficiente se
a10 a11
consigue con k = 11 y es 511 C1113 .

9. [1 + (n + 1)x](1 + x)2n1 = (1 + x)n (1 + x)n + nx(1 + x)n1 (1 + x)n .


Luego, el coeficiente de xn en el primer miembro debe ser igual al de xn en el segundo
miembro.
Coeficiente de xn en

2n1 (n + 2)(2n 1)
[1 + (x + 1)x](1 + x)2n1 = Cn2n1 + (n + 1)Cn1 =
n!(n 1)!

Coeficiente de xn en

(1 + x)n (1 + x)n = C0n Cnn + C1n Cn1


n n
+ + Ckn Cnk + + Cnn C0n

= (C0n )2 + (C1n )2 + + (Cnn )2

Coeficiente de xn en

nx(1 + x)n1 (1 + x)n = n(C0n1 Cn1


n + C1n1 Cn2
n +

+Ckn1 Cnk1
n n1 n
+ + Cn1 C0 )

Pero nCkn1 Cnk1


n n )2 .
= (k + 1)(Ck+1
Luego, el coeficiente de xn en el segundo miembro es

(C0n )2 + 2(C1n )2 + + (n + 1)(Cnn )2

10. Igual al anterior, pero buscando el coeficiente de xn1 en la expresion

(1 + x)2n = (1 + x)n (1 + x)n .


114

11.

(2 + 3)7 + (2 3)7 = 2(C07 27 + C07 27 + C27 25 3 + C47 23 32 + C67 2 33 ) = 10084

(2 + 3)7 (2 3)7 = ((2 + 3)(2 3))7 = 1

(2 + 3)7 = 10084 (2 3)7 .

Pero 0 < (2 3) < 1. Luego, (2 3)7 < 1. Entonces, la parte entera de (2 + 3)7 =
10083.
 
n
 n! n1
12. k =k
k =n .
k!(n k)! k1
Luego, la suma a calcular es igual a
     
n1 n1 n1
n + + + = n2n1 .
0 1 n1

13. Se deben igualar los coeficientes de xp en ambos miembros. Es similar al 9 o 10.


n   n  
X n k
X n k
14. a) aq = a q = a(1 + q)n
k k
k=0 k=0
n
X n   n   n  
X n X n
b) (a + kd) = a +d k = 2n a + dn2n1
k k k
k=0 k=0 k=0

15. Por induccion

n+1
X k 1 n+1 (n + 2) (n + 1) 1
=1 + =1 =1
(k + 1)! (n + 1)! (n + 2)! (n + 2)! (n + 2)!
k=1

s   
n
 X m n
16. Si k > n p se tiene que p+k = 0. Luego, con s n p
k p+k
k=0
np
X   np
X m     
m n n m+n m+n
= = = =
k p+k k npk np m+p
k=0 k=0
Captulo 4

ELEMENTOS DE
COMBINATORIA

4.1. NOCIONES BASICAS


Los problemas de combinatoria surgen en las mas diversas especialidades. As, por ejemplo,
los fsicos, qumicos, ingenieros, linguistas, especialistas en computacion, etc., y obviamente los
matematicos se ven enfrentados continuamente a problemas de esta naturaleza. En general
podemos decir que ellos resultan de situaciones en que es necesario contar el numero de
selecciones que pueden hacerse de algunos objetos de un cierto conjunto o el numero de
ordenaciones, tambien llamados arreglos, en que pueden ordenarse sus elementos.
Los dos ejemplos siguientes ilustran ambos casos.

Ejemplo 4.1
De cuantas maneras se pueden elegir dos libros, de idiomas diferentes, entre 5 libros en latn,
7 libros en griego y 10 libros en frances?.

Ejemplo 4.2
Cuantas placas diferentes de automoviles se pueden especificar usando 4 letras en los prime-
ros lugares, seguidas de 2 dgitos?.

En todos estos problemas partimos de un conjunto dados de elementos, que pueden ser
numeros, letras, personas, libros, etc. y debemos encontrar el numero de selecciones o de arre-
glos que cumplan ciertas condiciones.

La diferencia que hay entre selecciones y arreglos, es que en las primeras no importa el orden
mientras que en estos ultimos si importa. As, el primer problema es de selecciones ya que
es igual elegir los libros A y B que elegir B y A, mientras que el segundo es de arreglos u
ordenaciones, porque se consideran distintas las patentes ABAB12 con la BABA12 o aun con
la ABAB21, aunque estan compuestas de los mismos elementos.

Antes de intentar resolver estos problemas es necesario precisar nuestros conceptos e introdu-
cir algunos resultados. Con este fin, reduciremos las nociones involucradas a expresiones de
la teora de conjuntos aunque usaremos siempre el lenguaje tpico de la combinatoria. Jus-
tamente, este lenguaje tpico considera como sinonimo las palabras arreglos y ordenaciones

115
116

por una parte y selecciones y combinaciones por otra. Nosotros mantendremos estos sinonimos.

Todos los conjuntos que intervengan seran finitos. Denotaremos por N (S) el numero de ele-
mentos de un conjunto S y por S r el producto cartesiano de S, r veces consigo mismo. Daremos
por conocidas las dos propiedades que a continuacion se enuncian.

A) Si S T = entonces N (S T ) = N (S) + N (T )

B) N (S T ) = N (S) N (T )

Los siguientes resultados, denominados como Regla de la suma y Regla del producto son
las piedras fundamentales en las que se basan los argumentos combinatorios. Dicen as:

4.1.1. REGLA DE LA SUMA


Si un suceso P puede ocurrir de m manera y otro Q de n maneras, todas ellas distintas
de las primeras, entonces hay m + n maneras en que puede ocurrir uno y solo uno de estos
sucesos. Esto ultimo, en forma mas tecnica dice que hay m + n maneras en que puede ocurrir
el suceso P Q. (P o Q pero no ambos).

4.1.2. REGLA DEL PRODUCTO


Si un suceso P puede ocurrir de m maneras y si cada una de estas ocurrencias hay n
maneras en que puede ocurrir otro suceso Q, entonces hay mn maneras en que puede ocurrir
el suceso P Q, es decir ambos sucesos.

Para aclarar el uso de estas reglas, consideremos el siguiente caso: Se exhiben siete pelcu-
las y cuatro obras teatrales. A cuantos espectaculos distintos se puede asistir?.

Esta pregunta cae en el ambito de la regla de la suma ya que de su enunciado se desprende


que se asistira solo a un espectaculo sea pelcula u obra teatral. Luego, la respuesta es 11.
Pero la pregunta De cuantas maneras se puede elegir una pelcula y una obra teatral?
correspondera responderla segun la regla del producto, en este caso sera 28.

Como una receta practica, aunque no infalible, podemos decir que los enunciados que usan la
conjuncion o corresponden a la regla de la suma, mientras los con y la del producto. Pero
los problemas se enuncian con expresiones bien formadas idiomaticamente y parte de la difi-
cultad de su resolucion estriba en convertir ese enunciado en otro equivalente que se exprese
solo mediantes y y o. Por ejemplo, los enunciados de los ejemplos 1 y 2 se expresaran
del modo siguiente: De cuantas maneras se pueden elegir un libro en latn y otro en griego
o uno en latn y otro en frances o uno en griego y otro en frances, de entre 5 libros en latn y
7 en griego y 10 en frances?.

El del ejemplo 2 rezara as: Cuantas placas diferentes se pueden especificar usando cuatro
letras en los primeros lugares y dos dgitos en los lugares quinto y sexto?.

Antes de seguir avanzado, queremos justficar estas reglas. La de la suma no es mas que la
propiedad A) enunciada anteriormente. La del producto necesita alguna elaboracion.
117

Sean p1 , p2 , , pm las formas en que puede ocurrir P y sean qj,1 , qj,2 , , qj,n las formas
en que puede ocurrir Q si es que ocurrio pj . Entonces podemos representar las distintas
maneras en que pueden ocurrir P y Q con parejas del tipo (pj , qj,i ). La coleccion de todas
estas parejas es

T = {(p1 , q11 ), , (p1 , q1n )} {(p2 , q21 ), , (p2 q2n )}


{(pm , qm ), , (pm qmn )}
A partir de las propiedades A y B anteriores es inmediato que N (T ) = m n.

Vamos a ilustrar el uso de estas dos reglas resolviendo los problemas de los ejemplos 4.1 y 4.2.

Para el primero notemos que hay 5 maneras de elegir un libro en latn y 7 en elegir uno en
griego entonces, por la regla del producto, hay 5 7 maneras de elegir un libro en latn y uno
en griego. De igual modo hay 5 10 maneras de elegir uno en latn y uno en frances y hay
7 10 maneras de elegir uno en griego y uno en frances. Finalmente, por la regla de la suma,
hay 5 7 + 5 10 + 7 10 = 155 maneras de elegir dos libros de idiomas diferentes.

El problema del segundo ejemplo conviene pensarlo del siguiente modo. Imaginemos que
tenemos 6 lugares, uno al lado de otro, y que en el primero debemos colocar una letra, y en el
segundo una letra y en el tercero, una letra, y en el cuarto una letra, y en el quinto un dgito,
y en el sexto, un dgito. Hay 27 letras y 10 dgitos disponibles. Entonces, el numero de placas
distintas, de acuerdo con la regla del producto, es:

27 27 27 27 10 10 = 53,144,100.
Como dijimos anteriormente, queremos dar definiciones mas rigurosas de los conceptos de or-
denacion y seleccion. Notemos que el primer problema consiste en que dado un conjunto S de
22 libros de los cuales hay 5 libros en latn, 7 en griego y 10 en frances, hay que contar cuantos
subconjuntos de dos libros tiene S que cumplan con la condicion de estar escritos en idiomas
distintos. En general se trata de problemas que hay que contar el numero de subconjuntos
que cumplen con alguna condicion.

En el segundo problema la situacion es distinta. Si designamos por L el conjunto de las letras


y por N el de los dgitos, tenemos que una patente no es nada mas que un elemento del
producto cartesiano L L L L D D.

Esta consideraciones nos llevan a dar las siguientes definiciones:

Definicion 4.1

a) Una r-combinacion de n objetos es un subconjunto de r elementos del conjunto formado


por los n objetos. Al numero total de r-combinaciones lo designaremos por C(n, r).

b) Una r-permutacion de n objetos es un elemento, con componentes diferentes, del pro-


ducto cartesiano S S S tomado r veces, donde S representa al conjunto de los
n objetos. Al numero total de r-permutacion lo designaremos por P (n, r).
118

Volviendo a un lenguaje mas coloquial, tenemos que una r-combinacion de n objetos es


una seleccion de r objetos elegidos de entre los n y que una rpermutacion de n objetos es
una ordenacion en fila de r objetos elegidos entre los n.

Generalmente se subentiende cuales son los numeros n y r. Cuando es as, se abrevian las
expresiones refiriendose a ellos como combinaciones sin repeticion y arreglos u ordenaciones
sin repeticion. Se reserva la palabra permutaciones para el caso en que n = r.

Teorema 4.1
El numero de arreglos sin repeticion de r objetos tomados de un total de n, es decir P (n, r),
es n!/(n r)!.

Demostracion
Recordemos que cada r-arreglo es una r-tupla con componentes distintas. La primera compo-
nente se puede elegir de n maneras; la segunda de n 1 maneras ya que debe ser distinta de
la primera; la tercera de n 2 maneras ya que debe ser distinta de las dos anteriores, etc., la
ultima de n r + 1 maneras. Por la regla del producto tenemos que el total de posibilidades
es n(n 1) . . . (n r + 1) = n!/(n r)!.

Teorema 4.2
El numero de combinaciones sin repeticion de r elementos tomados de un total de n, es decir
C(n, r) es nr .

Demostracion
Podemos formar todos los arreglos sin repeticion de r objeto de un total de n, eligiendo r
objetos del total y despues ordenandolos de todas las formas posibles. El numero de formas
en que se pueden ordenar r objetos es r!. Por lo tanto, tenemos que P (n, r) = C(n, r) r!. Del
n!
teorema anterior sabemos que P (n, r) = . Reemplazando resulta que
(n r)!
 
n! n
C(n, r) = =
r!(n r)! r

Ejemplo 4.3
Cuantas senales diferentes pueden hacerse izando 6 banderas de colores distintos, una sobre
otra, si se puede izar cualquier numero de ellas a la vez?

El numero de senales con una sola bandera es P (6, 1), el de dos banderas es P (6, 2), etc.
Luego, el numero total de senales sera:

P (6, 1) + P (6, 2) + + P (6, 6) = 6 + 30 + 120 + 360 + 720 + 720 = 1956.

Ejemplo 4.4
De cuantas maneras n personas pueden sentarse alrededor de una mesa circular?

Notemos que hay diferencia entre ordenaciones lineales y circulares. En estas ultimas, las
personas no tienen posiciones absolutas sino solo interesa las posiciones relativas entre ellas.
As, por ejemplo, si las personas las designamos por p1 , p2 , p3 , , pn , entonces la ordena-
cion (p1 p2 p3 pn ) no es distinta de (p2 p3 pn p1 ) ni de (p3 p4 pn p1 p2 ). En general la
119

ordenacion (p1 p2 pn ) no es distinta de las n ordenaciones (pk pk+1 pn p1 pk1 ) con


k = 1, 2, 3, , n.

P (n, n)
Luego, el numero de ordenaciones circulares es: = (n 1)!
n
Por la palabra ordenaciones siempre entenderemos que son ordenaciones lineales.

Un ejemplo de naturaleza un tanto distinta, ya que aparecen repeticiones es el siguiente:

Ejemplo 4.5
Cuantos numeros de 4 dgitos se pueden formar con los dgitos 1, 2, 3, 4, 5, 6 si cada dgito se
puede usar las veces que se quiera?

Si designamos por S = {1, 2, 3, 4, 5, 6} entonces cada numero de 4 dgitos es un elemento de


SxSxSxS. Por la regla del producto tenemos que

N (SxSxSxS) = 64 .

Este problema no debe confundirse con los de combinaciones y permutaciones con repeticion
que los veremos mas adelante.

Continuaremos con algunos ejemplos de combinaciones.

Ejemplo 4.6
Si hay 18 equipos de futbol cuantos partidos deben jugarse para completar la primera rueda?

Hay que formar todas las posibles selecciones de 2 equipos. Entonces la solucion es
 
18
= 153
2
El siguiente ejemplo corresponde a combinaciones con restricciones, al igual que el ejemplo
4.2 anterior. Para su solucion conviene separarlos en casos.

Ejemplo 4.7
Cuantas comisiones de 6 alumnos se pueden formar entre 5 alumnos del ultimo ano y 6
alumnos de otros cursos si cada comision debe contener por lo menos 3 alumnos del ultimo
ano?

Las comisiones pueden ser de los siguientes tipos:

5 6
 
1. 3 alumnos del ultimo ano y 3 de otros cursos. El numero de comisiones es: 3 3 .
5 6
 
2. 4 alumnos del ultimo ano y 2 de otros cursos. De este tipo hay: 4 2 .
5 6
 
3. 5 del ultimo ano y 1 de otros cursos: 5 1 .
120

Luego, el numero total de comisiones es


           
5 6 5 6 5 6
+ + = 281
3 3 4 2 5 1

Ejemplo 4.8
De cuantas maneras pueden repartirse las 28 fichas de un domino entre 4 jugadores?

El primer jugador puede elegir sus 7 dominos de C728 maneras. Una vez seleccionados hay
C721 maneras en que el segundo jugador puede escoger los suyos. Posteriormente quedan C714
posibilidades para el tercero y por ultimo, C77 maneras para el cuarto. Por la regla del producto
el total de maneras es:
       
28 21 14 7 28!
=
7 7 7 7 (7!)4

Ejemplo 4.9 Prueba combinatoria de la identidad de Vandermonde.

De un grupo de m mujeres y r varones, de cuantas maneras se puede seleccionar un co-


mite de n personas?

Haremos el calculo de 2 maneras que, como deben dar resultados iguales, demostraran la
identidad citada.

La primera consiste en contar el numero de maneras de elegir n personas de un total de


m+r. Su valor es Cnm+r .
La segunda, cuenta las maneras de elegir k varones y n-k mujeres, con k recorriendo desde 0
n
X
a n. Resulta ser m
Ckr Cnk . Como estos valores deben ser iguales obtenemos:
k=0
n
X
Cnm+r = m
Ckr Cnk
k=0

En el siguiente ejemplo aparece un nuevo concepto: el de permutaciones con repeticion:

Ejemplo 4.10
De cuantas maneras se pueden ordenar 3 rayas y 2 puntos?

Pensemos primero, que las rayas y los puntos son distintos entre s. Estos 5 elementos se pueden
ordenar de 5! maneras. Ahora, si los dos puntos son indistinguibles, entonces la ordenacion en
que el primero esta en el lugar i y el segundo en el lugar j es igual a la que el primero esta en j
y el segundo en i supondiendo que las rayas permanecen en sus mismos lugares. Por eso, debe-
mos dividir el numero de ordenaciones por el numero en que se pueden ordenar dos elementos.

Luego, el numero de ordenaciones con los puntos indistinguibles pero las rayas distintas es

5!
.
2!
121

En estas ultimas, considerando las rayas iguales entre s por un argumento similar al anterior
llegamos a la conclusion que debemos dividir por 3!. Luego, el numero buscado es

5!
.
2! 3!
Definicion 4.2
Si tenemos n1 objetos de una clase, n2 de otra, , nk de una ultima y n1 + n2 + + nk = n,
designaremos por P (n, n1 , n2 , , nk ) el numero de permutaciones de estos n objetos. Estas
permutaciones se conocen como permutaciones con repeticion, ya que no todos los objetos
son distintos.

Teorema 4.3

n!
P (n, n1 , n2 , , nk ) =
n1 !n2 ! nk !
La demostracion es un argumento similar al del ejemplo anterior. La dejamos al lector.

Ejemplo 4.11
Se tienen n clases de objetos y de cada clase hay tambien n objetos. De cuantas maneras se
pueden ordenar estos n2 objetos?

Del teorema 3 obtenemos de inmediato el resultado

(n2 )!
.
(n!)n
(n2 )!
Este ejemplo, de paso prueba que es un numero natural.
(n!)n

El ejemplo que sigue es de indudable valor teorico.

Ejemplo 4.12
Cuantas soluciones enteras no negativas tiene la ecuacion x + y + z = 20?

Tomemos una coleccion de 20 numeros 1 y dos guiones separadores y ordenemos estos 22


objetos en una fila. Convengamos que la cantidad de 1 que hay desde el inicio hasta el primer
guion representa el valor de x, desde este al segundo el valor de y, y los restantes el valor para
z. Es facil ver que toda solucion queda representada por alguna de estas ordenaciones. Por lo
tanto el numero de soluciones sera

22!
= 231.
20! 2!
Si se pidiera que todas las soluciones fueran positivas, su numero sera el de soluciones no
negativas de la ecuacion x + y + z = 17 ya que agregandole una unidad a cada uno de los tres
numeros que representa una solucion, se obtiene una positiva para la ecuacion original.

Este ejemplo se generaliza sin ninguna dificultad para la ecuacion

x1 + x2 + + xm = n
122

obteniendose que el numero de soluciones enteras no negativas es

(n + m 1)!
n!(m 1)!

En el ejemplo con que continuamos, aparece un nuevo concepto:


Combinaciones con repeticion.

Ejemplo 4.13
En una confitera se venden 3 tipos de pasteles. De cuantas maneras se pueden comprar 7?.

Numeramos los tipos de pasteles con los dgitos 1, 2 y 3. Cada seleccion que hagamos la po-
demos caracterizar por una lista creciente de 7 cifras de estos numeros. As, por ejemplo la
lista 1, 1, 2, 2, 2, 3, 3 indica que se compraron 2 del primer tipo, 3 del segundo y 2 del tercero,
mientras que la lista 2, 2, 2, 2, 3, 3, 3 muestra una compra de 4 del segundo tipo y 3 del tercero.
Es claro que habra tantas selecciones posibles como listas de esta naturaleza. Para contar
cuantas listas hay usaremos el siguiente truco: Sumemos 0 al primer numero de la lista, 1 al
segundo, 2 al tercero, etc., 6 al ultimo. As todas las listas se transformaran en listas de 7
numeros de entre el 1 al 9 estrictamente creciente; es decir, todos los numeros seran diferen-
tes y ordenados de menor a mayor. Por lo tanto, habra tantas listas como selecciones de 7
numeros se pueden hacer con los del 1 al 9; a saber C79 .

El siguiente teorema generaliza este argumento.

Teorema 4.4
El numero de combinaciones con repeticion de r objetos que se pueden formar con n clases si
cada uno de ellos se puede repetir las veces que se quiera, lo designaremos por C(n, r) y vale

Crn+r1 .

Demostracion
Es absolutamente similar al ejemplo anterior y lo seguiremos paso a paso.

Identifiquemos los tipos de objetos con los numeros 1, 2, 3, , n y especifiquemos cada sele-
ccion por una lista de los correspondientes enteros arreglados en orden creciente. Por ejemplo,
la lista 1, 1, 1, 3, 4, 5, 5, indica que el primer objeto se tomo 3 veces; el segundo no se tomo; el
tercero una vez; el cuarto tambien una vez; el quinto dos veces; etc. En cada una de estas listas
sumemos el numero 0 al primer entero, 1 al segundo entero, r 1 al r-esimo entero. As la
lista 1, 1, 1, 3, 4, 5, 5, se convierte en 1, 2, 3, 6, 8, 10, 11, en la cual todos los numeros son
distintos. Es facil ver que cada una lista de este ultimo tipo se puede reconstruir la seleccion
que la origino. Finalmente el numero de listas del ultimo tipo es el de la selecciones de r
numeros que se pueden hacer con los numeros 1, 2, 3, , n + r 1 es decir

Crn+r1 .

Ejemplo 4.14
Cuantas selecciones de 6 monedas se pueden hacer con monedas de 10, 50, 100, y 500 pesos?
123

Se trata de seleccionar 6 objetos de entre 4 con repeticiones. Luego, la solucion es

C64+61 = C69 = 84.

El cuadro que sigue resume los resultados hasta aqu obtenidos.

Permutaciones Combinaciones

n! n!
Sin repeticion P (n, r) = C(n, r) =
(n r)! r!(n r)!

n! (n + r 1)!
Con repeticion P (n, n1 , n2 , , nk ) = C(n, r) =
n1 !n2 ! nk ! r!(n 1)!

Ademas de estos resultados, esta el del ejemplo 4.5 que se generaliza del modo siguiente:

Teorema 4.5
El numero de ordenaciones lineales con repeticion de r objetos, en que cada uno de ellos puede
elegirse entre n tipos distintos, es nr .

La demostracion es igual al razonamiento usado en ese ejemplo.

Para terminar esta seccion, generalizaremos el teorema del binomio. En el teorema 4.3
aparecieron numeros de la forma n!/n1 ! n2 ! nl ! con los nk no negativos y tales que
n1 + n2 + + nl = n. Se llaman coeficientes multinomiales y se designan por
 
n
.
n1 n2 nl
Notese que el caso en que n se descompone en dos sumandos corresponde a los coeficientes
binomiales.

Teorema 4.6 (Generalizacion del teorema del binomio).

Sean x1 , x2 , , xl reales y n natural. Entonces


X n

n
(x1 + x2 + + xl ) = xn1 1 xn2 2 xnl l
n1 n2 nl
donde la suma recorre todos los coeficientes multinomiales que corresponden a n.

Demostracion
Escribamos el primer miembro como un producto de n factores todos iguales a (x1 + x2 +
+ xl ). Al efectuar todos los productos se formara una sumatoria en que cada sumando es
de la forma xn1 1 xn2 2 xnl l con los ni 0 y tales que n1 + n2 + + nl = n. Cuantos de estos
124

sumandos son de la forma xp1 xq2 xtl ?

Como estos factores se genera tomando una x de cada una de las expresiones (x1 + x2 +
+ xn ), entonces hay que elegir p de ellas para formar xp1 , q de ellas para xq2 y as sucesiva-
mente.

n np nps
  
Esto se puede hacer de p q t .

Pero este producto vale:

n! (n p)! (n p s)!

p!(n p)! q!(n p 1)! t! 0!
que al cancelarse queda

n!
.
p! q! t!

Ejemplo 4.15

Desarrollar (x1 + x2 + x3 )4 .

Los coeficientes son todos los valores de p 4q r con p + q + r = 4 y no negativos. Esta




ecuacion es del tipo de las estudiadas en el ejemplo 4.12, lo que nos permite calcular cuantas
soluciones posee. Son 15. Ellas son:

4, 0, 0 0, 4, 0 0, 0, 4

3, 1, 0 3, 0, 1 1, 3, 0 1, 0, 3 0, 3, 1 0, 1, 3

2, 2, 0 2, 0, 2 0, 2, 2

2, 1, 1 1, 2, 1 1, 1, 2

Entonces el desarrollo es
4 4 4 4 4 4
 4  4
x43 + x31 x2 + x31 x3 + x1 x32
   
400 x1 + 040 x2 + 004 310 301 130

4 4 4 4 4
x1 x33 + x32 x3 + x2 x33 + x21 x22 + x21 x23
    
+ 103 031 013 220 202

4 4 4 4
x22 x23 + x21 x2 x3 + x1 x22 x3 + x1 x2 x23
   
+ 022 211 121 112

= x41 + x42 + x43 + 4x31 x2 + 4x31 x3 + 4x1 x32 + 4x1 x33 + 4x2 x34

+6x21 x22 + 6x21 x23 + 6x22 x23 + 12x21 x2 x3 + 12x1 x22 x3 + 12x1 x2 x23 .
125

4.1.3. EJERCICIOS
1. Hallar cuantos numeros diferentes de dos cifras se pueden formar con los dgitos 1, 2, 4, 7, 8
si

a) no se permite la repeticion;
b) se permite la repeticion.

2. Hallar, en cada caso del problema 1, los enteros pares e impares que pueden formarse.

3. Se forman senales colocando banderas de diferentes colores una sobre otra en un asta.
Si se tienen 5 banderas diferentes, hallar el numero de senales que pueden formarse.

a) con 3 de las banderas;


b) con 4 de las banderas;
c) con todas las banderas.

4. En el ejercicio 3, hallar el numero total de senales que pueden formarse usando una o
mas de las cinco banderas.

5. Al tirar una moneda al aire, puede caer en cara o sello. Encuentre el numero de maneras
diferentes en que pueden caer las siguientes cantidades de monedas:

a) 2 monedas;
b) 3 monedas;
c) n monedas.

6. Obtener el numero de palabras (no necesariamente pronunciables) de cuatro letras que


pueden formarse con 7 consonantes diferentes y 3 vocales diferentes si las consonantes
y vocales deben ir alternadas y no se permiten repeticiones.

7. De cuantas maneras diferentes pueden sentarse 5 personas en una fila de 8 sillas?

8. En cuantas formas pueden ordenarse en un estante 6 libros diferentes si dos libros


determinados deben estar contiguos?

9. Hallar el numero de comites de 4 miembros que se pueden formar con un conjunto de


15 personas.

10. En el ejercicio 9, hallar el numero de comites que

a) incluyen a una persona determinada


b) no incluye una persona determinada

11. Se tienen 10 puntos en un plano, de manera que no hay tres en la misma recta.

a) Cuantas rectas se pueden trazar por cada dos puntos?


b) Cuantas rectas de las de a) pasan por un punto fijo A de los dados?
c) Cuantos triangulos determinan los 10 puntos?
126

d ) Cuantos triangulos de c) tienen un vertice en A?

12. Cuantos numeros de 4 cifras, inferiores a 5000 y que sean multiplos de 5 pueden for-
marse con las cifras 0, 2, 3, 4, 5?

13. De cuantas maneras pueden disponerse en torno a una mesa redonda 8 matrimonios
con la condicion que cada esposo tenga siempre a su lado a su mujer?

14. Una senora invita a almorzar cada vez a 7 personas que pertenecen a dos familias, una
de 8 personas y la otra de 6. Para no agraviar a ninguna familia cree conveniente invitar
cada vez por lo menos a 2 personas de cada familia. Cual es el numero total de grupos
que pueden formarse?

15. De cuantas maneras 2n personas pueden ser divididas en n parejas?

16. De un gran numero de fichas blancas, rojas, azules y verdes, de cuantas maneras pueden
elegirse 6 de ellas?

17. Se tiene n fichas identicas que deben distribuirse en r cajas numeradas de 1 a r. De


cuantas maneras puede hacerse?

18. Un da puede ser clasificado de bueno, malo o indiferente. Cuantos anos deben pasar
para asegurarse que hay una repeticion de la descripcion de los das de una semana?

19. Cuantos divisores tiene el numero

m = 26 34 52 7 113 ?

20. Ocho personas van a jugar 4 partidos de tenis. De cuantas maneras pueden elegirse los
rivales?

21. Cuantas circunferencias se pueden trazar que pasen por lo menos por tres puntos de 9
puntos dados?, si

a) no hay tres colineales ni 4 cocclicos;


b) no hay tres colineales pero hay 4 cocclicos.

22. De 2m fichas blancas y 2n fichas rojas se colocan en una recta con m + n a cada lado
de una marca central.Cuantos de estos arreglos son simetricos?

23. De cuantas maneras pueden ordenarse n ninos en una lnea si 3 de ellos deben estar
separados?

24. a) De cuantas maneras pueden ponerse en fila r ninos y s ninas?


b) De cuantas maneras pueden hacerlo, si los ninos deben estar juntos y las ninas
deben estar juntas?
c) De cuantas maneras si las ninas deben estar juntas?

25. Cuantas diagonales tiene un polgono regular de n lados?

26. Calcular la suma de todos los numeros de cinco cifras distintas que se pueden formar
con las cifras 1, 2, 3, 4 y 5.
127

27. Dados 3n numeros distintos se pide indicar de cuantas maneras puede descomponerse
su producto en n factores cada uno de los cuales esta formado por el producto de tres
de los numeros dados.

28. Para transmitir senales se dispone de 6 luces verdes y 6 rojas colocadas en los vertices
de un hexagono. En cada vertice no puede haber encendida mas de una luz y el numero
mnimo de luces encendidas debe ser tres. Calcular el numero de senales distintas que
se puede hacer.

29. Calcular de cuantos modos distintos pueden ser pintadas con siete colores las caras de
tetraedro regular correspondiendo a cada cara un color no necesariamente distinto de
las restantes.

30. Un estudiante debe responder solamente 7 de las 10 preguntas de una prueba. De


cuantas formas puede hacer su eleccion si:

a) Debe contestar las preguntas 1, 3 y 5?


b) Debe responder exactamente 3 de las primeras 6 preguntas?

31. Siete libros deben ubicarse en cuatro casilleros distintos, sin ninguna restriccion. De
cuantas maneras pueden distribuirse importando el orden de los libros en cada casillero?

32. Cuantas soluciones enteras en que cada variable sea mayor a 4 tiene la ecuacion x +
y + z = 22?

33. Hay 2n objetos de una clase, 2n de una segunda clase y 2n de una tercera. De cuantas
formas pueden repartirse estos 6n objetos en dos grupos iguales de modo que cada grupo
tenga por lo menos dos objetos de cada clase? (2 < n).

34. Cual es el coeficiente de x31 x23 x24 en el desarrollo de (2x1 3x2 3x3 + x4 )7 ?

4.2. LA FORMULA DE INCLUSIONES Y EXCLUSIONES.


Sabemos que es valida la expresion N (A B) = N (A) + N (B) si A y B son conjuntos
disjuntos. Una generalizacion de esta propiedad es la que se conoce como la formula de in-
clusiones y exclusiones, que resulta util para una cierta gama de problemas combinatorios
difciles de resolver por otros metodos. Antes de dar la formula necesitamos establecer dos
teoremas y dar un ejemplo.

Teorema 4.7
Sean A y B conjuntos. Entonces

1. N (A B) = N (A) N (A B)

2. N (A B) = N (A) + N (B) N (A B)

Demostracion
128

1. A = (A B) (A B)
Como A B y A B son disjuntos se tiene que

N (A) = N (A B) + N (A B)

2. A B = (A B) (B A) (A B).
Como estos tres conjuntos son disjuntos, dos a dos, podemos escribir que

N (A B) = N (A B) + N (B A) + N (A B)

y aprovechando 1) obtenemos el resultado.

Ejemplo 4.16
De un grupo de 70 personas se sabe que 38 hablan aleman, 29 hablan frances y 21 hablan
estos dos idiomas. Cuantas no hablan ninguno de ellos?

Sea A el conjunto de los que hablan aleman y F el de los que hablan frances. Entonces
A F representa a las personas que hablan por lo menos uno de estos idiomas y A F a
los que hablan los dos. Del teorema anterior tenemos que el numero de los que hablan por lo
menos uno de estos idiomas es

N (A F ) = N (A) + N (F ) N (A F )

Entonces los que no hablan ninguno de estos idiomas son

70 N (A F ) = 70 N (A) N (F ) + N (A F ) = 70 38 29 + 21 = 24

Definicion 4.3
Sean p, q, r, etc. propiedades que pueden tener los elementos de un conjunto S. Designare-
mos por N (p) el numero de elementos de S que tienen la propiedad p. Por N (p0 ) designaremos
el numero de los que no tienen la propiedad p. Por N (p q) la cantidad de los que tienen
las propiedades p y q. Por N (p0 q 0 ) la cantidad de los que no tienen las propiedades p y
q simultaneamente. Por N (p q) al numero de los que tienen, por lo menos, una de las dos
propiedades, y as sucesivamente.

Teorema 4.8
Sea N el numero de elementos de un conjunto. Entonces, con la notacion de la definicion
anterior tenemos

1. N (p q) = N (p) + N (q) N (p q)

2. N (p0 q 0 ) = N N (p) N (q) + N (p q)

3. N (p0 q 0 r0 ) = N N (p) N (q) N (r) + N (p q)


+ N (p r) + N (q r) N (p q r).
129

Demostracion
La demostracion de 1) es inmediata del caso 2) del teorema 4.7. La de 2) es identica al
argumento del ejemplo anterior.
Para 3) tenemos que, por De Morgan, N (p0 q 0 r0 ) = N ((p q)0 r0 ). Pero, por 2) podemos
escribir

N ((p q)0 r0 ) = N N (p q) N (r) + N ((p q) r)


Aplicando 1) y la distribuidad del con el

= N N (p) N (q) + N (p q) N (r) + N ((p r) (q r))


nuevamente por 1)

= N N (p) N (q) N (r) + N (p q) + N (p r) + N (q r) N ((p r) (q r))

Pero (p r) (q r) = p q r, lo cual establece 3).

Teorema 4.9 (Formula de inclusiones y exclusiones).

Sean p, q, r, s, propiedades que pueden tener los elementos de un conjunto. Sea N el


numero de sus elementos. Entonces, con la notacion de la definicion tenemos que

N (p0 q 0 r0 s0 ) = N N (p) N (q) N (r) N (s)

+N (p q) + N (p r) + N (p q r) N (p q s)

+N (p q r s) +

Notemos que aparecen restandose los numeros que corresponden a un numero impar de pro-
piedades.

Demostracion
Para simplificar la notacion, sea m = r s t. A continuacion observemos que

N ((p q) r t) = N ((p q) m)

= N ((p m) (q m))
que por 1) del teorema anterior

= N (p m) + N (q m) N ((p m) (q m))

= N (p m) + N (q m) N (p q m)

Ahora procedemos por induccion sobre el numero de propiedades. El teorema 8 probo que
la formula vale para 2 o 3 propiedades. Supongamosla valida para n propiedades y sean
p, q, r, , s, r + 1 propiedades que pueden tener los elementos del conjunto. Entonces

N (p0 q 0 r0 s0 ) = N ((p q)0 r0 s0 )


130

que, por la hipotesis inductiva (considerando a (p q) como una sola propiedad) tenemos que

= N N (p q) N (r) N (s) + N ((p q) r)

+ + (1)n N ((p q) r s).


Pero, por el teorema anterior y por la observacion inicial:

N (p q) = n(p) + N (q) N (p q)

N ((p q) r) = N (p r) + N (q r) N (p q r)

..
.
N ((p q) r s) = N (p r s) + N (q r s)

N (p q r s)
y reemplazando estas expresiones en la igualdad anterior, obtenemos la formula para n + 1
propiedades.

Nota
Se acostumbra en la escritura de la formula de inclusiones y exclusiones, suprimir los
smbolos entre las letras que representan propiedades. Entonces, con esta simplificacion, el
teorema queda

N (p0 q 0 r0 s0 ) = N N (p) N (q) N (s)+

N (pq) + + N (rs) + (1)n1 N (pq rs)


A continuacion veremos algunos ejemplos que muestran aplicaciones de esta formula.

Ejemplo 4.17
Cuantos numeros primos hay entre 1 y 100?

Notemos primero, que si un numero entre 1 y 100 no es primo debe tener alguno de los factores
primos 2, 3, 5, 7. (Por que?)

Sea pi la propiedad ser divisible por i y S = {2, 3, 4, 5, , 100} Entonces, como el numero
100
de multiplos de i en S es la parte entera de tenemos que
i

N (p2 ) = 50 N (p3 ) = 33 N (p5 ) = 20

N (p7 ) = 14 N (p2 p3 ) = N (p6 ) = 16 N (p2 p5 ) = N (p10 ) = 10

N (p2 p7 ) = N (p14 ) = 7 N (p3 p5 ) = N (p15 ) = 6 N (p3 p7 ) = N (p21 ) = 4

N (p5 p7 ) = N (p35 ) = 2 N (p2 p3 p5 ) = N (p30 ) = 3 N (p2 p3 p7 ) = N (p42 ) = 2

N (p2 p5 p7 ) = N (p70 ) = 1 N (p3 p5 p7 ) = N (p105 ) = 0 N (p2 p3 p5 p7 ) = N (p210 ) = 0


131

Luego,

N (p2 p3 p5 p7 ) = 99 50 33 20 14 +16 + 10 + 7 + 6
+4 + 2 3 2 1 = 21

Pero en esta cuenta no estan los numeros 2, 3, 5, 7. Luego, son 25 los primos menores que 100.

Ejemplo 4.18
Cuantos numeros menores que 21560 y primos relativos a el hay?

La descomposicion en primos de 21560 es 23 5 72 11. Por lo tanto, se trata de contar cuantos


numeros menores que 21560 y que no tengan ninguno de los factores 2, 5, 7 u 11.

Con la notacion del ejemplo anterior tenemos que

21560
N (p2 ) = = 10780 N (p5 p11 ) = 392
2

N (p5 ) = 4312 N (p7 p11 ) = 280

N (p7 ) = 3080 N (p2 p5 p7 ) = 308

N (p11 ) = 1960 N (p2 p5 p11 ) = 196

N (p2 p5 ) = 2156 N (p2 p7 p11 ) = 140

N (p2 p7 ) = 1540 N (p5 p7 p11 ) = 56

N (p2 p4 ) = 980 N (p2 p5 p7 p11 ) = 28

N (p5 p7 ) = 616

El numero pedido es

N (p02 p05 p07 p011 ) = 21560 (10780 + 4312 + 3080 + 1960)

+(21156 + 1540 + 980 + 616 + 392 + 280)

(308 + 196 + 140 + 56) + 28 = 6720.

La generalizacion de este ejemplo, cuya demostracion no la daremos, lleva a la funcion de


Euler que se enuncia as:
Si p1 , p2 , , pl son los distintos factores primos de un natural m entonces el numero de
enteros positivos menores que m y primos relativos con el, es decir (m), viene dado por
    
1 1 1
(m) = m 1 1 1 .
p1 p2 pl
132

4.3. DESPLAZAMIENTOS
El Problema de los Desplazamientos se plantea con la pregunta: Cuantas permutacio-
nes sin repeticion hay de los numeros 1, 2, 3, , n de modo que el numero 1 no esta en el
primer lugar; el 2 no esta en el segundo lugar . . . el n no esta en el n-esimo lugar?

Designemos por D(n) este numero. El siguiente teorema da su valor.

Teorema 4.10

(1)n
 
1 1
D(n) = n! 1 + +
1! 2! n!

Demostracion
Sabemos que el numero total de permutaciones sin repeticion de los numeros 1, 2, , n es n!.

Sea pi la propiedad la permutacion tiene el numero i en el i-esimo lugar. Con la notacion


de costumbre tenemos que

D(n) = N (p01 , p02 , p03 , , p0n ).

Entonces, por la formula de inclusiones y exclusiones obtenemos que:

n
X n1
X
D(n) = n! N (pi ) + N (pi , pj )
i=1 i=1
( )
j>i

+ (1)n N (p1 , p2 , , pn ).

Es inmediato que

N (pi ) = (n 1)!

N (pi pj ) = (n 2)!

N (p1 p2 pn ) = 0!

n
X
Ademas, el numero de sumandos que hay en una sumatoria del tipo es C1n ; las del tipo
i=1
n1
X
es C2n , ya que equivale a elegir 2 numeros de un total de n. En forma similar, tenemos
(i=1
j>i)
n2
X
que tiene C3n sumandos, etc.
i=1
(k>j>i)
De donde resulta que:
133

D(n) = n! C1n (n 1)! + C2n (n 2)! + (1)n Cnn 0!

(1)n
 
1 1
= n! 1 + + .
1! 2! n!
Nota
1 1 (1)n
En cursos de calculo se prueba que la expresion 1+ + vale aproximadamente
1! 2! n!
e1 y esta aproximacion es mejor mientras mayor sea n. Usando esta observacion podemos
n!
escribir que D(n) es aproximadamente . Recomendamos al lector calcular para algunos
e
numeros n, el valor exacto y el aproximado de D(n).

Ejemplo 4.19
Se deben mandar 6 tarjetas de Pascua a 6 personas distintas. Ademas, ya estan escritos los
sobres con sus direcciones. De cuantas maneras se pueden confundir los sobres de modo que
ninguna persona reciba su propia tarjeta?

La respuesta es, simplemente, D(6) que calculado exactamente da 265. Calculado aproxima-
damente resulta 264,87.

Ejemplo 4.20
Se reparten n libros diferentes entre n ninos. Despues de ledos se retiran y se vuelven a re-
partir entre ellos de modo que a ningun nino le toque dos veces el mismo libro. De cuantas
maneras distintas se puede hacer?

La primera vez, los libros se pueden repartir de n! maneras. La segunda vez se pueden repar-
tir de D(n) maneras. Entonces, por la regla del producto, el proceso total puede hacerse de
n!D(n) maneras.

Profundizando en el problema de los desplazamientos llegamos a la siguiente situacion:


En las n-tuplas con elementos distintos que estamos considerando, diremos que hay una
coincidencia en el lugar l si en ese lugar se encuentra el numero l. Designaremos por D(n, k)
el numero de n-tuplas con exactamente k coincidencias.

Teorema 4.11
D(n, k) = Ckn D(n k).

Demostracion
Las tuplas con k coincidencias se pueden construir seleccionando k numeros de entre los n y
ubicandolos de modo que se produzcan las coincidencias y distribuyendo los n k restantes
de modo que todos estan fuera de sus puestos.

Por la regla del producto se obtiene el resultado enunciado.

Ejemplo 4.21
Una prueba consiste en cinco preguntas y sus cinco respuestas. Se pide emparejar las pre-
guntas con sus correspondientes respuestas. De cuantas formas se puede hacer de modo que
134

halla exactamente cero, una, dos, tres, cuatro y cinco parejas correctas?

Es evidente que el problema corresponde a una sucesion de cinco elementos en que se pide
que tenga cero, una, dos, tres, cuatro y cinco coincidencias.

Cero coincidencias se tiene en D(5) = 44 sucesiones. Del teorema anterior tenemos que con
solo una pareja correcta hay
 
1 1 1 1
D(5, 1) = C15 D(4) = 5 4! 1 + + = 45
1! 2! 3! 4!

De igual forma se calculan las restantes. Notemos que D(5, 4) = 0.

4.3.1. EJERCICIOS
1. Por convenio internacional se han elegido como bisiestos, los anos cuyo numero sea
divisible por 4 pero no por 100, a menos que lo sea por 400. Por ejemplo, 1900 no fue
bisiesto pero 2000 s.
Cuantos anos bisiestos hay entre 1884 y 4004 ambos incluidos?

2. Cuantos enteros entre 1 y 1.000.000 son cuadrados perfectos, cubos perfectos?

3. Una encuesta revela que de 100 mujeres adultas 59 fuman, 62 son casadas y 29 son
fumadoras y casadas. Cuantas solteras no fumadoras hay?

4. Encontrar el numero de permutaciones de los numeros del 1 al 12 en que los numeros


del 1 al 6 ocupen los 6 primeros lugares.

5. Escriba una tabla para los numeros D(n, k) con n 8.

6. Cuantos numeros entre 1 y 10.000 hay que no son divisibles por 5, 6 u 8?

7. En una fiesta 7 caballeros entregan sus sombreros. De cuantas maneras pueden serles
devueltos si:

a) Ninguno recibe el suyo?


b) Por lo menos uno recibe el propio?
c) Por lo menos dos reciben los suyos?

8. Se tiene 2n objetos a1 , a1 , a2 , a2 , , an , an , con ai 6= aj cuando i 6= j. En cuantas


permutaciones lineales de estos 2n objetos ninguna pareja estara junta?

9. Calcule el problema anterior para los casos n = 2 y n = 3.

10. Probar que m + n objetos distintos se pueden permutar de modo que exactamente n
queden en su lugar de (m + n)!D(m)/(m!n!), maneras.
135

4.4. PARTICIONES
Para terminar este captulo estudiaremos algunos aspectos de la problematica de las par-
ticiones. Estos son problemas en que los elementos que intervienen deben particionarse en 2
o mas conjuntos que cumplan ciertas condiciones. Hay una gran variedad de ellos dependiendo
si se considera o no el orden de los elementos en cada conjunto, o si hay elementos repetidos.
Nosotros estudiaremos solo algunos casos.

Para facilitar la exposicion conviene, en vez de dividir los elementos en conjunto, pensar que
deben ser guardados en cajas. Cuando sea necesario distinguirlas, las supondremos numeradas.

Teorema 4.12
El numero de maneras en que se pueden dividir n objetos distintos en l cajas, de modo que
la primera contenga n1 elementos, la segunda n2 , ,la ultima nl elementos, donde n1 + n2 +
+ nl = n es

n!
n1 ! n2 ! nl !

Demostracion
Debemos seleccionar n1 objetos de los n. Esto se puede hacer de Cn1 n maneras; de los res-
nn
tantes, debemos elegir n2 . Esto se realiza de Cn2 1 maneras; y as sucesivamente. Por la regla
del producto el numero buscado es
nn1 n2 nl1
Cnn1 Cnnn
2
1 C
nl =

n! (n n1 )! (n n1 n2 )!
=
n1 !(n n1 )! n2 (n n1 n2 )! n3 !(n n1 n2 n3 )!

nl ! n!
=
nl ! 0! n1 !n2 ! nl !

Ejemplo 4.22
De cuantas maneras se pueden ubicar 12 personas en 4 oficinas si en la primera oficina deben
haber 2 personas, en la segunda 3, en la tercera tambien 3 y en la cuarta las restantes?

Se puede aplicar directamente el teorema anterior. El resultado es

12!
= 277,200
2! 3! 3! 4!
Notemos que en estas cuentas se consideran como casos distintos en el que las personas A, B
y C estan en la oficina dos y las personas X, Y, Z en las tres con el caso en que las personas
X, Y y Z estan en la dos A, B y C en la tres.

Si no se especifica cuantos objetos deben haber en cada caja, tenemos el siguiente resultado:

Teorema 4.13
El numero de maneras en que se pueden dividir n objetos distintos en l cajas diferentes es nl .
136

Demostracion
Cada objeto tiene l maneras distintas de ubicarse. Por la regla del producto obtenemos el
resultado.

Otra situacion que aparece con frecuencia la da el siguiente teorema.

Teorema 4.14
El numero de maneras en que se pueden repartir n objetos iguales en l conjuntos ordenados
es

n+l1
Cl1 .

Demostracion
Agreguemosle a los n objetos l 1 barra y ordenemos estas n + l 1 cosas en una fila.
Convengamos en que los objetos que hay desde el principio de la fila hasta la primera barra
corresponden a la primera caja; desde la primera barra a la segunda corresponden a la segunda
caja; y as sucesivamente. Entonces, por cada ordenacion de los n + l 1 objetos y barras
corresponde a una division de los n objetos del tipo requerido. Como estas ordenaciones son
permutaciones con repeticion tenemos que el numero de ellas es

(n + l 1)! n+l1
= Cl1 .
n!(l 1)!

Ejemplo 4.23
De cuantas maneras se pueden repartir 8 manzanas, 5 peras y 2 duraznos entre 4 ninos?

Repartamos primero las manzanas. Segun el teorema anterior puede hacerse de C311 maneras.
De igual modo las 5 peras de C38 maneras y los dos duraznos de C35 maneras. Por la regla del
producto, tenemos que el numero de maneras de repartir las 15 frutas es

C311 C38 C35 = 92,400.


Hasta ahora no hemos tenido en cuenta el orden de los elementos dentro de cada conjunto.
El siguiente teorema es un aporte en esa direccion.

Teorema 4.15
El numero de maneras de distribuir n objetos distintos en l cajas diferentes, teniendo en
n+l1
cuenta el orden en el interior de cada una de ellas es n!Cl1 .

Demostracion
Al igual que en el teorema anterior agreguemosle a los n elementos l 1 barras y ordenemos
estos n+l1 objetos en fila. Convengamos en que los objetos que estan desde el principio de la
fila hasta la primera barra corresponden en orden y cantidad a los que habra en la primera caja;
los que hay entre la primera barra y la segunda corresponden en orden y cantidad a la segunda
caja; etc. Entonces para cada ordenacion de los n objetos y l1 barra habra una distribucion de
las contadas. Luego, aprovechando la formula para permutaciones con repeticion, obtenemos
que el numero de estas distribuciones es
137

(n + l 1)! n+l1
= n!Cl1 .
(l 1)!

Ejemplo 4.24
Cuantas senales se pueden hacer con 5 banderas distintas y 3 mastiles si pueden ocuparse
tantas banderas como se quiera?
Se consideraran senales diferentes intercambiar entre s las banderas en cada mastil.

Por el teorema anterior, el numero de senales en que intervengan las 5 banderas es


5+31
5! C31 = 2520

Con 4 banderas el numero de senales es:


4+31
C45 4!C31 = 1,800

ya que las 4 banderas se pueden elegir de C45 maneras.

El numero de senales en que intervienen 3, 2 o 1 bandera es respectivamente.

C35 3! C25 = 600

C25 2! C24 = 120

C15 1! C23 = 15
Luego, el numero total de senales es 5055.

Algunos casos mas de particiones seran propuestos como ejercicios.

4.4.1. EJERCICIOS
1. Con las letras de la palabra POLIINSATURADO, cuantas ordenaciones pueden reali-
zarse de modo que se conserve el orden en que estan las vocales?

2. Un tren tiene 9 coches. De cuantas maneras pueden acomodarse 4 personas si deben


viajar en coches diferentes?

3. Se dispone de n1 objetos de un tipo, n2 del otro, . . . , nl de un ultimo tipo.

a) De cuantas maneras se pueden dividir entre dos personas?

b) De cuantas maneras se pueden dividir entre dos personas de modo que cada una
reciba por lo menos si objetos del tipo i? (2si n).

4. a) De cuantas maneras pueden repartirse 40 manzanas, 20 naranjas, 12 limones y 24


peras entre 3 ninos?
b) De cuantas maneras, si cada uno de ellos debe recibir por lo menos la cuarta parte
de cada tipo de frutas?
138

5. De cuantas maneras pueden repartirse n objetos distintos en l conjuntos ordenados si


ninguno debe quedar vaco?

6. Se propone el siguiente problema: Por el envo de una encomienda se debe pagar N


pesos. Se dispone de estampillas de 4, 6 y 10 pesos. De cuantas formas pueden ordenarse
en una fila estampillas de modo que sumen N pesos, si dos formas que se diferencian
solo en el orden se consideran distintas?
Pruebe que :
F (N ) = F (N 4) + F (N 6) + F (N 10)
donde F (N ) representa la cantidad de ordenaciones de estampillas cuya suma es N .

7. De cuantas formas se puede cambiar una moneda de 50 pesos en monedas de 1, 5 y 10


pesos?

8. De cuantas formas puede descomponerse el numero 15 en sumandos distintos si ninguno


de ellos debe superar a 10?. Ecuentre un metodo sistematico para el conteo.

9. Igual al problema anterior sin la condicion que sean distintos.

10. Se tiran n dados iguales. Cuantos resultados distintos pueden obtenerse?

11. De cuantas formas pueden dividirse 30 personas en 3 grupos de 10?

12. Se dan 2n elementos distintos. De cuantas maneras se pueden particionar en n conjuntos


de dos elementos cada uno?

13. De cuantas maneras pueden distribuirse 4 fichas blancas, 4 negras y 4 rojas en 6


conjuntos si se permiten que algunos queden vacos?
139

4.5. SOLUCIONES O SUGERENCIAS A LOS EJERCICIOS


DEL CAPITULO
Pagina 125

1. a) 5 4 b) 5 5

2. Dividamoslos en dos grupos: Los de la forma impar par y los de la forma par par.
Sin repeticion: par,par 6. impar,par 6.
Con repeticion: par,par 9. impar,par 6.

3. Como importa el orden en que la banderas estan en el mastil son ordenaciones sin
repeticion.

a) 5 4 3
b) 5 4 3 2
c) 5!

4. Es la suma de todos los casos.

5 4 3 2 1 + 5 4 3 2 + 5 4 3 + 5 4 + 5 = 325

5. c) Puede haber 0 caras, 1 cara, 2 caras , n caras. Luego, son n + 1 maneras.

6. Dos grupos: Palabras que empiezan en vocal y las que empiezan en consonante.

3 7 2 6 + 7 3 6 2 = 504.

7. Se eligen 5 sillas de las 8 y en ellas se ordenan a las 5 personas.

C58 5! = 6720

8. Tomemos los dos que deben estar juntos como uno solo. Hay dos formas en que pueden
adozarse. Luego, el numero de ordenaciones es

2 5! = 240

9. C415 = 1365.

10. a) Se debe elegir tres personas de las restantes C314 = 364


b) Se debe elegir cuatro personas de entre 14. C414 = 1001.

11. a) Se debe elegir dos puntos, C210 = 45


b) Hay que elegir un punto de los restantes; son 9
c) C310 = 240
d ) C29 = 36
140

12. Deben ser numeros terminados en 0 o 5. La primera cifra no puede ser 5 o 0. Con estas
restricciones se obtiene el resultado 3 5 5 2 = 150.

13. Los ocho matrimonios pueden sentarse de 7! maneras. Pero cada matrimonio tiene dos
formas de sentarse, a saber el varon a la izquierda o a la derecha de su mujer. Luego, el
numero total es 28 7!

14. Puede invitar 2 de una familia y 5 de la otra o 3 de la primera y 4 de la otra, etc.

C28 C56 + C38 C46 + C48 C36 + C58 C26

15. Sea f (n) el numero de maneras en que 2n personas pueden ser divididas en n parejas.
Saquemos la persona 1. Cada vez que seleccionemos una persona para formar pareja
con 1 las restantes 2n 2 personas pueden emparejarse en f (n 1) maneras. Como la
pareja para 1 se puede elegir de 2n 1 formas tenemos la siguiente relacion.

f (n) = (2n 1)f (n 1)

Por recurrencia obtenemos,

f (n) = (2n 1)(2n 3)(2n 5) 1

Este problema tambien puede pensarse del siguiente modo: Se eligen 2 personas del
total. De las restantes se eligen otras 2, etc. Esto nos lleva al resultado :
1 2n 2n2
C C C22 .
n! 2 2

16. C64+61 = C69 = 84

17. Sea xk el numero de fichas que se pondra en la caja de orden k En estas condiciones el
problema es equivalente a encontrar todas las soluciones no negativas de la ecuacion

x1 + x2 + + xr = n

Usando el mismo argumento dado para las combinaciones con repeticion, se obtiene
como resultado: Cnn+r1 .

18. Cada da de una semana se puede elegir de tres maneras. Por lo tanto hay 37 semanas
distintas. Para que se repita necesariamente alguna debe haber una mas. En total (37 +1)
semanas = 42 anos.

19. Un divisor de m debe estar formado solo por los factores primos de m, de la siguientes
manera. El numero 2 puede tenerlo de 0 a 6 veces, el 3 de 0 a 4 veces, etc.
Luego, el numero de divisores es 7 5 3 2 4 = 840.
En esta cuenta esta considerado el 1 como divisor.

20. Ver problema 15. La respuesta es 7 5 3 1 = 105.

21. a) C39 ; b) C39 3.


141

22. Por cada ordenacion de las fichas de un costado, pueden ordenarse simetricamente las
(m + n)!
del otro. Luego, el resultado es .
m!n!
23. Los restantes (n 3) ninos pueden ordenarse de (n 3)! maneras. Debemos intercalar
los tres ninos. Hay (n 2) lugares donde intercalar uno.
por lo tanto,el resultado es

(n 3)!C3n2 3! = (n 3)(n 4)((n 2)!)

24. a) (r + s)!
b) 2r!s!
c) Al igual que el problema anterior despues de ordenados los ninos debemos elegir
un espacio donde intercalar las ninas. La respuesta es r!(r + 1)s!.

25. Toda recta que une dos vertices es o un lado o una diagonal

C2n n.

26. La cantidad de numero es 5!. Si disponemos los numeros en una columna para sumarlos,
cada una de sus subcolumnas tendra 24 unos, 24 dos, , 24 cincos. Luego, la suma de
cada subcolumna es 360. Luego, el resultado es 3999960.

27. Supondremos que los 3n numeros son distintos. Se trata de formar todas las colecciones
de n tros distintos. (Ver problema 15).

1 3n 3n3 1
C3 C3 C33 = n (3n 1)(3n 2)(3n 4)(3n 5) 4 5 2 1.
n! 2

28. Numero de senales de 3 luces 23 C36


Numero de senales de 4 luces 24 C46 . Etc.

29. Con un color hay solo una manera de pintarlo. Por lo tanto, el numero de posibilidades
es C17 . Con dos colores hay dos maneras. Dos de un color y otras dos del otro, o tres de
un color y una del otro. Se originan 3 posibilidades: Si las caras se pintaron a, bbb, o
aa, bb, o aaa, b, donde a y b representan los colores seleccionados.
En este caso el numero de variantes es 3C27 .
Con tres colores hay una manera de pintarlo: Dos caras llevan el mismo color y las otras
2, colores distintos. Tambien originan 3 posibilidades. El numero de casos en 3C37 . Con
cuatro colores el numero de posibilidades es C47 .

30. a) C47
b) C47 C36 .
c) C36

31. Son todas las permutaciones de 7 objetos distintos mas 3 separadores.


Resulta 10!/3! = 604 800.
142

32. Son 3 variables y cada una debe tener el valor 5 por lo menos. Solucionaremos la ecuacion
x+y+z = 7 en enteros no negativos. Si a, b, c es una solucion de ella entonces a+5, b+5 y
c+5 es solucion de la original en las condiciones pedidas. Luego, el numero de soluciones
del problema inicial es el de soluciones enteras no negativas de x + y + z = 7 que son
9!/2!7!.

33. Separemos 4 objetos de cada clase. Quedan 6n 12. Estos debemos dividirlos en dos
grupos con igual numero de objetos, que equinale a seleccionar 3n 6 del total.

6n12
Respuesta C3n6 .
7!
34. Es 23 (3)2 = 15120.
3!2!2!

Pagina 134

1. 515 anos

2. 1000 cuadrados y 100 cubos perfectos incluidos 1 y 1000000.

3. Sea F el conjunto de las fumadores y C el de las casadas. Entonces

N (F C) = 59 + 62 29 = 92

Luego, 8 son solteras no fumadoras.

4. 6! 6! = 518400

5.
n\k 0 1 2 3 4 5 6 7 8
1 0 1

2 1 0 1

3 2 3 0 1

4 9 8 6 0 1

5 44 45 20 10 0 1

6 265 264 135 40 15 0 1

7 1854 1855 924 315 70 21 0 1

8 14833 14832 7420 2464 630 112 28 0 1

6. Es similar al ejemplo 4.16 del texto.

7. a) D(7, 0) = 1854
143

b) D(7, 1) + D(7, 2) + + D(7, 7)


c) D(7, 2) + D(7, 3) + + D(7, 7)

8. El numero de permutaciones sin ninguna condicion es (2n)!/2n . Sea N (i) el numero de


permutaciones en que la pareja i esta junta; N (i, j) el de permutaciones en que tanto
la pareja i y j estan juntas, etc. Tenemos que, considerando a la pareja (i,i) como una
(2n 1)!
entidad, N (i) = .
2n1
(2n 2)!
En forma similar: N (i, j) = .
2n2
(2n 1)! P (2n 2)!
= C1n ; i,j N (i, j) = C2n
P
Luego, i N (i) n1
, etc.
2 2n2
Finalmente, por la formula de inclusiones y exclusiones, nuestro resultado es:

(2n)! (2n 1)! (2n k)!


n
C1n n1
+ + (1)k Ckn +
2 2 2nk
9. Para n = 2 es 2 y para n = 3 es 30.

10. De los m + n se deben elegir n para dejarlos en su lugar y los restantes desplazarlos de
todas las maneras posibles. En virtud de la regla del producto, obtenemos el resultado.

Pagina 137

1. Como las vocales deben conservar el orden, solo podemos variar sus ubicaciones, por lo
tanto podemos considerarlas todas iguales. Representemoslas a cada una de ellas por x.
El problema se convierte en ordenar en una fila 7 consonantes distintas y 7 x.
Resp. 14!/7!.

2. 4!C49

3. a) Del primer grupo de objetos, una persona puede no tener ninguno, obtener uno, etc.
Luego, ese grupo puede dividirse entre dos personas de (n1 + 1) maneras. Lo mismo
para los demas grupos. Luego, el numero pedido es (n1 + 1)(n2 + 1) (nl + 1).
b) Se reparte a ambas personas si objetos de cada grupo. Los sobrantes se dividen
como forma arbitraria. Luego, la solucion es

(n1 2s1 + 1)(n2 2s2 + 1) (nl 2sl + 1)

4. a) Agreguemosle a las 40 manzanas dos barras separadoras y ordenemos estos 42


objetos en una fila. Dandole al primer nino todas las manzanas desde el principio
de la fila hasta la primera barra, al segundo nino las manzanas que estan entre
las dos barras y al tercero las restantes, es inmediato que habra tantas formas de
dividir las manzanas como ordenaciones de estos 42 objetos se puedan hacer. Lo
mismo con las demas frutas. Por lo tanto, la solucion es C242 C222 C214 C226 .
b) Se reparte primero la cuarta parte de cada grupo de fruta a cada nino. En este
caso la solucion es C212 C27 C25 C28 .
144

5. El numero de maneras de repartir los objetos sin ninguna condicion es ln . En cuantas


de estas quedo la caja i vaca? La respuesta es (l 1)n . La caja i y la j quedan vacas es
(l 2)n casos. Et. Luego, usando la formula de inclusiones y exclusiones, la respuesta es

ln C1l (l 1)n + C2l (l 2)n + (1)l1 Cl1


l
1n

6. Si a cada ordenacion que suma N p le agregamos al final una estapilla de valor p ,


se convierte en una ordenacion de suma N . Ademas, si a cada ordenacion de suma N
le quitamos la ultima estampllla se convierte en una de los tres tipos anteriores. Por lo
tanto:
F (N ) = F (N 4) + F (N 6) + F (N 10)

7. Designemos por F (1, 5, 10; n) el numero de maneras en que se puede reunir la cantidad
n con monedas de 1, 5 y 10 pesos.
Notemos que

F (1, 5, 10; 50) = F (1, 5; 50) + F (1, 5; 40) + F (1, 5; 30) + F (1, 5; 20) + F (1, 5; 10) + F (1, 5; 0)

puesto que representan la formas en que se pueden reunir 50 pesos sin ninguna moneda
de 10; con 1 moneda de 10; con 2; etc. respectivamente.
As el calculo se reduce a otros mas sencillos que se pueden calcular directamente.
Finalmente
F (1, 5, 10; 50) = 11 + 9 + 7 + 5 + 3 + 1 = 36

8. Designemos por F (n1 , n2 , , np ; m) las formas de dividir m en sumando distintos, to-


mando estos del conjunto {n1 , n2 , , np }. El problema nos pide calcular F (1, 2, , 10; 15).
Los casos a contar se pueden dividir en dos clases: Que intervenga el 10 o no interven-
ga. Luego, F (1, 2, , 10; 15) = F (1, 2, , 9; 10) + F (1, 2, , 9; 15) y as reducir el
problema a casos mas sencillos que se pueden calcular directamente.

9. Este problema es del mismo tipo que el problema 7 de esta seccion.

10. Los resultados seran de la forma (n1 , n2 , , n6 ) = n donde ni indica el numero de dados
que marcaron el numero i. Este problema es del tipo del ejemplo 4.12. La solucion es
C511+5 .
30 C 20 C 10 .
11. Al igual que el problema 1 de esta seccion, se tiene que la solucion sera C10 10 10
1 30!
Pero, ahora no importa el orden entre los grupos. Luego, la solucion es .
3! (10!)3
(2n)!
12. En forma similar al problema anterior se tiene que el resultado es .
2n n!
13. Agreguemos a las 4 fichas blancas 5 separadores y ordenemos estos 9 objetos en una
fila. Convengamos en que las fichas que hay desde el principio hasta el primer separador
corresponden al primer conjunto; desde este al segundo separador al segundo conjunto,
etc. El numero de posibles distribuciones sera C59 y por lo tanto el resultado final es
(C59 )3 .
Captulo 5

ELEMENTOS DE
PROBABILIDAD

5.1. GENERALIDADES
En este captulo haremos una introduccion elemental a la teora de la probabilidad. Este
concepto, que mas adelante definiremos con todo rigor, es absolutamente natural al pensa-
miento, refiriendose a la mejor o peor oportunidad que tiene un resultado de ocurrir con
respecto a otro. Por ejemplo, al lanzar un dado, nos parece que todas sus caras tienen la
misma posibilidad de salir y esto lo decimos afirmando que la probabilidad de cada cara es
1/6. Si lanzamos dos dados, nuestra intuicion y la experiencia nos dice que la suma 7 ocurre
con mayor frecuencia que cualquier otra suma, tambien que las sumas 6 y 8 parecieran tener
la misma probabilidad y que es mejor que la de 3. Pero nos damos cuenta que debemos ser
mas precisos, no conformandonos solo con estimaciones cualitativas, lo cual implica tener que
asignar valores numericos a la probabilidad de cada suma. El como hacerlo sera lo que veremos
en este captulo.

Para seguir avanzando, debemos precisar algunos conceptos generales propios del tema y es-
tablecer algunas observaciones.

En el contexto de este captulo entenderemos por experimento cualquier quehacer que cada
vez que se efectua arroja un resultado de entre varios posibles, todos conocidos de antemano.
Son ejemplos de experimentos tirar un dado, sacar una carta de un naipe, lanzar una moneda,
etc. En general nos limitaremos a experimentos que tienen un numero finito de resultados.

Definicion 5.1
Sean r1 , r2 , , rn los diferentes resultados de un experimento. Al conjunto formado por ellos
lo llamaremos el Espacio Muestral y a cada uno de sus subconjuntos, un evento o suceso.
En este texto nos limitaremos a espacios muestrales finitos. Se hace necesario aclarar que,
ocasionalmente, el espacio muestral de un experimento puede no ser unico y queda al criterio
del experimentador determinar que espacio muestral usara. El siguiente ejemplo aclara este
hecho.

Ejemplo 5.1

145
146

Consideremos el experimento de lanzar dos dados. Se pueden tomar como espacios muestrales
los siguientes:

i) U1 = {2, 3, , 12} que son las diferentes sumas que pueden producirse. Tiene 11 ele-
mentos.

ii) U2 = {(i, j) : 1 i 6 1 j 6 i j} que son los resultados obtenidos, anotados


como parejas, sin distinguir entre ambos dados. Posee 21 parejas.

iii) U3 : {(i, j) : 1 i 6 1 j 6} que son los resultados obtenidos anotados como


parejas de dos numeros, en que se distingue entre ambos dados. Consta de 36 elementos.

La diferencia entre U2 y U3 se entiende mejor si se piensa que los dos dados son de colores
diferentes, digamos blanco y rojo. Anotando el numero que indica el dado blanco como la pri-
mera componente de cada pareja, mientras que el senalado en el dado rojo como la segunda
componente se obtiene el espacio muestral U3 . Si independientemente de los colores, se toma
como primera componente el menor de los numeros en el caso que salgan distintos y el otro
como segundo o, si son iguales, ambas componentes toman ese valor, se obtiene el espacio U2 .

Antes de dar nuestra definicion de probabilidad creemos necesario decir algunas palabras so-
bre su significado.

Que queremos decir cuando afirmamos que la probabilidad que salga cara al lanzar una
moneda es 1/2?

Observemos, en primer lugar, que esta afirmacion lleva implcita otra, a saber. La probabili-
dad existe, es decir cada evento tiene asociado un numero que mide su probabilidad y que
para el ejemplo citado es 1/2. Pero, que es lo que mide este numero?

Basicamente hay dos interpretaciones distintas a las cuales recurrir para contestar esta pre-
gunta. La primera de ellas, que podramos llamar frecuentista dira que su significado es: Si
se lanzara muchas veces una moneda, aproximadamente arrojara cara en la mitad de los
lanzamientos y que la aproximacion es mejor cuando mayor es el numero de lanzamientos.
Dicho en forma mas tecnica:
c 1
lm =
n n 2
donde c es el numero de veces que salio cara y n el numero total de intentos.
c
Esta interpretacion adolece de algunos defectos: En primer lugar el numero lm no se
n n
puede determinar porque es imposible lanzar infinitas veces una moneda. En el fondo lo que
hacemos, es creer que ese lmite es 1/2.

En segundo lugar, si para determinar la probabilidad decidimos hacer series finitas de lan-
zamientos, por ejemplo 20 series de 100 lanzamientos cada una y contar cuantas caras se
produjeron en cada serie, obtendremos una sucesion de numeros, todos ellos mas o menos
147

cercanos a 50, pero casi todos distintos, lo que nos dejara en la duda de si la probabilidad es
0, 5 o 0, 49 o 0, 513, etc.

En realidad, la posicion frecuentista no permite asignar probabilidades a los eventos aun pre-
suponiendo que ella existe en el mundo real; es decir, que aun aceptando que la probabilidad
es una propiedad intrnseca del experimento realizado, no podramos determinar sus valores
para los diferentes eventos por metodos estadsticos.

La segunda interpretacion, que podra caracterizarse como la posicion subjetiva, dice que la
probabilidad mide el grado de creencia que cada persona tiene, en un momento dado, sobre el
resultado de un experimento y este grado de creencia se ha formado por el conocimiento que
ella tiene del experimento. As, por ejemplo, si digo que la probabilidad de obtener cara es
1/2, quiero decir que, por lo que se de la experiencia de lanzar una moneda y de como estan
construidas las monedas, etc., creo, en este momento, que el numero que mide la probabilidad
de salir cara es 1/2.

Esta posicion, aunque tambien tiene defectos, estimamos que es mas aceptable que la frecuen-
tista. De ella hay muchas variantes tratando de mejorar las dificultades que presenta y que
no corresponde describirlas en este texto. De todas maneras, ambas posiciones son usadas,
dependiendo del problema a tratar. As por ejemplo, si un fabricante quiere demostrar las
bondades de las ruletas que fabrica, desarrollara largas experiencias con ellas que prueben
que los numeros de 0 a 36 aparecen todos, casi con la misma frecuencia. Pero si esta misma
persona apuesta en una carrera de caballos, tendra que informarse lo mas posible, evaluar esa
informacion y as decidir a que caballo arriesgar su dinero. Su posicion en este caso sera sub-
jetiva.

La definicion de probabilidad que daremos a continuacion no arroja ninguna luz sobre como
deben asignarse probabilidades a los diferentes resultados de un experimento. Ella solo dice
cuales son los requisitos que debe cumplir cualquier asignacion que se haga para que, desde el
punto de vista de la matematica, sea una asignacion de probabilidad correcta. Pero tambien
recoge las propiedades que nuestra intuicion exige que tenga cualquier nocion que pretenda
ser una probabilidad.

Definicion 5.2
Sea U un espacio muestral. Una probabilidad P para U es una funcion de los subconjuntos
de U en R tal que:

i) Se A y B son dos eventos de disjuntos, entonces P (A B) = P (A) + P (B).


ii) Para todo evento A de U se tiene que P (A) 0.
iii) P (U ) = 1.

Ni esta definicion ni otras que se den de probabilidad, puede decir cuales son sus valores
numericos para los diferentes eventos de un experimento. Estos valores deben ser fijados
por el investigador en forma totalmente libre, con la unica limitacion que se satisfagan las
condiciones impuestas en esta definicion .

Teorema 5.1
Sea U = {r1 , r2 , , rn } un espacio muestral y P una probabilidad definida en el.
148
X
i) Si A U entonces P (A) = P ({ri })
ri A
X
ii) P ({ri }) = 1
ri U

iii) Si A y B son eventos y A B entonces P (A) P (B).

iv) P () = 0.

Demostracion

i) Es inmediata por induccion sobre el numero de elementos de A. ii) y iii) son consecuen-
cias directas de i).

iv) Como A y son disjuntos, entonces P (A) = P (A ) = P (A) + P ().


Luego, necesariamente P () = 0.

A la luz de este teorema vemos que para asignar una probabilidad P a un espacio muestral
U = {r1 , r2 , , rn } basta definir los valores de P para los singleton {r1 }, {r2 }, , {rn } y
definir la probabilidad de un evento como la suma de las probabilidades de sus elementos.

Para fijar estos valores adoptaremos una actitud subjetiva. Procederemos del siguiente modo:
Fijaremos, en los posible, como espacio muestral un conjunto de resultados que a nuestro
entender nos parezcan que todos tienen la misma oportunidad de ocurrir; es decir, espacios
muestrales con resultados equiprobables. A continuacion, si el numero de resultados del espacio
muestral es n, a cada uno de ellos le asignaremos la probabilidad 1/n. De esta manera quedan
satisfechas las condiciones de la definicion 1.
Este tipo de espacios aparecen con frecuencia, cuando los elementos que intervienen presentan
simetras fsicas, por ejemplo dados simetricos, ruletas equilibradas, naipes parejos, etc. Pero
tambien usaremos espacios muestrales no equiprobables cuando las condiciones del problema
lo sugieran.
En los espacios muestrales equiprobables se cumple la siguiente propiedad usandose ademas,
el lenguaje que a continuacion se indica:

Sea A U . Es costumbre llamar a los elementos de U los casos posibles y a los de A, los
casos favorables. Entonces por la parte i) del teorema tenemos que

Numeros casos favorables


P (A) =
Numeros de casos posibles

Ejemplo 5.2
Consideremos el experimento que consiste en lanzar dos dados. Tomemos como espacio mues-
tral el conjunto U3 = {(i, j) : 1 i 6 1 j 6} que se obtiene distinguiendo los
dos dados. Este espacio muestral nos permite describir como eventos todas las preguntas que
queramos hacer sobre el resultado que se consigue al tirar los dados. Por ejemplo, obtener una
suma de 9 queda descrito por el evento A = {(6, 3)(5, 4)(4, 5)(3, 6)}; obtener un numero 2 y
un numero 3 queda descrito por el evento B = {(2, 3)(3, 2)}; obtener dos numeros 6 queda
149

descrito por el evento C = {(6, 6)}, etc. Ademas, por razones de simetra de los dados nos
parece que todas las parejas deben tener la misma probabilidad; como son 36, le asignamos a
1
cada una la probabilidad de . Ahora podemos calcular las probabilidades de los diferentes
36
eventos. Como ejemplo, calculemos las siguientes probabilidades:

1. Obtener una suma de 8.


Queda descrito por el evento {(6, 2), (5, 3), (4, 4), (3, 5), (2, 6)}. Luego,

casos favorables 5
=
casos posibles 36

2. Obtener una suma de por lo menos 8.


Queda descrito por el evento

{(6, 2)(5, 3), (4, 4), (3, 5), (2, 6)} {(6, 3), (5, 4), (4, 5), (3, 6)}

{(6, 4), (5, 5), (4, 6)} {(6, 5), (5, 6)} {(6, 6)}.

15 5
Luego, la probabilidad es = .
36 12

3. Obtener un numero 2 y un numero 3 simultaneamente. El evento es {(2, 3), (3, 2)}.


1
Entonces esta probabilidad es .
18
4. Obtener una suma impar. El evento queda descrito por las parejas cuya suma es impar.
Son 18; por lo tanto la probabilidad es 1/2.

Ejemplo 5.3
Se extraen 13 cartas de un mazo de 52.

Cual es la probabilidad que en la muestra se incluyan los cuatro ases?

Los elementos del espacio muestral son todas las selecciones de 13 cartas que se pueden hacer
de un mazo de 52. Luego, el numero de casos posibles es 52 13 . Los casos favorables son las
selecciones de 13 cartas que tienen los 4 ases; es decir, sacando aparte estas cuatro cartas, hay
que seleccionar nueve cartas de las restantes. Luego, el numero de casos favorables es 48

9 .
Por lo tanto, la probabilidad pedida es:
48

9 48 47 40 13 12 2
52 = 52 15 40 9 8 2 = 0, 00264

13

Ejemplo 5.4
Se reparten todas las cartas de un mazo de 52 entre 4 jugadores. Cual es la probabilidad que
alguno de ellos tenga los 4 ases?

Numeremos los jugadores de 1 a 4. Tomemos como espacio muestral S un conjunto de tuplas


de 4 componentes en que la primera componente es el conjunto de 13 cartas que recibe el
150

jugador 1; la segunda, las 13 cartas que recibe el jugador 2; etc., cuando todas las cartas son
repartidas entre los 4 jugadores.

52 39 26 13
   
El total de estas tuplas es 13 13 13 13 .

Los casos en que los 4 ases estan en una mano quedan representados por los casos en que los
tiene el primer jugador, mas los casos en que los tenga el segundo jugador, etc. Ellos son en
total
48 39 26 13 48 35 26 13
       
9 13 13 13 + 13 9 13 13

48 35 22 13 48 35 22
       9
+ 13 13 9 13 + 13 13 13 9
Estos 4 numeros son iguales como se comprueba facilmente.

Luego, la propiedad pedida es


48 39 26 13
   
9 13 13 13
4 52
 39
 26
 13

13 13 13 13
que al simplificar, resulta 4 veces el valor del ejemplo anterior.

Ejemplo 5.5
El juego del Kino es equivalente al siguiente problema: En una caja hay 15 fichas blancas y
10 negras. Se extraen 15 al azar. Cual es la probabilidad que las 15 sean blancas?

Vamos a resolver el problema en forma mas general. Cual es la probabilidad que exactamente
p de ellas sean blancas?

Sea U = {b1 , b2 , , b15 , n1 , , n10 }. Debemos seleccionar p fichas blancas de las 15 y 15 p


fichas negras de las 10. El numero total de casos es C15 25 ; por lo tanto, la probabilidad pedidad

es

Cp15 C15p
10

25
C15
1
Para el caso particular p = 15 el resultado es .
3 268 760
Dejamos al lector calcular la probabilidad de obtener: 14 aciertos; 5 aciertos.

El siguiente teorema muestra algunos resultados que pueden facilitar el calculo de probabili-
dades.

Teorema 5.2
Sean A y B eventos de un espacio muestral U . Entonces

i) P (A0 ) = 1 P (A). En palabras: La probabilidad que no ocurra A es 1 menos la


probabilidad que ocurra.

ii) P (A B) = P (A) + P (B) P (A B)


151

Demostracion
U = A A0 . Pero A y A0 son disjuntos. Luego, P (U ) = 1 = P (A) + P (A0 ) de donde se obtiene
la primera afirmacion. Para la segunda notemos que

A B = (A B) (B A) (A B)
y estos 3 conjuntos son disjuntos dos a dos. Luego,

P (A B) = P (A B) + P (B A) + P (A B)
Pero

A = (A B) (A B) y

B = (B A) (A B)
y estas descomposiciones son disjuntas. Entonces

P (A) = P (A B) + P (A B) y

P (B) = P (B A) + P (A B).
Reemplazando estos valores en P (A B) se obtiene el resultado. Posteriormente generaliza-
remos esta formula. (Ver: Formula de Inclusiones y Exclusiones).

Ejemplo 5.6
Una caja contiene 3 fichas blancas y 6 negras. Se sacan 2 fichas. Cual es la probabilidad de
que entre ellas se encuetre, por lo menos, una blanca?

C26 5
Es mas facil calcular la probabilidad que no halla alguna blanca. Resulta ser = . Pero
C29 12
5 7
la probabilidad pedida es la del complemento, por lo tanto es 1 = .
12 12

El siguiente ejemplo ilustra como construir algunos espacios muestrales.

Ejemplo 5.7
Un edificio consta de 3 ascensores. Cada uno de ellos tiene probabilidad p de presentar una
falla en los siguientes 12 meses. Designemos por a el hecho que el ascensor no falla en el ano
y por b el que falle. Nuestro espacio muestral sera el siguiente:

{aaa}, {aab}, {aba}, {baa}, {abb}

{bab}, {bba}, {bbb}


Por ejemplo, {a b b } significa que el primer ascensor no falla en el ano mientras que los dos
restantes presentan fallas. Sus probabilidades son (1 p)3 , (1 p)2 p, . . . p3 . Verifiquemos que
la suma de las probabilidades es 1.

(1 p)3 + 3(1 p)2 p + 3(1 p)p2 + p3 = ((1 p) + p)3 = 1


152

5.1.1. EJERCICIOS
1. Cual es la probabilidad de obtener:

a) 9, 10 u 11 puntos al tirar 3 dados?


b) Dos caras y dos sellos al tirar 4 monedas?

2. 10 cartas son sacadas de un naipe ingles. Cual es la probabilidad que entre ellas halla:

a) Por lo menos un as?


b) Por lo menos dos ases?

3. Dos fichas son sacadas de una caja que contiene 5 fichas blancas y 3 negras. Cual es la
probabilidad de que:

a) ambas sean blancas;


b) ambas sean del mismo color;
c) por lo menos una sea blanca?

4. El mismo problema anterior con la diferencia que las extracciones se hacen con reposi-
cion, vale decir, se saca una ficha, se anota su color y se devuelve a la caja.

5. Una caja contiene 10 fichas numeradas de 0 a 9. Se sacan 3 fichas en forma sucesiva con
reposicion (sin reposicion). En el orden en que salen van formando un numero entre 0 y
999. Cual es la probabilidad que ese numero sea divisible por 39?

6. Se ordenan 4 fichas blancas y 4 negras en una fila. Se le pide a una persona, que sin ver
las fichas, adivine los colores en que estan ubicadas en la fila. Cual es la probabilidad
que adivine el color de k fichas?
(Se supone que la persona nombra solo 4 veces blanca y 4 rojas).

(0 k 8).

7. Se reparten n fichas identicas en N cajas numeradas.


Cual es la probabilidad que la primera caja tenga h fichas?

8. Cual es la probabilidad que al sacar 6 cartas de un naipe salgan 3 rojas y 3 negras?

9. Cuatro fichas numeradas de 1 a 4 se distribuyen al azar en 4 lugares numerados de 1 a


4, una en cada lugar. Cual es la probabilidad en que por lo menos una ficha ocupe su
propio lugar?

10. Una caja contiene a fichas blancas y b negras. Se sacan fichas, una tras otra, hasta
que en la caja solo queden fichas de un mismo color. Cual es la probabilidad que sean
blancas?

11. Se tiran n monedas simultaneamente. Cual es la probabilidad:

i) que todas salgan caras?


ii) que todas sean iguales?
153

iii) que salgan exactamente una cara?

12. Una caja contiene 50 hojas de afeitar de las cuales 5 han sido usadas. Se sacan 5 hojas.
Cual es la probabilidad que las 5 sean nuevas?

13. Una caja contiene 52 fichas numeradas de 1 a 52. Se sacan 13. Cual es la probabilidad
que entre estas 13 estan los numeros del 1 al 7?

14. Una caja contiene a fichas rojas, b azules y c verdes. Se saca una muestra de 3. Cual es
la probabilidad que las tres sean del mismo color? Las tres sean de colores distintos?

15. Una caja contiene 3 fichas rojas y 7 negras. Dos personas A y B sacan una ficha con-
secutivamente hasta que alguien extraiga roja. Cual es la probabilidad que la persona
que inicio las extracciones sea la que saque roja por primera vez?

16. Dos dados se tiran n veces. Cual es el menor valor de n para que la probabilidad de
obtener en alguno de los lanzamientos un doble seis sea por lo menos de 1/2?

17. Se tienen 12 fichas de las cuales 6 son rojas y 6 blancas. Se separan al azar en dos grupos
de seis. Cual es la probabilidad que en cada grupo hallan 3 rojas y 3 blancas?

18. Se tiene n cajas cada una con a fichas blancas y b negras. Se traslada una ficha de la
primera a la segunda despues de la segunda a la tercera y as sucesivamente. Calcule la
probabilidad que la k-esima ficha trasladada sea blanca.
Sugerencia: Exprese pk en funcion pk1 donde pk es la probabilidad pedida.

5.2. PROBABILIDAD CONDICIONADA


Introduciremos este concepto empezando con un ejemplo.

Ejemplo 5.8
Se saca una carta de un naipe ingles. Cual es la probabilidad que sea un 7 sabiendo que es
mayor que 3?

El experimento tiene como espacio muestral S las 52 cartas de la baraja, todas con igual
probabilidad.
Si no tuviesemos la informacion que la carta extraida es mayor que 3, la probabilidad de ser
7 sera, simplemente, 1/13; pero el conocimiento adicional que poseemos de la extraccion,
nos sugiere que debemos reevaluar las probabilidades originales para aprovechar esta nueva
informacion. Para decidir como lo haremos, analizaremos con mayor profundidad.
Sea A el evento, La carta es mayor que 3 y B La carta es un 7. Entonces la informacion
adicional nos dice que el resultado del experimento es un elemento de A y el problema se
transforma en Como reevaluar la probabilidad de B sabiendo que A ocurrio? Anotemos este
nuevo valor por P (B|A).

Es inmediato que B ocurrio si y solo si la carta extraida es un elemento de B A.

Esto nos sugiere que conviene considerar a A como el espacio muestral y definir la probabili-
dad condicionada por:
154

Definicion 5.3

P (B A)
P (B|A) = conP (A) 6= 0
P (A)
P (B|A) se lee la probabilidad de B dado A.

40 4
Para concluir con el ejemplo, debemos calcular P (A) y P (B A). Sus valores son 52 y 52
1
respectivamente. Luego, P (B|A) =
10
40
El conjunto A esta formado por las cartas mayores que 3. Son 40. por lo tanto, P (A) = .
52
1
B A esta formado por los 4 siete. Luego, P (B|A) = .
10

Teorema 5.3
P (B|A) es una probabilidad definida en A.

Demostracion
Es inmediato que P (A|A) = 1. Como A B A se tiene que P (A B) P (A) lo que prueba
que 0 P (B|A) 1.

Sean B y C disjuntos. Entonces

P ((B C) A) P ((B A) (C A))


P ((B C)|A) = =
P (A) P (A)

P (B A) + P (C A)
= = P (B|A) + P (C|A)
P (A)
lo cual demuestra que P (B|A) es una probabilidad definida en A.

Ademas, como era de esperar, se cumple que:

P (A0 A) P ()
P (A0 |A) = = = 0.
P (A) P (A)

Como consecuencia de este teorema tenemos que todas las propiedades establecidas, en los
teoremas 5.1 y 5.2 valen para esta probabilidad. En particular tenemos que:

a) B C P (B | A) P (C | A)

b) P (B 0 | A) = 1 P (B | A)

c) P (B C | A) = P (B | A) + P (C | A) P (B C | A)

Ejemplo 5.9
Se tira un dado dos veces. Cual es la probabilidad de obtener una suma mayor que 8, sa-
biendo que en el primer tiro salio un 5? Cual es la probabilidad que en el primer tiro haya
155

salido un 5, sabiendo que la suma es mayor que 8?

Sea U el espacio muestral:

U = {(i, j) : 1 i 6 1 j 6}
Sea A el evento Sacar en el primer tiro un 5 y B Obtener en los dos tiros una suma mayor
que 8.

A = {(5, 1), (5, 2), . . . , (5, 6)}

B = {(3, 6), (6, 3), (4, 6), (6, 4), (5, 6), (6, 5), (6, 6), (4, 5), (5, 4), (5, 5)}

A B = {(5, 6), (5, 5), (5, 4)}


Debemos calcular P (B | A) y tambien, P (A | B).
1 5 1
Es inmediato que P (A) = , P (B) = y P (A B) =
6 18 12
Luego,
1
P (A B) 12 1
P (B | A) = = 1 =
P (A) 6
2
1
P (A B) 3
P (A | B) = = 12
5 =
P (B) 18
10
Notemos que, si no sabemos nada del resultado del primer tiro, la probabilidad de obtener
una suma mayor que 8 es 5/18.

Ejemplo 5.10
Una caja tiene 5 fichas blancas, 7 rojas y 8 negras. Se saca una. Cual es la probabilidad que
sea blanca sabiendo que no es negra?

Podemos tomar como espacio muestral equiprobable el conjunto de las 20 fichas de la caja. El
evento B Sacar ficha blanca es el subconjunto formado por las 5 fichas blancas y N Sacar
ficha negra es el subconjunto de las 8 negras. Entonces, lo que se pide es P (B | N 0 ).

P (B N 0 ) P (B)
P (B | N 0 ) = =
P (N 0 ) 1 P (N )

5/20 5
= =
1 8/20 12

La probabilidad condicionada aplicada a los elementos del espacio muestral S tiene los
siguientes valores. Sea x S y A S con P (A) 6= 0.
Entonces:

P (x)
P (A) si x A

P (x | A) =
si x A0

0
156

Este hecho permite calcular P (B | A) de una manera distinta a la que implica la definicion,
manera que puede ser util cuando es difcil calcular la probabilidad de A B.

El siguiente ejemplo ilustra esta observacion.

Ejemplo 5.11
Sea S el conjunto de todos los numeros naturales entre 1 y 1010 tales que en su escritura se
ocupen exactamente dos cifras 3 y supongamos que todos tienen igual probabilidad de ser
seleccionados. Se elige uno al azar. Cual es la probabilidad que sea par sabiendo que tiene 4
cifras?

Sea A el subconjunto de los numeros de 4 cifras y B el subconjunto de sus pares. Entonces se


pide calcular P (B | A).

El conjunto B A esta formado por los numeros de 4 cifras y que ademas son pares. Son de
las formas 33xy o 3x3y o x33y con y una cifra par o cero y x 6= 3.

Luego, N (B A) = 45 + 45 + 40 = 130

El numero de elementos de A, es decir N (A) es 81 + 81 + 72 = 234. Estos valores se obtienen


recordando que x 6= 3 y que y, en el caso de A B debe ser par mientras que en el caso de A
debe ser distinto a 3.

Por la observacion anterior, se obtiene finalmente


130 65
P (B | A) = = .
234 117

El siguiente resultado suele ser util.

Teorema 5.4
Sean A y B eventos. Entonces

P (B) = P (B | A)P (A) + P (B | A0 )P (A0 ).

Demostracion

B = (B A) (B A0 )
Como son disjuntos se tiene que

P (B) = P (B A) + P (B A0 )
De la definicion de probabilidad condicionada se obtiene que

P (B A) = P (B | A)P (A) y P (B A0 ) = P (B | A0 )P (A0 )


Reemplazando en la igualdad anterior establecemos el resultado.
157

Ejemplo 5.12
En una prueba de seleccion multiple de cinco alternativas, un alumno decide contestar al azar
en el caso de no conocer la respuesta. Sea p la probabilidad de que el alumno conozca la
respuesta. En estas condiciones, si una pregunta fue bien contestada, cual es la probabilidad
de que supiera la respuesta?

Sea A el evento la pregunta fue bien contestada y B el evento conoca la respuesta.


Entonces, lo que debemos calcular es P (B | A).

P (A B) P (A | B)P (B)
P (B | A) = =
P (A) P (A)
Es obvio que P (A | B) = 1. Como P (B) = p, solo tenemos que calcular P(A). Pero:

1
P (A) = P (A | B)P (B) + P (A | B 0 )P (B 0 ) = p + (1 p)
5

1
El numero 5 corresponde a la probabilidad de contestar al azar correctamente. Luego,:

p 5p
P (B | A) = 1 =
p+ 5 (1 p) 4p + 1

Ejemplo 5.13 Se conoce con el nombre La Ruina del jugador.


Fue propuesto por Cristian Huygens en 1656 y resuelto, independientemente, por Abraham
de Moivre en 1710 y Niklaus Bernoulli en 1711.
Dice asi: Dos jugadores A y B lanzan, sucesivamente, una ficha, la cual tiene una cara roja y
la otra negra. Si la cara que queda a la vista es roja, entonces B paga a A una moneda; en
caso contrario, el pago es a la inversa. Supongamos que A comienza el juego con i monedas y
B con N i. El juego termina cuando un jugador consigue las N monedas, es decir, cuando
el otro se arruina.
Cual es la probabilidad que A sea el ganador?

Sea p la probablidad que aparezca la cara roja y q la de la cara negra. Aceptaremos que
p + q = 1.Designaremos porP (i)la probabilidad de ganar cuando se tienen i monedas, que ob-
viamente, es equivale a la de comenzar con i monedas. Es inmediato que P (0) = 0 y P (N ) = 1.
Sea H el evento Salio cara roja. Entonces:

P (i) = P (i|H)P (H) + P (i|H 0 )P (H 0 ) = pP (i + 1) + qP (i 1)

Reemplazando el coeficiente 1, tacito de P (i), por p + q, se deriva facilmente


q
P (i + 1) P (i) = (P (i) P (i 1))
p

que corresponde a una relacion de recurrencia para P (i + 1).


Sustituyendo en esta ecuacion i por 1, 2, ,(i 1), , (N 1), obtenemos:
158

1) P (2) P (1) + [q/p](P (1) P (0)) = [q/p]P (1)

2) P (3) P (2) = [q/p](P (2) P (1)) = [q/p]2 P (1)


..
.
i 1) P (i) P (i 1) = [q/p]i1 P (1)
..
.
N 1) P (N ) P (N 1) = [q/p]N 1 P (1)
Sumando estas ecuaciones:
P (N ) P (1) = P (1)(q/p + (q/p)2 + + (q/p)N 1 )
de donde, despejando P (N ) y sumando la P.G. que aparece:
1 [q/p]N

P (1) si p 6= 1/2


1 q/p

P (N ) = 1 =


P (1)/N si p = 1/2

Esta ultima ecuacion nos permite despejar P (1).



1 q/p
si p 6= 1/2


1 [q/p]N

P (1) =


1/N si p = 1/2

Sumando las ecuaciones anteriores desde la 1) hasta la i-1)obtenemos una expresion explci-
ta para P (i).

1 [q/p]i

si p 6= 1/2


1 [q/p]N

P (i) =


i/N si p = 1/2

Aplicaremos estos resultados a un caso concreto. Jugar al color rojo en una ruleta ameri-
cana (tiene 0 y 00). Bajo estas condiciones p = 0, 4737 aproximadamente. Supongamos que
comenzamos con 10 monedas y decidimos jugar hasta doblar el capital o perderlo totalmente.
Esto nos obliga a suponer que el casino tiene tambien 10 monedas y por lo tanto N = 20. Con
estos valores, obtenemos que P (10) = 0, 2587. Este resultado es muy inferior a la probabilidad
de duplicar el capital jugando las 10 monedas en un solo intento, ya que con esta modalidad
su probabilidad es 0,4737.

Ejemplo 5.14
Un examen de sangre detecta, en el 95 % de los casos, una cierta enfermedad si es que la
persona realmente la posee. Pero el test da resultados positivos en el 1 % de las personas
sanas. Se sabe que el 0, 5 % de la poblacion tiene la enfermedad. Cual es la probabilidad que
una persona tenga la enfermedad dado que su test es positivo?

Es obvio que el espacio muestral sera toda la poblacion. Sea E el evento La persona tiene
la enfermedad y T El test resulta positivo. Entonces, se pide calcular P (E | T ). Nuestros
datos nos dicen que P (T | E) = 0, 95; P (T | E 0 ) = 0, 01; P (E) = 0, 005. Luego,
159

P (E T ) P (T | E) P (E)
P (E | T ) = =
P (T ) P (T | E)P (E) + P (T | E 0 )P (E 0 )

0,95 0,005
= 0,323
0,95 0,005 + 0,01 0,995

Este resultado, sorprendentemente bajo, se puede explicar con el siguiente argumento. Suponga-
mos que la poblacion fuese de 1000 personas. Habra 5 personas enfermas y 995 sanas. El
examen habra detectado, de las 5 enfermas, solo a 4,75 mientras que habra diagnosticado
como enfermas a 9,95 personas sanas.
Por lo tanto, la probabilidad de estar realmente enfermo con test positivo es

4, 75
0,323.
4, 75 + 9, 95

El teorema 5.4 se generaliza del modo siguiente:

Teorema 5.5 Formula de Bayes

Sean H1 , H2 , . . . , Hn una particion del espacio muestral S y sea B un evento. Entonces

P (B | Hi )P (Hi )
P (Hi | B) = n
X
P (B | Hj ) P (Hj )
j=1

Demostracion
Es inmediato que
n
[
B= (B Hj )
j=1

de donde resulta que


n
X n
X
P (B) = P (B Hj ) = P (B | Hj )P (Hj )
j=1 j=1

y finalmente

P (Hi B) P (B | Hi ) P (Hi )
P (Hi | B) = = n
P (B) X
P (B | Hj ) P (Hj )
j=1

Los eventos Hi se conocen como las hipotesis del evento B y la formula anterior se enuncia
como la probabilidad de la hipotesis Hi dado que ocurrio B.
160

Ejemplo 5.15
En una caja hay 3 tarjetas de formas identicas, pero una tiene ambas caras rojas, la segunda,
una roja y otra negra, mientras que la ultima ambas caras negras. Se extrae una al azar y se
coloca sobre una mesa, viendose que muestra una cara roja. Cual es la probabilidad que la
otra cara sea negra?

Designemos por RR, RN y N N las tres tarjetas. Aparentemente podra pensarse que la proba-
bilidad es 1/2, porque es, o la tarjeta RR o la RN la que ha extrado. Pero es necesario ser mas
cuidadoso. Sea R el evento, La cara a la vista es roja. Lo que se pide es calcular P (RN | R).
Pero

P (R | RN ) P (RN )
P (RN | R) =
P (R | RN )P (RN ) + P (R | RR)P (RR) + P (R | N N )P (N N )
1 1
2 3 1
= 1 1 1 1 =
2 3 +1 3 +0 3
3

Analizando la fraccion se puede comprender el resultado que a primera vista es tan paradojal.
El producto 31 21 representa la probabilidad de sacar la tarjeta RN por la probabilidad de que
la cara roja quede a la vista. En forma analoga se explican los productos que aparecen en el
denominador.

Volviendo a la Formula de Bayes, los eventos H1 , H2 , , Hn que particionan el espacio


muestral suelen llamarse, como ya dijimos, las hipotesis del evento B. Las probabilidades
P (H1 ), P (H2 ), , P (Hn ) son las probabilidades a priori de dichas hipotesis, mientras que
P (H1 | B), P (H2 | B), son las probabilidades a posteriori. La comparacion de estas
probabilidades para una hipotesis fija indicara como la evidencia que aporta la ocurrencia
del evento B, cambia la probabilidad de dicha hipotesis.
Ejemplo 5.16
Una caja contiene 10 fichas. Ellas fueron puestas segun la siguiente pauta. Se tiro 10 veces una
moneda. Cada vez que salio cara se coloco una ficha blanca, en caso contrario se coloco una
negra. Habiendo terminado este proceso, se confecciona una muestra de 10 fichas del modo
siguiente. Se extrae una ficha, se anota su color y se devuelve a la caja. Esto se repite por 10
veces completandose as la muestra. Cual es la probabilidad que en la caja halla solo blancas
sabiendo que en la muestra todas fueron de ese color?

Sea Hi (i = 0, 1, . . . , 10) la hipotesis En la caja hay i fichas blancas y B el evento La mues-


tra de 10 fichas con reposicion, tiene solo fichas blancas.

Entonces lo que se pide es P (H10 | B).


 10
1
Es facil ver que P (Hi ) = Ci10 . En particular P (H10 ) = 0, 00098, aproximadamente.
2
Tambien es inmediato que
 n
i
P (B | Hi ) =
10
161

ya que se extrajeron 10 fichas blancas de una caja con i fichas de ese color. Luego,:

P (B | H10 P (H10 1
P (H10 | B) = 10
= 10  10 0, 0702
X X i
P (B | Hi )P (Hi ) Ci10
10
i=0 i=0

5.3. EVENTOS INDEPENDIENTES


Definicion 5.4
Sean A y B dos eventos. Diremos que B es independiente de A si y solo si P (B | A) = P (B).

Intuitivamente, esta definicion significa que el saber que el evento A ocurrio no afecta en nada
la probabilidad de B.

Teorema 5.6
Si B es independiente de A, entonces

i) P (A B) = P (A) P (B)

ii) A es independiente de B.
(Se dice que A y B son independientes)

iii) B y A0 son independientes

iv) A0 y B 0 son independientes.

Demostracion

P (A B)
i) P (B) = P (B | A) =
P (A)
Luego, P (A B) = P (A) P (B)

P (B A) P (A) P (B)
ii) P (A | B) = = = P (A)
P (B) P (B)

iii) B = (B A) (B A0 )
P (B) = P (B A) + P (B A0 )
Luego,

P (B A0 ) = P (B)(1 P (A))

= P (B)P (A0 )
162

Como

P (B A0 )
P (B | A0 ) =
P (A0 )

= P (B)

iv) Por iii) B es independiente de A0 . Por ii) A0 es independiente de B y por iii) A0 y B 0


son independientes.

Nota
Es facil ver que si P (A B) = P (A) P (B) entonces A y B son eventos independientes.

Ejemplo 5.17
Se extrae una carta de un naipe ingles. Sea A el evento La carta extrada es un as. Sea B el
evento La carta extrada es de corazon. Probaremos que estos eventos son independientes.
1
Es inmediato que P(A) = 13 ; P(B)= 14 y P (A B)= 52
1
.

Entonces
P (A B) 1
P (A|B) = = = P (A)
P (B) 13

Ejemplo 5.18
Se tira un dado 2 veces. Probemos que el evento Obtener una suma de 7 es independiente
con el evento Obtener en el primer tiro, un numero determinado; por ejemplo 5.

Sea A el evento Obtener suma de 7 y B Obtener en el primer tiro el numero 5. Al igual


1
del calculo del ejemplo 5.3 tenemos que P (A) = . Es inmediato que P (B) = 1/6. Como
6
1
A B = {(5, 2)}, se tiene que P (A B) = . Luego, P (A B) = P (A) P (B).
36

5.4. EXPERIMENTOS INDEPENDIENTES


El concepto de experimentos independientes debe ser considerado como primitivo y se
refiere a la intuicion que se tiene cuando se realizan dos o mas experimentos, uno tras otro,
que son de tal naturaleza que nos llevan a suponer que el resultado que cada uno arroja no
influye en los resultados que daran los siguientes. Por ejemplo, estamos dispuestos a aceptar
como independientes las experiencias Tirar un dado y Sacar una carta de una baraja
porque pensamos que en nada puede influir el numero que marque el dado, en la carta que
posteriormente extraeremos.

Cuando es un mismo experimento que se repite varias veces, considerando cada ejecucion
como un experimento independiente de los anteriores se habla de Intentos repetidos.
163

Ejemplos de estos son: Tirar un dado n veces; sacar una carta de un naipe, restituirla a este
y volver a repetir la operacion hasta completar n extracciones, tira una moneda n veces, etc.
Todos estos ejemplos han sido considerados tradicionalmente como intentos repetidos, es decir,
intentos en que los resultados obtenidos no influyen en el siguiente resultado. Pero son solo
consideraciones de tipo filosofico y psicologico los que nos inducen a pensar de esta manera. Por
ejemplo, en el analisis matematico del juego Punto y Banca cada jugada es considerada por el
tecnico como un experimento independiente, pero la psicologa de la mayora de los jugadores
los induce a creer que cuando se ha producido Banca unas cuatro o cinco veces seguidas, la
probabilidad que la siguiente jugada sea Banca aumenta. Esto lo expresan diciendo Hay que
seguir la Banca. En resumen, para el tecnico el juego Punto y Banca es de experimentos
independientes pero para muchos jugadores no es as.

Definicion 5.5
Sean E1 y E2 dos experimentos independientes con espacios muestrales U1 = {x1 , x2 , , xm }, U2 =
{y1 , y2 , , yn } y probabilidades P1 y P2 respectivamente.

Sea E el experimento, realizar E1 y E2 . Tomaremos como espacio muestral para E el


conjunto U = U1 U2 y en el definimos una probabilidad P de la forma siguiente:

i) P (xi , yj ) = P1 (xi ) P2 (yj )


P
ii) Sea A U . P (A) = P (xi , yj ) donde se suma sobre todas las parejas de A.

Teorema 5.7
Con la notacion de la definicion anterior tenemos que

i) P es una funcion de probabilidad definida en U .

ii) Sea R el evento {xi } E2 y S el evento E1 {yj }. R y S son eventos independientes.

Demostracion

i) Como

0 P1 (xi ) 1 y

0 P2 (yj ) 1 se tiene que

0 P1 (xi ) P2 (yj ) 1. Luego,

0 P (xi , yj ) 1.

Ademas,
164

X m X
X n
P (U ) = P (x, y) = P (xi , yj )
(x,y)U i=1 j=1

m X
X n
= P1 (xi )P2 (yj )
i=1 j=1

m
! n
X X
= P1 (xi ) P2 (yj )
i=1 j=1

=1

Para concluir la demostracion de i), notemos que inmediato que que si A y B son
disjuntos entonces P (A B) = P (A) + P (B). Con esto queda probado que P es una
funcion de probabilidad.

ii) Es facil ver que


P (R) = P1 (xi )P2 (E2 ) = P1 (xi )

De igual forma se tiene que

P (S) = P2 (yi )

Por otra parte R S = {(xi , yj )}


Por lo cual
P (R S) = P1 (xi )P2 (yj ) = P (R) P (S)

lo que prueba que son independientes.

La definicion y el teorema anterior se extienden sin ninguna dificultad para 3 o mas expe-
rimentos independientes teniendo en cuenta que la definicion de eventos independientes
debe generalizarse de la manera siguiente:

Definicion 5.6
Los eventos A1 , A2 , , An son mutualmente independientes si y solo si:

i) Son mutuamente independientes toda seleccion de n 1 de ellos.

ii) P (A1 A2 An ) = P (A1 )P (A2 ) P (An )

Ejemplo 5.19
Se lanzan n veces dos dados. Cual es la probabilidad que en algun tiro se obtenga un doble 6?

El espacio muestral U son las n-tuplas en que cada componente es una pareja ordenada de
numeros entre 1 y 6. Es facil ver que N (U ) = 36n . Ademas, como nos parece que lanzar dos
dados cada vez son intentos independientes le asignaremos a cada tupla la probabilidad que
indica la definicion 5.4. En este caso resulta 1/36n para cada una. Ademas, los eventos Ai
165

Obtener doble seis en el i-esimo lanzamiento son independientes. Luego, A0i tambien lo son.
Sea A el evento Obtener un doble seis en algun lanzamiento.

Entonces

A = A1 A2 An . Luego,

A0 = A01 A02 A0n , y como resulta que

p(A) = 1 p(A0 )

= 1 p(A01 A02 A0n )

= 1 p(A01 ) p(A02 ) p(A0n ). Pero

P (Ai ) = 1/36. Luego,

P (A) = 1 (35/36)n

Ejemplo 5.20
Una caja tiene 5 bolas blancas y 8 negras. Se realiza la siguiente experiencia. Se tira un dado
y despues se sacan de la caja tantas bolas como el numero indicado por el dado.Cual es la
probabilidad que todas las bolas extradas sean blancas?

Podemos suponer que el experimento esta formado por dos independientes: Tirar el dado y
sacar tantas bolas como el numero indicado. Entonces, el espacio muestral estara formado por
parejas en que el primer elemento es un numero del 1 a 6 y el segundo es una seleccion de
tantas bolas de la caja como indica la primera componente. La probabilidad de cada pareja
sera 1/6 por la probabilidad de la seleccion que tiene como segunda componente. El evento
pedido es

A = {(1, B), (2, BB), , (5, BBBBB)}


cuya probabilidad es
5
 5
 5
!
1 1 2 5
13
 + 13
 + + 13

6 1 2 5

Para terminar el captulo desarrollaremos, a modo de repaso, algunos ejemplos mas gene-
rales.

Ejemplo 5.21
Una caja contiene a fichas blancas y b negras. Se sacan n de ellas. Cual es la probabilidad
que entre esas n hallan exactamente m fichas blancas?

Hay Cna+b maneras distintas e igualmente probables de sacar n fichas del total. Hay Cma
b
maneras de sacar m blancas y Cnm a Cb
de sacar negras las restantes. Luego, hay Cm nm
166

maneras de que entre las n hallan exactamente m fichas blancas. Entonces la probabilidad
buscada es
a Cb
Cm nm
p=
Cna+b
Ejemplo 5.22
Una primera caja contiene a fichas blancas y b negras. Una segunda caja contiene c blancas
y d negras. Se saca una ficha de la primera caja y se deposita en la segunda. Despues se saca
una ficha de esta segunda caja. Cual es la probabilidad que esta segunda ficha sea blanca?

Tenemos como espacio muestral, con elementos de igual probabilidad, las parejas de fichas
en que la primera representa la ficha trasladada de la caja uno a la dos y la segunda la ficha
sacada. Es facil ver que este espacio muestral tiene(a + b)(c + d + 1) elementos. Ahora debemos
contar el numero de parejas en que la segunda componente es blanca. Las dividiremos en dos
grupos, las que ambas componentes son blancas y las negras blancas.

En el primero hay a(c + 1) y en el segundo hay bc. Luego, la probabilidad buscada es

a(c + 1) + bc
(a + b)(c + d + 1)

Ejemplo 5.23
Se tienen k cajas, cada una con n fichas numeradas de 1 a n. Se saca una ficha de cada caja.
Cual es la probabilidad que el mayor numero sacado sea m?

Sea Pi la probabilidad que el mayor numero sacado sea i y P la probabilidad que el mayor
numero sacado sea menor o igual a m.

Por ser los eventos El mayor numero sacado es i disjuntos para distintos i se tiene que

P = P1 + P2 + + Pm .
Por otra parte, podemos suponer que los eventos sacar una ficha de las diferentes cajas son
independientes.

Bajo este supuesto podemos calcular el valor de P .

Se tiene que
 m k
P =
n
Luego,
 m k
P1 + P2 + + Pm = .
n
Usando el mismo argumento resulta
 k
m1
P1 + P2 + + Pm1 =
n
167

Entonces restando obtenemos que


 k
 m k m1
Pm =
n n

Ejemplo 5.24
Se van destapando simultaneamente las cartas de 2 naipes ingleses. Cual es la probabilidad
que se encuentre, por lo menos, una coincidencia en numero y pinta entre las dos cartas?

Las cartas de un naipe se pueden ordenar de 52! maneras distintas. El numero de maneras en
que se puede ordenar el segundo naipe para que no se encuentren coincidencias es D52 que
vale aproximadamente 52!/e.

Luego, la probabilidad de que se presenten coincidencias es

1
1 = 0, 632.
e

Nota
Se ha intentado asignarles una medida de probabilidad a algunos conceptos del lenguaje
corriente. Existe una cierta aceptacion en los siguientes valores.

1 Posible. P > 0

2 Probable. P > 0,5

3 Imposible. P = 0

4 Inviable. P < 1050

5.4.1. EJERCICIOS
1. Se lanzan dos dados. Cual es la probabilidad que alguno marque 6 sabiendo que dieron
numeros distintos?

2. Un matrimonio tiene dos hijos. Cual es la probabilidad que ambos sean varones dado
que:

a) Por lo menos uno es varon


b) El mayor es varon

3. Una caja contiene 12 fichas de las cuales 8 son blancas. Se sacan 4 en forma sucesiva.
Cual es la probabilidad que la primera sacada haya sido blanca si entre las 4 hay 2
blancas?
168

4. Se tienen 3 cajas. La primera con 2 fichas blancas y 4 rojas. La segunda con 8 blancas
y 4 rojas, y la tercera con 1 blanca y 3 rojas. Se saca una ficha de cada caja. Cual es
la probabilidad de que la ficha sacada de la segunda caja sea blanca sabiendo que en la
muestra hay 2 blancas?

5. Se sacan 3 cartas de una pila de 8 cartas formada por los 4 ases y los 4 reyes de un naipe.
Cual es la probabilidad que en la muestra hallan, por lo menos, dos ases sabiendo:

i) que hay un as?


ii) que esta el as de corazon?

6. Con las mismas cajas del problema 4 se hace la siguiente operacion: Se traslada una
ficha de la primera caja a la segunda. Despues se traslada una de la segunda a la tercera
y por ultimo se saca una ficha de la tercer. Cual es la probabilidad que esta ultima
ficha sea blanca?

7. Pruebe que si E1 , E2 , , En son eventos independientes entonces

P (E1 E2 En ) = 1 (1 P (E1 )(1 P (E2 )) (1 P (En ))

8. Se tienen 10 monedas de tal naturaleza que al tirar la i-esima la probabilidad de cara


es i/10. Se lanza una moneda mostrando cara. Cual es la probabilidad que esa moneda
fuera la quinta?

9. Se tienen dos cajas identicas exteriormente. La caja A contiene dos monedas de plata
mientras que la B una de plata y otra de oro. Se elige una caja al azar y se extrae una
moneda resultando que es de plata. Cual es la probabilidad de que la otra moneda de
la caja sea de plata?

10. Cual es la probabilidad que un evento A que tiene probabilidad p de ocurrir, ocurra
por lo menos, 2 veces en una serie de n independientes intentos?

11. Sean los eventos A1 , A2 , , An independientes con probabilidades pi respectivamente.


Sea p0 la probabilidad de que ninguno ocurra. Pruebe que:

p0 = (1 p1 )(1 p2 ) (1 pn ).

12. Si en el problema anterior p1 = p2 = = pn = p, calcule la probabilidad de que


exactamente k eventos ocurran.

13. Pruebe que las dos afirmaciones siguientes son falsas:

P (A | C) + P (A | C 0 ) = 1 P (A | C) + P (A0 | C 0 ) = 1

14. Sean E y F eventos excluyentes de un experimento (E F = ).


Pruebe que si intentos independientes del experimento se realizan, entonces la probabili-
dad que ocurra E antes que F es P (E)/(P (E) + P (F )).
169

15. El contenido de 3 cajas iguales es el siguiente:

Caja 1 : 3 bolas blancas y 2 negras

Caja 2 : 2 bolas blancas y 2 negras

Caja 3: 4 bolas blancas y 2 negras

Se elige una caja y se extrae una bola resultando blanca. Cual es la probabilidad que
la caja elegida fuera la i(i = 1, 2, 3).

16. A un concursante de un programa de television se le pida que elija una puerta de tres
que tiene a la vista, de las cuales solo una tiene premio. El opta por la puerta numero
1. El animador, que sabe cual es la puerta premiada, abre la segunda, comprobandose
que no tiene premio. Cual es la probabilidad que la tercera puerta sea la premiada?
170

5.5. SOLUCIONES O SUGERENCIAS A LOS EJERCICIOS


DEL CAPITULO

Pagina 152

1. a) Por calculo directo encontramos que los casos favorables son 79.
Resp: 79/63
b) N (U ) = 16.
4!
El numero de casos favorables es
2!2!
Resp: 3/8.
52 .
2. N (U ) = C10
El numero de elementos de U que tienen exactamente un as es C14 C948 ; dos ases C24 C848 ;
tres ases C34 C748 y cuatro ases C44 C648 .
3. N (U ) = C28 .

C25 C25 + C23 C23


a) b) c) 1 .
C28 C28 C28

4. N (U ) = 82

52 + 3 2 52 + 5 32 2
a) 52 /82 b) c)
82 82

5. a) Con reposicion. Contando el 0, hay 26 numeros menores que 1000 divisibles por
39.
b) Sin reposicion. En este caso los numeros deben tener cifras distintas. Por inspeccion
directa los multiplos de 39 que tienen cifras distintas son 21.
8!
6. El numero de ordenaciones posibles es = 70. El numero de casos favorables para
4! 4!
cada k es el siguiente:

k 0 1 2 3 4 5 6 7 8
C.F. 1 0 16 0 36 0 16 0 1

7. Numero de casos posibles:

(n + N 1)!
n!(N 1)!
Para calcular los casos favorables, retiramos h fichas. Hay que distribuir las restantes
en N 1 cajas. El numero de maneras de hacerlo es

(n h + N 2)!
(n h)!(N 2)!
171

8. C326 C326 /C652


D(4)
9. 1
4!
10. Los casos posibles son todas las ordenaciones de las a + b fichas en una fila. Son (a +
b)!/a! b!.
Los casos favorables son las ordenaciones que terminan en ficha blanca. En total son
(a + b 1)!/(a 1)! b!
11. El espacio muestral S, es el conjunto de todas las n-tuplas en que cada componente es
cara o sello. En total son 2n .

i) 1/2n ii) 2/2n iii)n/2n


12. C545 /C550
13. C645 /C13
52

14. a) Las tres del mismo color:


 
C3a + C3b + C3c /C3a+b+c
b) Las tres de colores distintos:

abc/C3a+b+c
15. R o NNR o NNNNR etc. En total son 4 sumandos. Llevados a probabilidades:
3/10 + 7/10 6/9 3/8 +
 n
35 1
16. Es el menor valor que satisface la desigualdad 1 . Este valor es 25.
36 2
17. C36 C36 /C612
a+1 a a + pk1
18. pk = pk1 + (1 pk1 ) =
a+b+1 a+b+1 a+b+1
a
a a + a+b a a
Pero p1 = , de donde p2 = = ; ; pk = .
a+b a+b+1 a+b a+b
Pagina 167
1. 1/3
2. a) Casos posibles: (v,v), (v,m), (m,v). Probabilidad de (v,v) es 1/3 ; b) 1/2.
3.
4!C28 C24
P (A) =
12 11 10 9

2 3!C28 C24
P (B A) =
12 11 10 9
2 3! 1
P (B|A) = =
4! 2
172

4.
10
Resp: P (B|A) =
11
5. Resp: i) 7/13; ii) 15/52.
6. Resp: 7/16
7. Pruebe el resultado para n = 2 y despues use induccion.
8. Aplique Bayes. Resp: 0,1707...
9. El espacio muestral U esta formado por parejas en que sus componentes son la caja y
la moneda que se extrae. Aplique Bayes.
Resp: 2/3.
10. La probabilidad que no ocurra nunca es (1p)n ; la de que ocurra una vez es pn(1p)n1 .
Luego, el resultado es:

1 (1 p)n1 (1 p + pn).
11. p0 = P (A01 A02 A0n ). Pero los eventos A01 , A02 , , A0n son independientes.
12. Ckn ph (1 p)nk
13. Se debe mostrar un caso donde esas igualdades no ocurran. Por ejemplo: Se tiran dos
dados. Sea A el evento: La suma de los dados es 5.
Sea C el evento: Los numeros que mostraron los dados son menores que 5. Entonces es
facil ver que P (A|C) + P (A|C 0 ) = 1/4 y que P (A|C) + P (A0 |C 0 ) = 5/4.
14. Sea Hn el evento: En los primeros (n 1) intentos no ocurrio ni E ni F ; pero en el
n-esimo E.

P (Hn ) = (1 P (E F ))n1 P (E)


m
X 1 (1 P (E F ))n
Luego, P (Hn ) = P (E)
P (E F )
n=1
La probabilidad buscada es el lmite cuando m . Ella es

P (E) P (E)
=
P (E F ) P (E) + P (F )
15. Aplique Bayes.
Resp: 18/53.
16. Representemos los eventos basicos por parejas (x, y), donde x es el numero de la puerta
premiada e y el de la que se abre. Entonces, nuetro espacio muestral es el conjun-
to {(1, 2), (1, 3), (2, 3), (3, 2)} con probabilidades 1/6, 1/6, 1/3 y 1/3 respectivamente.
Cuando el animador abre la puerta numero 2 el problema se transforma en: Cual es la
probabilidad de (3, 2) sabiendo que ocurrio (1, 2) o (3, 2)?.
Resp: 2/3
Captulo 6

LOS NUMEROS REALES


1a. PARTE

Existen dos maneras de construir el sistema de los numeros reales. La primera, que no
sera la que adoptaremos en este texto, consiste en partir con un sistema axiomatico que de
cuenta de la existencia de los numeros naturales, despues ampliar este sistema numerico a los
enteros, una nueva ampliacion de los enteros para obtener los racionales y por ultimo, con
una ampliacion de estos obtener los reales.

Este proceso, que es muy claro desde un punto de vista conceptual al reflejar mejor la forma
en que los humanos han captado el concepto de numero tiene, a nuestro modo de ver, dos
dificultades que no lo hacen apto en todos los niveles. Es largo de desarrollar y el paso de Q
a R no es elemental.

Preferiremos en esta ocasion, la siguiente presentacion que salva las dos dificultades anteriores
aunque sacrifica comprension intuitiva. Ella consiste en describir los numeros reales como un
todo, basandose en un conjunto de axiomas que los caracteriza completamente, apareciendo
los numeros naturales, enteros y racionales como ciertos subconjuntos de este total. Esta
presentacion se acostumbra resumirla en la siguiente frase: El sistema de los numeros reales
es un campo, ordenado y completo. Esta expresion indica los tres aspectos que se tendran
que estudiar y el orden en que dicho estudio se hara. Empezaremos cn los axiomas de campo
que daran cuenta de toda la operatoria algebraica de los reales, seguiremos con los axiomas
de orden y algunas de sus consecuencias, terminando el captulo con las construcciones de los
numeros naturales, enteros y racionales. Dejaremos para un captulo posterior el problema de
la completitud y sus aplicaciones.

6.1. AXIOMAS DE CAMPO


Existe un conjunto que designaremos por R y a cuyos elementos llamaremos numeros
reales, en el cual estan definidas dos operaciones binarias llamadas suma y producto y anotadas
por + y que tienen las siguientes propiedades:
(Frecuentemente el smbolo de producto se omite . Por ejemplo a b y a (b + c) se abrevian
por ab y a(b + c)).

i) Tanto la suma como el producto son operaciones asociativas y conmutativas.

173
174

ii) El producto se distribuye con respecto a la suma. Esta propiedad en smbolos se enuncia:

x, y, z[x(y + z) = xy + xz]

iii) Existe un elemento neutro para la suma. En smbolos: y x(x + y = x). Este elemento,
cuya existencia se asegura, se designara por 0.

iv) Todo elemento de R tiene inverso aditivo:

xy(x + y = 0)

El inverso aditivo de x lo designaremos por x.

v) Existe un elemento neutro para el producto. Este elemento no es el cero. En smbolos:


y(y 6= 0 x(xy = x)). Este elemento cuya existencia se afirma, se designara por 1.

vi) Todo elemento de R distinto de 0 tiene inverso multiplicativo

x(x 6= 0 y(xy = 1)).

El inverso multiplicativo de x lo designaremos por x1 .

Estos son los axiomas de campo de R. Se llaman de esa manera porque corresponden a los
axiomas de la estructura llamada campo conmutativo en Algebra Moderna. Toda propiedad
o concepto que establezcamos a partir de ellos, tendra valor en cualquiera de dichos campos.
Notemos que tanto los numeros racionales Q como los complejos C satisfacen estos axiomas,
como veremos mas adelante.

La siguiente definicion introduce las operaciones de resta y division en R.

Definicion 6.1
Sean x, y R. Definimos:

a) x y = x + (y)
x
b) = xy 1 siempre que y 6= 0.
y

El siguiente teorema, bastante largo de enunciar, da una lista de las propiedades adicionales
y de uso frecuente que tienen estas cuatro operaciones con numeros reales. Omitiremos los
smbolos y , dandolos por subentendidos.

Teorema 6.1
Sean x, y, z, w R. Entonces

1. El cero es unico

2. x + y = x + z y = z

3. x 0 = 0
175

4. x 6= 0 y 6= 0 xy 6= 0

5. xy = 0 x = 0 y = 0

6. x 6= 0 x1 6= 0
 
x
7. y 6= 0 = 0 x = 0
y
8. El inverso aditivo de un numero es unico

9. -0 = 0

10. (x) = x

11. (x + y) = x y

12. (x y) = y x

13. El 1 es unico

14. xy = xz x 6= 0 y = z

15. El inverso multiplicativo de un numero es unico.

16. 11 = 1

17. El cero no tiene inverso multiplicativo

18. x 6= 0 (x1 )1 = x

19. x 6= 0 y 6= 0 (xy)1 = x1 y 1
x xz
20. y 6= 0 z 6= 0 =
y yz
x z xz
21. y 6= 0 w 6= 0 =
y w yw
x z xw + yz
22. y 6= 0 w 6= 0 + =
y w yw
23. (1)(1) = 1

24. (1)x = x

25. (x)(y) = xy

26. (xy) = (x)y = x(y)

27. x(y z) = xy xz

28. x2 y 2 = (x y)(x + y) (x2 significa xx)

Demostracion
Haremos algunas dejando las restantes la lector.
176

1. Supongamos que tanto 0 como 00 son neutros para la suma. Entonces, por la conmuta-
tividad y las propiedades del neutro tenemos que

0 = 0 + 00 = 00 + 0 = 00 .

2. Sumando el inverso aditivo de x, que sabemos que existe, a la igualdad x + y = x + z


obtenemos:

x + (x + y) = x + (x + z)

Por la asociatividad es equivalente a (x + x) + y = (x + x) + z, de donde concluimos


que y = z.

3. Por ser 0 neutro aditivo tenemos que 0 = 0 + 0. Esto nos permite escribir x 0 = x(0 + 0).
Por la distributividad

x0=x0+x0

Sumando a ambos miembros el inverso aditivo de x 0 tenemos que 0 = x 0.

4. Supongamos que xy = 0. Como x 6= 0 existe x1 . Luego, x1 (xy) = x1 0. Por la aso-


ciatividad del producto y la propiedad 3) del teorema 6.1, podemos escribir (x1 x)y = 0.
Pero esto nos conduce a que y = 0 contradiciendo la hipotesis.

5. Sea xy = 0. Si tanto x como y fueran distintos de cero tendramos por 4) que xy 6= 0 lo


que contradice la hipotesis.

6. Si x1 = 0 tendramos que 1 = xx1 = x0 = 0 lo que contradice que 1 y 0 son distintos.


Las restantes demostraciones las dejamos al lector. Conviene destacar que entre los
axiomas de campo y este teorema se encuentran todas las propiedades que se usan en la
operatoria aritmetica de los numeros. Desde ahora las usaremos sin mayores comentarios.
El siguiente ejemplo aprovecha las propiedades establecidas en este teorema; el es una
generalizacion de lo que se acostumbra llamar composicion y descomposicion de razones.

Ejemplo 6.1
Sean a, b, c, d, w, x, y, z numeros reales tales que ninguno de los denominadores que aparezcan
sea cero. Entonces:

a c aw + bx cw + dx
= =
b d ay + bz cy + dz
a c
Para establecer la igualdad, sea m = = . Entonces a = bm y c = dm. Luego,
b d
aw + bx bmw + bx mw + x
= =
ay + bz bmy + bz my + z
De igual modo resulta que

cw + dx mw + x
=
cy + dz my + z
177

lo que prueba la afirmacion.

Los siguientes casos particulares, de uso mas frecuente, reciben los siguientes nombres:

i) Componer con respecto al consecuente:


w = x = z = 1, y = 0.

a c a+b c+d
= =
b d b d

ii) Descomponer con respecto al consecuente: w = z = 1, x = 1, y = 0.

a c ab cd
= =
b d b d

iii) Componer y descomponer a la vez: w = x = y = 1, z = 1.

a c a+b c+d
= =
b d ab cd

Terminaremos esta seccion enunciando tres teoremas que generalizan las propiedades asocia-
tiva, conmutativa y distributiva. El lector interesado en las demostraciones, puede consultar
el libro de John M. H. Olmsted, The Real Number System.

Teorema 6.2 Asociatividad Generalizada.

El resultado de cualquier suma de los numeros x1 , x2 , , xn es independiente de las


formas de asociar sus sumandos.

Teorema 6.3 Conmutatividad Generalizada.

El resultado cualquier suma de los numeros x1 , x2 , , xn es independiente del orden de


los sumandos.

Nota
Estos 2 teoremas siguen siendo validos si se cambia suma por producto.

Teorema 6.4 Distributividad Generalizada.

x(y1 + y2 + + yn ) = xy1 + xy2 + + xyn


178

6.2. AXIOMAS DE ORDEN


Existe un subconjunto de R que designaremos por R+ , cuyos elementos llamaremos nume-
ros reales positivos y que satisfacen las siguientes condiciones:

i) Si x e y son reales positivos entonces x + y y xy tambien lo son.

ii) Para cada numero real x, una y solo una de las siguientes alternativas es verdadera: x
es real positivo, x es real positivo o x = 0.

La primera propiedad simplemente dice que R+ es cerrado con respecto a la suma y al


producto; la segunda, conocida como Ley de la Tricotoma, asegura que si x es un real
entonces, o pertenece a R+ o su inverso aditivo es elemento de R+ o x = 0; y solo una de
estas tres alternativas es verdadera.

Definicion 6.2
Definimos el subconjunto de los reales negativos de la forma siguiente

R = {x : x R+ }

Nota
De la ley de la tricotoma se deduce inmediatamente que R = R+ R {0} y que estos
conjuntos son disjuntos de a pares.

Definicion 6.3
Definimos en R la relacion binaria menor que, que denotamos con el smbolo < por la
formula:

i) x < y si y solo si y x R+

Se lee x menor que y. Usando esta, definimos las relaciones menor o igual (),
mayor(>) y mayor o igual () por las formulas siguientes:

ii) x y si y solo si x < y o x = y.

iii) x > y si y solo si y < x.

iv) x y si y solo si x > y o x = y.


En el siguiente teorema concentramos las propiedades mas usadas que tienen estas re-
laciones.

Teorema 6.5
Sean x, y, z y w numeros reales. Entonces

1. x < 0 si y solo si x R .

2. x > 0 si y solo si x R+ .

3. x y si y solo si es falso que x > y.

4. x y si y solo si y x.
179

5. x < y e y < z implica x < z. En palabras, la relacion menor es transitiva.

6. x y e y z implica x z. La relacion menor o igual es transitiva.

7. Una y solo una de las tres afirmaciones siguientes es verdadera:

x < y; x > y; x = y

Es otra forma de la ley de la tricotoma.

8. Sea x < y. Entonces, para cualquier real z se tiene que x + z < y + z.


Se acostumbra referir a esta propiedad diciendo que A ambos miembros de una de-
sigualdad se le puede sumar o restar un mismo numero sin que cambie su sentido.

9. Sea x < 0, y < 0. Entonces xy > 0.


En palabras. El producto de dos reales negativos es positivo.

10. Sean x < 0, y > 0. Entonces xy < 0.


El producto de dos reales de signo distinto es negativo.

11. 1 R+ .
1
12. x R+ implica que R+
x
1 1
13. 0 < x < y implica < .
y x
14. Sea z > 0. Entonces (x < y) (xz < yz).
Esta propiedad se acostumbra enunciarla por:
Si los dos miembros de una desigualdad se multiplican por un real positivo su sentido
no cambia.

15. Sea z R . Entonces (x < y) (xz > yz).


Si ambos miembros de una desigualdad se multiplican por un real negativo su sentido
se invierte.

16. x < y y z < w implica que x + z < y + w.


En palabras Dos desigualdades del mismo sentido se pueden sumar sin que cambie el
sentido

17. Sean x, y, z y w reales no negativos y tales que x < y y z < w. Entonces xz < yw.

Nota
Las propiedades 8), 14), 15), 16) y 17) tambien son validas cambiando < por .

Demostracion
Son todas faciles. Las expondremos en forma sucinta.

1. x < 0 (0 x) R+ x R+ x R
180

2. x > 0 0 < x x 0 R+ x R+ .

3. x y x < y x = y y x R+ x = y
x y R x = y q(x y R+ ) q(x > y)

4. x y x > y x = y y < x x = y y x

5. x < y y < z y x R+ z y R+
Al sumar se obtiene z x R+ que equivale a x < z

6. Analoga a 5)

7. Por el axioma de la tricotoma tenemos que y x R+ o x y R+ o x = y que es


equivalente a x < y o x > y o x = y
(La o tiene el significado de excluyente).

8. x < y y x > 0 y x + z z > 0 y + z (x + z) > 0 x + z < y + z

9. x < 0 y < 0 x R+ y R+ (x)(y) R+ xy R+ xy > 0.

10. x < 0 y > 0 x R+ y R+ (x)y R+ (xy) R+ xy < 0

11. 1 R 1 R+ (1)(1) R+ 1 R+ . (Contradiccion).


Luego, 1 R+ .
1 1
12. 1 = x . Si R por 10) tendramos que 1 R , (Contradiccion con 11).
x x
1 1 yx 1 1
13. = = (y x) . Por 12), es positivo y como y x tambien lo es, tenemos
x y xy xy xy
1 1
que es positivo.
x y

14. Sea x < y y z > 0. Luego, y x R+ y z R+ . Entonces (y x)z R+ ; de donde,


xz < yz.
1 1 1
Sea ahora, xz < yz y z > 0. Por 12) R+ y, por lo ya probado, (xz) < (yz) ; es
z z z
decir, x < y.

15. Sabemos que y x R+ y z R+ . Luego, usando 10) obtenemos que (y x)z < 0, de
donde cocluimos que xz > yz.

16. Por hipotesis tenemos que yx R+ wz R+ . Sumando, se tiene que y+w(z+z)


R+ , lo que implica que x + z < y + w.

17. Multiplicando x < y por z y z < w por y obtenemos, segun 14) que xz < yz y yz < yw.
Por la transitividad logramos el resultado.

Antes de seguir avanzado, desarrollaremos algunos ejemplos con el fin de familiarizarnos


mas profundamente con las propiedades hasta aqu establecidas.
181

Ejemplo 6.2
Pruebe que la suma de dos numeros negativos es negativa.

Sean x e y negativos. Entonces x e y son positivos. Luego, (x) + (y) = (x + y) es


positivo, lo que implica que x + y es negativo.

Ejemplo 6.3
Pruebe que a b y b a es equivalente con a = b.

a b y b a (a = b o a < b) y (a = b o b < a) (a = b) o (a < b y b < a)

Pero la segunda parte de la conjuncion es falsa. Por lo tanto

a=b

Ejemplo 6.4
Sea x < y. Entonces x < c < y si y solo si existen a y b positivos con a + b = 1 y tales que
c = ax + by.
yc cx
Sea x < c < y. Tomemos a = yb= . Entonces
yx yx
yc+cx yc cx
a+b= =1 y ax + by = x+ y=c
yx yx yx

Inversamente, supongamos que a y b son positivos y suman 1 y sea c = ax + by. Debemos


probar que x < c < y. Si c y tenemos que, al reemplazar c por ax + by, se transforma en
y ax + by. Usando el hecho que a + b = 1 podemos escribir que (a + b)y ax + by, lo que
al simplificar implica que y x. Pero esto contradice la hipotesis inicial que y > x. Luego,
c < y. De igual forma se prueba que c > x.
Un estudio mas profundo de las desigualdades se hara en el captulo octavo.

6.2.1. EJERCICIOS
1. Complete la demostracion del teorema 6.1

2. Pruebe que la ecuacion ax + b = 0 con a 6= 0 tiene solo una solucion.

3. Pruebe que si x es un numero tal que para todo numero  positivo, se cumple que x < 
entonces x 0.

a+b
4. Pruebe que si a y b son numeros distintos, entonces su promedio aritmetico esta en-
2
tre ellos.
182

6.3. NUMEROS NATURALES


Corresponde construir los numeros naturales. Ellos apareceran como un cierto subconjunto
de R. Previamente, necesitamos establecer el concepto de conjunto inductivo.

Definicion 6.4
Sea S R. Diremos que S es un conjunto inductivo si satisface las dos propiedades siguientes:

i) 1 S

ii) x S x + 1 S
Como ejemplos de conjuntos inductivos tenemos R, R+ ,

1 3
{0, , 1, , 2, }, {x : x = 1 x 2}, {x : x 1}.
2 2
Definicion 6.5
Diremos que un numero es natural si y solo si es miembro de cualquier conjunto inductivo.
Al conjunto de todos los numeros naturales lo designaremos por N.

Teorema 6.6
N es inductivo. Ademas si S es un conjunto inductivo entonces N S.

Demostracion
Por la definicion 6.4 tenemos que 1 es miembro de cualquier conjunto inductivo. Luego, 1 N.

Sea n N. Por la definicion 6.5 tenemos que n es miembro de todo conjunto inductivo.
Entonces, nuevamente por 6.4, n + 1 tambien es miembro de esos mismos conjuntos. Luego,
n + 1 N, lo que termina de probar que N es inductivo. El resto de la demostracion es inme-
diato de la definicion de N.

El siguiente teorema es el que permite justificar las definiciones y demostraciones inductivas,


que fueron introducidas en el Captulo 2.

Teorema 6.7 (Principio de Induccion).

Sea S N tal que:

i) 1 S

ii) Si n S entonces n + 1 S.
Entonces, en estas condiciones S = N.

Demostracion
Por la hipotesis se tiene que S es inductivo. Entonces, por el teorema anterior N S. Pero
como S N, se obtiene la igualdad.
183

Notemos que este teorema se puede enunciar diciendo que N es el unico conjunto inductivo
formado solamente por numeros naturales.

En el siguiente teorema se exponen las propiedades mas caractersticas de los numeros


naturales.

Teorema 6.8

i) 1 es el menor de todos los numeros naturales.

ii) La suma de dos numeros naturales es un numero natural.

iii) El producto de dos numeros naturales es un numero natural.

iv) Si m y n son numeros naturales con m < n entonces n m tambien es natural.

v) Sea m natural. Entonces no hay ningun natural n entre m y m + 1.

Demostracion

i) El conjunto S = {x : x 1} es inductivo. Luego, N S. Pero el menor elemento de S


es el 1, lo que prueba que este es tambien el menor elemento de N.

ii) Probaremos que m + n es natural cuando m y n lo son, por induccion en n.

Como m es natural, m es miembro de todo conjunto inductivo. Entonces, por la de-


finicion de inductivo, m + 1 tambien es miembro de dichos conjuntos. Luego, por la
definicion de N, se tiene que m + 1 es natural, lo que prueba que la afirmacion es valida
para n = 1. Supongamos ahora que m + n es natural cuando m y n lo son. En forma
similar al argumento anterior se prueba que m + n + 1 tambien lo es, lo que completa
la demostracion inductiva.

iii) Probaremos que mn es natural cuando m y n lo son, por induccion en n.

Sea natural. Como m 1 = m tenemos que la proposicion es valida para 1. Supongamos


que mn N. Entonces m(n + 1) = mn + m es, por ii), un numero natural.

iv) Se probara por induccion sobre m. El caso m = 1 se hara por contradiccion. Suponga-
mos que existe un natural t mayor que 1, tal que t 1 no es natural. Sea S = N {t}.
Vamos a probar que S es inductivo. Como t 6= 1 se tiene que 1 S.
Sea n S, si n + 1 6 S entonces n + 1 = t. Luego, como n = t 1 se tendra que
n no es natural en contradiccion con la definicion de S. Luego, hemos probado que si
n S entonces n + 1 S. Del teorema 4 concluimos que N = S. Pero esto es una
contradiccion, lo que establece iv) para el caso en que m = 1.

Supongamos ahora que para todo natural n mayor que m se cumple que n m es na-
tural. Debemos probar que cualquiera sea el natural n mayor que m + 1, tambien se
cumple que n (m + 1) es natural.
184

Sea n natural tal que n > m + 1. Tenemos que n > m y por la hipotesis inductiva
obtenemos que d = nm es un natural. Como n > m+1 se tiene, que nm > 1, es decir,
d > 1. Luego, por lo ya probado tenemos que d 1 es natural. Pero n (m + 1) = d 1,
lo que termina la demostracion de iv).
v) Se probara por induccion en m. En primer lugar debemos probar que no hay un natural
entre 1 y 2. Para esto consideremos el conjunto S = {1} {x : x 2}. Es inmediato
que S es inductivo y no tiene elementos entre 1 y 2. Como N S se tiene la afirmacion.
Supongamos, ahora, que no hay numeros naturales entre m y m + 1. Debemos probar
que tampoco los hay entre m + 1 y m + 2. Lo haremos por contradiccion. Sea n un
natural tal que m + 1 < n < m + 2. Se tiene que n > 1. Luego, por el punto anterior,
n 1 N. Pero la desigualdad de arriba implica m < n 1 < m + 1 lo que contradice
la hipotesis inductiva.

El siguiente teorema, conocido con el nombre de Principio del Buen orden, muestra
una propiedad fundamental del conjunto N.

Teorema 6.9
Cualquier subconjunto de N no vaco tiene, un menor elemento.

Demostracion
Por contradiccion. Supongamos que existe A N no vaco y tal que no tiene un menor
elemento. Sea S N definido por: n S si y solo si n es menor que cualquier elementos de A.
Vamos a probar que S es inductivo. Es evidente que 1 6 A porque sera su menor elemento.
Entonces 1 es menor que cualquier numero de A. Luego, 1 S. Sea n S. Entonces para
todo a A se tiene que a > n. Luego, por la propiedad v) del teorema anterior, a n + 1.
Pero n + 1 6 A porque sera su menor elemento. Por lo tanto a > n + 1 para todo elemento
de A. Luego, n + 1 S lo que prueba que es inductivo. En estas condiciones sabemos que
S N y como S A = , tenemos que A = . Pero esta conclusion contradice la hipotesis y
el teorema queda establecido.

6.4. NUMEROS ENTEROS Y RACIONALES


Definicion 6.6
Diremos que un numero m es un entero si y solo si m es un natural o m es natural o m = 0.

Al conjunto de todos los enteros lo designaremos por Z.

Notemos que si m es un entero, una y solo una de las tres alternativas de esta definicion, es
valida.

Teorema 6.10
La resta de dos naturales es un entero.

Demostracion
Sean n y p numeros naturales. Entonces

i) Si p > n se tiene, por la definicion de <, que p n es un natural.


185

ii) Si p = n entonces p n = 0, que es un entero.

iii) Si p < n, entonces p n = (n p). Pero n p es un numero natural y por lo tanto


(n p) es entero.

Teorema 6.11
Z es cerrado con respecto a la suma y a la resta.

Demostracion
Es sencilla a partir del teorema anterior.

Teorema 6.12
Z es cerrado con respecto al producto.

Demostracion
Sean m y n Z. Si m y n son naturales, entonces mn tambien lo es.

Si m = 0 entonces mn = 0, el cual es entero.

Si m, n N entonces mn = (m)n.

Pero, (m)n es natural. Luego, mn es un entero. Por ultimo, si m y n N entonces


mn = (m)(n) que pertenece a los naturales.

En el captulo 2 definimos an con n N y a R. Vamos a ampliar esta definicion a


exponentes enteros del modo siguiente:

Definicion 6.7
Sea a un real distinto de 0 y n un entero no positivo: Definimos

1. a0 = 1
1
2. an =
an
Las propiedades de esta operacion estan dadas en el siguiente teorema.

Teorema 6.13
Sean a y b reales distintos de cero y m y n enteros. Entonces

i) am an = am+n
am
ii) = amn
an
iii) (am )n = amn

iv) (ab)n = an bn
 a n a n
v) = n
b b
186

Demostracion

i) Es por casos.

a) Si m y n son positivos fue propuesto como ejercicio en el ejemplo 2.7.


1 1
b) Si m y n son negativos. Entoces am an = m n donde m y n son enteros
a a
positivos. Nuevamente, por el ejemplo 2.7, tenemos que:

1 1
= = = am+n
am+(n) a(m+n)
am
c) Si m > 0 y n < 0. Entonces am an = . Pero, por las propiedades de los
an
exponentes enteros positivos = am(n) = am+n , siempre que m > n.

am
Si m = n, entonces = 1 = a0 = am+n .
an
Por ultimo, si m < n, por el ejemplo ya citado tenemos que:

am 1 1
= nm = (m+n) = am+n .
an a a

ii) Por i) am = amn an . Dividiendo por an obtenemos el resultado.

Las restantes demostraciones las dejamos al lector.

Ejemplo 6.5 i) 40 = 1.
 1
1
ii) = 31 = 3.
3
1
iii) (32 )2 = 34 =
34
 2  2
1 3
iv) 32 52 2
=3 =
5 5

Ejemplo 6.6
Sea a > 1 y m y n dos enteros. Entonces m < n si y solo si am < an .

Sea m < n. Entonces n m es un entero positivo que llamaremos d. Entonces n = n + d. Por


la propiedad 17) del teorema 2 se prueba facilmente que ad > 1. Luego, multiplicando por
am , se obtiene la desigualdad.

Supongamos ahora, que an > am . Si m > n, por lo ya probado tenemos que am > an . Con-
tradiccion.

Si m = n es inmediato que an = am . Contradiccion.


187

Luego, m < n.

Nota
Al conjunto Z de los enteros podemos considerarlo como una ampliacion del conjunto N de los
naturales en el siguiente sentido; Z es un subconjunto de R que contiene a N, que es cerrado
con respecto a la suma, resta y producto y que si T es otro subconjunto de R con estas mismas
caractersticas, entonces Z T .

Es facil ver que Z no es cerrado con respecto a la division. Esto nos lleva a tratar de buscar
un subconjunto de R, que sea una ampliacion de Z y que ademas sea cerrado con respecto a
la division. Este resultara ser Q, el conjunto de los numeros racionales.

Definicion 6.8
Diremos que q es un numero racional si y solo si existen dos enteros m y n tales que q = m/n.
Al conjunto de todos los numeros racionales lo designaremos por Q. Si un numero no es ra-
cional, diremos que es irracional.

Las propiedades 21) y 22) del teorema 6.1 nos muestran que Q es cerrado con respecto a la
suma, resta, producto y division. Es inmediato que si z es un entero, entonces es racional
ya que z = z/1. Ademas, si T es un subconjunto de R que es cerrado con respecto a estas
cuatro operaciones y que contiene a Z, entonces Q T . Todo esto nos prueba que Q es
una extension de Z cerrada con respecto a la division. Mas aun, como 0 y 1 son numeros
enteros y por lo tanto racionales y la distributividad, que al ser valida en R vale tambien para
cualquier subconjunto de este, se tiene que Q es un campo. Todava, aprovechando el orden
de R, concluimos que Q es un campo ordenado.

1
Sabemos que 1 6 N lo que nos muestra que N Z. Tambien 6 Z lo que prueba que
2
Z Q. Pero conlos axiomas hasta aqu desarrollados, es imposible probar que Q y R son
distintos, porque en rigor aun no hemos caracterizado completamente al conjunto de los reales.

6.4.1. EJERCICIOS
1. Pruebe que la suma o el producto de n numeros naturales tambien es natural.

2. Pruebe que si d N y a > 1 entonces ad > 1.

3. Pruebe que si 0 < a < 1 y m y n naturales. Entonces m < n es equivalente cn am > an .

4. Sea a positivo y distinto de 1 y m y n enteros. Pruebe que am = an implica que m = n.

5. Sea T R tal que es cerrado con respecto a la suma, resta y producto. Supongamos
ademas que N T . Pruebe que Z T .

6. Sea U R tal que es cerrado con respecto a la suma, resta, producto y division.
Supongamos ademas que Z U . Pruebe que Q U .

7. Complete la demostracion del teorema 6.14


188

6.5. SOLUCIONES O SUGERENCIAS A LOS EJERCICIOS


DEL CAPITULO
Pagina 177

1. (7) Por (6) y 1 6= 0. Luego, por 5) xy 1 = 0 x = 0.

En el otro sentido. Si x = 0 entonces por 3) xy 1 = 0.


(8) Sean x0 y x00 inversos aditivos de x. Entonces

x0 = (x + x00 + x0 = (x + x0 ) + x00 = x00 .

(9) 0 = 0 + (0) = 0
(10) x = x + ((x) + ((x))) = (x + (x)) + (x) = (x)
(11) (x y) + (x + y) = 0. Luego, por 8, (x + y) = x y.
(12) Analoga a 11)
(13) Sean 1 y 1neutros para el producto.
Entonces 1 = 1 10 = 10
(14) x 6= 0 x1 existe. Multiplicando la igualdad por este se obtiene y = z.
(15) Sean u y v inversos multiplicativos para x. Entonces u = u(x v) = (ux)v = v.
(16) 11 = 1 11 = 1. La primera igualdad se debe a las propiedades de 1 mientras que
la segunda al hecho que x x1 = 1.
(17) Sea 01 inverso multiplicativo de 0. Entonces, 1 = 0 01 = 0. Contradiccion.
(18) x x1 = 1. Luego, por 15, x = (x1 )1 .
(19) (x1 y 1 )(xy) = (x1 x)(y 1 y) = 1. Luego, x1 y 1 = (xy)1 .
(20) x/y = x y 1 = xy 1 zz 1 = (xz)(yz)1 = xz/yz.
x z xz
(21) = (xy 1 )(z w1 ) = (xz)(yw)1 =
y w yw
(22)

x/y + x/w = xy 1 + zw1 = (xw)(w1 y 1 ) + (zy)(y 1 w1 )

= (xw + yz)(yw)1

(23) (1)(1) = (1)(1) + 1 + (1) = (1)((1) + 1) + 1 = (1) 0 + 1 = 1.


(24) x + (1)x = x(1 + (1)) = 0
(25) De 23) y 24)
(26) (x)y + xy = (x + x)y = 0 y = 0. Luego, (x)y = (xy)
(27) x(y z) = x(y + (z)) = xy + x(z) = xy xz.
(28) Distribuyendo dos veces se obtiene la igualdad.
b
2. Sea q una solucion de ax + b = 0. Luego, aq + b = 0; aq = b; q = .
a
189

3. Sea x > 0. Entonces x/2 > 0 y por el enunciado x < x/2, de lo que resulta que 2 < 1.
Restandole 1 se obtiene 1 < 0 que contradice el hecho de que 1 es positivo.

4. Sea a < b, entonces (a + b)/2 a = (b a)/2 > 0.

b (a + b)/2 = (b a)/2 > 0

Pagina 183

1. Es inmediato por induccion.

2. Por induccion en d.

3. Sea 0 < a < 1 y sea n = m + p con p > 0. Probaremos que am > an por induccion en p.
Como a < 1 multiplicando por am se tiene que am+1 < am . Sea am+p < am . Entonces

am+p+1 < am+1 < am .

Inversamente. Supongamos ahora que am > an .


Si m = n entnces am = an . Si m > n entonces, por lo ya probado am < an . Pero. Luego,
m < n.

4. Supongamos que m n. Sabemos que 0 = an am = am (anm 1). Pero am 6= 0.


Luego, anm = 1.
Pero como a 6= 1 la unica posibilidad es que n = m.

5. 1 T . Luego, 1 1 = 0 T .
Sea p Z. Entonces p N. Luego, 0 (p) T . Luego, p T .

6. Es similar a 5.

7. Demostracion de iii) del teorema 6.14. Si n es natural es facil probar por induccion en
n que

(am )n = amn

Si n = 0 entonces (am )n = 1 y amn = a0 = 1.


Si n es un entero negativo

1 1
(am )n = = = amn
(am )n amn

Demostracion de iv).
1
Sea n < 0. Entonces (ab)n =
(ab)n
1 1 1
= = = an bn .
an bn an bn
190

Demostracion de v)
 a n  a  n
an = b = bn .
b b
 a n an
Luego, = .
b bn
Captulo 7

ALGO MAS SOBRE


RACIONALES

7.1. DECIMALES
El smbolo a/b con (a, b) = 1 es la forma habitual de representar los numeros racionales.
Esta representacion es unica, hecho que se prueba facilmente a partir de la factorizacion en
primos. Tambien pueden ser escrito, por medio del algoritmo de la division, en forma decimal.
El desarrollo que resulta es finito o periodico, como se ilustra en los siguientes ejemplos.

1. 3/4 = 0, 75 2. 33/20 = 1, 65
3. 4/9 = 0, 444 . . . 4. 17/11 = 1, 545454 . . .
5. 21/74 = 0, 2837837 . . . 6. 799/220 = 3, 631818 . . .
En los ejemplos 1.- y 2.- el desarrollo decimal es finito, mientras que en los otros ejemplos el
desarrollo es periodico. En 3.- y 4.- el periodo empieza inmediatamente despues de la coma
decimal, mientras que en 5.- y 6.- comienza mas alla de la coma decimal.

El siguiente teorema caracteriza los desarrollos decimales finitos.

Teorema 7.1
El desarrollo decimal de a/b es finito si y solo si b = 2 5 .

Demostracion
Supongamos que a/b = p, q1 q2 . . . qn . Entonces

q1 q2 qn 10n p + 10n1 q1 + + qn
a/b = p + + 2 + + n =
10 10 10 10n
Luego,

10n a/b = 10n p + 10n1 q1 + + qn


de donde b debe ser un factor de 10n . Por lo tanto, b = 2 5 .

Inversamente, sea b = 2 5 . Entonces


a
a/b =
2 5

191
192

Sea n = max{, }. Es inmediato que 10n = 2 5 2n 5n = bq, para algun natural q, de


donde b = 10n /q. Entonces, a/b = qa/10n , expresion que tiene un desarrollo decimal finito.

De este teorema concluimos que si b no es primo con 10, pero posee un factor distinto de 2
o de 5, entonces su desarrollo decimal es periodico.

A continuacion probaremos que los casos del tipo 3) y 4) se producen cuando (b, 10) = 1,
es decir, cuando b no tiene el factor 2 o el factor 5. Ademas, encontraremos una caracterizacion
para la longitud del periodo, que designaremos por (b).

Veamos primero un ejemplo: La division de 7 por 41 se muestra en detalle en las siguientes 6


ecuaciones:

1) 7 = 0 41 + 7; r1 = 7
2) 70 = 1 41 + 29; r2 = 29 q1 = 1
3) 290 = 7 41 + 3; r3 = 3 q2 = 7
4) 30 = 0 41 + 30; r4 = 30 q3 = 0
5) 300 = 7 41 + 13; r5 = 13 q4 = 7
6) 130 = 3 41 + 7; r6 = 7 q5 = 3
Como el resto 7 ya aparecio, estas ecuaciones se repetiran en forma identica a partir de la
segunda.

Este proceso queda resumido en el siguiente esquema:

7/41 = 0, 17073
7 29 3 30 13 7
donde la primera fila de numeros corresponde a los sucesivos cuocientes y la segunda a los co-
rrespondientes restos. Notemos que, como el numerador es menor que el denominador, siempre
este sera el primer resto. Ademas, como todos los restos son menores al denominador, alguno
de ellos forzosamente debera repetirse. En este caso fue el 7, por lo tanto, desde ese punto en
adelante se repetiran los mismos decimales y restos.

En el siguiente teorema nos dicen cual sera el primer resto en repetirse.

Teorema 7.2
Sea a/b un racional con a < b y (b, 10) = 1.Entonces, el primer resto en repetirse es a; es
decir, es el primer resto que aparece.

Demostracion
Aplicando el algoritmo de la division, obtenemos las siguientes ecuaciones:

1. a = 0 b + a. a es el primer resto. Designemoslo por r0 .

2. 10r0 = 10a = q1 b + r1 con r1 < b.

3. 10r1 = q2 b + r2 .
..
.
n+1. 10rn1 = qn b + rn
193

De paso, notemos que todos los restos son numeros primos con b. Esto se debe a que como
(a, b) = 1, entonces de la ecuacion 2) obtenemos que (r1 , b) = 1, y as sucesivamente. Este
resultado nos sera util mas adelante.

Sea rn = rp , p < n, el primer resto en repetirse.

Tenemos que

rn = 10rn1 qn b
rp = 10rp1 qp b
Restando

10(rn1 rp1 ) = b(qp1 qn )


Como (b, 10) = 1, se tiene que b | (rn1 rp1 ). Pero rn1 rp1 es un numero entre b y b.
Luego, debe ser 0 y, por lo tanto, rn1 = rp1 .

La unica manera de evitar la contradiccion es aceptando que rp es un resto que no tiene


anterior. El unico con esta cualidad es a. Luego, rp = a.

De este resultado concluimos que el periodo comienza inmediatamente despues de la coma


decimal y, ademas, su longitud no es mayor que (b), pues los posibles restos son numeros
primos con b.

La siguiente observacion, que usaremos a continuacion, es inmediata al expresar, en las


ecuaciones anteriores, los restos en funcion de a y b.

rp es el resto de la division de 10p a por b00


Ahora estamos en condiciones de probar el teorema central de esta seccion.

Teorema 7.3
Sea (b, 10) = 1. Entonces, la longitud de su periodo es el menor l tal que b | (10l 1).

Demostracion
Sea rl el primer resto que se repite. Sabemos que rl = a

Entonces 10l a = qb + a.

De donde a(10l 1) = qb.

Pero (a, b) = 1.

Por lo tanto b | (10l 1).

Notemos que el valor de l depende solamente de b, por lo tanto, todos los racionales de deno-
minador b tienen periodos de igual longitud. Designaremos por (b) esta longitud.
194

Ejemplo 7.1

1/13 = 0, 0 7 6 9 2 3
1 10 9 12 3 4 1
2/13 = 0, 1 5 3 8 4 6
2 7 5 11 6 8 2
Observemos que entre los dos desarrollos aparecieron todos los posibles restos. En estas con-
diciones, es muy facil escribir otros cuocientes. Por ejemplo:

7/13 = 0, 5 3 8 4 6 1
7 5 11 6 5 2 7
ya que el primer resto que debe aparecer es el 7, que se encuentra en el desarrollo de 2/13.
Entonces, los restos y el orden en que apareceran sera 7 5 11 6 8 2 y su cuociente es 0, 538461.

La siguiente tabla muestra, para los primeros 16 valores de (b), el menor b cuyo periodo tiene
esa longitud.

(b) 1 2 3 4 5 6 7 8
b 3 11 27 101 41 7 239 73

(b) 9 10 11 12 13 14 15 16
b 81 451 21 649 13 53 2629 31 17

Consideraremos ahora el problema inverso, es decir, dado un decimal periodico 0, 1 2 . . . n


encontrar una fraccion que lo genere.

Teorema 7.4
1 , 2 . . . n
La fraccion genera el decimal 0, 1 , 2 , . . . n .
10n 1
Demostracion
Sea a/b dicha fraccion . Entonces, a/b = 0, 1 2 . . . n . Amplificando por 10n obtenemos
a a
10n= 1 2 . . . n + 0, 1 2 . . . n = 1 2 . . . n +
b b
De esta ultima igualdad, despejando a/b, finalmente obtenemos
a 1 2 . . . n 1 2 . . . n
= n
= (n nueves)
b 10 1 99 . . . 9
En esta fraccion pueden haber factores comunes que habra que simplificarlos para obtener el
racional que corresponde a ese periodo.

Ejemplo 7.2
Encontrar el racional cuyo desarrollo decimal es 0, 10701.

Por lo anterior tenemos que la fraccion 10 701/99 999 tiene ese desarrollo decimal. Ella, sim-
plificada, se reduce a 29/271.
195

Volviendo a los ejemplos con que empezamos este captulo, se tiene que los casos 1.- y 2.-
corresponde a los denominadores de la forma 2 5 . Los casos 3.- y 4.-, a denominadores que
son primos con 10. Por lo tanto, los casos 5.- y 6.- deben corresponder a la situacion en que
el denominador no es primo con b, pero tiene algun factor distinto de 2 y 5.

Ejemplo 7.3
El decimal 0, 2216 lo podemos expresar como:
1 216 41
0, 2 + 0, 0216 = + =
5 9990 185
cuyo denominador se factoriza en 5 37.

Que podemos decir de los decimales decimales infinitos no periodicos?; como, por ejem-
plo,
1, 101001000100001, . . . , donde la sucesion de ceros va aumentando de 1 en 1. Por el mo-
mento, solo podemos afirmar que no es un racional, pues estos tienen un desarrollo finito o
periodico. Por esta razon, los llamaremos numeros irracionales. Volveremos sobre ellos en el
ultimo captulo.

Los temas restantes de esta seccion los desarrollaremos a traves de ejemplos, que se ex-
trapolan facilmente a la situacion general.

Consideremos el denominador b = 21. El conjunto de sus restos es

{1, 2, 4, 5, 8, 10, 11, 13, 16, 17, 19, 20}


Ellos son 12. es decir, (b) = 12
1
= 0, 0 4 7 6 1 9
21
1 10 16 13 4 19 1
Como el resto 4 aparece, podemos deducir que 4/21 es 0, 190476, ya que 4 sera el primer resto
y, por lo tanto, el orden en que estos apareceran sera 4, 19,1, 10, 16, 13 y por lo tanto, sus
correspondientes cuocientes seran 1, 9, 0, 4, 7, 6.

Como el resto 2 no aparece en esta division, consideremos el racional 2/21 cuyo primer resto
sera 2. Esto obliga a que ningun resto de la division anterior podra aparecer en su desarrollo
decimal. De hecho,
2
= 0, 0 9 5 2 3 8
21
2 20 11 5 8 17 2
Como el numero de restos que aparecen en cada division es igual a la longitud del periodo
(21), se concluye facilmente que (21) es un divisor de (21).

Este resultado se generaliza facilmente como se enuncia en el siguiente teorema.

Teorema 7.5
Sea (b, 10) = 1. Entonces, (b) es un divisor de (b).
196

Usando este resultado probaremos el siguiente teorema

Teorema 7.6
Sea (b, 10) = 1. Entonces, b | (10(b) 1)

Demostracion
Existe un natural k, tal que (b) = k(b), por lo tanto,

10(b) 1 = 10k(b) 1 = (10(b) )k 1 = xk 1 = (x 1)(xk1 ) + xk2 + + 1)

donde x = 10(b) .

Pero b | (10(b) 1). Luego, b | (10(b) 1).

7.2. CAMBIOS DE BASE


El sistema de numeracion que actualmente usamos es en base 10 (Sistema Denario). Sig-
nifica que usamos diez smbolos (dgitos) para escribir los numeros. Por ejemplo, el smbolo
8 307, 52 es una abreviatura de

8 103 + 3 102 + 0 101 + 7 100 + 5 101 + 2 102


Este sistema de escritura es posicional, ya que la potencia de 10 que acompana a cada dgito
depende de su ubicacion dentro del grupo. Ademas tiene la ventaja de poseer algoritmos muy
sencillos para realizar las operaciones aritmeticas de suma, resta, multilplicacion y division.

No hay ninguna razon especial para que nuestro sistema numerico tenga ese numero de dgi-
tos. Se cree que se debe al numero de dedos de ambas manos, que lo ratifica la frecuencia con
que las personas usan sus dedos para ayudarse en las operaciones aritmeticas.

Supongamos que hemos entrado en contacto con habitantes de otro planeta identicos a no-
sotros, excepto que en cada una de sus manos tienen tres dedos. Entonces, probablemente
su sistema numerico sera en base seis. Aceptemos que sus dgitos sean 0, 1, 2, 3, 4, 5. En
el siguiente cuadro aparece la escritura de sus primeros 24 numeros y los equivalentes en el
sistema denario.

6 dedos 1 2 3 4 5 10 11 12 13 14 15 20
denarios 1 2 3 4 5 6 7 8 9 10 11 12

6 dedos 21 22 23 24 25 30 31 32 33 34 35 40
denarios 13 14 15 16 17 18 19 20 21 22 23 24

Observando esta tabla, podra producirse la siguiente conversacion:


197

- Denarios: Ustedes usan un sistema de numeracion en base 6.


- 6 dedos: No, nosotros usamos un sistema en base 10, pero ustedes usan el de base 14.

Queremos hacer notar que, desde el punto de vista del usuario, si su base es un natural,
siempre el sistema sera en base 10. Esto se debe a que tiene que tomarse como base el natu-
ral que sigue al ultimo dgito, el cual se escribira 10. Ademas conviene que los smbolos que
representan a los dgitos sean sencillos para facilitar la escritura.

Por ejemplo, en el sistema de base 12 se toman las letras A y B como los dgitos que reem-
plazan al numero 10 y al numero 11. Entonces, sus primeros numeros seran: 0, 1, 2, 3, 4, 5,
6, 7, 8, 9, A, B, 10, 11, 12, . . . , 19, 1A, 1B, 20, . . .

El numero 2A3BA, en esta base equivale al numero 2 124 + 10 123 + 3 122 + 11 12 + 10


que, en base denaria, corresponde a 59 326.

Debemos hacer notar que los smbolos tales como 1, 7, 29, 2715, etc. no son numeros, sino
smbolos que los representan o, si se prefiere, nombres que les damos a ellos. Por lo tanto, un
cambio de base es simplemente un cambio de nombre que no altera las propiedades intrnsecas
de los numeros. Por ejemplo, si un numero es primo en alguna base, seguira siendolo en cual-
quier otra base. Si se quiere una prueba mas objetiva de este hecho, notemos que el sistema de
la criba de Eratostenes, para determinar los numeros primos, de hecho fija los lugares donde
se ubican, independientemente del nombre que tengan.

Si un numero se factoriza en n primos en una base, su equivalente en la otra base se fac-


torizara en el mismo numero de primos. En general, todas las propiedades relativas a la
factoriabilidad son independientes de la base, pero los nombres de los numeros, es decir la
sucesion de dgitos que lo representa, va a cambiar al usar otra base. Por ejemplo, sucede con
frecuencia que un decimal es periodico en una base y finito en otra, lo que muestra que la
periodicidad o finitud de un decimal no es propiedad esencial del numero.

En lo que sigue desarrollaremos, a traves de ejemplos, los metodos para cambiar de bases.
Siempre usaremos la base denaria y la base 6, que la llamaremos 6 dedos. Los descom-
pondremos en cuatro casos: 1) natural a natural, 2) racional a racional, 3) decimal finito a
decimal, 4) decimal periodico a decimal.

1. Natural a natural.

a) De 6 dedos a denarios.
23 051 = (2 64 + 3 63 + 0 62 + 5 6 + 1) = 3 271
b) De denarios a 6 dedos.
Estudiaremos primero el caso mas general , de base 10 a base b. Dado un natural
A que en base b queda representado por Ab = an an1 a0 se tiene que su numero
en denario es:

A10 = an bn + an1 bn1 + a1 b + a0 = b(an bn1 + an1 bn2 + + a1 ) + a0


198

de donde obtenemos que a0 es el resto de la division de A por b.


Sea q el cuociente de la division. Entonces:
q = an bn1 + an1 bn2 + + a1 = b(an bn2 + + a2 ) + a1
que nos muestra que a1 es el resto de la division de q por b.

Continuando con este proceso hasta conseguir un cuociente 0, habremos determi-


nado todos los coefientes ak .

Ejemplo: Expresar 5 873 en 6 dedos

5 873 = 6 978 + 5 ; a0 =5
978 = 6 163 + 0 ; a1 =0
163 = 6 27 + 1 ; a2 =1
27 = 64+3 ; a3 =3
4 = 60+4 ; a4 =4
Luego, 5 87310 = 43 1056 . (Los numeros pequenos indican la base).

Notemos que los dgitos se obtienen en orden inverso al sistema de escritura.

2. Racional a racional.
Estos casos se remiten a los casos anteriores, cambiando de base tanto el numerador
como el denominador. Veremos un ejemplo, el cual, ademas, iluminara otros aspectos.
Ejemplo: Por lo anterior, podemos determinar que el denario 72/503 corresponde al 6
dedos 200/2 155. Podra pensarse que, en este ultimo, fuera posible una simplificacion,
pero esto queda descartado porque el denario ya era irreducible.

3. Decimal finito a decimal.

a) De 6 dedos a denario.
Ejemplo: 0, 512 = 5/6 + 1/62 + 2/63 = 47/54 = 0, 8703
b) De denario a 6 dedos.
Ejemplo: Expresar el denario 0, 35 en base 6 dedos.
a1 a2 a3
0, 35 = + 2 + 3 +
6 6 6
Amplificando por 6, obtenemos
a2 a3
2, 1 = a1 + + 2 +
6 6
lo que nos muestra que a1 = 2. Restando, y amplificando nuevamente por 6:
a3 a4
0, 6 = a2 + + 2 +
6 6
de la cual obtenemos que a2 = 0. Repitiendo el proceso:
a4 a5
3, 6 = a3 + + 2 +
6 6
199

lo que implica que a3 = 3. Al restar, nos aparece la ecuacion 0, 6 = a4 /6 + a5 /62 +


, la cual es la del paso anterior. Por lo tanto, 3 = a4 = a5 = . Entonces,
0, 3510 = 0, 2036 .

4. Periodico a decimal.

a) Ejemplo 1: Convertir el 6 dedos 0, 32 a base denaria. Haremos uso del teorema 7.4
de la seccion anterior.

0, 326 = 32/(102 1)6 = 32/556 = 20/3510 = 0, 57142810

Ejemplo 2: Convertir el 6 dedos 0, 4 a base denaria.

0, 46 = 4/(10 1)6 = 4/56 = 4/510 = 0, 810

Estos dos ejemplos nos ilustran que existen decimales periodicos que permanecen
periodicos al cambiar de base y otros, a los cuales el cambio los convierte en finitos.

b) Ejemplo: Expresar el denario 0, 46 en base 6 dedos.

0, 4610 = 7/1510 = 11/236

Ahora deberamos hacer esta division como numeros de base 6. Para esto, tendre-
mos que desarrollar las tablas de multiplicar de ese campo. Le dejaremos al lector
esta tarea. El resultado es 0, 24.

En la seccion anterior hemos demostrado el teorema Si (b, 10) = 1, entonces b | (10(b)


1).

Este teorema, obviamente, rige usando cualquier base; por lo tanto, traduciendo la base 6
dedos en denaria, dira Si b es primo con 6, entonces b es un divisor de 6(b) 1. Generalizado
a cualquier base, nos queda:

Teorema 7.7
Si b es primo con q, entonces b es un divisor de q (b) 1.

El caso particular en que b es un numero primo y, ademas, (b, q) = 1 se conoce como


Pequeno Teorema de Fermat. El dice que, en estas condiciones, b es un divisor de q b1 1.

Para concluir esta seccion, estudiaremos un caso en que la base es negativa. Elegiremos
como b al numero -10 y como dgito, a los corrientes (0, 1, . . . , 9).

El nombre de estas bases se construye anteponiendolo nega al nombre que tiene la base
cuando b es positivo. En este caso su nombre sera negadecimal o, tambien, negadenaria.

Veamos primero el paso de un numero negadecimal a denario.


200

Ejemplo 7.4
Expresar en base denaria al numero 357 042, 3810 .

Este numero corresponde al denario

3(10)5 + 5(10)4 + 7(10)3 + 0(10)2 + 4(10)1 + 2 + 3/ 10 + 8/(10)2


= 300 000 + 50 000 7 000 40 + 2 3/10 + 8/100
= 257 038, 22

Para pasar de denario a negadecimal, debemos dividir por -10, de modo tal que los restos
sean dgitos.

Ejemplo 7.5
Expresar el denario 13 659 en base negadecimal.

1. 13 659 = (10)(1 365) + 9


2. 1 365 = (10)137 + 5
3. 137 = (10)(13) + 7
4. 13 = (10) 2 + 7
5. 2 = (10) 0 + 2

Luego, 13 65910 = 27 75910

En los sistemas numericos de este tipo no es necesario usar el signo menos, lo cual puede
representar un simplificacion, pero los algoritmos para efectuar las operaciones aritmeticas
resultan complejos.

Ejemplo 7.6
Expresar el denario -457 en base negadecimal.

1. 457 = (10)46 + 3
2. 46 = (10)(4) + 6
3. 4 = (10)(1) + 6
4. 1 = (10) 0 + 1

por lo tanto,45710 = 166310

Otros aspectos de este tema apareceran en los ejercicios.

7.3. SUCESIONES DE FAREY


Definicion 7.1
Sea n un natural. Se define la sucesion de Farey de orden n, que anotaremos por F (n), como
la sucesion formada por todos los racionales irreductibles del intervalo [0, 1], ordenados de
menor a mayor, y cuyos denominadores no excedan a n.
201

Ejemplos

F (1) : 0, 1
F (2) : 0, 1/2, 1
F (3) : 0, 1/3, 1/2, 2/3, 1
..
.
F (7) : 0, 1/7, 1/6, 1/5, 1/4, 2/7, 1/3, 2/5
3/7, 1/2, 4/7, 3/5, 2/3, 5/7, 3/4, 4/5
5/6, 6/7, 1
Observemos que, en estos ejemplos, si a/b y c/d son 2 racionales sucesivos en alguna sucesion,
se cumple que bc ad = 1. La llamaremos la propiedad del producto cruzado. Si anotamos el
0 como 0/1 y el 1 como 1/1, esta propiedad tambien se cumple en los extremos. Probaremos
esta afirmacion para cualquier F (n), pero necesitaremos algunos resultados preliminares que
se establecera en los Teoremas 7.8 y 7.9.

En lo que sigue, las letras representan numeros naturales.

Teorema 7.8
Sean a/b y c/d dos fracciones no necesariamente irreductibles y tales que a/b < c/d. Entonces,
a a+c c
< <
b b+d d

Demostracion
a c
< es equivalente con ad < bc. Sumando ab y factorizando
b d
a(b + d) < b(a + c) de donde
a a+c
<
b b+d
En forma analoga se prueba la desigualdad restante.

Corolario
Sean p y q positivos. Entonces, se cumple que
a pa + qc c
< <
b pb + qd d

Demostracion
Basta aplicar el teorema a las fracciones pa/pb y qc/qd.

Definicion 7.2
Dados a/b y c/d, llamaremos su medianta al racional (a + c)/(b + d).
a c
Es inmediato, del teorema anterior, que si y son dos racionales sucesivos en una sucesion
b d
de Farey, entonces, su medianta no pertenece a dicha sucesion, ya que esta los separara.
202

Teorema 7.9
a x c
Sea < < .
b y d
Entonces existen p y q tales que

x pa + qc
=
y pb + qd

Demostracion
Sea p = cy dx y q = bx ay.

Es inmediato que p y q son naturales. Resolviendo el sistema para x e y obtenemos

ap + cq bp + dq
x= , y=
bc ad bc ad
de donde

x ap + cq
=
y bp + dq

De este teorema y del corolario anterior obtenemos que:

a x c
< <
b y d
si y solo si existen p y q naturales positivos tales que

x ap + cq
=
y bp + dq

Teorema 7.10
a c
Sean < y tales que bc ad = 1. Entonces su medianta hereda esta propiedad, es decir, la
b d
diferencia de los productos cruzados entre ella y cualquiera de sus padres tambien es 1.

Demostracion
Sabemos que

a a+c c
< <
b b+d d
Entonces
b(a + c) a(b + d) = bc ad = 1

y tambien
c(b + d) d(a + c) = bc ad = 1

Ahora estamos en condiciones de demostrar la propiedad central de estas sucesiones.


203

Teorema 7.11
a c
Si y son dos fracciones sucesivas de F (n), entonces, bc ad = 1.
b d
Demostracion
Por induccion. La idea general consistira en probar que las nuevas fracciones que aparecen en
F (n + 1) son solamente mediantas, de denominador n + 1, de parejas de F (n).

Por inspeccion, podemos comprobar que vale para F (1). Supongamos que se cumple para
F (n).
a c
Sean y dos sucesiones sucesivas de F (n), que aparecen separadas en F (n + 1). Podemos
b d
a c x
suponer que < y sea la fraccion que las separa. Probaremos que esta fraccion es su
b d y
medianta. Por lo anterior, sabemos que x/y es de la forma (ap + cq)/bp + dq para algun
p y q naturales. Por ser una fraccion de F (n + 1) y no de F (n), su denominador debe ser
n+1. Pero sabemos que b+d > n. La unica alternativa viable es que p = q = 1 y b+d = n+1.

Por lo tanto,

x a+b
=
y b+d
con lo cual se prueba que la fraccion que las separa es su medianta. Pero, por el teorema
anterior, sabemos que cumple con la propiedad del producto cruzado, con lo cual termina la
demostracion por induccion.
Estas propiedades nos permiten construir F (n + 1) a partir de F (n), simplemente agregandole
a F (n) las mediantas cuyos denominadores sean n + 1.
Ejemplo 7.7
Construccion de F (8) a partir de F (7).

F (7) : 0/1, 1/7, 1/6, 1/5, 1/4, 2/7, 1/3, 2/5, 3/7, 1/2,
4/7, 3/5, 2/3, 5/7, 3/4, 4/5, 5/6, 6/7, 1/1
Debemos intercalar las mediantas de las fracciones vecinas cuyos denominadores sumen 8.

Entre 0/1 y 1/7 va 1/8. Entre 1/3 y 2/5 se ubica 3/8. Entre 3/5 y 2/3 colocamos 5/8. Y,
finalmente, 7/8 entre 6/7 y 1/1.

F (8) : 0/1, 1/8, 1/7, 1/6, 1/5, 1/4, 2/7, 1/3, 3/8,
2/5, 3/7, 1/2, 4/7, 3/5, 5/8, 2/3, 5/7, 3/4,
4/5, 5/6, 6/7, 7/8, 1/1
notemos que el numero de fracciones que se intercalan en F (n) para formar F (n + 1) corres-
ponde a la cantidad de naturales menores a n + 1 y primos con este.

Una buena aproximacion al numero de fracciones que forman a F (n) viene dado por
3n2 / 2 ,
resultado que dejaremos sin prueba. Sirve solo para valores grandes de n. Por ejem-
plo, F (1000) tiene aproximadamente 303 000 fracciones.
204

Una aplicacion de estas sucesiones se encuentra en solucionar la ecuacion diofantica ax+by = 1


con a y b primos entre s.

Supongamos que b > a. Entonces la fraccion a/b se encuentra en todas las sucesiones de F (b)
en adelante.

l a s
Sean < < sus vecinos en alguna de ellas. Por la propiedad del producto cruzado
m b t
tenemos que

ma lb = 1 y sb at = 1
Por lo tanto, la ecuacion ax + by = 1 con (a, b) = 1 tiene, por lo menos, las soluciones x = m;
y = l; x = t; y = s.

Nota
Es facil probar que si x0 , y0 es una solucion de la ecuacion ax + by = 1, entonces todas su
soluciones vienen dadas por

x = x0 + bt ; y = y0 at

Como otra aplicacion, consideremos el siguiente teorema, que nos dice que no es necesario
denominadores muy grandes para obtener una buena aproximacion a un irracional por un
racional.

Teorema 7.12
Sea (0, 1) un numero irracional y N un natural. Entonces, existe un racional x/y con
y N , tal que

x 1

y (N + 1)y

Demostracion
En F(N) existen dos racionales sucesivos a/b y c/d, tales que
a c
<<
b d
considerando la medianta, se debe cumplir que solamente una, de las 2 desigualdades que
siguen, es verdadera.
a a+c a+c c
<< o <<
b b+d b+d d
Notemos que b + d N + 1.

En el primer caso se tiene que


a a+c a 1 1
0< < =
b b+d b b(b + d) (N + 1)b
En forma analoga, en el segundo caso se obtiene
205

c 1
<
d (N + 1)d
x
Eligiendo para el racional para el cual la desigualdad es verdadera, se obtiene
y

x
< 1
y (N + 1)y

Ejemplo 7.8
Sea los decimales de , es decir, = 3 + y elijamos para N el valor 10.

Se tiene que en F (10), 1/8 < < 1/7. Tomando la medianta y calculando se llega
2 1
<<
15 7
Luego, por el teorema anterior
1 1
<
7 77
de donde, sumando y restando 3
22 1
<
7 77
lo que nos muestra que la aproximacion de Arqumides, para el numero es bastante buena.

2
Mas aun, como en F (15) aun tenemos que 15 < < 17 , se cumple, haciendo N = 15, que

22 1
<
7 112
206

7.4. EJERCICIOS
22 1
1. Determine el mayor N , tal que < .
7 7(N + 1)
2. Para el numero e (e = 2, 7182818284 . . .) determinar el racional que mejor lo aproxima
y cuyo denominador sea menor que 18.

3. Expresar 18.995 en base 12 y negadecimal.

4. En F(200) hay un unico numero que separa a 133/200 de 133/199. Cual es?
5
5. Encuentre el numero que sigue a 6 en F (20).

6. Cual es la base b (b un natural), tal que 1 000b corresponde al denario 1 331.

7. En el sistema seis dedos, indique los numeros cuyo periodo es 3.

8. Explique el siguiente fenomeno: el numero 142 857, al ser multiplicado por cualquiera
de los dgitos 2, 3, 4, 5, 6 da como resultado una rotacion cclica de sus dgitos.

9. Escribir los siguientes numeros como racionales:


a)0, 45 b)0, 4554 c)4, 54
d)0, 9

10. Escriba las siguientes fracciones como decimales periodicos: 1/13, 2/13, 7/55, 11/25.

11. Calcule (41), (271), (239).

12. Calcule (15) y (21) y verifique que 15 no divide a 10(15) 1, pero 21 | 10(21) 1.

13. Pruebe el siguiente teorema, que claramente es mas fuerte que el Teorema
7.12. Sea
x 1
un irracional. Pruebe que hay infinitos racionales x/y, tales que < 2 .
y 2y
Sugerencia: En F (N ) existen ab y dc , tales que ab < < dc . Suponga que a
b 1
2b2
y
que dc 2d12 . Sume y llegue a una contradiccion.
207

7.5. SOLUCIONES
1. Resp.: N=111. Esto nos demuestra que la aproximacion de Arqumides para es menor
que 1/784.

2. Resp.: 19/7

3. Resp.: ABAB; 21015.

4. Resp.: 2/3.

5. En F(6), el siguiente a 5/6 es 1/1. En F(7) es 6/7. En F(13) es 11/13. En F(19) es 16/19
que se mantiene hasta F(24).

6. Resp.: 11

7. Resp.: Son los divisores de 555 que no son divisores de 55 o de 5. Ellos son 111 y 555.
El 5 tiene periodo 1.

8. 1/7=0,142857. por lo tanto,2/7, 3/7, etc. seran rotaciones cclicas de los decimales.
54 50
9. Resp.: c) 4 + 99 = 11 . d) 1.

10. Resp.: 0, 076923, 0, 153846, 0, 1272, 0,44.

11. Respuestas: (41) = 5, (271) = 5, (239) = 7

12. Resp.: (15) = 8 y (21) = 12.

13. Se puede suponer que b 6= d. En la suma de las dos supuestas desigualdades se aplica la
propiedad del producto cruzado, de donde se concluye que (b d)2 < 0.
208
Captulo 8

DESIGUALDADES, VALOR
ABSOLUTO, INECUACIONES

8.1. EJEMPLOS INTRODUCTORIOS


En este captulo profundizaremos el estudio de las desigualdades cuyos fundamentos fue-
ron presentados en el captulo 6, aplicandolas ademas, a la resolucion de inecuaciones.

Aprovecharemos tambien de desarrollar el concepto de valor absoluto en vista que sus pro-
piedades mas interesantes se expresan mediante desigualdades.

Empezaremos con algunos ejemplos que muestran metodos para establecer desigualdades.
En general, toda la operatoria con desigualdades esta empapada de triquinuelas algebraicas.
Algunos de los recursos mas usados son los siguientes: El producto de numeros positivos es
positivo; el cuadrado de un numero es no negativo; si a b y c > 0 entonces a c < b; si
a + b c y p < b entonces a + p < c; etc.

En los seis ejemplos que vienen a continuacion se mostraran algunos de los medios mas usados
en este tema.

Ejemplo 8.1 En este ejemplo se usa el recurso de sumar a una desigualdad una misma
cantidad.

Sean a y b numeros reales distintos. Entonces 4ab < (a + b)2 .

Es facil probar que 4ab = (a + b)2 (a b)2 . Sumandole, a la desigualdad 0 > (a b)2 la
expresion (a + b)2 se obtiene el resultado.

El siguiente ejemplo usa la propiedad que el producto de positivos es positivo.

Ejemplo 8.2
Sea a > 1 y 0 < b < 1. Entonces a + b > 1 + ab.

De la hipotesis tenemos que (a 1) > 0 y (1 b) > 0. Luego, (a 1)(1 b) > 0, donde,


multiplicando se obtiene que a 1 ab + b > 0. Sumandole a ambos miembros 1 + ab logramos

209
210

el resultado pedido.
1 1
En particular, si b = tenemos que a + > 2. Este resultado se usara posteriormente.
a a
El siguiente ejemplo ilustra el uso de la induccion para establecer desigualdades. Ademas,
ocupa el hecho que x + b > x si b es positivo, que se pruebe de inmediato al sumarle x a la
desigualdad b > 0.

Ejemplo 8.3
Sean a1 , a2 , , an reales positivos. Entonces
n
X
(1 + a1 )(1 + a2 ) (1 + an ) > 1 + ai .
i=1

La demostracion la hacemos por induccion. Para n = 2 tenemos que

(1 + a1 )(1 + a2 ) = 1 + a1 + a2 + a1 a2 > 1 + a1 + a2
ya que a1 a2 > 0. Luego, la afirmacion es valida para n = 2. Supongamosla valida para n,
entonces
n
X
(1 + a1 )(1 + a2 ) (1 + an )(1 + an+1 ) > (1 + ai )(1 + an+1 )
i=1

n
X Xn n+1
X
=1+ ai + an+1 + an+1 ( ai ) > 1 + ai
i=1 i=1 i=1
n
X
ya que an+1 ( ai ) > 0. Luego, la desigualdad es valida.
i=1

Ejemplo 8.4
Sean a1 , a2 , , an reales positivos, todos menores que 1. Entonces
n
X
(1 a1 )(1 a2 ) (1 an ) < 1/(1 + ai ).
1

Como 0 < ai < 1, es inmediato que 0 < < 1 que multiplicada por 1 nos da 1 < a2i < 0.
a2i
2
Sumandole 1 obtenemos 0 < 1 ai < 1 de donde concluimos que 0 < 1 + ai < 1/(1 ai ).

Efectuando todos los productos podemos escribir, por 17) del teorema 6.2, que:
1
(1 + a1 )(1 + a2 ) (1 + an ) <
(1 a1 )(1 a2 ) (1 an )
Del ejemplo 8.3 y la transitividad obtenemos
n
X 1
1+ ai <
(1 a1 )(1 a2 ) (1 an )
i=1

y finalmente por 13) del mismo teorema


211

n
X
(1 a1 )(1 a2 ) (1 an ) < 1/(1 + ai )
i=1

Dejamos al lector las siguientes dos desigualdades:


Pn
(1 a1 )(1 a2 ) (1 an ) > 1 k=1 ak si ak < 1
Pn Pn
(1 + a1 )(1 + a2 ) (1 + an ) < 1/ ( k=1 ak ) si k=1 ak <1

Estas desigualdades, mas las de los ejemplos 8.3 y 8.4, se conocen como desigualdades de
Weierstrass.

El siguiente ejemplo ilustra el metodo, que podramos llamar de las equivalencias, para
probar desigualdades. Consiste en ir transformando la desigualdad que se pide verificar en
otras equivalentes, hasta conseguir una conocida.

Ejemplo 8.5
Probar que

(b + c a)2 + (c + a b)2 + (a + b c)2 bc + ca + ab.


Desarrollando y pasando al primer miembro vemos que es equivalente a

3a2 + 3b2 + 3c2 2ab 2ac 2bc 0


que equivale a

(a b)2 + (a c)2 + (b c)2 + a2 + b2 + c2 0


Pero esta ultima es verdadera porque cada sumando, al ser un cuadrado, es no negativo.

Otro ejemplo del mismo tipo.

Ejemplo 8.6
Sean a, b, x numeros positivos. Entonces,
a a+x
< si y solo si a < b.
b b+x

a < b ax < bx ab + ax < ab + bx a(b + x)

a a+x
< b(a + x) < .
b b+x
a a+1
En particular, tomando x = 1, tenemos que a < b < , que sera aplicado en el
b b+1
ejemplo siguiente.

El siguiente ejemplo es un tanto complejo.


212

Ejemplo 8.7
1 3 5 2n 1
Sea L = .
2 4 6 2n
1 1
Probar que <L< .
2 n+1 2n + 1
Por el ejemplo anterior tenemos que

1 2 3 4 2n 1 2n
< ; < ; ; < .
2 3 4 5 2n 2n 1
Entonces,

2 4 2n
L< .
3 5 2n + 1
Multiplicando por L tenemos que

1 3 2n 1 2 4 2n 1
L2 < =
2 4 2n 3 5 2n + 1 2n + 1
De donde sale que

1
L< .
2n + 1
Por otra parte

1 3 5 2n 1 3 5 2n + 1
(2n + 1)L = (2n + 1) =
2 4 6 2n 2 4 2n
pero, por el ejemplo anterior aplicado a los inversos tenemos que

3 4 5 6 2n + 1 2n + 2
> ; > , > .
2 3 4 5 2n 2n + 1
Elevando al cuadrado la igualdad anterior y reemplazando nos queda que:

3 5 2n + 1 4 3 2n + 2
(2n + 1)2 L2 >
2 4 2n 3 5 2n + 1
de donde sale que

(2n + 1)2 L2 > n + 1

y finalmente

n+1 n+1 1
L> > = .
2n + 1 2n + 2 2n + 1
Por lo tanto

1 1
<L<
2 n+1 2n + 1
213

8.2. ALGUNAS DESIGUALDADES IMPORTANTES


Teorema 8.1
Sean a y b numeros reales no negativos y n un numero natural.
Entonces:

a < b si y solo si an < bn .

Demostracion
Primero demostraremos por induccion que a < b an < bn .

La proposicion es obviamente verdadera si n = 1. Supongamos que a < b an < bn , debemos


probar que a < b an+1 < bn+1 .

Sea a < b. Por la hipotesis inductiva tenemos que an < bn .

Multiplicando estas dos desigualdades se obtiene que an+1 < bn+1 , lo que termina la induccion.

A continuacion demostraremos que:

an bn a < b.
Sea an < bn .

Por la tricotoma tenemos que una y solo una de las siguientes afirmaciones es verdadera

a < b o a = b o a > b
si a = b es obvio que an = bn .

Si a > b, por la primera parte de la demostracion se tendra que an > bn . En ambos casos se
contradice la hipotesis que an < bn . Luego, a < b.

Corolario

n
n
Sean a y b no negativos y n un natural. Entonces a < b si y solo si a< b.

Demostracion

n
a < n b si y solo si ( n a)n < ( n a)n si y solo si a < b.

Nota
Este teorema y su corolario tambien valen si < se reemplaza por .

Teorema 8.2 Desigualdad de Bernoulli

Sea a > 1 y n un numero natural. Entoces (1 + a)n 1 + na.


214

Demostracion
Por induccion. La proposicion es obvia si n = 1. Supongamos valida la desigualdad para n,
es decir: (1 + a)n 1 + na. Multipliquemos esta desigualdad por (1 + a), que es un numero
mayor que cero. Tenemos:

(1 + a)n+1 (1 + na)(1 + a)
Luego,

(1 + a)n+1 1 + na + a + na2
Como na2 es mayor o igual que cero, se cumple:

(1 + a)n+1 1 + (n + 1)a.

Nota
En forma analoga a este teorema se puede demostrar que:

1
a) (1 + a)n 1 + na + n(n 1)a2 si a 0
2
1
b) (1 + a)n 1 + na + n(n 1)a2 si 1 a 0.
2
A continuacion vamos a establecer uno de los resultados mas usado en desigualdades. Lo
llamaremos la Desigualdad de los Promedios. Necesitaremos, previamente, algunas defini-
ciones.

Definicion 8.1
Sean a1 , a2 , , an numeros reales positivos.

Definimos su promedio cuadratico y lo designaremos por C, al valor


r
a1 2 + a2 2 + + an 2
C=
n
Definimos su promedio aritmetico y lo designaremos por A, al valor
a1 + a2 + + an
A=
n
Definimos su promedio geometrico y lo designaremos por G, al valor

G= n
a1 a2 an
Definimos su promedio armonico y lo designaremos por H, al valor
n
H= 1 1 1
a1 + a2 + + an

Teorema 8.3
Sean los ai no todos iguales. Entonces C > A > G > H.
215

La demostracion la haremos separadamente para cada desigualdad.

Demostracion de C > A
Probaremos la desigualdad a partir de (a1 A)2 + (a2 A)2 + + (an A)2 > 0. Ella es
obviamente verdadera, ya que por lo menos un sumando es distinto de 0. Desarrollando y
reordenando se obtiene que

a1 2 + a2 2 + + an 2 > 2A(a1 + a2 + + an ) nA2


Pero a1 + a2 + + an = nA
Reemplazando y simplificando se obtiene

a21 + a22 + + a2n


> A2
n
Extrayendo la raz, obtenemos C > A

Demostracion de A > G
Necesitaremos un resultado previo, que lo establecemos en el lema siguiente.

LEMA . Sean a1 , a2 , , an numeros positivos no todos iguales a 1 y tales que

a1 a2 an = 1.
Entonces a1 + a2 + + an > n.

Demostracion
Por induccion. Para n = 1 la hipotesis es falsa, luego el teorema vale. Aunque esta forma de
proceder es correcta vamos a demostrar el teorema para el caso n = 2, aunque este paso es
innecesario.

Sea a1 > 0, a2 > 0, tales que a1 a2 = 1 y a1 6= 1.


Entonces a2 6= 1 y como a1 a2 = 1 podemos suponer que a1 > 1 y a2 < 1. Entonces, por el
ejemplo 1, se tiene que a1 + a2 > 2.

Supongamos el teorema probado para cualquier coleccion de n numeros que cumpla con la
hipotesis y sean a1 , a2 , , an , an+1 , (n + 1) numeros que la satisfacen. Por las condiciones de
ella debe haber alguno que sea mayor que 1 y otro menor que 1. Sean a1 y a2 tales numeros.
Sea a = a1 a2 .Luego, a; a3 ; ; an+1 , son n numeros que satisfacen la hipotesis o todos son
iguales a 1. En ambos casos se tiene que:

a + a3 + + an+1 n.
Pero por el ejemplo 8.2 tenemos que

a = a1 a2 < a1 + a2 1
e introduciendo esta desigualdad en la anterior nos queda que

a1 + a2 + + an+1 > n + 1.
216

que concluye la demostracion del Lema.

Ahora podremos probar que A > G.


ai
Definimos bi = para i = 1, 2, , n.
G
Entonces los bi son positivos y no todos iguales y ademas

b1 b2 bn = 1.
Por el lema anterior, tenemos que

b1 + b2 + + bn > n.
Luego,
a1 + a2 + + an
>n
G
de donde obtenemos finalmente

A > G.

Demostracionde G > H
Aplicando el teorema anterior a los numeros

1 1 1
, , , ,
a1 a2 an
tenemos que
1 1 1
+ +
r
a1 + a2 an n 1 1 1 1
> =
n a1 a2 an n a1 a2 an
reordenando
n
n
a1 a2 an > 1 1 1
a1 + a2 + + an

es decir, G > H.

Ejemplo 8.8
Sean a, b, c 0. Probar que (a + b + c)(bc + ca + ab) 9abc.

El promedio aritmetico de a, b, c es mayor o igual que el geometrico.

Luego,

a+b+c 3
abc
3
Tomando el medio aritmetico y geometrico de bc, ca y ab, tenemos que
217

bc + ca + ab p
3 (bc)(ca)(ab)
3
Multiplicando miembro a miembro las dos desigualdades obtenemos

(a + b + c)(bc + ca + ab)
3
a3 b3 c3
9
Luego,

(a + b + c)(bc + ca + ab) 9abc

Ejemplo 8.9
Sea n N. Probar que nn > 1 3 5 (2n 1).

Comparando los promedios aritmetico y geometrico de 1, 3, 5, , (2n 1), tenemos

1 + 3 + 5 + (2n 1) p
n 1 3 5 (2n 1)
n
Pero

1 + 3 + 5 + + (2n 1) = n2 .
Luego,

n2 p
n 1 3 5 (2n 1)
n
Por el teorema 8.1, elevando a la potencia n

nn 1 3 5 (2n 1)
que es el resultado pedido.

El ejemplo que continua es de interes teorico. Muestra que la sucesion que aparece es estric-
tamente creciente, resultado que sera usado posteriormente.

Ejemplo 8.10
1 n
   n+1
1
Sea n N entonces: 1 + < 1+ .
n n+1
1
Sea a1 = 1, a2 = a3 = = an+1 = 1 + . Comparemos los promedios aritmeticos y
n
geometricos de a1 , a2 , , an+1 . Tenemos que

1 + n(1 + n1 )
r
n+1 1
> (1 + )n
n+1 n
que equivale a:
r
1 n+1 1 n
1+ > (1 + )
n+1 n
218

Luego, elevando a la n + 1-esima potencia.


n+1
1 n
  
1
1+ > 1+
n+1 n

Ejemplo 8.11
Si a, b, c son numeros positivos y distintos, demostrar que:

ab bc ca 1
+ + < (1 + b + c).
a+b b+c c+a 2
El promedio aritmetico de a y b es mayor o igual que el armonico. Luego,

a+b 2ab
>
2 a+b
Analogamente

b+c 2bc
>
2 b+c
a+c 2ac
>
2 a+c
Sumando miembro a miembro las tres desigualdades
 
ab bc ca
a+b+c>2 + +
a+b b+c c+a
Luego,

1 ab bc ca
(a + b + c) > + +
2 a+b b+c c+a

El siguiente teorema involucra a dos sucesiones de numeros positivos. Se conoce con el


nombre de Desigualdad del Reordenamiento o de La Palanca.

Teorema 8.4 Desigualdad del Reordenamiento

Sean a1 a2 an y b1 b2 bn reales positivos y sea una permutacion de los


ndices. Entonces

a1 bn + a2 bn1 + + an b1 a1 b(1) + a2 b(2) + + an b(n) a1 b1 + c2 b2 + + an bn

En palabras: Una vez ubicados los ai en orden creciente, el menor valor se obtiene cuando los
bj se ubican en forma decreciente y el mayor valor cuando van en orden creciente.

Demostracion
219

Vamos a probar que si se intercambian bi con bj , el valor de la suma aumenta si el menor de


ellos queda a la izquierda del mayor y disminuye en caso contrario.

Sea S = a1 b(1) + + al b(l) + + ak b(k) + + an b(n) con l < k.

Sea S 0 la sumatoria que se obtiene al intercambiar b(l) con b(k) .

S 0 = a1 b(1) + + al b(k) + ak b(l) + + an b(n)


S S 0 = al b(l) ak b(l) + ak b(k) al b(k)
= (ak al )(b(k) b(l) )
Entonces, S > S 0 si b(k) > b(l) y S < S 0 si b(k) < b(l) .

Por lo tanto, el menor valor de la suma se produce cuando no existen intercambios que lo
disminuyan. Esto ocurre cuando

S = a1 bn + a2 b( n 1) + + an b1 .
El mayor valor se produce cuando no hay intercambios que lo aumenten. Esto ocurre cuando

S = a1 b1 + a2 b2 + + an bn .

Esta desigualdad tiene una interpretacion fsica interesante. Supongamos que a1 , a2 , . . . , an


son que posiciones fijas en el brazo de una palanca; y b1 , b2 , . . . , bn son pesas que se ubicaran
en esas posiciones, como indica la figura.

a1 a2 an a1 a2 an
... ...
? ?
b1 b1
?
b2 ?
? ? bn1
bn bn
Mayor torque Menor torque.
Figura 7.1

Entonces, el mayor torque se produce segun el diagrama A y el menor, segun B.

Como una aplicacion, probaremos la desigualdad conocida como Desigualdad de Tchebys-


hev.

Teorema 8.5
Sea a1 a2 an y b1 b2 bn reales positivos. Entonces

a1 + a2 + + an b1 + b2 + + bn a1 b1 + a2 b2 + + an bn

n n n

Demostracion
220

Sea a1 b1 + a2 b2 + + an bn = S
Entonces a1 b2 + a2 b3 + + an b1 S
a1 b3 + a2 b4 + + an b2 S
..
.
a1 bn + a2 b1 + + an bn1 S

Sumando y factorizando:

(a1 + a2 + + an )(b1 + b2 + + bn ) nS
Dividiendo por n2

(a1 + a2 + + an ) (b1 + b2 + + bn ) a1 b1 + a2 b2 + + an bn

n n n

En forma totalmente analoga se prueba que:

Si a1 a2 an y b1 b2 bn

Entonces

a1 + a2 + + an b1 + b2 + + bn a1 b1 + a2 b2 + + an bn

n n n

Ejemplo 8.12
Sean a, b, c numeros naturales y sea d = a + b + c. Entonces

1a + 2a + + na 1b + 2b + + nb 1c + 2c + + nc 1d + 2 d + + nd
<
n n n n
Por lo anterior tenemos que

1a + 2a + + na 1b + 2b + + nb 1a+b + 2a+b + + na+b


<
n n n
Luego, el primer miembro de la desigualdad que se pretende probar, es menor que

1a+b + 2a+b + + na+b 1c + 2c + + nc



n n
Aplicando, de nuevo el teorema 8.5, se obtiene el resultado.

El siguiente ejemplo puede considerarse como un teorema.

Ejemplo 8.13
Sea a1 , a2 , , an reales positivos no todos iguales y m un numero natural. Entonces

a1 + a2 + + an m am + am m
 
2 + + an
< 1
n n
Para su demostracion, podemos suponer que a1 a2 an . Entonces el resultado se
logra de inmediato por induccion, aplicando la desigualdad de Tchebychev y empezando con
m = 2.
221

Ejemplo 8.14
Probar que, si a, b y c son positivos y no todos iguales, entonces a2 (b+c)+b2 (c+a)+c2 (a+b) <
2(a3 + b3 + c3 ).

Podemos suponer que a b c. Entonces

a + b + c a2 + b2 + c2 a3 + b3 + c3
<
3 3 3
que es equivalente a (a + b + c)(a2 + b2 + c2 ) < 3(a3 + b3 + c3 ); que al multiplicar da el resultado
pedido.
222

Cuadro Resumen de las desigualdades mas importantes que hemos presentado


1. Si a 6= 0, entonces a2 > 0
2. 0 < a < b y n racional, entonces an < bn
3. Desigualdad de Bernoulli
(1 + a)n 1 + na Con a > 1 y n natural
4. Desigualdades de Weiertrasse
n
X
a) (1 + a1 )(1 + a2 ) (1 + an ) > 1 + ai . Con ai > 0.
i=1
n
X
b) (1 a1 )(1 a2 ) (1 an ) < 1/(1 + ai ). Con 0 < ai < 1.
1
Pn
c) (1 a1 )(1 a2 ) (1 an ) > 1 k=1 ak , si ak < 1

d ) (1 + a1 )(1 + a2 ) (1 + an ) < 1/ ( nk=1 ak ), si nk=1 ak < 1


P P

5. Desigualdad de los Promedios


CAGH
donde
r
a21 + a22 + + a2n a1 + a2 + + an
C= A=
n nn

G = n a1 a2 . . . an H= 1 1 1
a1 + a2 + + an

6. Desigualdad del Reordenamiento


a1 bn + a2 bn1 + + an b1 a1 b(1) + a2 b(2) + + an b(n) a1 b1 + c2 b2 + + an bn

con los ai ybi positivos y ordenados en forma creciente.


7. Desigualdad de Tchebyshev

a1 + a2 + + an b1 + b2 + + bn a1 b1 + a2 b2 + + an bn

n n n

con los ai y bi ordenados en forma creciente o decreciente.


8. Desigualdad de Cauchy-Schwarz

n
!2 n
! n
!
X X X
ar br a2r b2r
r=1 r=1 r=1

9. Desigualdad Triangular

|a + b| |a| + |b|
Estas dos ultimas las estableceremos en la siguiente seccion.
223

Para facilitar la memorizacion algunas formulas que incluyen sumatorias o sucesiones de


productos se pueden escribir omitiendo los lmites y los subndices, dandose por subentendidos.
Veremos como ejemplos algunas expresiones de la pagina anterio.

4.a (1 + a) > 1 + a. Con a > 0

4.c (1 a) > 1 a. Con 0 < a < 1

7. Desigualdad de Tchebyshev.
a b ab
, con a y b en orden creciente o decreciente.
n n n
8. Desigualdad de Cauchy-Schwarz.
(ab)2 (a2 )(b2 )
224

8.2.1. EJERCICIOS
1. Sean a, b y c numeros reales, positivos y distintos. Probar que:

a) a3 b + ab3 < a4 + b4
b) (a + b + c)(ab + bc + ca) > 9abc
a2 + b2 b2 + c2 c2 + a2
c) + + >a+b+c
a+b b+c c+a
d ) Si a + b + c = 6 entonces a2 + b2 + c2 12
9 2 2 2 1 1 1
e) < + + < + +
a+b+c a+b b+c c+a a b c

f) a+b < a+ b

2. Pruebe que:
n
X
a) (1 a1 )(1 a2 ) (1 an ) > 1 ai si ai < 1
i=1
n
1 X
b) (1 + a1 )(1 + a2 ) (1 + an ) < n si ai < 1
X
1 ai i=1

i=1

3. a) (a2 + b2 )(a3 + b3 )(a6 + b6 ) < 4(a11 + b11 )


b) (a + b + c + d)3 < 16(a3 + b3 + c3 + d3 )
c) (ab + bc + ca)(a + b + c)4 < 27(a3 + b3 + c3 )2

4. Si n 2 es un natural entonces
    
3n 1 1 2 n1 2n
< 1+ 1 + 2 1 + <
2n n2 n n2 n+1

5. Sea n natural. Probar que

a) (n + 1)n+1 < 2n+1 nn si n > 1


b) 2n+4 > (n + 4)2
1 2 1
c) (n + 1) < (11 22 33 nn )2/(n +n) < (2n + 1)
2 3
2
d ) (n!) > nn n>2
Xn
e) 2k < (n + 1)n
k=1

f ) 2n > 1 + n 2n1

6. Sean m y n naturales. Probar que


1 m 1 n
   
1+ < 1+ si m < n
m n
225

7. Sean a1 , a2 , , an reales positivos no todos iguales y s su suma. Probar que:


n
X s n2
a) >
sai n1
i=1
n
X ai n
b) >
s ai n1
i=1
n1
X n1
c) ai aj < s
2
i=1
i<j

8. Sean a1 , a2 , , an , b1 , b2 , , bn reales positivos. Pruebe que:


   
ai a1 + a2 + + an ai
a) mn max
bj b1 + b2 + + bn bj
  r  
ai a1 a2 an ai
b) mn n max
bj b1 b2 bn bj

9. Pruebe que:

3 1 1 1 1 1 1
a) + < 2 + 2 + + 2 < 2
2 n 2n2 1 2 n n
1 1 1 5 1
b) 2
+ 2
+ + 2
<
1+1 1+2 1+n 4 n

10. Si a es positivo, encontrar el mayor valor para (a + x)5 (a x)3 , cuando a < x < a.

11. Encontrar el mayor valor de:

5
a) (2x + 5)5 (7 x)2 si < x < 7
2
b) xy 2 z 3 t4 cuando x2 + y 2 + z 2 + t2 = 1
226

8.3. VALOR ABSOLUTO


El valor absoluto es una funcion de multiples aplicaciones en matematica. Conviene tra-
tarla en este captulo porque sus propiedades mas interesantes se expresan mediante desigual-
dades. Se define de la manera siguiente:

Definicion 8.2
El valor absoluto de un numero real x, se escribe |x| y se define

x si x 0
x=
x si x < 0

De la definicion es inmediato que el valor absoluto es una funcion de los numeros reales en
los reales no negativos. Ejemplo de algunos de sus valores son:

|5| = 5; |0| = 0; | 3| = 3; | 2/9| = 2/9


El valor absoluto de un numero puede interpretarse geometricamente como su distancia al
origen y |x y| como la distancia entre los puntos x e y.

Teorema 8.6 Propiedades del valor absoluto

1. |x| 0

2. |x| = 0 si y solo si x = 0

3. |xy| = |x| |y|



x |x|
4. = (y 6= 0)
y |y|
5. Si  > 0 entonces |x| <   < x < 

6. Si  > 0 entonces |x|   x 

7. |x| x |x|

8. |x + y| |x| + |y| (Desigualdad triangular)

9. | x| = |x|

10. |x y| = |y x|

11. |x|2 = x2

12. ||x| |y|| |x y|

13. |x| = |y| x = y x = y

Demostracion
227

1) y 2) son inmediatas de la definicion.

El hecho que la definicion de valor absoluto fuera dada por casos, a saber x 0 y
x < 0, obliga muchas veces que algunas demostraciones de sus propiedades sea necesa-
rio tambien separarlas en casos. Esta situacion se presentara en la prueba de la tercera
propiedad.

3) a) Si x e y son no negativos. Entonces |x| = x; |y| = y y como xy tambien es no


negativo se tiene que |xy| = xy. Luego, |xy| = |x||y|.

b) Si x e y son negativos. Entonces |x| = x, |y| = y. Pero como xy es positivo


tenemos que |xy| = xy. Luego, |xy| = xy = (x)(y) = |x||y|.

c) Si x 0 e y < 0. Entonces |x| = x, |y| = y y |xy| = (xy) = x(y) = |x||y|.

Luego, en todos los casos se cumple que |xy| = |x||y|.


x x |x| x
4) |x| = y , que por 3) es igual a |y|, de donde resulta que
= .
y y |y| y

5) Sea |x| < . Si x 0 se tiene que |x| = x < ; y como  < 0 podemos escribir que
 < x < .

Si x < 0, entonces |x| = x < , de donde  < x. Como  > 0 podemos concluir que
 < x < . Entonces, en ambos casos |x| <   < x < . Supongamos, ahora, que
 < x < . Si x 0 entonces |x| = x < . Si x < 0 entonces |x| = x.

Pero de  < x se concluye que x < . De donde |x| < . Luego, en ambos casos si
 < x <  se tiene que |x| < .

La demostracion de 6) es analoga a la anterior.

7) Se deduce de 6) tomando  = |x|. Dejamos los detalles al lector.

8) De 7) tenemos que |x| x |x|; |y| y |y|. Sumando miembro a miembro nos
queda que |x| |y| x + y |x| + |y|. Entonces por 6) |x + y| |x| + |y|.

9) | x| = |(1)x| y por 3) = | 1||x| = |x|.

10) |x y| = |x y|| 1| = |(x y)(1)| = |y x|

11) |x|2 = |x||x| = |x2 | = x2 ya que x2 0.

12) |x| = |x y + y| |x y| + |y|, de donde se obtiene que |x| |y| |x y|. De igual
forma se prueba que |y| |x| |y x| = |x y|. Multiplicando, esta ultima por (1)
tenemos que |x| |y| |x y| que combinada con la primera, nos permite escribir
|x y| |x| |y| |x y|. Entonces por 6) se obtiene el resultado.
228

13) Es inmediato de la definicion y de 9.

Nota
Como ya dijimos, el valor absoluto es una funcion de R R+ {0}, tal que para x > 0, |x| = x,
de manera que en este caso su grafico es la bisectriz del primer cuadrante. Para x < 0, |x| = x,
el grafico es la bisectriz del segundo cuadrante, como se observa en la figura.

Ejemplo 8.15
2 3 1 3
Dibuje el grafico de f (x) = x + + x .
3 2 3 2
1 3 1 3
Debemos estudiar los casos: x > 0 y x < 0.
3 2 3 2
1 3 9
Si x 0, es decir, si x entonces
3 2 2
2 3 1 3
f (x) = x + + x = x.
3 2 3 2
9 2 3 3 1 1
Si x < entonces f (x) = x + + x = x + 3.
2 3 2 2 3 3
Luego, el grafico de f (x) es:

6


3 
 

 -

-9 4.5


2 3 1 3
Grafico de f (x) = x + + x
3 2 3 2
229

Ejemplo 8.16
Resolver la ecuacion |x2 2x 3| = |x + 1|.

Como los valores absolutos son iguales, hay dos posibilidades, a saber: x2 2x 3 = x + 1
o x2 2x 3 = (x + 1).

La primera ecuacion tiene como soluciones 4 y -1 y la segunda 2 y -1. Por lo tanto, las solu-
ciones a la ecuacion en valor absoluto propuesta son 4, 2 y -1.

Mas ejemplos se veran en la seccion siguiente.

8.4. INECUACIONES
Antes de introducirlas, es necesario desarrollar el concepto de intervalo. Ellos son una
forma conveniente y tradicional de referirse a ciertos subconjuntos de R.

Definicion 8.3
Sean a y b numeros reales con a b.

i) El intervalo abierto de extremos a y b, anotado por (a, b), se define por (a, b) = {x
R : a < x < b}.

ii) El intervalo cerrado de extremos a y b, anotado por [a, b], se define [a, b] = {x R : a
x b}.

iii) El intervalo semiabierto por la izquierda de extremos a y b, anotado por (a, b], se define
por

(a, b] = {x R : a < x b}.

Notemos que si a = b entonces (a, b) = (a, b] = y [a, b] = {a}.

Definicion 8.4
Tambien se llaman intervalos con un extremo en el infinito a los siguientes conjuntos:

(a, ) = {x R : a < x}

(, a) = {x R : x < a}

[a, ) = {x R : a x}

(, a] = {x R : x a}
Los dos primeros son abiertos mientras que los otros dos, cerrados.

Ahora explicaremos la idea de inecuacion. Por esta, entenderemos una relacion definida en los
numeros reales, con una sola variable, digamos x, y expresada por alguna de las relaciones
<, , >, . Algunos ejemplos de inecuaciones son:
230

1 p
3x 2 > 0; x2 5x + 1 |x 1|; < x2 + 1
x2 + 1
Se dice que un numero real r satisface una inecuacion ssi la desigualdad que resulta al re-
emplazar x por r es verdadera. Por resolver una inecuacion se entiende encontrar todos los
valores reales r que satisfacen la inecuacion. En general, hay infinitos valores para x que la
satisfacen y no es posible escribirlos todos. Habitualmente la forma de proceder es transformar
la inecuacion en otras equivalentes hasta poder expresarla por medio de intervalos.

Ilustraremos el metodo por medio de ejemplos.

Ejemplo 8.17
Resolver la inecuacion 3x 2 > 0.

2
3x 2 > 0 3x > 2 x > .
3
 
2
Esta ultima expresion es equivalente a poner x , pero normalmente se omite esta
3
2
parte y se da como solucion x > . Este mismo procedimiento se usa para resolver las
3
inecuaciones del tipo:

ax + b > 0; ax + b 0; ax + b < 0, ax + b 0.

Ejemplo 8.18
x2 x+2
Resolver la inecuacion < .
x1 x+1
Es muy frecuente que se cometa el error de efectuar el producto cruzado y escribir (x 2)(x +
1) < (x 1)(x + 2). Esta forma de proceder es incorrecta pues los factores por los cuales se
multiplico la inecuacion, a saber (x 1) y (x + 1), no tienen un signo fijo, es decir, seran
mayores o menores que 0 dependiendo del valor de x. El metodo correcto es el siguiente:

x2 x+2
<
x1 x+1
es equivalente a

x2 x+2
<0
x1 x1
que es equivalente a

2x
> 0.
(x 1)(x + 1)
Esta ultima expresion sera verdadera si y solo si los tres factores 2x, (x + 1), (x 1) son
positivos o uno positivo y los otros dos negativos. Entonces, debemos estudiar el signo de
cada uno de ellos separadamente y despues encontrar los intervalos donde se cumplen estas
condiciones. Un buen diagrama para este calculo es el siguiente:
231

x 1 0 1
2x + +
x1 +
x+1 + + +

La explicacion de este cuadro es: La primera fila representa el campo de variacion de x, que en
este caso es el intervalo (, ). Despues, se agregan tantas filas como factores a estudiar.
La funcion 2x cambia de signo en el valor 0. Se traza la recta vertical en 0 y en la segunda
fila se ponen signos - a la izquierda de 0 y signos +, a su derecha. La funcion x 1 cambia
de signo en 1. Se traza la lnea vertical en ese valor y se ubican signos - a la izquierda de 1 y
signos +, a su derecha. En forma analoga, se completa la cuarta fila. Aqu el cambio de signo
se produce en -1. Finalmente la solucion a nuestro problema es x > 1 o 1 < x < 0 porque
esos son los unicos intervalos en que estas tres funciones son positivas o dos negativas y una
positiva.

Este mismo metodo puede usarse para resolver inecuaciones que pueden expresarse como pro-
ductos o cuocientes de expresiones cuyos signos es posible estudiar separadamente.

A continuacion, haremos un estudio del signo de la funcion f (x) = ax2 + bx + c, conocida


con el nombre de Trinomio de Segundo Grado.

Debemos distinguir dos casos: si el trinomio es o no factorizable en dos binomios.

Sabemos que es factorizable cuando el discriminante 4 es mayor o igual que 0, (recordemos


que 4 = b2 4ac) y se resuelve por el metodo explicado anteriormente. Veamos ejemplo.

Ejemplo 8.19
Resolver la inecuacion: 4x2 + 3x x2 2x + 2.

Esta inecuacion es equivalente a

3x2 + 5x 2 0
con 4 = 25 + 24 = 49 > 0.

Entonces se puede factorizar, quedando


 
1
3(x + 2) x 0
3
1
donde 2 y son las races de la ecuacion 3x2 + 5x 2 = 0. El diagrama para encontrar la
3
solucion es:

x 2 1/3
x+2 + +
x 1/3 +
232

Entonces su solucion es 2 x 1/3. Notemos que son soluciones 2 y 1/3 porque la


inecuacion propuesta puede tomar el valor 0.

Si la inecuacion inicial hubiese sido 4x2 + 3x < x2 2x + 2, entonces su solucion sera


1
2 < x < . Si ella hubiese sido 4x2 + 3x > x2 2x + 2 su solucion habra sido x < 2
3
o x > 1/3.

En general, si los numeros reales y , con < son las races de la ecuacion ax2 +bx+c =
0, la inecuacion ax2 +bx+c > 0, tendra soluciones de la forma < x < si a < 0 y soluciones
de la forma x < x > si a > 0.

Dejamos al lector comprobar esta afirmacion, recordandoles que la factorizacion del trinomio
es ax2 + bx + c = a(x )(x ).

Ahora trataremos el caso en que el trinomio no es factorizable en los reales. Se produce


cuando 4 < 0, es decir, cuando b2 4ac < 0. Esta situacion la trataremos en forma general
en el siguiente teorema.

Teorema 8.7
La expresion ax2 + bx + c con b2 4ac < 0 tiene siempre el signo de a. Es decir si a > 0
entonces x(ax2 + bx + c > 0) y si a < 0 entonces x(ax2 + bx + c < 0).

Demostracion

b2 b2 b 2
   
b
ax2 2
+ bx + c = a x + x + 2 + c =a x+
a 4a 4a 2a

b 2
4a2 x + + 4ac b2

4ac b2 2a
+ =
4a 4a
b 2
4a2 x + 2a

4
=
4a
El numerador de esta expresion es siempre positivo independiente del valor que pueda tomar x,
ya que 4 > 0. Por lo tanto ax2 +b+c sera positivo si a es positivo y negativo cuando a lo sea.

Nota
El grafico de T (x) = ax2 + bx + c es una parabola de eje perpendicular al eje x. En la figura
se muestra para a positivo y para 4 > 0 , 4 = 0 y 4 < 0, respectivamente.

 -
Figura 7.3
233

Ejemplo 8.20
Para que valores de k se cumple que x(x2 + kx + 1 > 0).
En este caso a = 1. Debemos conseguir que 4 sea menor que cero.

4 = b2 4ac = k 2 4 < 0

k 2 4 < 0 (k 2)(k + 2) < 0.


y la solucion es

2 < k < 2.

Cuando una inecuacion contiene expresiones de grado mayor que 2, es siempre posible
factorizarla en factores de grado 1 o 2, como lo probaremos en el Captulo 11.

Ejemplo 8.21
x4 5x2 36
Resolver 3 < 0.
x x2 + x 1
Factorizando el primer miembro se transforma en

(x2 + 4)(x 3)(x + 3)


<0
(x 1)(x2 + 1)
Ahora construimos el cuadro

x 3 1 3
x2 + 4 + + + +
x3 +
x+3 + + +
x1 + +
x2 + 1 + + + +

As, obtenemos la solucion que es

x < 3 o 1 < x < 3.

Desgraciadamente no hay metodos generales para obtener factorizaciones y el unico recurso


es desarrollar una gran habilidad, la que se consigue resolviendo muchos ejercicios.

Ejemplo 8.22
Resolver la inecuacion x2 + 2x 3 < x + 8.

En primer lugar, debemos limitar los valores de x a aquellos que hacen que la cantidad
subradical sea positiva o cero; y esto se consigue resolviendo la inecuacion.

x2 + 2x 3 0.
Al factorizarla se obtiene
234

(x + 3)(x 1) 0
y luego sus soluciones son

x 3 o x 1.
Por otra parte, el primer miembro de la inecuacion es siempre positivo o cero, por lo tanto no
puede haber solucion si

x < 8.
Entonces limitamos los valores de x a

8 x 3 o x > 1.
En este rango, ambos miembros son positivos y podemos elevar al cuadrado. Nos queda

x2 + 2x 3 < x2 + 16x + 64
que es equivalente a

14x > 67
de donde resulta que

67
x> .
14
Luego, la solucion es

67/14 < x 3 o x 1.

Ejemplo 8.23
Resolver la inecuacion |x + 2| + |x 1| 5.

Los cambios de signos de las expresiones en el interior de los valores absolutos se producen en
-2 y 1, por lo tanto debemos estudiar la inecuacion en los 3 casos:

x < 2; 2 x < 1 y x 1.
Primer Caso:

x < 2. Entonces |x + 2| = (x + 2) y |x 1| = (x 1) y la inecuacion se reduce a


(x + 2) (x 1) 5 que da como solucion x 3. Encuadrando este resultado dentro del
rango que tenemos para x nos queda que la solucion de este caso es

3 x < 2.
Segundo Caso:

2 x < 1. Aqu |x + 2| = x + 2 y |x 1| = (x 1) y la inecuacion es x + 2 (x 1) 5


que se convierte en 1 5 que es siempre verdadero independiente del valor de x. Esto quiere
235

decir que todos los x en el rango propuesto satisfacen la inecuacion. Luego, su solucion es
2 x < 1.

Tercer Caso:

x 1. Ahora |x + 2| = x + 2 y |x 1 = x 1 y la inecuacion es x + 2 + x 1 5 que se


reduce a x 2. Nuevamente encuadrado esta solucion dentro del rango queda que 1 x 2.

Finalmente la solucion a la inecuacion |x + 2| + |x 1| 5 se consigue uniendo las soluciones


obtenidas en cada caso. Por eso, la solucion final es 3 x 2.

Aprovechando los propiedades del trinomio de segundo grado, probaremos la desigualdad de


Cauchy-Schwarz.

Teorema 8.8 Sean {a1 a2 an } y {b1 b2 bn } dos conjuntos de numeros reales. Entonces:
n
!2 n
! n !
X X X
2 2
ar br ar br
r=1 r=1 r=1

y la igualdad se cumple si y solo si


a1 a2 an
= = =
b1 b2 bn
Demostracion

Pn 2
Consideremos la funcion F (x) = r=1 (ar x + br ) . Por ser una suma de cuadrados, nunca
es negativa. Desarrollandola:
n
X n
X n
X
2
F (x) = x a2r + 2x ar br + b2r 0
r=1 r=1 r=1

Por lo tanto, su discriminante es negativo. De donde obtenemos:


n
!2 n
! n !
X X X
ar br a2r b2r
r=1 r=1 r=1

Supongamos que existe un real t 6= 0, para el cual F (t) = 0. (El caso t = 0 conduce a 0 = 0).
Entonces, por la definicion de F concluimos que, para todo r, ar t + br = 0 lo que demuestra
que:
a1 a2 an
= = = = 1/t
b1 b2 bn
finalizando la demostracion.

Ejemplo 8.24 Sean a, b, c y d reales positivos. Probar que

(a2 + b2 + c2 + d2 ) (a + b + c + d)(a3 + b3 + c3 + d3 )
236

Aplicando la desigualdad anterior a los conjuntos



{ a, b, c, d} y { a3 , b3 , c3 , d3 },

se obtiene el resultado.
237

8.4.1. EJERCICIOS
1. Resolver las siguientes inecuaciones:

i) |2x 1| 3 vi) |x 1|2 + 2|x 1| 3 > 0

ii) |3x + 5| > 2 vii) |x + 2| |x + 3 1

iii) x + |x| 4 viii) |3x + 2| |x + 1| + |2x + 1|


2
x 1
iv) 2
>2 ix) |x2 x| + |x| |x 1| 0
x 4

v) |1 + |x 1| + x| < 4 x) ||x| |x + 2|| < 2


2. Resolver las siguientes inecuaciones:

i) x 1 < x+1 vi) x2 x6<8

ii) 1 x 2 x > 1 vii) 2x 1 > x2 3x 2

3x 5 2x 5 x3 1
iii) < viii)
x+1 x+5 (x 2)2 4

x2 6x 7 2 3
iv) <3 ix) > 2
x2 + 2x + 1 x2 + 9 x + 2x + 5

x3 (x 4)
v) (x + 1)(x3 x2 + x 1) > 0 x) > 2
x2 +x+1 x x+1
3. Encuentre los valores de m de manera que:

i) x2 (m 1)x + 2 sea positivo para todo real x.


ii) (m 5)x2 4mx + m 2 < 0 para todo real x.

4. i) Pruebe que si 0 < b < c, entonces ||x a| b| < c es equivalente con |x a| < b + c.
ii) Resuelva ||x 2| 8| < 4.
1
5. Pruebe que el mayor entre los dos numeros x e y es (x + y + |x y|) y el menor es
2
1
(x + y |x y|).
2
6. (Desigualdad de Minkowski).
Si {a1 , a2 , , an } y {b1 , b2 , , bn } son dos conjuntos de numeros reales, probar que
v v v
u n n
u n 2
uX uX uX
t (ak + bk )2 t 2
u
ak + t bk
k=1 k=1 k=1
238

8.5. SOLUCIONES O SUGERENCIAS A LOS EJERCICIOS


DEL CAPITULO
Pagina 224

1. a) (a3 b3 )(a b) > 0



3
b) (a + b + c)/3 > 3 abc y (ab + bc + ca)/3 > a2 b2 c2
a2 + b2 a+b
c) a2 + b2 > 2ab 2(a2 + b2 ) > (a + b)2 >
a+b 2
d ) a2 + b2 > 2ab, etc. Sumando resulta que 2(a2 + b2 + c2 ) > 2 y como a2 + b2 + c2 +
2(ab + bc + ca) = 36 reemplazando se obtiene el resultado.
e)
  s
1 1 1 1 3
1
+ + >
3 a+b b+c c+a (a + b)(b + c)(c + a)

1 p
((a + b) + (b + c) + (c + a)) > 3 (a + b)(b + c)(c + a)
3
multiplicando resulta la desigualdad de la izquierda.
Para la segunda desigualdad pruebe que 1/a + 1/b > 4(a + b).

f ) 2 ab > 0. Luego, a + b + 2 ab > a + b. De donde ( a + b)2 > a + b.

2. a) Es inmediato por induccion.


b) Como 1 + a1 < 1/1 a1 se tiene que

(1 + a1 )(1 + a2 ) (1 + an ) < 1/(1 a1 )(1 a2 ) (1 an )

usando a) se obtiene el resultado.

3.a)-b) Use la desigualdad de Tchebychef (Teorema 5).


a + b + c 3 a3 + b3 + c3
 
c) Por es ejemplo 13 se tiene que < y por el teorema 5 que
3 3
a2 + b2 + c2 a + b + c a3 + b3 + c3
<
3 3 3
Multiplicando y observando que ab + bc + ca < a2 + b2 + c2 se obtiene el resultado.
d) La primera desigualdad sale con el ejemplo 3, la segunda usando el problema 2.b).

4. Use Weiertrasse.
1 1 1
5. a) Tomar G < A para los n + 1 numeros 1, , , , .
n n n
b) Por induccion. El meollo es

(n + 5)2 = (n + 4)2 + 2n + 9 < 2n+4 + 2n + 9

Pero 2n + 9 < 2n+4 . Luego, (n + 5)2 < 2n+4 + 2n+4 = 2n+5 .


c) H < G < A aplicado a los numeros 1, 2, 2, 3, 3, 3, , n, n, , n.
239

d ) (n!)2 = (1 n)(2 (n 1))(3 (n 2)) (n 1) pero p(n p + 1) > n si 1 < p < n


pues

p p2
   p 
p(n p + 1) = n p + >n p+ p >n
n n n
Luego, (n!)2 > nn .
e) Aplicar G < A a los n numeros 2, 4, 6 , 2n.
f ) A > G aplicado a los n numeros 1, 2, 22 , , 2n1 .

6. Por induccion a partir del ejemplo 10.

7. a) A > H aplicado a los n numeros s/(s ai ).


ai s (s ai ) s
b) = = 1 y use la parte a.
s ai s ai s ai

c) ( ai ai )2 0. Luego, 2 ai aj ai + aj
y sumando se obtiene el resultado.

8. Sea ap el menor de los ai y bq el mayor de los bj .


Entonces n ap a1 + a2 + + an
y n bq b1 + + bn
 
ai ap a1 + + an
Luego, mn =
bj aq b1 + + bn

Los demas casos son similares.

9. Tanto a) como b) salen por induccion.


a+x ax
10. Sea a1 = a2 = = a5 = y a6 = a7 = a8 = y apliquemos A G.
5 3
Resulta que
 5  3  8
a+x ax 1
a
5 3 4

Por lo tanto, para todo valor de x se tiene que

(a + x)5 (a x)3 r5 33 a8 /48

como el mayor valor de G es A y este se produce cuando todos los ai son iguales, entonces
a+x ax
el mayor valor de (a+x)5 (ax)3 sera 55 33 a8 /48 y se alcanzara cuando = ,
5 3
es decir, cuando x = a/4.

5 5
 
5 2
11. a) (2x + 5) (7 x) = 2 x +5 (7 x)2 .
2
5 5
 
Entonces se debe encontrar el mayor valor de x + (7 x)2 cuando 5/2 <
2
x < 7. El resto es analogo al problema anterior. El maximo se alcanza en x = 30/7.
240

b) Aplicando A G a los numeros, y, y, y, z, z, z, t, t, t, t se obtiene que

(x + 2y + 3z + 4t)10
xy 2 z 3 t4
1010
Como la igualdad se alcanza cuando todos son iguales, esta se produce cuando
forman el valor 1/2.

Pagina 237

1. i) 3 2x 1 3 2 2x 4 1 x 2.
ii) (3x + 5 > 2) (3x + 5 < ) , etc.
iii) (2x 4 x 0) (x < 0)
2
x 1 2
x 1 x2 1 x2 1
iv) 2 >2 2 < 2 2 2>0 2 + 2 < 0, etc.
x 4 x 4 x 4 x 4
Se llega a que 3 < x < 7 x 6= 2 7 < x < 3 x 6= 2.
v)
(| 2x |< 4 x 1) (| 2 |< 4 x < 1)

(2 x < 2 x 1) (x < 1)

x < 2.

vi)
x2 2x 2 + 2|x 1| > 0

(x2 4 > 0 x 1) (x2 4x > 0 x < 1)

x>2 x < 0.

vii) Usando |a| |b| |a b| resulta que |x + 2| |x + 3| | 1| = 1. Por lo tanto,


vale para todo x.
viii) |3x + 2| = |(x + 1 + (2x + 1)| |x + 1| + |2x + 1|.
Por lo tanto, vale para todo x.

ix) Por casos


a) Si x 1 entonces es equivalente con x2 x + 1 0 que vale para todo x 1.
b) Si 0 x < 1 es equivalente con x2 3x + 1 < 0 de lo que resulta que

3 5
x<1
2
c) Si x < 0 entonces es equivalente con x2 x 1 0 que vale para

1 5
x
2

3 5 1 5
En resumen, la inecuacion vale para x o x .
2 2
241

x) Notemos que ||x| |x + 2|| |x x + 2| = 2. Por casos


a) Si x 0 se tiene que |x| |x + 2| = 2. Luego, la inecuacion no se cumple.
b) Si 2 x < 0. En este caso ||x| |x + 2|| < 2 equivale a |x + 1| < 1 que se
cumple para 2 < x < 0.
c) Si x < 2. En este caso , otra vez ||x| |x + 2|| = 2.

2. i) Condicion inicial x > 1. Es equivalente a x2 2x + 1 < x + 1;


Finalmente se obtiene 0 < x < 3.

ii) Condicion inicial: x 1.



1 x > 1 + 2 x equivale a 2 > 2 2 x que es siempre falso.
2x 5 3x 5 x2 + 13x 20
iii) 0 < < 0.
x+5 x+1 (x + 5)(x + 1)
Sea

13 249 13 + 249
= =
2 2
entonces la inecuacion equivale a

(x )(x )(x + 1) < 0

R
+ 5 + 1 +
la cual vale para

( < x < 5) (1 < x < )


x2 6x 7 x2 + 5
iv) = 3 < 0 >0
x2 + 2x + 1 (x + 1)2
la cual vale para todo x 6= 1.

v) Equivale a (x + 1)(x 1)(x2 + 1) > 0 como x2 + 1 > 0 para todo x, equivale a


x2 1 > 0 cuya solucion es (x > 1) (x < 1).

vi) Condicion inicial: x 6



x 2 < 8 + x 6 elevando al cuadrado equivale a 30 < x 6 que es valido
para todo x 6.

3 17 3+ 17
vii) Condicion inicial: x2 3x 2 0 que equivale a x < o x > .
2 2
Elevando al cuadrado la inecuacion, se obtiene que equivale a 3x2 x + 3 > 0, la
cual es verdadera para todo x pues su discriminante es negativo. Luego, la solucion
es la condicion inicial.
242

viii) Condicion inicial: x 6= 2.


x3 1
0 equivale a (x 4)2 0, que vale para todo x.
(x 2)2 4
2 3
ix) 2 2 > 0 x2 4x + 17 < 0 la cual es falsa para todo x.
x + 9 x + 2x + 5
x3 x4
x) 2 > 0 x2 7x 1 < 0 que tiene por solucion:
x2
+x+1 x x+1

7 53 7 + 53
<x<
2 2

3. i) Resp. 1 8 < m < 1 + 8.
ii) Resp. 10/3 < m < 1.

4. i) ||x a| b| < c c < |x a| b < c b c < |x a| < b + c| |x a| < b + c,


pues b c < 0.
ii) Resp. 10 < x < 4 o 6 < x < 14

5. Por la simetra del valor absoluto, podemos suponer que x > y, con lo cual la prueba es
inmediata.
n
X n
X n
X n
X
6. (ak + bk )2 = a2k + 2 ak bk + b2k
k=1 k=1 k=1 k=1

Pero, por teorema 8.8 se cumple


v v
Xn u n u n
uX 2 uX
ak bk t ak t b2k
k=1 k=1 k=1

Finalmente

v v v v 2
n
X n
X
u n
X uX
u n n
X
u n
X
u n
X
(ak + bk )2 a2k + 2t a2k t b2k + b2k = t a2k + t b2k
u u u

k=1 k=1 k=1 k=1 k=1 k=1 k=1

y extrayendo la raz se obtiene la desigualdad.


Captulo 9

LOS NUMEROS COMPLEJOS

9.1. INTRODUCCION
Existen ecuaciones que no son satisfechas por ningun numero real. Por ejemplo, la ecua-
cion x2 + 1 = 0 tiene ese caracter puesto que x2 + 1 siempre es positivo, cualquiera sea el
numero real que le asignemos a x. Este tipo de consideraciones sugiere la necesidad de inten-
tar construir un nuevo sistema numerico que ample a R y el cual no presente esta anomala.
Como veremos posteriormente, tal intento no puede tener exito. Es imposible conseguir un
sistema numerico que extienda tanto la estructura de campo como la de orden de R y que
posea la propiedad de que toda ecuacion tenga, por lo menos, una solucion.

Con el fin de familiarizarnos mas con la idea de ampliacion o extension de un sistema numerico
y tambien por su interes intrnseco recordaremos muy brevemente algunos ejemplos tpicos
de este proceso.

a) El conjunto Z de los numeros enteros es una ampliacion tanto del aspecto algebraico
como de la estructura de orden del sistema de los numeros naturales. Su objetivo fue
conseguir que la resta sea siempre posible, cualquiera sean los numeros con que se opere.
En estas condiciones N queda limitado a ser un subconjunto de Z y aunque las opera-
ciones y el orden que rigen en el son las impuestas por la estructura de Z, no producen
ninguna modificacion en su propia estructura, puesto que coinciden plenamente con las
operaciones y el orden que el tiene cuando es considerado como un sistema numerico
aislado.

b) El conjunto Q de los numeros racionales es una extension de Z hecha con el fin de


lograr que la division sea siempre posible excepto en los casos en que el divisor es el
cero. Analogamente al caso anterior, Z se convierte en un subconjunto de Q, pero sus
operaciones y su orden coinciden con los que le impone Q.

c) El conjunto R de los numeros reales puede ser considerado como una ampliacion de
Q con la finalidad de conseguir que ciertas sucesiones tengan lmite. En el captulo 12
estudiaremos algunos aspectos de este tema.

El sistema de los numeros complejos que construiremos a continuacion, resultara ser


una extension solo de la estructura algebraica de los numeros reales, en la cual toda

243
244

ecuacion que pueda definirse tendra, por lo menos una solucion. Esta propiedad conlleva
la existencia de raices de numeros negativos, pero, desgraciadamente, esta ampliacion
nos hara perder la estructura de orden.

9.2. EL CAMPO DE LOS NUMEROS COMPLEJOS


Definicion 9.1
Designaremos por C al conjunto R R provisto de las dos operaciones siguientes:

i) (a, b) + (c, d) = (a + c, b + d)

ii) (a, b)(c, d) = (ac bd, ad + bc)

C se llama el campo de los numeros complejos y sus elementos, numeros complejos. La prime-
ra operacion se llama suma y la segunda, producto y son anotadas con los mismos smbolos
que tienen en R las operaciones con dichos nombres.

Nota
Conviene destacar que, al ser los complejos parejas ordenadas se tiene que (a, b) = (c, d) si y
solo si a = c y b = d. (Teorema 1.2).

El siguiente ejemplo ilustra estas operaciones. Los casos ii), iii) y iv) seran usados en el
teorema siguiente.

Ejemplo 9.1

i) [(5, 3) + (2, 1)](3, 2) = (7, 2)(3, 2) = (21 + 4, 14 6) = (25, 8)

ii) (a, b)(1, 0) = (a, b)

iii) (a, b)(0, 1) = (b, a)


a2 b2
   
a b ab ab
iv) (a, b) , = + , + = (1, 0) siempre
a2 + b2 a2 + b2 a2 + b2 a2 + b2 a2 + b2 a2 + b2
que a2 + b2 6= 0.

Nota
El unico caso para el cual a2 + b2 = 0 es (0,0).
El siguiente teorema establece las propiedades que tienen las operaciones de los complejos.
Ellas son las que justifican el que se hable del campo C.

Teorema 9.1

i) Tanto la suma como el producto de complejos son operaciones asociativas y conmutati-


vas.

ii) El producto se distribuye con respecto a la suma.


245

iii) (0, 0) es elemento neutro para la suma y (1, 0) para el producto.


 
a b
iv) El inverso aditivo de (a, b) es (a, b) y el inverso multiplicativo de (a, b) es , ,
a2 + b2 a2 + b2
pero en este caso, con la condicion que (a, b) 6= (0, 0).

Demostracion
Son muy elementales. A modo de ejemplo desarrollaremos los casos que nos parezcan menos
sencillos.

El producto es asociativo.

[(a, b)(c, d)](e, f ) = (ac bd, ad + bc)(e, f )

= (ace bde adf bcf, acf bdf + ade + bce)

= (a(ce df ) b(cf + de), a(cf + de) + b(cd df ))

= (a, b)(ce df, cf + de)

= (a, b)[(c, d)(e, f )]


El producto se distribuye con respecto a la suma

(a, b)[(c, d) + (e, f )] = (a, b)(c + e, d + f )

= (ac + ae bd bf, ad + af + bc + be)

= (ac bd, ad + bc) + (ae bf, af + be)

= (a, b)(c, d) + (a, d)(e, f )


Parte de las demostraciones de iii) y iv) estan en el ejemplo 1.

Nota
Sean w = (a, b) y z = (c, d) dos numeros complejos. Anotaremos:

i) w + z por (a, b) + (c, d)

ii) wz por (a, b)(c, d)

iii) w por (a, b), el inverso aditivo de w


 
1 a b
iv) w por , , el inverso multiplicativo de w.
a2 + b2 a2 + b2

Definicion 9.2
Sean w y z complejos. Definimos su resta y division del modo siguiente:

i) w z = w + (z)
246

ii) w/z = wz 1 , con z 6= (0, 0).

Ejemplo 9.2
 
(1, 5) (3, 3) 5 3
= [(1, 5) + (3, 3)] ,
(5, 3) 34 34
 
5 3
= (2, 8) ,
34 34
 
7 23
= ,
17 17

Definicion 9.3
Sea w = (a, b). Se definen la parte real y la parte imaginaria de w y se anotan Re w y Im w
respectivamente, como los numeros reales a y b. Es decir: Re w = a, Im w = b.

Ejemplo 9.3
Sea w = (a, b) y z = (c, d). Probaremos que:

i) Re (w + z) = Re(w) + Re(z)

ii) Im(w + z) = Im(w) + Im(z)

iii) Re(wz) = Re(w)Re(z) Im(w)Im(z)

iv) Im(wz) = Im(w)Re(z) + Re(w)Im(z)

Demostracion
Para a) y b) tenemos que w + z = (a + c, b + d). Luego, por la definicion anterior

Re(w + z) = Re(w) + Re(z) y

Im(w + z) = Im(w) + Im(z)

Para c) y d) sabemos que wz = (ac bd, ad + bc). Entonces, por la misma definicion:

Re(wz) = Re(w)Im(z) Im(w)Re(z) y

Im(wz) = Im(w)Re(z) + Re(w)Im(z)

Nota
Identificando el real x con el complejo (x, 0) es bastante obvio que la estructura de campo de
R puede considerarse como un subcampo de C, puesto que los resultados de las operaciones
con reales efectuadas como operaciones de C, coinciden con los resultados al realizarlas en R.
247

9.3. FORMA CANONICA DE LOS COMPLEJOS


Usando las siguientes abreviaturas se logra simplificar enormemente la operatoria con los
complejos.

I. (x, 0) anotaremos x. Por lo tanto, (1, 0) queda como 1. Sera omitido en los productos.

II. (0, 1) anotaremos i.

Por lo tanto, el complejo (x, y), que equivale a (x, 0)(1, 0) + (y, 0)(0, 1), se simplifica a x + yi.

En esta forma aditiva sera como escribiremos los complejos en el resto del captulo. Ella tiene
ventaja con respecto a la notacion como pareja, porque permite operar con los complejos
como binomio, con la unica salvedad de tener que reemplazar i2 por 1.

Ejemplo 9.4

i) x + yi = yi + x = iy + x

ii) (x + iy) = x iy (Inverso Aditivo)

iii) (x + yi) + (a + bi) = x + a + yi + bi = (x + a) + (y + b)i

iv) (x + yi)(a + bi) = xa + xbi + yai + ybi2 = (xa yb) + i(xb + ya)

v) (x + iy)1 = (x iy)/(x2 + y 2 )

Las dos primeras nos permite escribir los complejos con una gran libertad, mientras que las
dos siguientes nos autorizan a tratarlos como si fueran binomios para los efectos de sumarlos
y multiplicarlos. Ademas, tomando en cuenta el hecho que i2 = 1 no necesitaremos escribir
potencias mayores de i. La ultima, nos da una forma practica para operar con el inverso
multiplicativo

Ejemplo 9.5
1 + 2i 2 i
Simplificar + .
3 4i 5i
La expresion se puede escribir como

(1 + 2i)(3 4i)1 + (2 i)(5i)1 .


Pero
3 + 4i 1
(3 4i)1 = , (5i)1 = i.
25 5
Entonces
   
1 + 2i 2 i 3 + 4i 1
+ = (1 + 2i) + (2 i) i
3 4i 5i 25 5
2
que al efectuar las multiplicaciones y reducir los terminos se obtiene .
5
248

9.4. MODULO Y CONJUGADO


Definicion 9.4
Sea z = a + bi. Llamamos complejo conjugado de z, en smbolos, z, al complejo z = a bi.

Definicion 9.5
Sea z = a + bi. Llamamos modulo de z, en smbolos, |z|, al numero real
p
|z| = a2 + b2
Nota
Si z es real su modulo coincide con su valor absoluto.

En el siguiente teorema se resumen las propiedades de la conjugacion.

Teorema 9.2 (Propiedades del conjugado)

Sean z = a + bi, w = c + di dos numeros complejos. Entonces valen las siguientes propiedades:

1. z = z

2. z + w = z + w

3. z w = z w
z z
4. = w 6= (0, 0)
w w
5. z + z = 2Re(z) y z z = 2iImz

6. z real si y solo si z = z

7. z z = |z|2

Demostracion
Son todas sencillas. Demostraremos solo algunas dejando las restantes como ejercicio.

3.
z w = (a + bi)(c + di)

= (ac bd) + i(ad + bc) = (ac bd) i(ad + bc)


= (a ib)(c id) = z w

z w z
4. z = , pero por 3) del teorema anterior es igual a w. Dividiendo por w se
w w
obtiene el resultado.
249

Teorema 9.3 (Propiedades del moludo)

Sean z = a + bi; w = c + di dos numeros complejos. Entonces

i) |Re z| |z|; |Im z| |z|

ii) z = 0 ssi |z| = 0

iii) | z| = |z| = |z|

iv) |zw| = |z| |z|


z |z|
v) = w 6= 0

w |w|
vi) |z + w| |z| + |w| (Desigualdad triangular).

Demostracion


i) |Re z| = |a| = a2 a2 + b2 = |z|.
La demostracion para Im z es identica.

ii) Sea |z| = 0, es decir, a2 + b2 = 0. Entonces a2 + b2 = 0. Pero la suma de dos positivos
es positiva. Luego, a2 = b2 = 0, lo que implica que z = 0. Inversamente, si z = 0,
entonces z = 0 + 0i lo que fuerza que |z| = 0.
p
iii) z = a bi. Luego, | z| = (a)2 + (b)2 = a2 + b2
p
z = a bi. Luego, |z| = a2 + (b)2 = a2 + b2
p
zw = ac bd + (ad + bc)i. Luego, 2 2
iv) p |zw| = (ac bd) + (ad + bc) , que desarrollando y
factorizando se transforma en (a2 + b2 )(c2 + d2 ). Pero esta expresion es |z||w|.
z z
v) |z| = w = |w|. Dividiendo por |w| se obtiene el resultado.

w w
vi) Por el teorema anterior se justifican las siguientes igualdades:

|z + w|2 = (z + w)(z + w) = (z + w)(z + w)

= zz + ww + zw + zw

= |z|2 + |w|2 + zw + zw Por 5. del Teorema 3

= |z|2 + |w|2 + 2Re(zw)

Por i) de este teorema, aplicado a Re(zw) se obtiene la desigualdad

|z + w|2 |z|2 + |w|2 + 2|zw|

Por iii) y iv) recien probadas, el segundo miembro


250

|z|2 + |w|2 + 2|z||w| = (|z| + |w|)2

Luego, extrayendo raz cuadrada se obtiene el resultado.

Ejemplo 9.6
1 z
Demostrar que = 1 con z 6= 1.
z 1
En efecto, por los dos teoremas anteriores las siguientes igualdades son validas:

1 z |1 z| |1 z| |1 z| |1 z| |1 z|
z 1 = |z 1| = |z 1| = |z 1| = |z 1| = |1 z| = 1.

Ejemplo 9.7
Demostrar que |z 1| < |z + 1| si y solo si Re z > 0.

Sea z = a + bi. Por lo estudiado hasta este momento son equivalentes las siguientes desigual-
dades:

|z 1| < |z + 1| |a + bi 1| < |a + bi + 1|
p p
(a 1)2 + b2 < (a + 1)2 + b2

(a 1)2 + b2 < (a + 1)2 + b2 .

Simplificando esta ultima resulta equivalente con 4a > 0, lo cual implica a > 0. Luego,
Re z > 0.

Ejemplo 9.8
Probar que

|z + w|2 + |z w|2 = 2|z|2 + 2|w|2 .

Sabemos que

|z + w|2 = (z + w)(z + w) = (z + w)(z + w)

En forma similar

|z w|2 = (z w)(z w)

Entonces, multiplicando y sumando

|z + w|2 + |z w|2 = zz + wz + zw + ww + zz wz zw + ww

= 2|z|2 + 2|w|2

El siguiente ejemplo debe ser considerado como parte del desarrollo de la teora.
251

Ejemplo 9.9
Sea z = a + bi. Determnese un complejo w tal que w2 = z. En otras palabras, w sera la raz
cuadrada de z.

Sea w = x + iy. Entonces w2 = z implica que (x + iy)2 = a + bi. Luego, x2 y 2 + 2xyi = a + bi.

Pero dos complejos son iguales si y solo si sus partes reales y sus partes imaginarias son iguales
respectivamente.

Consecuentemente, x e y deben satisfacer el sistema de ecuaciones

1) x2 y 2 = a y 2) 2xy = b

Usando la identidad, por lo demas muy facil de establecer,

(x2 + y 2 )2 = (x2 y 2 )2 + 4x2 y 2

y combinandola con 1) y 2) se obtiene que



3) x2 + y 2 = a2 + b2 .

1) y 3) nos
da
a2 + b2 + a 2 a2 + b2 a
4) x2 = , y = y
2 2
extrayendo raz cuadrada logramos dos valores para x y dos para y.

No todas las combinaciones de estos cuatro valores dan soluciones del problema. Hay que
tener en cuenta que 2xy = b. Por lo tanto, si b > 0 se deben tomar x e y con el mismo signo
y si b < 0 deben ser de signo contrario. En resumen si b 6= 0 las soluciones son
!
a2 + b2 + a a2 + b2 a
w= +i si b > 0 y
2 2

!
a2 + b2 + a a2 + b2 a
w= i si b < 0
2 2

Si b = 0, de 4) obtenemos que x2 = a, y 2 = 0 que nos da para w los valores a. Notemos
que si a 0, w es un real, mientras que a < 0 implica que w es imaginario puro, es decir
su

parte real es cero. En este caso conviene calcular w haciendo uso de la igualdad a = i a.

Ejemplo 9.10
Resolver la ecuacion (1 i)x2 + (3 + i)x (4 + 6i) = 0.

b b2 4ac
La formula x = para la ecuacion ax2 + bx + c = 0 es valida en el campo de
2a
los complejos, porque se obtiene a partir de la estructura algebraica sin usar propiedades del
orden. Luego, las soluciones vienen dadas por:
252

p
(3 + i) (3 + i)2 + 4(1 i)(4 + 6i)
x=
2(1 i)

Simplificando se obtiene que la raz que aparece en el numerador es 48 + 14i. Calculemosla
segun el ejemplo anterior.

Tenemos que a = 48 y b = 14

Luego, a2 + b2 = 50, de donde sale que

a2 + b2 + a a2 + b2 a
+i =7+i
2 2
Entonces

3(3 + i) (7 + i)
x=
2(1 i)
que da las soluciones 1 + i y (2 + 3i).

Para cerrar esta seccion, probaremos que en los complejos no existe un orden que sea
compatible con sus operaciones algebraicas, vale decir, no hay una relacion de orden total,
que podramos anotar por <, que tenga las siguientes caractersticas:

1. Si w < z entonces w + t < z + t ;

2. Si w < z y 0 < t entonces wt < zt

Recordemos que una relacion de orden total queda definida por las propiedades de tran-
sitividad, antisimetra y comparabilidad entre cualquier par de elementos.

Teorema 9.4
En C no existe una relacion de orden que sea compatible con sus operaciones algebraicas.

Demostracion
Por contradiccion. Supongamos que existe una relacion de orden total compatible con sus
operaciones algebraicas. Anotemosla por <. Por la comparabilidad, como i 6= 0, se tiene que
i < 0 o 0 < i.

1. Si i < 0, entonces sumando i tenemos que 0 < i. Multiplicando por i, ya que es


positivo, nos queda que 0 < 1. Si esta ultima relacion la multiplicamos por 1, que
es positivo, se obtiene que 0 < 1, pero si a 0 < 1 le sumamos 1 se logra 1 < 0, lo que
establece una contradiccion.

2. Si 0 < i entonces podemos multiplicar por i llegandose nuevamente a 0 < 1.


253

9.4.1. EJERCICIOS
1. Dados z = 3 + 7i y w = 2i 5, calcular:

a) z + w, b) zw, c) z/w, d) zw e) |z|

2. Expresar en la forma a + bi los siguientes complejos:

(3 + 5i)(2 i)3
3
a) 7 + 4i b) c) i
1 + 4i

3. Complete la demostracion del teorema 3.

4. Calcular los modulos de:

(2 3i)4 (1 i)3
a) b) (1 i)10 c) 1i
5+i

5. Resolver la ecuacion,

a) ix2 + ix + 1 + i = 0
1 2
b) + =1+i
x x
a + bi
6. Pruebe que la condicion para que sea un real es que ad cb = 0.
c + di
7. Sea U = {z C : |z| = 1}. Pruebe que este conjunto, con la operacion multiplicacion,
forma un grupo.

8. Probar que:
   
z w
a) Re + Re =1
z+w z+w
   
z w
b) Im + Im =0
z+w z+w

1
9. Probar que si z + es real, entonces Im z = 0 o |z| = 1.
z
10. Probar que:

n n n n
! !
X X Y Y
a) zk = zk b) zk = zk
1 1 1 1


Xn X n
c) zk |zk |


1 1
254

11. Resolver las ecuaciones:

a) x3 = 1 b) x4 = 1

c) x3 = 1 d) x4 = 1

12. Encontrar un complejo z tal que:

1
|z| = = |1 z|
|z|
13. Resuelva las siguientes ecuaciones:

a) |z| z = 1 + 2i b) |z| + z = 2 + i

14. Pruebe que para todo z 6= 1 existe un real t tal que:

1 + it
z=
1 it

9.5. LOS COMPLEJOS COMO VECTORES


Aprovechando el hecho que un numero complejo z = a + bi queda definido por el par
ordenado de numeros reales (a, b) al igual que los puntos del plano, podemos representar cada
complejo (a, b) por el punto del plano con dichas coordenadas. Pero resulta tambien conve-
niente representarlo, no por el punto, sino por el vector que va del origen a dicho punto y
considerar a estos vectores como deslizantes. Con vectores deslizantes queremos decir que cada
uno de los vectore que es paralelo al vector que va de (0, 0) a (a, b) y con su misma longitud y
sentido sera considerado como una representacion vectorial del complejo z = a + bi. Por eso,
cada complejo z no tiene solamente un vector que lo representa, sino una infinidad compues-
ta de todos aquellos que son paralelos, de iguales longitudes y sentidos a un representante de z.

4 4
6 6
 3

3



2
2


1

H
Y
H 1 *




 HH 




- H -
 H
-2 -1 1
2

-1 -2-1-1 H1 2
H




 H
j
H

-2

-2

Figura 8.1 Figura 8.2

En la figura 8.1 se han dibujado 4 vectores cada uno de los cuales representa al mismo
complejo a + bi. En la 8.2, aparecen los vectores z, z, z y z.
255

Nota
Cuando se habla del vector z, entenderemos el vector representante de z que nace en el origen.
Por el punto z entenderemos el otro extremo de dicho vector.
En estas representaciones, se acostumbra llamar al eje de las abscisas, eje real y al de las
ordenadas, eje imaginario, porque las abscisas y ordenadas de los puntos son las partes reales
e imaginarias de los complejos respectivamente. Notemos que la longitud de cualquier vector
representante de z, corresponde a |z|.

El siguiente teorema muestra la interpretacion vectorial de la suma y resta de complejos.

Q

6
   
6
R  
R

   
  
  Q 
~z  ~z + w
~  @
I
 @ ~
z 
  
  @ 
 ~ zw~@
1
 
 
1
P
   
  w  
~ @   w ~

 -  @ -
O   O
 
  w~
Figura 8.3 
)
 Figura 8.4
P

Teorema 9.5
Sean z y w numeros complejos. El vector z + w queda representado por la diagonal que
nace en el origen del paralelogramo, del cual dos de sus lados son los vectores z y w.

Demostracion
Sea z = a + bi y w = c + di. Se pueden presentar varios caso en la ubicacion de los dos
vectores. Nos limitaremos al de la figura 8.3, en la cual todas las coordenadas son positivas.

Como OPQR es un paralelogramo es inmediato que la abscisa de Q es a + c y su ordenada



es b + d. Luego, OQ = (a + c) + (b + d)i = ~z + w.
~

Nota
En la figura 8.4 aparece la imagen de ~z w~ que simplemente se realiza, efectuando el

dibujo para ~z + w. Observemos, ademas, que |z w| es la distancia entre los puntos z y w.

Nota
Dejamos al lector comprobar las siguientes afirmaciones: Sea m R z C. Entonces los
vectores mz y z estan sobre la misma recta. Ademas la longitud de mz es m veces la de z y
su sentido coincide con el de este cuando m es positivo. Si m es negativo, entonces su sentido
es el contrario al de z.
256

Estas afirmaciones son un caso particular de un resultado que expondremos posteriormente,


cuando interpretemos geometricamente el producto de dos complejos.

a a
A BMB aa C
aa 6 6
B 6 aa B
B aa
B 
z
'$
B 
B 
B  
B 

&%
B 
z0
B 
)
B 
O
B - -

Figura 8.5 Figura 8.6 Figura 8.7

Ejemplo 9.11
Sea C un punto en el interior del trazo AB tal que AC : BC = m : n, y sea O un punto del
plano como indica la figura 8.5. Probar que:

(m + n)OC = n(OA) + m(OB)

donde P Q indica el vector que nace en P y termina en Q.

De la figura tenemos que:


1. OC = OA + AC

2. OC + CB = OB

y de la hipotesis

3. nAC = mCB

Multiplicando 1) por n, 2) por m y sumando resulta




4. (m + n)OC + mCB = nOA + mOb + nAC
y usando 3) se obtiene el resultado.

En particular, si C es el punto medio tenemos que m = n = 1 y por lo tanto,

1
OC = (OA + OB)
2
257

Ejemplo 9.12
Probar que una circunferencia con centro en el complejo z0 y radio r, tiene una ecuacion de
la forma zz z0 z z 0 z + k = 0, donde k es un numero real. (Figura 8.6)

Como la diferencia entre cualquier punto de la circunferencia y su centro z0 viene dada por
|z z0 | tenemos que los puntos de la circunferencia satisfacen la relacion:

|z z0 | = r
que es equivalente con

|z z0 |2 = r2
Transformandola segun los pasos siguiente

(z z0 )(z z0 ) = r2

(z z0 )(z z 0 ) = r2

zz + z0 z 0 zz 0 zo z = r2

zz zz 0 z0 z + (|z0 |2 r2 ) = 0
se obtiene finalmente que es equivalente con

zz zz 0 z0 z + k = 0
donde k = |z0 |2 r2

9.6. FORMA TRIGONOMETRICA DE LOS COMPLEJOS


Sea el angulo orientado medido, en grados sexagesimales o en radianes, desde el eje real
al vector z, en sentido antihorario como indica la figura 8.7. Es facil ver que cualquiera sea la
posicion de z se cumple, tanto en magnitud como en signo, las siguientes igualdades:

Re z = |z|cos, Im z = |z|sen
de donde obtenemos que

z = |z|(cos + i sen)
que es la formula conocida como la forma trigonometrica o polar del complejo z.

Nota
En el texto que continua usaremos solo radianes como medida de los angulos, aunque en
los ejemplos y ejercicios pueden aparecer grados.

Nota
Por ser las funciones seno y coseno periodicas de periodo 2 , hay una infinidad de angu-
los que satisfacen la ecuacion anterior, pero todos tienen la propiedad que la diferencia entre
258

cualquier par de ellos es un multipo de 2. Cada uno de estos angulos se conoce con el nombre
de argumento o amplitud de z.

Llamaremos Argumento principal de z al argumento cuyo valor numerico se encuentra en


el intervalo [0, 2). Lo anotaremos por Argz.

Nota
El intervalo [0, 2) tiene el defecto de presentar una discontinuidad en el sentido positivo
del eje x, al pasar del cuarto cuadrante al primero. Este salto no afecta el desarrollo algebraico
de los complejos, pero si el desarrollo analtico de la trigonometra. Por esto, es preferible usar
el intervalo (/2, /2], donde el salto aparece en la parte negativa del eje x. Pero en este
texto, por razones de sencillez, usaremos el intervalo [0, 2).

Las ventajas de la forma trigonometrica se aprecia en el producto de complejos como lo


establece el siguiente teorema.

Teorema 9.6
Sea z = |z|(cos + i sen) y w = |w|(cos + i sen). Entonces

1. zw = |z||w|(cos( + ) + i sen( + ))
z |z|
2. = (cos( ) + i sen( ))
w |w|

Demostracion

zw = |z|(cos + i sen)|w|(cos + i sen).

Efectuando los productos y reagrupando se obtiene que

zw = |z||w|(cos cos sen sen + i(sen cos + cos sen)

que por formulas conocidas de la trigonometra nos da

zw = |z||w|(cos( + ) + i sen( + ))

Con respecto al cuociente

z |z|(cos + i sen) |z|(cos + i sen) cos i sen


= =
w |w|(cos + i sen) |w|(cos + i sen) cos i sen

Efectuando los productos y reagrupando se obtiene que

z |z| cos cos + sen sen + i(sen cos cos sen)


=
w |w| cos2 + sen2
259

De donde se obtiene finalmente

z |z|
= (cos( ) + i sen( )
w |w|

Como casos particulares de este teorema, tenemos que:

3. z n = |z|n (cos n + i sen n); formula que se establece facilmente por induccion. Se co-
noce como Formula de Moivre.

1
4. w1 = (cos i sen)
|w|

Esta formula se prueba de inmediato haciendo en 2)

z = 1 = cos0 + i sen0
1
5. z n = (cos n i sen n)
|z|n

cuya demostracion es la siguiente:

z n = (z 1 )n

que por 4) podemos escribir


 n
1 1
= (cos i sen) = n (cos() + i sen())n
|z| |z|

que por 3)

1
= (cos(n) + i sen(n))
|z|n

que finalmente

1
= (cosn i senn).
|z|n

5) tambien se puede escribir


5) z n = |z|n (cos(n) + i sen(n))

que tiene la misma estructura de 3).

Por este resultado, podemos afirmar que la formula de De Moivre vale para exponentes
enteros.
260

Ejemplo 9.13
Escribir en forma trigonometrica los siguientes complejos:

a) 1 + i

b) i
1 1
c) + 3i
2 2
Im z
Conviene calcular el angulo usando tg = .
Re z
Para el caso a) tg = 1, por lo tanto = /4 o 5/4. (45 o 225 ). Pero 1 + i es un complejo
del primer cuadrante, por ser su parte real e imaginaria positivas. Luego, el valor de es /4.
( 45 ).

Entonces

1 + i = |1 + i|(cos /4 + i sen /4)



= 2(cos /4 + i sen /4)
b) en este caso = /2 y |i| = 1. Luego, i = cos /2 + isen /2.

c) En este
caso usaremos grados.
tg = 3 que corresponde a 120 o 300 . Como el complejo tiene parte real negativa
pero imaginaria positiva debe ser un vector del segundo cuadrante. Luego, = 120 .

Entonces
1 1
+ 3i = (cos 120 + i sen120 )
2 2

El siguiente ejemplo usa un metodo que conviene conocer, util en problemas de trigo-
nometra. Este se basa en las siguientes identidades que se desprenden de inmediato de la
formula de De Moivre:

Sea z = cos + i sen. Entonces

1. 2 cos n = z n + z n

2. 2i sen n = z n z n

Ejemplo 9.14
1
Probar que cos5 = (cos 5 + 5 cos 3 + 10 cos )
24
Haciendo n = 1 tenemos que

2 cos = z + z 1
261

Elevando a la quinta potencia y desarrollando

25 cos5 = z 5 + 5z 4 z 1 + 10z 3 z 2 + 10z 2 z 3 + 5zz 4 + z 5

que simplificando y reordenado

= (z 5 + z 5 ) + 5(z 3 + z 3 ) + 10(z + z 1 )

usando 1)

= 2 cos 5 + 10 cos 3 + 20 cos

de donde obtenemos finalmente que

1
cos5 = (cos 5 + 5 cos 3 + 10 cos)
24
El proximo ejemplo aplica directamente De Moivre.

Ejemplo 9.15
Expresar cos 5 y sen 5, en funcion de cos y sen .

Sabemos que

(cos + i sen)5 = cos 5 + i sen5.

Desarrollando el primer miembro por el teorema del binomio y calculando los coeficientes
binomiales, nos queda que

cos5 + 5 icos4 sen 10 cos3 sen2 10i cos2 sen3


+5 cos sen4 + i sen5 = cos 5 + i sen 5
Igualando partes reales e imaginarias tenemos finalmente,

cos 5 = cos5 10cos3 sen2 + 5 cos sen4

sen 5 = 5 cos4 sen 10 cos2 sen3 + sen5

Ejemplo 9.16
Simplificar la expresion A = (1 i)2n (1 + i)2n donde n es un numero natural.

En el ejemplo 9.13 vimos que


 
1+i= 2 cos + i sen .
4 4
En forma analoga se obtiene que


 
7 7
1i= 2 cos + i sen .
4 4
Entonces
262

A = (1 i)2n (1 + i)2n

2n
 
7 7  
= ( 2) cos 2n + i sen 2n ( 2)2n cos 2n + i sen 2n
4 4 4 4
  
7 7
A = 2n cos n cos n + i sen n sen n
2 2 2 2
Aplicando las formulas trigonometricas que expresan sumas en productos, se tiene que
     
n 3n 3n
A = 2 2 sen(2n) sen + i 2 sen cos(2n)
2 2
3n
Pero sen 2n = 0, y cos 2n = 1. Ademas, sen = 0 si n es par, 1 si n = 3, 7, 11, y 1
2
si n = 1, 5, 9, .

Luego,


0 si n par



A= 2n+1 i si n = 3, 7,




2n+1 i si n = 1, 5,

El siguiente teorema nos dara un metodo para calcular las races n-esimas de un complejo
a partir de su forma trigonometrica.
Teorema 9.7
Sea z = |z| (cos + i sen).

Entonces, los n-numeros


 
p
n + 2k + 2k
|z| cos + i sen con k = 0, 1, , n 1
n n
son las races n-esimas de z.

Demostracion
Debemos probar que cada uno de estos numeros elevado a n da z.
  n
p
n
+ 2k + 2k
|z| cos + i sen
n n
 
p
n
+ 2k + 2k
= ( |z| cos n + i sen n
n n

= |z| (cos( + 2k) + i sen( + 2k))

= |z| (cos + i sen) = z.


263

Nota
+ 2k
Es facil ver que todos los angulos con k = 0, 1, , n 1 estan comprendidos entre
n

y + 2 siendo menores a este ultimo valor. Luego, cualquier par de ellos se diferencian
n n
en menos de 2. Esto fuerza que dos angulos de estos deben tener diferente, por lo menos,
uno de los dos valores seno o coseno. Entonces los n numeros que son las races n-esimas de z
son distintos. En el siguiente captulo se probara que un numero tiene exactamente n races
de ndice n, lo que determina que los numeros definidos en el teorema anterior son todas las
races n-esimas de z.

Ejemplo 9.17
Encontrar las races cuartas de z = 1 + i.

En el ejemplo 9.13 probamos que z = 2 (cos /4 + i sen /4). Entonces

4
q  
4
/4 + 2k /4 + 2k
z= 2 cos + i sen ,
4 4
con k = 0, 1, 2, 3.

Sean z0 , z1 , z2 , z3 las races buscadas. Tenemos que


 
z0 = 8 2 cos + isen
16 16

 
8 /4 + 2 /4 + 2
z1 = 2 cos + i sen
4 4


 
8 /4 + 4 /4 + 4
z2 = 2 cos + i sen
4 4


 
8 /4 + 6 /4 + 6
z3 = 2 cos + isen
4 4

Ejemplo 9.18
Encontrar las races quintas de la unidad y calcular, a partir de estas, el lado del pentagono
regular.
Sabemos que: 1 = cos0 + i sen0

   
5 5 2 2
1 = cos0 + i sen0 = cos 0 + k + i sen 0 + k con k = 0, 1, 2, 3, 4.
5 5

Los diferentes valores son:

z0 = cos0 + i sen0 = 1

z1 = cos 2 i sen 2
5 5
264

-
1

Figura 8.8


z2 = cos 4 + i sen 4
5 5

z3 = cos 6 + i sen 6
5 5

z4 = cos 8 + i sen 8
5 5
Estos cinco complejos son de modulo 1 y mirados como vectores forman entre s angulos de
72 . Por eso son los vertices de un pentagono regular inscrito en la circunferencia de centro
(0, 0) y radio 1, como indica la figura 8.8. El lado de este pentagono queda dado por |z1 z0 |.

Luego,

|z1 z0 | = cos 2 + i sen 2 1

5 5
r
2
= cos 2 1 + sen2 2
5 5
r

= cos2 2 2 cos2 + sen2 2 + 1
5 5 5


r

= 2 1 cos 2 .
5

Nota
Partiendo del teorema 9.7, es inmediato determinar que las races n-esimas de 1 vienen dadas
por

2k 2k
wk = cos + i sen
n n
donde k vara de 0 a n 1.

Este resultado sera aplicado en el siguiente ejemplo.


265

Ejemplo 9.19
Resuelva la ecuacion (x + 1)n = (x 1)n .

Podemos escribir la ecuacion en la forma

(x + 1)n = (x 1)n 1
y extrayendo raz n-esima

x + 1 = (x 1)wk
que nos permite escribir

x(wk 1) = wk + 1
para k = 0, wk = 1 y la ecuacion queda x 0 = 2 que no tiene solucion.

Para k > 0, wk 6= 1, y tenemos que

wk + 1 cos 2 k k
n + i sen 2 n + 1
x= =
wk 1 cos2 k k
n + i sen2 n 1
Notemos que la ecuacion propuesta es en realidad de grado n 1 lo que hace que tenga
n 1 soluciones, segun probaremos en el captulo 11.

Teorema 9.8
2 2
Sea w = cos + i sen , es decir, w es la raz n-esima de 1 que anteriormente designamos
n n
por w1 . Entonces:

i) Todas las races n-esimas de 1 son w0 , w, w2 , , wn1 .

ii) 1 + w + w2 + + wn1 = 0.

iii) Sea r un real positivo. Entonces los numeros n
rwk con 0 k n 1 son todas sus
races n-esimas.

Demostracion

i) Sea k un natural tal que 0 k n 1. Entonces

2 k
 
k 2 2 2
w = cos + i sen = cos k + i sen k
n n n n

ii) Sabemos, del captulo 2, que xn 1 se puede factorizar en (x 1)(xn1 + xn2 + + 1).
Entonces, como wn 1 = 0, tenemos que (w 1)(wn1 + wn2 + + 1) = 0. Pero
w 6= 1, luego wn1 + wn2 + + 1 = 0.

iii) Todos los numeros n rwk son distintos para los diferentes k porque los wk = wk lo son,

segun la nota que sigue al Teorema 8.7. Ademas: ( n rwk )n = ( n r)n (wk )n = r, lo que
demuestra la afirmacion.
266

Ejemplo 9.20
Calcular las races cuartas de 81.

2 2
En este caso w = cos + i sen = cos90 + i sen90 = i. Luego, las races cuartas de 1 son
4 4
0 2 3
i = 1, i, i = 1, i = i y las de 81, los complejos 3, 3i, 3, 3i.

Ejemplo 9.21 Consideremos el complejo z = 1 + i. Por el teorema del binomio se tiene:

z n = C0n C2n + C4n C6n + + i(C1n C3n + C5n C7n + . . . )

Por otra parte z = 21/2 (cos(/4) + isen(/4)) de donde z n = 2n/2 (cos(n/4) + isen(n/4)).
Igualando, finalmente obtenemos:

C0n C2n + C4n C6n + = 2n/2 cos(n/4)

y
C1n C3n + C5n C7n + = 2n/2 sen(n/4)

A continuacion nos referiremos a la interpretacion geometrica del producto y cuociente de


complejos.

t
D
 D
6  D 6
 D
 D
 D
 Dw w



*

z D
D  *
z

   
D 
    D 
  
   ) D 

 )  -  D  -
O OPPPP
1 PP 1
Figura 8.9 Figura 8.10
q
P
P t

Consideremos la figura 8.9. Sean z = |z|(cos + isen) y w = |w|(cos + i sen) los com-
plejos cuyo producto interpretaremos. Construyamos el triangulo de vertices O 1 z y posterior-
mente, sobre el vector w, el triangulo O w t directamente semejantes al primero. Afirmamos
que t = zw. Para probarlo tenemos, por la semejanza de los triangulos:

|1| |w|
= de donde|t| = |z||w|.
|z| |t|
267

Nuevamente por la semejanza,


wOt = . Luego, Arg t = + . Entonces

t = |z||w|(cos( + ) + i sen( + ))

= |z|(cos + i sen)|w|(cos + isen)

= zw

Para el cuociente z/w debemos proceder a la inversa. Construyamos primero el triangulo Ow1
y despues el triangulo Ozt directamente semejante al primero. Figura 8.10.

|w| |z|
Entonces =
|1| |t|

|z|
de donde |t| =
|w|

Ademas Art t = lo que prueba


z
que t = .
w

Un caso particular interesante de la interpretacion geometrica del producto se produce


cuando |w| = 1; es decir, cuando w = cos + i sen. En estas condiciones

zw = |z|(cos( + ) + i sen( + ))

Esta formula interpretada geometricamente nos lleva a que el producto de z por el complejo
de modulo uno, w = cos + i sen , produce una rotacion del vector z en el angulo . Como el
conjunto de todos los complejos de modulo uno forman un grupo cuando se considera provisto
de la operacion producto, dicho conjunto recibe el nombre de El grupo de las rotaciones.

Un segundo caso particular que conviene destacar en esta interpretacion geometrica, se


produce cuando = 0 o 180, es decir, si w es un real. En estas condiciones |t| = |z||w| donde
|w| es el valor absoluto de w y Argt = o + 180. Por lo tanto, t que por definicion es zw,
es un vector colineal con z, de igual sentido o sentido contrario dependiendo si w es mayor o
menor que cero. Su largo es |w| veces el largo de z.

9.7. APENDICE
9.7.1. EXTENSIONES AL CAMPO C DE ALGUNAS FUNCIONES REALES
A.- La funcion exponencial
La formula de De Moire, que convierte el producto de complejos unitarios en una suma,
sugiere la siguiente definicion para la funcion exponencial en el campo de los complejos:

Definicion 9.6
Sea z = a + bi. Definimos:
268

ez = ea (cos b + i sen b)

donde b se interpreta como la medida en radianes de un angulo.


Notemos que si z es un real, entonces b = 0 y la definicion coincide con la funcion
exponencial definida en R.

El siguiente teorema prueba la caracterstica fundamental de esta funcion.

Teorema 9.9
ez ew = ez+w

Demostracion

Sea z = a + bi y w = c + di. Entonces,


ez ew = ea (cos b + i sen b)ec (cos d + i sen d) = ea+c (cos b cos sen b sen d + i(sen b cos d +
sen d cos b)) = ea+c (cos(b + d) + i sen(b + d)).
Por otra parte, z + w = (a + c) + i(b + d).

Luego, ez+w = ea+c (cos(b + d) + i sen(b + d))


lo que concluye la demostracion.

Teorema 9.10
Algunas propiedades de la funcion exponencial.

1.- ez 6= 0 z C
2.- ez = 1 si y solo si z = 2ki con k entero.
3.- ez es periodico de periodo 2i

Demostracion

Sea z = a + bi

1.- ez = ea (cos b + i sen b)


Como ea > 0 y (cos b + i sen b) 6= 0, se obtiene el resultado.
2.- Sea ez = 1. Entonces, por la definicion se tiene que ea (cos b + i sen b) = 1 Esto es
posible si y solo si a = 0 y b = 2k. De lo cual se concluye que z = 2ki.
3.- Sea ez+w = ez ez ew = ez
ew = 1. Pero por 2) equivale a
w = 2ki
269

Observacion.

Sea z = ai. Es inmediato de la definicion que ez = cos a + i sen a. Reemplzando a por


nos queda ei = 1. Se acostumbra escribirla:

ei + 1 = 0

que muestra una relacion sorprendente entre las constantes e, i, y los neutros para la
suma y el producto. Se conoce como Formula de Cotes - Euler.

B.- Las funciones seno y coseno


Sea a R. Sabemos que

eai = cos a + i sen a

eai = cos a i sen a

Sumando obtenemos que

1
cos a = (eai + eai )
2
Restando

1 ai
sen a = (e eai )
2i
A base de estos resultados, ampliamos las funciones seno y coseno al campo C del modo
siguiente:

Definicion 9.7
Sea z C

1 iz
sen z = (e eiz )
2i

1
cos z = (eiz + eiz )
2

Las principales propiedades de esta extension de las funciones trigonometricas se resu-


men en el siguiente teorema, cuya demostracion dejamos al lector.

Teorema 9.11

1.- cos(z) = cos z y sen(z) = sen z


2.- cos(z + w) = cos z cos w sen z sen w
3.- sen(z + w) = sen z cos w + cos z sen w
270

4.- cos z = sen( 2 z) = sen( 2 + z)


5.- cos2 z + sen2 z = 1
6.- cos z = cos(z) y sen z = sen(z)
7.- sen z = 0 ssi z = k
8.- sen z y cos z son periodicas de periodo 2k

C.- La funcion Logaritmo


Es facil ver que la ecuacion ex = z con z 6= 0 tiene soluciones que difieren entre s en
2i. (Ver Teorema 9.10. Tercer Punto). A cada una de estas soluciones le llamaremos
un logaritmo de z. Pero definiremos su logartimo principal, que lo anotaremos por Lnz,
a la solucion de la ecuacion que se encuentra en el intervalo (, ].

En los siguientes ejemplos, x = p + qi representa el logaritmo que se pide encontrar.

Ejemplo 9.22
Calcular Ln(i).

ex = e(p+qi) = ep (cos q + i sen q) = i = cos 2 + i sen 2


de donde p = 0 y q = 2 .

i
Entonces, Ln(i) = 2

Ejemplo 9.23
Calcular Ln(1).

Sea ex = 1. Reemplazando, obtenemos que:

ep (cos q + i sen q) = 1.

De donde, p = 0 y q = .

Luego, Ln(1) = i.

En general: Sea z = |z|(cis), donde (, ] es su argumento.

Entonces, ex = ep+qi = ep (cos q + i sen q) = |z|(cis)


de donde p = Ln|z| y q =arg z.

Finalmente Ln(z) = x = Ln|z| + i arg z

El siguiente teorema lo dejaremos sin prueba.


271

Teorema 9.12
Sean z y w 6= 0. Entonces:

Ln(z w) = Ln(z) + Ln(w) mod 2i.

Nota
i
En general, Ln(z w) 6= Ln(z) + Ln(w). Por ejemplo, Ln(i) = .
2
i 3i
Pero Ln((1)i) = Ln(1) + Ln(i) = i + = .
2 2
3
Pero 6 (, ]. Por lo tanto, debe ajustarse por mod 2i.
2
D.- Potencias complejas
Nos limitaremos a la definicion.

Definicion 9.8
Sean z y w C con z 6= 0.

Definimos z w = ewLn(z)

Ejemplo 9.24
1
i(1+i) = e(1+i) 2 i = ie 2
272

9.7.2. EJERCICIOS
1. Expresar en forma trigonometrica los siguientes complejos:

a) 3 + i b) 3 4i

c) 14 + i d) 2i

2. Dados z = 3 cis 150 y w = 6 cis30 . Calcular:

a) zw b) z/w

c) z 6 d) w3

(Cis abrevia cos + i sen )

3. Demostrar que si z es un complejo tal que |z| < 1, entonces (1 + z) es un punto a la


derecha del eje imaginario.

4. Determinar los puntos z del plano complejo que satisfacen la relacion:

a) |z z0 | < r con r R+
b) |z 1| 2|z + 1|

z z1
c) Re =0
z z2
d ) Re z + Im z = 1

e) < arg(z z0 ) <

5. Probar que

n
(1 + i)n + (1 i)n = 2( 2 +1) cos
4
!n !n
1 + 3 i 1 i 3
6. Sea A = +
2 2

Probar que:

2 si n es multiplo de 3
A=
1 en los demas casos

7. Resolver la ecuacion:
273

1 3
a) x4 1 b) x4 = +i
2 2
1i
c) x6 = 8 d) x6 =
3+i

8. Probar que

a) sen 7 = 7 sen 56sen3 + 112 sen5 64 sen7 .


1 + sen + i cos n
   
b) = cis n
1 + sen i cos 2

9. Tomando z = cos + i sen en la identidad

1 zn
1 + z + z 2 + + z n1 = .
1z

Probar que:

sen n 21

a) 1 + 2cos + 2 cos 2 + + 2 cos(n 1) =
sen 12
cos 12 cos n 12

b) sen + sen 2 + + sen(n 1) =
2 sen 12

10. Demostrar que uniendo los puntos z1 y z2 con el origen se forma un angulo recto si y
solo si:

a) z1 /z2 es imaginario puro


b) z1 z 2 + z 1 z2 = 0

11. Calcular la longitud del lado del octagono regular inscrito en una circunferencia de radio
r usando las races octavas de 1.

12. Sea c un numero real. Describir todos los puntos que satisfacen las siguientes condiciones:

1
a) Re =c
2
b) Re z 2 = c

z z1
c) =c
z z2

13. Pruebe que:


 
npi n
a) (1 + i)n
= 2n/2cos + i sen
4 4
 n n 
b) (1 + cos + i sen)n = 2n cosn cos + i sen
2 2 2
274

14. Encuentre todos los valores de las siguientes races:


6

5
a) 8 b) 4 + 3i

1+i
c) P
3i

15. Expresar en terminos de sen y cos las funciones cos 5 y sen 6.

9.8. SOLUCIONES O SUGERENCIAS A LOS EJERCICIOS


DEL CAPITULO
Pagina 82

1. a) 2 + 9i
b) 29(1 + i)
1
c) (1 + 41i)
29
d ) 29(1 i)

e) 58
r r !
65 7 65 + 7
2. a) +i .
2 2
b) (9 + 13i)

i 3
c) i,
2
3. i) Sea z = a + bi, entonces z = a bi y z = a + bi.
ii) z + w = a + c + (b + d)i = a + c (b + d)i = a bi + c di
v) z + z = a + bi + a bi = 2a
z z = a + bi (a bi) = 2bi
vi) z real ssi b = 0 ssi z = z
vii) z z = (A + bi)(a bi) = a2 + b2

|2 3i|4 |1 i|3 132 ( 2)3
4. a) =
|5 + i| 26
10
10
b) |1 i| = ( 2) = 32

c) Sea z = 1 i. Entonces z 2 = 1 i de donde |z|2 = |1 i|. Luego, |z| = 4 2.
p
i i2 4i(1 + i)
5. a) x = = i 1 i
2i
275

x + 2x 3a + bi
b) = 1 + i. Sea x = a + bi. Entonces la ecuacion queda en 2 = 1 + i.
xx a + b2
Luego,

3a
=1
a2 + b2
b
=1
a + b2
2

De lo que resulta a = 3/10 y b = 9/10.

a + bi a + bi c di ac + bd + i(bc ad)
6. = = .
c + di c + di c di c2 + d2
Entonces es real ssi bc ad = 0.

7. Sean z y w U . Entonces |zw| = |z||w| = 1. Luego, z + w U .


1 |1| 1
z |z| = 1. Luego,
= U.
2
 
z w z z w w
8. a) 2 Re + Re = + + + =2
z+w z+w z+w z+w z+w z+w

   
z w z z w w
b) 2i Im + Im = 2i + =0
z+w z+w z+w z+w z+w z+w

9. Sea

z = a + bi

1 a bi
z+ = a + bi + 2
z a + b2

b
y como es real se tiene que b = 0.
a2 + b2
Pero esto equivale a b = 0 a2 + b2 = 1.

10. Es inmediato por induccion.

11. a) x3 1 = (x 1)(x2 + x + 1)
b) x4 1 = (x2 1)(x2 + 1)
c) x3 + 1 = (x + 1)(x2 x + 1)
d ) x4 + 1 = (x2 + i)(x2 i)
276

1
12. De |z| = se obtiene que |z| = 1.
|2|

1 3
Sea z = a + bi. De 1 = |1 z| sale que a = y por lo tanto, b = .
2 2

13. a) Sea z = a + bi.



Entonces
la ecuacion es a2 + b2 a bi = 1 + 2i de lo que resulta que b = 2 y
a2 + b2 a = 1 y finalmente se obtiene b = 2 y a = 3/2.
b) Es similar

14. Sea |z| = 1.


1 + it z1
Entonces z = implica que t = . Amplificando por z + 1 se tiene que
1 it i(z + 1)
Im z
t = 2 que es un real.
|z + 1|2
z
Para el caso general, notemos que z = w|z| donde w = es unitario.
|z|

Pagina 97

1. a) 2 cis 150
b) 5 cis 307

c) 197cis 4

d ) 3cis325

2. a) zw = 18cis 180 = 18
1 1
 
1 1
b) z/w = cis 120 = + 3i
2 2 2 2
c) z 6 = 36 cis 180 = 36
1 i
d ) w3 = 3 cis (90) = 3
6 6
3. |z| < 1 1 < Rez < 1. Luego, R(1 + z) > 0

4. a) Interior de la circunferencia con centro en z0 y radio r.


b) Sea z = x + yi.
Entonces |z 1| 2|z + 1| ssi 0 3x2 + 3y 2 + 10x + 3. Todos los puntos excepto
los del interior de la circunferencia.
 2
5 16
x+ + y2 = .
3 9
c) Sean z1 = a + bi, z2 = c + di y z = x + iy, entonces

z z1 (x a)(x c) + (y b)(y d)
Re =
z z2 (x c)2 + (y d)2
277

z z1
Luego, Re = 0 ssi (xa)(xc)+(y b)(y d) = 0 que equivale a la ecuacion
z z2
de una circunferencia.
d ) La ecuacion es x + y = 1, que representa una recta.

5.
 
1+i= 2 cos + i sen
4 4
 
1 i = 2 cos i sen
4 4

6.
A = cos120n + i sen 120n

+cos240n + i sen 240n

Si n = 3p;

A = cos0 +i sen0 + cos0

isen0 + 1

Si n = 3P + 1;

A = cos120 + i sen 120 + cos 240

+isen 240 = 1

Si n = 3p + 2
A = cos 240 + i sen240 + cos120 + i sen120 = 1.

7. a) 1 = cis(180 + 360k). Luego, x = cis(45 + 90k), (k = 0, 1, 2, 3)


p
b) 1/2 + i 3/2 = cis(120 + 360k)
x = cis(30 + 90k), (k = 0, 1, 2, 3)
c) 8 = 8cis (180 + 360k)

x = 6 8cis(30 + 60k), (k = 0, 1, , 5)
d)

1i 2cis 325
=
3+i 2 cis 30

1
= cis(285 + 360k)
2
1
cis(47, 5 + 60k)
x = 12 (k = 0, 1, , 5)
2
278

8. a)
(cos + i sen)7 = cos7 + i sen7

7
= cos7 + 7icos6 sen cos5 sen2

2

7 7
i cos4 sen3 + cos3 sen4
 
3 4

7 7
i cos2 sen5 cos sen6
 
+ 5 6

i sen7

Luego,
sen7 = 7cos6 sen

35 cos4 sen3 + 21cos2 sen5

sen7

Expresando cos en funcion de sen se obtiene el resultado.


b)
1 + sen i cos
= sen + i cos
1 + sen icos
 
= cis
2

9. Reemplazando tenemos que

1 + cos + cos 2 +

(1 z n )(1 z)
+cos(n 1) + i(sen + sen2 + + sen(n 1)) =
(1 z)(1 z)

1 cos + cos(n 1) cos n sen sen n + sen(n 1)


= +i
2(1 cos) 2(1 cos)

Igualando partes reales y partes imaginarias y haciendo uso de las identidades trigo-
nometricas:


a) 1 cos = 2 sen2
2
 
1
b) cos(n 1) cos n = 2 sen n sen
2 2
 
1
c) sen(n 1) sen n = 2 cos n sen
2 2

Se obtienen los resultados.


279

z1 |z1 | z1
10. a) Sea z1 = |z1 |cis z2 = |z2 |cis. Entonces = cis( ). Luego, ima-
z2 |z2 | z2
ginario puro si y solo si cos( ) = 0 que equivale a | | = 90.

b)
z1 z2 + z1 z2 = 0 |z1 ||z2 |(cis( ) + cis( )) = 0

cos( ) = 0 | | = 90.

11. Si un vertice es el complejo z0 = 1 el siguiente vertice es el complejo z1 = cis45 =


1 i
2+ 2.
2 2
Luego, el lado del octagono es


q
|z1 z0 | = 2 2

1 x
12. a) Sea z = x + iy entonces Re = 2
z x + y2
1 x
re = c ssi 2 = c que, en general, es la ecuacion de una circunferencia.
z x + y2
b) En este caso Rez 2 = x2 y 2 . Entonces Rez 2 = c ssi x2 y 2 = c que, en general, es
la ecuacion de una hiperbola.
c) Sea z1 = x1 + iy1 y z2 = x2 + iy2 . Entonces

z z1
z z2 = c ssi
(x x1 )2 + (y y1 )2 = c2 ((x x2 )2 + (y y2 )2 )

que, en general, es la ecuacion de una circunferencia.


 
13. a) 1 + i = 2 cos + i sen .
4  4 n
n n/2 n 
Luego, (1 + i) = 2 cos + i sen
4 4

15.
(cos + i sen)5 = cos 5 + i sen 5

= cos5 + 5icos4 sen 10cos3 sen2

10i cos2 sen3 + 5 cos sen4 + i sen5

Luego,

cos 5 = cos5 10cos3 sen2 + 5cos sen4


280
Captulo 10

POLINOMIOS Y FUNCIONES
RACIONALES

10.1. INTRODUCCION
Presentaremos los polinomios como ciertas funciones del campo de los numeros complejos
en s mismo. Este modo de proceder tiene la desventaja sobre otras formas de presentacion del
tema, de dificultar la definicion de grado de un polinomio, pero en cambio se gana en menores
prerequisitos y aplicaciones mas directas sin sacrificar rigurosidad ni sencillez. Ademas, esta
presentacion se puede usar tambien en cualquier campo infinito.

Vamos a exponer las ideas generales que conducen a la definicion de polinomio que daremos
posteriormente.

Sea I(x) la funcion identidad en C; es decir, para todo x C se tiene I(x) = x. Ademas,
para cada c C consideremos la funcion c(x) definida por c(x) = c para todo complejo x.
Recordemos que este tipo de funciones se llaman constantes.

Entonces, un polinomio consistira en cualquier funcion, que se genera a partir de I(x) y


de las funciones constantes, por las operaciones de suma, resta y producto realizadas entre
ellas. Estas funciones tendran una cierta forma que sera de la que nos aprovecharemos para
definirlas.
Definicion 10.1
Diremos que P es un polinomio si y solo si P es una funcion de C en C tal que P (x) ad-
mite una representacion de la forma a0 + a1 x + + an xn donde n es un numero natural y
a0 , a1 , , an son numeros complejos a los que llamaremos coeficientes del polinomio. El de
mayor subndice que sea distinto de cero se llamara el coeficiente principal de la representacion.

Nota
Observemos que dos polinomios P y Q son iguales si y solo si, son iguales como funciones, es
decir, para todo x complejo P (x) = Q(x).

Esta definicion acepta como polinomio la funcion cero porque ella admite entre otras las si-
guientes representaciones: 0; 0 + 0x; 0 + 0x + 0x2 ; etc.

281
282

Como un ejemplo de polinomio tenemos la siguiente funcion de C en C.

P (x) = 2 + (3 + i)x + 0x2 + (5)x3 + (2i)x4 + 0x5


El coeficiente principal de la representacion es 2i. Esta funcion se puede escribir mas breve-
mente

2 + (3 + i)x 5x3 + 2ix4


y tambien en la forma

2 + 3x 5x3 + i(x + 2x4 )


Nuestro primer objetivo sera probar que un polinomio no puede tener dos representaciones
distintas de la forma indicada en la definicion salvo, posibles coeficientes del valor cero y de
orden mayor al del coeficiente principal. Para lograrlo necesitaremos desarrollar un teorema
y algunos corolarios.

Teorema 10.1
Sea P un polinomio que admite una representacion de la forma a0 + a1 x + + an xn , en la
cual los coeficientes son numeros reales, n 1 y an > 0. Sea m R+ . Entonces existe b R
tal que x > b implica P (x) > m.

Demostracion
Lo demostraremos por induccion sobre n. Sea P (x) = a0 + a1 x con a1 > 0.
m a0
Tomemos b =
a1
m a0
Si x > b se tiene que x > y luego a1 x + a0 > m.
a1
por lo tanto,el teorema vale para n = 1.

Supongamos el teorema vale para los polinomios que tienen una representacion del tipo a0 +
a1 x + + an xn con an > 0 y sea P un polinomio que tiene una representacion del tipo

a0 + a1 x + + an xn + an+1 xn+1 con an+1 > 0.


Sea Q(x) = a1 + a2 x + + an+1 xn .

Por la hipotesis de induccion, existe un b0 tal que x > b0 implica Q(x) > m.

Sea b un real mayor o igual que b0 y tambien mayor que (m a0 )/m. Sea x > b.

Como P (x) = a0 + x Q(x), tenemos que, por ser Q(x) > m, P (x) > a0 + xm. Como
m a0
x> , podemos escribir
m
m a0
P (x) > a0 + m = m,
m
con lo cual queda demostrado el teorema.
283

El siguiente ejemplo muestra como se puede obtener el numero b siguiendo la metodologa


dada en este teorema.

Ejemplo 10.1
Sea P (x) = 2x3 + 3x2 7x 30 y supongamos que se nos pide encontrar un numero real b tal
que x > b implique P (x) > 10. Para estos efectos aplicaremos el teorema recien demostrado.

Por ser su demostracion inductiva, debemos encontrar b1 para el polinomio P1 (x) = 2x + 3,


despues un b2 para P2 (x) = 2x2 + 3x 7 y por ultimo, b para P (x).

Es facil ver que P1 (x) > 10 cuando x > 3, 5. Luego, b1 = 3, 5. Segun la demostracion del
teorema, b2 puede ser el
 
10 + 7
max 3, 5,
10
Podemos tomar para b2 = 3, 5 con lo cual obtenemos que x > 3, 5 implica que P2 (x) > 10.
10 + 30
Finalmente, b puede ser el max 3, 5, , que da 4. Luego, si x > 4 se tiene que P (x) > 10.
10
El teorema 1 muestra que si an > 0 el polinomio P crece sin lmite, vale decir, dado cualquier
numero m por muy grande que sea, siempre existe un intervalo de la forma (b, ) tal que
para todo x en ese intervalo, P (x) > m. Esta idea se acostumbra a resumir en la formula
lm P (x) = que se lee:
x
El lmite de P (x) cuando x tiende a infinito es infinito

Corolario 10.1.

Sea P (x) = a0 + a1 x + + an xn con n N, a0 , a1 , , an numeros reales y an < 0. Sea


m R . Entonces existe b R tal que x > b implica P (x) < m.

Demostracion
La funcion H(x) = P (x) es un polinomio con su ultimo coeficiente positivo. Por el teorema
anterior existe b R tal que x > b implica H(x) > m, es decir, x > b implica P (x) > m,
que equivale a P (x) < m.

Corolario 10.2.

Sea P un polinomio que admite una representacion de la forma c0 + c1 x + + cn xn donde


los coeficientes son numeros complejos, n N y cn 6= 0. Entonces existe un real r tal que
P (r) 6= 0.

Demostracion
Sea ck = ak + ibk . Entonces

P (x) = a0 + a1 x + + an xn + i(b0 + b1 x + + bn xn )
284

Como cn 6= 0 se tiene que an 6= 0 o bn 6= 0. Por el teorema 1 o el corolario 1, existe un real r


tal que a0 + a1 r + + an rn 6= 0 o b0 + b1 r + + bn rn 6= 0.

Luego, P (r) = a0 + a1 r + + an rn + i(b0 + b1 r + + bn rn ) es un complejo cuya parte real


o parte imaginaria no es cero, de donde P (r) 6= 0.

Corolario 10.3.

Sea P un polinomio con la representacion a0 + a1 x + + an xn , con coeficientes complejos y


tal que para todo x C se tiene que P (x) = 0. Entonces a0 = a1 = a2 = = an = 0.

Demostracion
Si alguno de los coeficientes no fuera cero, por el corolario 2, habra un r real tal que P (r) 6= 0.

El siguiente resultado es uno de los mas utiles de la teora de los polinomios. Aparte
de permitirnos definir lo que entederemos por su grado , da origen al metodo llamado de
los Coeficientes Indeterminados, de multiples aplicaciones en la resolucion de problemas.
Nosotros hemos usado este metodo, pero sin mencionarlo en el ejemplo 2.12 del captulo 2,
cuando hemos consentido en igualar los coeficientes de xn de las dos expresiones.

Corolario 10.4.

Sean

P (x) = a0 + a1 x + + an xn con an 6= 0

Q(x) = b0 + b1 x + + bm xm con bm 6= 0

dos polinomios con coeficientes complejos tales que para todo x complejo se tiene que P (x) =
Q(x). Entonces m = n y a0 = b0 , a1 = b1 , , an = bm .

Demostracion
Podemos suponer que n m.

Entonces P (x) Q(x) tiene la representacion

(a0 b0 ) + (a1 b1 )x + + (am bm )xm + am+1 xm+1 + + an xn .


Por la hipotesis, es igual a cero para todo x C. Luego, por corolario 3,

a0 b0 = a1 b1 = = am bm = 0
y am+1 = = an = 0.

Como, por la hipotesis an 6= 0, solo se evita la contradiccion aceptando que m = n. Entonces,


de la primera cadena de igualdades sale que

a0 = b0 , a1 = b1 , , an = bn .
285

Este corolario nos dice que cada polinomio distinto de cero, tiene una unica representacion
del tipo a0 + a1 x + + an xn . Esto nos permite dar la siguiente definicion:

Definicion 10.2
Sea P (x) = a0 + a1 x + + an xn un polinomio. Definimos el grado de P como el exponente
de la mayor potencia de x con coeficiente distinto de cero. Este coeficiente se llama coeficiente
principal del polinomio. Si todos los coeficientes son cero, el polinomio es la funcion cero, a la
cual no le asignaremos grado. Al grado del polinomio P lo designaremos por grado P .

Ejemplo 10.2

1. P (x) = 2 + 3x2 + 5x3 7x5 es de grado 5.

2. P (x) = 2 + 3i es de grado cero

3. P (x) = 0 + 0x + 0x2 no tiene grado

Teorema 10.2
Sean

P (x) = a0 + a1 x + + an xn

Q(x) = b0 + b1 x + + bm xm
dos polinomios con coeficientes complejos de grado n y m respectivamente y supongamos que
n es mayor o igual que m. Entonces:

(P Q)(x) es un polinomio de grado a lo mas n, o bien es el polinomio cero.

(P Q)(x) es un polinomio de grado n + m.

Las demostraciones son sencillas. Las dejamos al lector.

Ejemplo 10.3
Sea P (x) = a0 + a1 x + a2 x2 con a2 6= 0 y Q(x) = b0 + b1 x + b2 x2 + b3 x3 + b4 x4 con b4 6= 0,
entonces

i) (P Q)(x) = (a0 b0 ) + (a1 b1 )x + (a2 b2 )x2 b3 x3 b4 x4 y grado (P Q) = 4.

ii) (P Q)(x) = c0 + c1 x + c2 x2 + c3 x3 + c4 x4 + c5 x5 + c6 x6
donde
c0 = a0 b0
c1 = a0 b1 + a1 b0
c2 = a0 b2 + a1 b1 + a2 b0
c3 = a0 b3 + a1 b2 + a2 b1
c4 = a0 b4 + a1 b3 + a2 b2
c5 = a1 b4 + a2 b3
c6 = a2 b4 .

Notemos que c6 6= 0 y por lo tanto, grado P Q = 6.


286

10.2. PROPIEDADES DE LOS POLINOMIOS


En esta seccion, estudiaremos algunas propiedades de los polinomios que son totalmente
analogas a las del los numeros naturales.

Teorema 10.3
Sean P (x) y Q(x) dos polinomios con coeficientes complejos tales que
P (x) Q(x) = 0 para todo x C. Entonces P (x) = 0 o Q(x) = 0.

Demostracion
Si P y Q son ambos distintos del polinomio cero entonces tanto P como Q tienen grado.
Por el teorema anterior, P Q tiene grado. Luego, P Q no es el polinomio 0, contradiciendo la
hipotesis.

Teorema 10.4
Sean P, Q, S polinomios con coeficientes complejos y P 6= 0. Supongamos, ademas, que
P (x) Q(x) = P (x) S(x) para todo x complejo, entonces Q = S.

Demostracion
P (x) Q(x) = P (x) S(x) es equivalente a P (x)(Q(x) S(x)) = 0. Como P (x) 6= 0 se tiene,
por el teorema 3, que Q(x) S(x) = 0.

Definicion 10.3
Sean P, Q polinomios con coeficientes complejos con Q distinto del polinomio cero. Diremos
que Q divide a P o que Q es un factor de P si y solo si existe un polinomio S tal que
P (x) = S(x) Q(x).

Nota
Es inmediato de la definicion que S tambien divide a P . Simbolizaremos por Q|P el hecho
que Q es un divisor de P . Notemos que, como P 6= 0, se tiene que Q y S tambien lo son.

Nota
Cualquier constante distinta de cero es un divisor de cualquier polinomio. Para comprobarlo
notemos que si P (x) = a0 + a1 x + + an xn y c es un complejo distinto de cero, entonces
a a1 an n 
0
P (x) = c + x + + x
c c c
Ejemplo 10.4
Los polinomios (x2 + 1) y (x 1) son factores de P (x) = x3 x2 + x 1, ya que P (x) =
(x2 + 1)(x 1). Tambien (x + i) es factor de P (x), pues es factor de x2 + 1.

Nota
Algunas factorizaciones de uso frecuente son las siguientes:

1. xn an = (x a)(xn1 + xn2 a + + an1 )

2. x2n a2n = (xn an )(xn + an )

3. x2n1 + a2n1 = (x + a)(x2n2 x2n3 a + x2n4 a2 + a2n2 )


287

4. x2n a2n = (x + a)(x2n1 x2n2 a + a2n1 ).

Las demostraciones son por induccion. Fueron vistas en el captulo 2.

Nota
En algunos casos es posible factorizar la suma de dos cuadrados. Se basa en la siguiente
identidad:

Sea c = 2ab. Entonces a2 + b2 = (a + b + c)(a + b c).
Por lo tanto, cuando cuando c es un cuadrado la factorizacion es racional. Por ejemplo
36x4 + 9 = (6x2 + 3 + 6x)(6x2 + 3 6x).

En la definicion siguiente nos referiremos a polinomios con coeficientes reales, que para
simplificar los llamaremos polinomios reales.

Definicion 10.4
Diremos que un polinomio real no constante es primo si y solo si no puede factorizarse en
polinomios reales no constantes.

Ejemplo 10.5

a) El polinomio x2 1 no es primo ya que se puede escribir como (x + 1)(x 1).

b) El polinomio x2 +1 es primo. En efecto, supongamos que no lo fuese. Entonces existiran


dos polinomios ax + b con a 6= 0 y cx + d con c 6= 0, tal que: x2 + 1 = (ax + b)(cs + d)
con a, b, c y d numeros reales. Multiplicando y usando el Corolario 10.4:

1. ac = 1

2. ad + bc = 0

3. bd = 1

Multiplicando 1) y 3):

4. adbc = 1

De 2) y 4) obtenemos que (ad)2 = 1, lo cual contradice el hecho que el cuadrado de


un numero real siempre es no negativo.

Teorema 10.5 Teorema del Resto para Polinomios

Sean P y Q dos polinomios con coeficientes complejos. Supongamos ademas que Q 6= 0.


Entonces existen dos unicos polinomios S y R tales que P = SQ + R donde el grado de R es
menor que el grado de Q o R es el polinomio 0.

Demostracion
Primero demostraremos la existencia. Es por casos.
288

Caso 1:

Si Q divide a P , es decir, si existe un S tal que P = SQ. En este caso R es simplemente


el polinomio 0.

Caso 2:

Si grado P < grado Q. En este caso tomando S = 0 y R = P se cumple la existencia.

Caso 3:

Si Q no divide a P y grado P grado Q.

Sea A = {grado (P T Q) : P ara todo polinomio T }.

A esta bien definido puesto que P T Q 6= 0 para cualquier T porque hemos supuesto que Q
no divide a P .

Como A N0 , tiene un menor elemento. Sea s este menor elemento. Entonces existe un
polinomio S tal que grado (P SQ) = s.

Sea R = P SQ.

Luego, grado R = s.

Debemos probar que grado R < grado Q.

Supongamos que s grado Q y sea

R(x) = a0 + a1 x + + as xs y

Q(x) = b0 + b1 x + + bn xn .

Tenemos que as 6= 0 y bn 6= 0.
 
as sn as
Entonces, P (x) S(x) + x Q(x) = R(x) xsn Q(x) es un polinomio cuyo grado
bn bn
es elemento de A por ser de la forma P T Q.
 
as
Pero grado R(x) xsn Q(x) < s porque al efectuarse las operaciones se cancelan los
bn
terminos en xs .

Esto contradice el hecho que s es el menor elemento de A. Luego, s < grado Q.

Demostraremos ahora, la unicidad de S y R. Sea P = SQ + R = S1 Q + R1 dos descom-


posiciones de P con
289

grado R < grado Q

y grado R1 < grado Q.


Restando se tiene

R1 R = (S S1 )Q
Si S S1 6= 0, obtenemos, de la igualdad anterior que

grado (R1 R) = grado (S S1 ) + grado Q grado Q.


Por esto contradice el teorema 2. Consecuentemente S = S1 y por lo tanto, R = R1 .

Nota
El polinomio R recibe el nombre de Resto y S el Cuociente en la division de P por Q. Una
forma de encontrarlos sera explicada en el siguiente ejemplo.
Ejemplo 10.6
Sea P (x) = 3x3 4x2 + 7x + 5 y Q(x) = x2 + 2x 1. Encontrar R(x) y S(x).

El procedimiento es el siguiente:
i) Los polinomios P y Q deben estar escritos en orden decreciente de sus potencias.
ii) El termino de mayor grado de P (x) es 3x3 y de Q(x) es x2 . Entonces determinamos el
factor que multiplicado por x2 da 3x3 . En este caso es 3x.
iii) Calculamos P (x) = Q(x)(3x) = P1 (x). Si grado P1 < grado Q el proceso termina y P1
es el resto. Si no se cambia P por P1 y se empieza otra vez de i).
iv) El polinomio S(x) es la suma de los factores que aparecen en el punto ii).
Habitualmente los calculos se realizan bajo el siguiente esquema:

a) 3x3 4x2 + 7x + 5 : x2 + 2x 1 = 3x 10
b) 3x3 + 6x2 3x
c) 10x2 + 10x + 5
d) 10x2 20x + 10
e) 30x 5

En la fila a) se coloca P (x), Q(x) y, a medida que se calculen, los factores que forman
S(x). En b) esta (3x)Q(x). En c) P (x) (3x)Q(x) = P1 (x). Se calcula el nuevo factor
que es 10. En d) 10Q(x).

En e) P1 (x) = 10Q(x) = P2 (x). Como grado P2 < grado Q se tiene que P2 = R.


Finalmente, podemos escribir que

P (x) = (3x 10)Q(x) + 30x 5.


El siguiente corolario, es el caso particular de la division en que Q(x) es de grado 1. Se
conoce como teorema del resto y sera bastante usado.
290

Corolario (Teorema del resto)

El resto al dividir P (x) por (x a) es P (a).

Demostracion
Como grado (x a) = 1 el resto es una constante. Sea r. Entonces P (x) = S(x)(x a) + r y
de esta ecuacion, dandole a x el valor a, obtenemos que P (a) = r.

Ejemplo 10.7

i) El resto al dividir P (x) = 2x4 x3 + 2x2 5 por (x 1), esta dado por P (1) =
2 14 13 + 2 12 5 = 2.

ii) El resto al dividir P (x) = x4 ix3 + (2 + i)x2 5 por (x + i) es P (i) = (i)4 i(i)3 +
(2 + i)(i)2 5 = (5 + i).

Nota
Hay un metodo, llamado division sintetica, que permite encontrar el cuociente y el resto
al dividir un polinomio por (x a). Es el siguiente:

Sea P (x) = a0 +a1 x+ +an1 xn1 +an xn . Entonces, por el teorema del resto podemos
escribir

P (x) = (x a)(b0 + b1 x + + bn1 xn1 ) + P (a),

de donde, igualando los coeficientes, obtenemos las relaciones:

bn1 = an
bn2 = an1 + abn1
..
.
bnk = ank+1 + abnk+1
..
.
b1 = a2 + ab2
b0 = a1 + ab1

P (a) = a0 + ab0

Una forma practica de efectuar el calculo de los coeficientes bi se muestra a continuacion:

an an1 ank+1 a2 a1 a0

a abn1 abnk+1 ab2 ab1 ab0

Sumas bn1 = an bn2 bnk b1 b0 P (a)


291

El cuadro se obtiene mediante el siguiente proceso:

Se escriben los coeficientes de P (x) en la primera lnea. En la tercera lnea se escribiran


sucesivamente los coeficientes del cuociente, el primero de los cuales, bn1 , es an . Se
calcula abn1 y se escribe en la segunda lnea debajo de an1 . La suma de ellos dos es
bn2 y se ubica en la tercera lnea. Se continua este proceso hasta obtener a0 + ab0 que
es el resto.

Ejemplo 10.8
Dividamos P (x) = 5x4 (2 + i)x2 + 3ix + 2 por (x 2) usando la division sintetica:

5 0 (2 + i) 3i 2

2 10 20 36 2i 72 + 2i

5 10 18 i 36 + i 74 + 2i

Entonces se obtiene el cuociente 5x3 + 10x2 + (18 i)x + (36 + i) y el resto 74 + 2i.

A menudo es conveniente expresar un polinomio en potencias de (x c). Para realizarlo


conviene usar el metodo que sigue.

Sea P (x) un polinomio de grado n y supongamos que lo hemos expresado en la forma

c0 + c1 (x c) + c2 (x c)2 + + cn (x c)n .

Entonces P (x) = c0 + (x c)P1 (x)


donde P1 (x) = c1 + c2 (x c) + + cn (x c)n1 ;
a su vez P1 (x) = c1 + (x c)P2 (x)
donde P2 (x) = c2 + + cn (x c)n2 ;

y as sucesivamente. De donde resulta que c0 y P1 (x) son, respectivamente, el resto y el


cuociente al dividir P (x) por (x c). Analogamente, c1 y P2 (x) son el resto y el cuociente al
dividir P1 (x) por (x c), etc. La forma practica de disponer el calculo la ilustraremos con el
siguiente ejemplo.

Ejemplo 10.9
Expresemos P (x) = x4 2x3 x 3 en potencias de (x 2).
292

1 2 0 1 3
2 2 0 0 2

1 0 0 1 5 = c0
2 2 4 8

1 2 4 7 = c1
2 2 8

1 4 12 = c2
2 2

1 6 = c3

1 = c4

Luego,:

x4 2x3 x 3 = (x 2)4 + 6(x 2)3 + 12(x 2)2 + 7(x 2) 5.


El teorema 10.5 tiene muchas consecuencias interesantes. A continuacion expondremos
algunas que se refieren a divisibilidad entre polinomios reales. En los dos teoremas siguientes
se supondra que todos los polinomios que intervienen son reales.

Teorema 10.6
Sea P un polinomio primo y L y M dos polinomios ambos de grado menor que grado P .
Entonces P no es un factor del producto LM .

Demostracion
Supongamos que no, es decir supongamos que existen polinomios L y M tales que ambos son
de grados menores que el de P y ademas P |LM .

Sea H un polinomio de menor grado posible y tal que P |LH. Es facil ver que tal polinomio
debe existir ya que no es vaco el conjunto de los polinomios que multiplicados por L producen
uno divisible por P .

H no es una constante pues sera grado L grado P .

Luego, grado H 1.

Por el teorema 10.5, existen S y R tales que P = SH + R con grado R < grado H. R 6= 0
porque en caso contrario H sera un divisor de P , el cual es primo. Entonces, multiplicando
por L y reordenando se obtiene que: P L SLH = LR.

El primer miembro de esta igualdad es divisible por P , luego P |LR. Pero esta conclusion
contradice el hecho que H era un polinomio de grado mnimo que satisfaca esa relacion. Esta
contradiccion establece el teorema.
293

Ahora es sencillo establecer el siguiente resultado.

Teorema 10.7
Si P es primo y divide al producto LM entonces P |L o P |M .

Demostracion
Por contradiccion. Supongamos que P divide al producto pero no divide ni a L ni a M . Por
el teorema 10.5 podemos escribir L = S1 P + R1 y M = S2 P + R2 con R1 y R2 distintos de
cero y con grados menores al de P .

Multiplicando LM = (S1 S2 P + S1 R2 + S2 R1 )P + R1 R2 .

Como P |LM tenemos que P |R1 R2 . Pero esta conclusion contradice al teorema anterior.

Otra de las consecuencias mas utiles del teorema 10.5 es el siguiente algoritmo aplicable
a cualquier par de polinomios.

Teorema 10.8 Algoritmo Euclideano.

Dados dos polinomios P y Q existe un proceso que conduce a la obtencion de un polinomio


que lo designaremos D, que tiene las tres propiedades que a continuacion se indican:

a) D|P y D|Q

b) Existen polinomios M y N tales que D = M P + N Q.

c) Si F |P y F |Q entonces F |D.

Demostracion
El proceso en cuestion es el siguiente:

1. Por el teorema 10.5 existen polinomios S1 y R1 tales que P = S1 Q + R1 . Si R1 = 0 se


toma D = Q y el proceso termina.

2. Si R1 6= 0 hay polinomios S2 y R2 tales que Q = S2 R1 + R2 . Si R2 = 0 se toma D = R1


y el proceso termina.

3. Si R2 6= 0 existen polinomios S3 y R3 tales que R1 = S3 R2 + R3 .

Si R3 = 0 se toma D = R2 y el proceso termina. Si no, se continua de la misma manera


hasta lograr resto cero. Este resto debe obtenerse en algun paso porque grado Q > grado
R1 > grado > R2 > 0 lo que prueba que, en el peor de los casos, se tendra un
resto Rl de grado 0, es decir Rl sera una constante. Como una constante es un divisor
de cualquier polinomio el resto siguiente sera cero.

Se tomara como D el ultimo resto distinto de cero que se haya obtenido.


294

Debemos probar que D cumple con las propiedades a), b) y c) de la hipotesis.

Probaremos que cumple con a) y b) por induccion sobre el numero de pasos del proceso.
Si el proceso termino en el paso 1 entonces D = Q, R1 = 0 y P = S1 Q. por lo
tanto,D|Q, D|P y D = O P + 1 Q.
Supongamos que a) y b) son validas para los procesos que terminan en n pasos y consi-
deremos un proceso que concluyo en n + 1 pasos. En este caso D = Rn y Rn+1 = 0. Si
volvemos al paso 2, notamos que D es el polinomio que corresponde a Q y R1 determi-
nado en un proceso de n pasos.

Entonces, por la hipotesis inductiva D|Q, D|R1 , y existen polinomios M1 y N1 tales


que D = M1 Q + N1 R1 . Pero, al ser P = S1 Q + R1 tenemos que D|P . Multiplicando
esta igualdad por N1 y reemplazando N1 R1 por D M1 Q, que se obtiene de la ecuacion
anterior, tenemos que D = N1 P + (M1 N1 S1 )Q lo que prueba la propiedad b).
Finalmente, c) es una consecuencia inmediata de b) ya que si divide a P y Q entonces
tambien divide a M P + N Q.

Ejemplo 10.10
Dados P (x) = x4 + x3 + 5x2 + 3x + 6 y Q(x) = x4 x3 + x2 3x 6 encontrar el polinomio
D(x) introducido en el teorema anterior.

Por division encontramos que

1. P (x) = Q(x) + 2x3 + 4x2 + 6x + 12 de donde obtenemos S1 (x) = 1 y R1 (x) = 2x3 +


4x2 + 6x + 12. La division de Q por R1 nos da el siguiente paso.
 
1 3 1 3
2. Q(x) = x R1 (x) + 4x2 + 12 de donde obtenemos S2 (x) = x y R2 (x) =
2 2 2 2
4x2 + 12.
El tercer paso proviene de la division de R1 por R2 .
 
1 1
3. R1 (x) = x + 1 R2 de donde obtenemos S3 (x) = x + 14 y resto 0. Luego, D(x) =
2 2
R2 (x) = 4x2 + 12.

Para expresar D(x) en funcion de P (x) y Q(x), como asegura la propiedad b) del teorema
anterior, procedemos de la siguiente manera:

De 1) obtenemos que

R1 (x) = P (x) Q(x)


Reemplazandolo en 2)
 
1 3
Q(x) = x (P (x) Q(x)) + R2 (x)
2 2
y en ella despejando R2 , que es D, se obtiene
295

   
1 3 1 1
D(x) = x+ P (x) + x Q(x).
2 2 2 2
Definicion 10.5

a) Un polinomio D distinto de cero, se llama un maximo comun divisor (en adelante,


m.c.d.) de dos polinomios P y Q, ambos distintos de cero, si y solo si tiene las siguientes
propiedades:

i) D|P y D|Q
ii) Si H es un divisor comun de P y Q entonces H tambien es divisor de D.

b) Se define el m.c.d. de P y Q como el polinomio monico, es decir, con coeficiente


principal 1, que es m.c.d. de P y Q.

El siguiente teorema, cuya demostracion dejamos de ejercicio, da propiedades de los maxi-


mos comunes divisores.

Teorema 10.9

i) El polinomio D descrito en el teorema 10.8 es un m.c.d. de P y Q.


ii) Si H y E son dos m.c.d. de P y Q entonces existe una constante c tal que H = cE.

Definicion 10.6
Diremos que dos polinomios P y Q son primos relativos si y solo si su m.c.d. es 1. Tambien
se dice que P es primo con Q, o bien P y Q son primos entre s.

Ejemplo 10.11
Sea D el m.c.d. de P y Q. Entonces existen polinomios P1 y Q1 tales que P = P1 D y Q = Q1 D.
Es facil ver que P1 y Q1 son primos relativos.

Teorema 10.10
Sea H un divisor del producto P Q y supongamos que H y P son primos relativos. Entonces
H|Q.

Demostracion
Por el teorema 10.8, existen dos polinomios M y N tal que 1 = M H + N P . Multiplicando
por Q nos queda que

Q = M HQ + N P Q.
El segundo miembro es divisible por H, luego H|Q.

Definicion 10.7
Un polinomio monico M se llama el mnimo comun multiplo de P y Q si y solo si tiene las
siguientes propiedades

i) P |M Q|M
ii) Si H es un polinomio distinto de 0 tal que P |H y Q|H entonces H|M .

Propiedades del mnimo comun multiplo se veran en los ejercicios.


296

10.2.1. EJERCICIOS
1. De un ejemplo de dos polinomios P y Q tales que el grado de P + Q sea menor que el
grado de cualquiera de los sumandos.

2. Pruebe que si P es primo y c es una constante, entonces c P (x) es primo.

3. Pruebe que x2 + x + 1 es primo.

4. Averigue si son primos los siguientes polinomios:

a) x2 + x + 2
b) x3 2
c) x4 + 1

5. Pruebe que:

a) x4 + 3x3 + 3x2 + 3x + 2 es divisible por x + 2.


b) x5 3x4 + x2 2x 3 es divisible por x 3.

6. Si a 6= b y P es divisible por x a y por x b pruebe que P (x) es divisible por el


producto (x a)(x b).

7. Encuentre el cuociente y el resto entre:

a) x7 + 3x6 + 2x3 + 3x2 x + 1 y x4 x + 1


b) x5 3x2 + 6x 1 y x2 + x + 1
c) (x + 1)7 x7 1 y (x2 + x + 1)2
d ) (n 1)xn n xn1 + 1 y (x 1)2

8. Pruebe el teorema 10.9.

9. Desarrolle:

a) x5 2 en potencias de x + 1.
b) x5 6x3 + x2 1 en potencias de x + 1.

10. Sean P (x) = 8x7 + 12x6 + 22x5 + 5x4 + 2x3 10x2 x 4 y Q(x) = 2x2 + x + 1.
Escriba P en la forma S3 Q3 + S2 Q2 + S1 Q + S0 donde los Si representan polinomios
de grado no mayor que 1. Indique como se generaliza el metodo para cualquier par de
polinomios.

11. Encontrar valores para a y b de manera que 3x3 4x2 + ax + b sea divisible por x2 1.

12. El resto cuando x2 3x + 2 divide a px4 + qx3 18x2 + 15x 5 es 4x 7. Demostrar


que p = 1 y q = 4.

13. Sea P (x) = axn b(n + 1)xn1 + x + 2.


Encontrar a y b de manera que P (x) sea divisible por x2 3x + 2.
297

14. Determinar el m.c.d. entre los siguientes polinomios:

a) 2x3 + x + 4 y x2 2x + 4
b) 2x4 3 y x3 + 2x2 + x + 1
c) x2 + x2 + x + 1 y x2 + x + 1

15. Determine un polinomio P (x) de grado mnimo que sea divisible por x2 + 1 y tal que el
polinomio P (x) + 1 sea divisible por x3 + 1.
PQ
16. Sea D el m.c.d. de los polinomios P y Q. Pruebe que el polinomio M = , donde k
kD
es una constante que se elige de modo que M sea monico, es el mnimo comun multiplo
de P y Q.

17. Factorice

a) (x x2 )3 + (x2 1)3 + (1 x)3


b) (xm+n )2 (xm an )2 (xn an ) + (am+n )2
c) x4 + a4 2(x2 + a2 ) + 1

10.3. FUNCIONES RACIONALES


En esta seccion estudiaremos las funciones racionales y su descomposicion en fracciones
parciales. Ocuparemos un resultado que sera establecido en el captulo siguiente, a saber: Un
polinomio de grado n no puede tener mas de n races, es decir, si P (x) es un polinomio de
grado n, entonces existen a lo mas n numeros complejos r1 , r2 , , rl tales que
P (ri ) = 0(i = 1, 2, , l). Los numeros r1 , r2 , , rl se llaman las races del polinomio.

Definicion 10.8
Diremos que una funcion es racional si y solo si es de la forma P (x)/Q(x) donde P y Q
son polinomios reales, este ultimo distinto de cero. El dominio de definicion de esta funcion
consistira de todos los numeros reales menos las races de Q. Designaremos por letras tales
como p, q, r, s, t, a las funciones racionales.

Nota
Recordemos que dos funciones f y g son iguales si y solo si tienen el mismo dominio y para
todo x elemento de este, se cumple que f (x) = g(x). Pero, para las funciones racionales,
como en general sus dominios son distintos, ya que pueden diferenciarse en las races de
sus denominadores, se aceptara que r y s son iguales si y solo si para todo x del dominio
de ambas, se cumple que r(x) = s(x). Rigurosamente se dice que son iguales en casi todas
partes, aludiendo a que el numero de puntos en que pueden discrepar es finito. Si r y s son
iguales en casi todas partes, las no coincidencias que pueden producirse son en puntos en que
una esta definida pero no la otra. El siguiente ejemplo aclarara este analisis.

Ejemplo 10.12
2(x2 1)x 2x3 2x 2x
Sea r(x) = 2 2
= 4
y s(x) = 2 .
(x + 1)(x 1) x 1 x +1
298

El dominio de definicion de r es R {1, 1} mientras que el de s es R. Notemos que si x


es cualquier punto del dominio de r se tiene que r(x) = s(x); pero se diferencian en que r
no esta definida en 1 ni en -1 mientras que s si lo esta (s(1) = 1, s(1) = 1). Bajo estas
circunstancias escribiremos que r = s y diremos que son iguales en casi todas partes.

Definicion 10.9
P (x)
Diremos que la funcion racional esta en forma canonica si y solo si P y Q son polinomios
Q(x)
primos relativos y Q es monico.

En el ejemplo 10.8, vimos dos representaciones distintas de una misma funcion racional. La
forma s(x) es monica.

Teorema 10.11
Cada funcion racional tiene una sola forma canonica.

Demostracion
M P
Si y son dos formas canonicas para una misma funcion racional, entonces para casi
N Q
M (x) P (x)
todos los x R se tiene que = , de donde M (x)Q(x) = N (x)P (x), es decir
N (x) Q(x)
(M Q N P )(x) = 0 para casi todos los x R.

Pero como un polinomio de grado n tiene a lo mas n races, segun probaremos en el siguiente
captulo, se obtiene que M Q N P es el polinomio cero. Luego, (M Q)(x) = (N P )(x) para
todo real x. Ademas, como N |N P se tiene que N |M Q pero por ser N y M primos relativos
se concluye que N |Q.

En forma simetrica se prueba que Q|N , lo que fuerza que Q = N . Pero, por ser monicos, su
coeficiente principal es 1. Luego, Q = N y por cancelacion se obtiene que M = P .

La forma canonica para la funcion racional S/T se consigue de inmediato simplificando por
bm D donde bm es el coeficiente principal de T y D es el m.c.d. entre S y T .

Es facil comprobar, y lo dejamos al lector, que las funciones racionales forman un campo.
Probaremos que este campo es ordenado aunque no arquimidiano. El interes de esto residira en
que las funciones racionales seran un ejemplo no trivial de esta naturaleza. Para introducir el
orden necesitamos previamente un teorema.

Teorema 10.12
P (x) M (x)
Sea r(x) = una funcion racional y su forma canonica. Sean p, q y m los coeficientes
Q(x) N (x)
principales de P, Q y M respectivamente. Entonces m es positivo si y solo si p y q tienen el
mismo signo.

Demostracion
P
Sea D el m.c.d. de P y Q. En el teorema anterior vimos que simplificandopor qD se obtiene
Q
la forma canonica. por lo tanto,podemos escribir que qDM = P . Igualando los coeficientes
299

p
principales de ambos miembros obtenemos que qm = p, es decir que = m, lo que prueba el
q
teorema.
p
Notemos que el valor es independiente de los polinomios P y Q elegidos para representar
q
la funcion r(x). Esto nos permite dar la siguiente definicion.

Definicion 10.10
P (x)
Diremos que la funcion racional r(x) = es positiva si y solo si los coeficientes principales
Q(x)
p y q de P y Q respectivamente, son de iguales signos. En caso contrario diremos que es
negativa.

La funcion racional 0 no sera ni positiva ni negativa.

Diremos que r(x) < s(x) si y solo si s(x) r(x) es positiva.

Ejemplo 10.13
2 3x2
Sea r(x) = 3 y s(x) = 2 .
x +1 x 1
Entonces tanto r(x) como s(x) son positivos. Ademas

3x2 2 3x2 (x2 x + 1) 2(x 1)


s(x) r(x) = =
x2 1 x3 + 1 (x 1)(x3 + 1)

3x4 3x3 + 3x2 2x + 2


= ,
x4 x3 + x 1
lo que prueba que s(x) > r(x).

Para que nuestra definicion de orden sea compatible con las operaciones algebraicas y
as tener un campo ordenado, debemos probar que sumas y productos de funciones racionales
positivas son positivas y que se cumple la tricotoma. Esto es lo que establece el siguiente
teorema cuya demostracion la dejaremos de ejercicio.

Teorema 10.13
Sea s(x) y r(x) funciones racionales positivas. Entonces s(x) + r(x) y s(x)r(x) son positivas.
Ademas, si t(x) es una funcion racional, una y solo una de las tres alternativas siguientes es
verdadera:

t(x) es positiva; t(x) es positiva; t(x) = 0.


Probaremos, ahora, que este campo ordenado no es arquimidiano. Primero definamos este
concepto.

Definicion 10.11
Diremos que un campo ordenado K es arquimidiano si y solo si para todo par de elementos
positivos a y b tales que a < b existe un numero natural n tal que na > b. (na significa
a + a + a + n veces).

El siguiente ejemplo prueba la afirmacion anterior.


300

Ejemplo 10.14
2x2 x2
Sea s(x) = 2 y r(x) = 2 .
x +x1 (x + x 1)(x + 1)
Es facil ver que ambas son positivas y que r(x) < s(x). Sea n un numero natural. Entonces

2x2 (x + 1) nx2 2x3 + (2 n)x2


s(x) nr(x) = =
(x2 + x 1)(x + 1) (x2 + x 1)(x + 1)
que es una funcion racional positiva. Luego, para ningun numero natural n se cumple que
nr(x) > s(x), lo que prueba que este campo ordenado no es arquimidiano.

10.4. FRACCIONES PARCIALES


En la ultima parte de este captulo desarrollaremos el tema que se acostumbra designar
por Descomposicion en Fracciones Parciales. Se trata de expresar una funcion racional como
una suma de funciones racionales mas simples. Basta limitarse a funciones racionales propias,
es decir funciones racionales en que el numerador es de grado inferior al del denominador. Si
P/Q no es propia, sabemos que existen S y R, este ultimo de grado inferior al de Q, tal que
P = SQ + R. Entonces P/Q = S + R/Q siendo esta ultima una funcion racional propia y la
descomposicion de P/Q sera S mas la que se obtenga para R/Q.

Dejamos como ejercicio probar que la suma, resta y producto de funciones racionales propias
es tambien propia y que r(x) no puede tener representaciones propias e impropias.

El teorema fundamental del tema es el siguiente.

Teorema 10.14
Sea P/Q una funcion racional propia y supongamos que Q = ST con S y T primos relativos.
Entonces existen funciones racionales propias X/S y Y /T tales que

P X Y
= +
Q S T

Demostracion
P
Por ser S y T primos relativos, existen M y N tales M S + N T = 1. Entonces =
Q
P (M S + N T ) PM PN
= + .
ST T S
Si estas dos ultimas fracciones no son propias podemos escribir que P N = U S + X P M =
P X Y
V T + Y y de donde obtenemos que = U + + V + . Como P/Q es propia tenemos que
Q S T
U + V = 0, con lo cual queda establecido el teorema.

Nota
X Y
Se acostumbrara decir que + es una descomposicion en fracciones parciales de P/Q.
S T

Ejemplo 10.15
301

3x2 + 8
Descomponer en fracciones parciales .
(x2 + 1)(x 2)
X Y
Como (x2 +1) y (x2) son primos relativos, la descomposicion sera de la forma +
x2+1 x2
con X de grado a lo mas uno e Y una constante.

Sea X = ax + b; Y = c. Entonces

3x2 + 8 ax + b c
1. 2
= 2 +
(x + 1)(x 2) x +1 x2

Para determinar las constantes a, b y c podemos proceder del modo siguiente:

Multiplicando 1) por (x2 + 1)(x 2) se tiene

2. 3x2 + 8 = (ax + b)(x 2) + c(x2 + 1).

Ahora hay dos maneras de continuar.

Una consiste en realizar los productos e igualar los coeficientes, obteniendose un sistema
de tres ecuaciones con tres incognitas. La otra manera reside en formar un sistema de
ecuaciones asignandole valores numericos a x, tantos como constantes desconocidas se
tengan. Procederemos de acuerdo a este metodo.

x = 0; 8 = 2b + c

x = 1; 11 = a b + 2c

x = 2; 20 = 5c

Este sistema tiene las soluciones

a = 1, b = 2, c = 4.

Luego, la descomposicion

3x2 + 8 4 x+2
= .
(x2 + 1)(x 2) x 2 x2 + 1

Veremos en el captulo siguiente, que siempre se puede factorizar un polinomio en expre-


siones de la forma (xa)n o (x2 +bx+c)n . Aplicaremos el teorema anterior a estos casos.

I Caso.

Si Q = (x a)n S con (x a) y S primos relativos, entonces

P a1 a2 an Y
= + + + + .
Q x a (x a)2 (x a)n S
302

Demostracion

Por el teorema anterior hay polinomios X e Y tales que

P X Y
= + con grado X < n.
Q (x a)n S

Al igual que en el ejemplo 10.8, expresemos X en potencias de (x a)

X(x) = a1 (x a)n1 + a2 (x a)n2 + + an .

No aparecen potencias mayores puesto que grado X < n. Reemplazando X en la igual-


dad anterior se obtiene el resultado.

II Caso.

Si Q = (x2 + bx + c)n S con x2 + bx + c y S primos relativos. En este caso

b1 x + c1 b2 x + c2 bn x + cn Y
X= + 2 + + 2 + .
x2 + bx + c (x + bx + c) 2 (x + bx + c)n S

Demostracion
P X Y
Al igual que anteriormente = 2 n
+ con grado X < 2n.
Q (x + bx + c) S

Segu el ejercicio 10 de la primera seccion podemos expresar X en la forma

X = (b1 x + c1 )(x2 + bx + c)n1 + (b2 x + c2 )(x2 + bx + c)n2 + + bn x + cn

y reemplazandolo en la igualdad inicial se obtiene el resultado.

Ejemplo 10.16
x3 19x 15
Expresar r(x) = como una suma de fracciones parciales.
(x2 + 1)(x + 2)3
Segun los dos casos, podemos esribir que

bx + c a1 a2 a3
r(x) = 2
+ + 2
+
x + 1 x + 2 (x + 2) (x + 2)3
Amplificando por el denominador

x3 19x 15 = (bx + c)(x + 2)3 + a1 (x2 + 1)(x + 2)2 + a2 (x2 + 1)(x + 2) + a3 (x2 + 1)

Debemos formar un sistema de 5 ecuaciones:


303

x = 0 ; 15 = 8c + 4a1 + 2a2 + a3
x = 1 ; 3 = cb + 2a1 + 2a2 + 2a3
x = 2 ; 15 = 5a3
x = 3 ; 15 = 3b c + 10a1 10a2 + 10a3
x = 1 ; 33 = 27b + 27c + 18a1 + 6a2 + 2a3

Los valores para x que hemos escogido son los que nos parecen que originan ecuaciones mas
simples.

Solucionando el sistema encontramos los valores:

b = 1, c = 2, a1 = 1, a2 = 1, a3 = 3.
Finalmente

x2 1 1 3
r(x) = 2
+ 2
+
x + 1 x + 2 (x + 2) (x + 2)3

Nota
En la descomposicion en fracciones parciales no es necesario suponer que los polinomios que
intervienen son reales. Todo lo dicho, sigue siendo valido para polinomios con coeficientes
complejos. Aceptada esta observacion, se puede simplificar bastante el sistema de ecuaciones
del ejemplo anterior, tomando para x el valor i y percatandose que los numeros buscados son
reales.

El siguiente ejemplo muestra una aplicacion de naturaleza distinta.

Ejemplo 10.17
Pruebe que

n! Cn C1n Cn
i) = 0 + + (1)n n
x(x + 1) (x + n) x x+1 x+n

y a partir de ella muestre que


C0n C1n Cn 1
ii) + + (1)n n =
1 2 n+1 n+1
C0n C1n Cn 1
iii) + + (1)n n =
2 3 n+2 (n + 1)(n + 2)

Por descomposicion en fracciones parciales podemos escribir que

n! a0 a1 ak an
1. = + + + + +
x(x + 1) (x + n) x x+1 x+k x+n

Multipliquemos 1) por x + k y despues demosle a x el valor k. Nos queda


n!
2. = ak que se transforma en
k(k + 1) 1 1 2 (n k)
304

n!
3. ak = (1)k = (1)k Ckn
k!(n k)!

Lo que prueba i). Si en esta se toma x = 1 resulta ii) y con x = 2 sale iii).

10.4.1. EJERCICIOS
1. Descomponga en fracciones parciales

2x2 + 1
a)
(x2 + 1)(x2 + x + 1)
1 + 4x + 16x2
b)
1 16x2
2x3 + 3x + 4
c)
(x + 1)(x2 + 1)

2. Descomponga en fracciones parciales

x
a)
(x a)(x b)
x2
b)
(x a)(x b)(x c)

3. Descomponga en fracciones parciales

x2
a)
(x a)(x b)
x4
b)
(x a)(x2 + a2 )

4. Descomponga en fracciones parciales

x+3
a)
(x 1)(x2 + 1)2
1
b)
(x + 1)2 (x2 + 1)2
x
c)
(x + 2)(x2 + 2x + 2)3
12x4
d)
(x2 1)(x3 1)

(x 1)(x 2) (x n)
5. Descomponga en fracciones parciales y de ella deduzca que
x(x + 1) (x + n)
(n + 1)! (n + 2)!
hasta completar n sumandos es igual a 1.
1! 2!(n 1)! 2! 3! (n 2)
305

10.5. SOLUCIONES O SUGERENCIAS A LOS EJERCICIOS


DEL CAPITULO
Pagina 126

4. a) Es primo

b) Tiene el factor (x 3 2)

c) x4 + 1 = (x2 + 2x + 1)(x2 2x + 1)

6. P (x) = (x a)Q(x) = (x b)S(x) como (x b) y (x a) son primos relativos se tiene


que Q(x) = (x b)Q0 (x).

8. i) Por el teorema 8 tenemos que D|P y D|Q y ademas existen M y N tales que
D = M P + N Q.
Si F es un polinomio que divide tanto a P como a Q, entonces, por la igualdad
anterior F |D. Luego, D es un m.c.d.

ii) Tenemos que H = SE y E = T F . Luego, E = T SE, de donde T S = 1. Por lo


tanto, T y S son constantes.

12. px4 + qx3 18x2 + 15x 5 = (x 1)(x 2)Q(x) + 4x 7.


Para x = 1 y para x = 2 resultan las dos ecuaciones

p + q 8 = 1

16p + 8q 47 = 1

cuya solucion es p = 1 y q = 4.

13. Similar al 12.

15. P (x) = (x2 + 1)Q(x)


1
(x2 + 1)Q(x) + 1 debe tener las races de x3 + 1 a saber 1, (1 i 3). Se puede suponer
2
Q(x) de grado 3.

Pagina 134

x1 x
1. a) + 2
x2
+x+1 x +1
1 3
b) 1
2(4x + 1) 2(4x 1)
5 3x 1
c) +2
2(x2 + 1) 2(x + 1)
306

x A B
2. a) = +
(x a)(x b) xa xb
a b
Resulta finalmente A = , B=
ab ab

b) En este caso:

a2
A=
(a b)(a c)

b2
B=
(b a)(b c)

c2
C=
(c a)(c b)

a2 b2
3. a) 1 +
(a b)(x a) (a b)(x b)
a2 a3 + a2 x
b) x + a +
2(x a) 2(x2 + a2 )
1 x+1 2x + 1
4. a) 2 2
x 1 x + 1 (x + 1)2
1 1 x 1 2x
b) + +
4(x + 1)2 2(x + 1) 2(x2 + 1)2 4(x2 + 1)
x+1 x x 1
c) 2 3
+ 2 2
+ 2

(x + 2x + 2) 2(x + 2x + 2) 4(x + 2x + 2) 4(x + 2)
4x 3 2 5
d) 2 + + +
x + x + 1 x + 1 (x 1)2 x 1
n
(x 1)(x 2) (x n) X Ak
5. =
x(x + 1) (x + n) x+k
k=0

Por lo tanto,

n
X
(x 1)(x 2) (x n) = (Ak x (x + k 1)(x + k + 1) (x + n)
k=0

Haciendo x = k

(1)n (k + 1) (k + n) = Ak (k)(k + 1) (1)(1) (n k)

= Ak (1)k (k!)((n k)!)

(n + k)!
Ak = (1)nk
(k!)(k!)(n k)!

Luego,
307

n
(x 1)(x 2) (x n) X (n + k)!
= (1)nk
x(x + 1) (x + n) (k!)(k!)(n k)!(x + k)
k=0

Haciendo x = 1 el primer miembro vale 0.

Luego,

n
n
X (n + k)!
0 = (1) + (1)nk
k!(k + 1)!(n k)!
k=1

como (1)n y (1)n1 son los signos distintos siempre tenemos que:

n
X (n + k)!
1= (1)k1
k!(k + 1)!(n k)!
k=1
308
Captulo 11

TEORIA DE ECUACIONES

11.1. GENERALIDADES
Dado un polinomio P , por resolver la ecuacion P (x) = 0, se entiende encontrar todos los
numeros complejos tales que P () = 0.

En este captulo desarrollaremos algunas ideas que ayudaran a encontrar dichos numeros.
Desgraciadamente no todos los teoremas que enunciaremos podremos demostrarlos y en otros
sus demostraciones requeriran de conocimientos de Calculo Elemental. No obstante, aparte
de esas demostraciones, el captulo es totalmente inteligible con las materias tratadas ante-
riormente.

No todas las ecuaciones son del tipo P (x) = 0. Una forma mas general en que con frecuencia
se presentan es r(x) = s(x) donde r y s son funciones racionales. Por resolverla se entiende
encontrar todos los numeros complejos tales que r() = s(). Estas ecuaciones, al ser de la
P (x) M (x)
forma = , tienen por soluciones las races del polinomio P (x)N (x) Q(x)M (x)
Q(x) N (x)
para las cuales N () y Q() no son ceros.

Aun hay otros aspectos bajo los cuales pueden presentarse ecuaciones y cuya resolucion con-
duzca a obtener las races de un polinomio. Las veremos en ejercicios.

Aunque el concepto de raz de un polinomio fue usado en el captulo anterior, en esa ocasion
no formalizamos su definicion. Con ella empezaremos.

Definicion 11.1
Sea P (x) = a0 + a1 x + + an1 xn1 + an xn un polinomio de grado n con coeficientes
complejos. Diremos que C es una raz de P (x) si y solo si P () = 0.

Las races de P (x) tambien se llaman las soluciones de la ecuacion P (x) = 0.

Ejemplo 11.1

a) Los numeros i y i son races del polinomio x2 + 1, ya que P (i) = P (i) = 0.



3 1 + i 3 1 i 3
b) Las races de x 1 son los numeros 1, y .
2 2

309
310

Teorema 11.1
es una raz de P (x) si y solo si (x ) es un factor de P (x).

Demostracion
Por el teorema del resto P (x) = (x )Q(x) + P (). Entonces

es raz si y solo si P () = 0

si y solo si P (x) = (x )Q(x)

si y solo si (x ) es factor de P (x).

El siguiente teorema es uno de los que quedara sin demostracion.

Teorema 11.2
Todo polinomio no constante tiene por lo menos una raz.

Es conocido con el nombre de Teorema Fundamental del Algebra.

Teorema 11.3
Un polinomio de grado n tiene a lo mas n races distintas.

Demostracion
Por induccion sobre el grado.

b
Es facil ver que el polinomio de grado 1, ax + b tiene como unica raz al numero . Supon-
a
gamos la afirmacion valida para los polinomios de grado n y sea P (x) un polinomio de grado
n+1. Por el teorema 2 existe raz de P (x) y por el teorema 1 tenemos que P (x) = (x)Q(x)
donde grado de Q(x) = n. Por la induccion, Q(x) tiene a lo mas n races distintas. Luego,
P (x) tiene a lo mas n + 1 races distintas.

Definicion 11.2
Sea una raz de P (x). Diremos que es de multiplicidad m(m N) ssi (x )m divide a
P (x) pero (x )m+1 no lo divide.

Ejemplo 11.2
Sea P (x) = 2x3 11x2 + 12x + 9 y supongamos que hemos encontrado que 3 es una raz. Para
averiguar la multiplicidad de 3 dividimos P (x) en forma sucesiva por (x 3), (x 3)2 , (x
3)3 , hasta encontrar un resto que sea distinto de cero. El siguiente esquema muestra estas
operaciones:

2 11 12 9
3 6 15 9
2 5 3 (0 = Resto)
3 6 3
2 1 (0 = Resto)
3 6
2 (7 = Resto)
311

Entonces (x 3)2 divide a P (x) pero (x 3)3 no lo divide. Luego, 3 tiene multiplicidad 2 y
P (x) = (x 3)2 (2x + 1).

Notemos que para dividir P (x) por (x 3)2 , en realidad dividimos el primer cuociente por
x 3. En forma similar se procedio para dividirlo por (x 3)3 .

Teorema 11.4
Sea P (x) un polinomio de grado n 1.

Sean 1 , 2 , , p todas sus races y sean m1 , m2 , , mp sus multiplicidades respectivas.


Entonces

m1 + m2 + + mp = n.

Demostracion
Sabemos que (x 1 )m1 , (x 2 )m2 , , (x p )mp son factores de P (x), luego:

P (x) = (x 1 )m1 (x 2 )m2 (x p )mp Q(x) (1)


Tenemos que i no es raz de Q(x), pues si lo fuera se tendra que (x i )mi +1 sera un factor
de P (x).

Ademas Q(x) no puede tener otros numeros como races, pues seran races de P (x). Luego,
para no contradecir el teorema 2, se debe tener que Q(x) es una constante. Por lo tanto, el
grado del segundo miembro de (1) es m1 + m2 + + mp y, como es igual al de P , se tiene
que esta suma vale n.

Este teorema se enuncia corrientemente diciendo que un polinomio de grado n tiene exacta-
mente n races aunque no necesariamente distintas.

Nota
Es facil ver que el polinomio Q(x) de la igualdad (1), que se demostro constante, no es mas
que el coeficiente principal de P .

En muchas ocasiones conviene escribir la igualdad (1) en la forma P (x) = an (x r1 )(x


r2 ) (x rn ) donde m1 de estos factores corresponden a (x 1 ), m2 de ellos son el factor
(x 2 ) etc. Esta factorizacion es la que justifica la expresion con que corrientemente se
enuncia el teorema 11.4.

Observacion.
La siguiente observacion es muy importante. Se conoce por Relaciones entre los coeficientes
de un polinomio y sus races.

Sea P (x) = a0 + a1 x + a2 x2 + + an xn un polinomio de grado n con coeficientes complejos


y sean r1 , r2 , , rn todas sus races no necesariamente distintas.
Sabemos que P (x) = an (x r1 )(x r2 ) (x rn )
y al igualar los coeficientes de las distintas potencias de x se obtienen las n formulas:
312

n
X an1
1. ri =
an
i=1

n1
X an2
2. ri rj = j = 2, , n 1
an
i=1
i<j

n2
X an3 j=2, ,n1
3. ri rj rk = k=3, ,n
an
i=1
i<j<k

..
.

a0
n. r1 r2 rn = (1)n
an

Las sumatorias que aparecen son respectivamente la suma de las races, la suma de los pro-
ductos de las races tomadas de dos en dos, la suma de los productos de las races tomadas
de tres en tres y as sucesivamente hasta terminar con el producto de todas las races.

A la inversa, si tenemos n numeros r1 , r2 , , rn no necesariamente distintos y formamos estas


sumatorias, los valores de ellas con los signos segun indican las formulas anteriores, podemos
considerarlos como los coeficientes b0 , b1 , , bn1 de un polinomio b0 +b1 x+ +bn1 xn1 +xn
que tendra por races los numeros r1 , r2 , , rn0 .

Ejemplo 11.3
2 2
Encontrar un polinomio que tenga por races los numeros , , i, i.
3 3
Hay dos formas de resolver este problema. La primera se basa en aprovechar la factorizacion
(1).

2 2
 
Entonces P (x) = 9 x (x i)(x + i).
3
Hemos tomado como coeficiente principal de P el numero 9 para que la ecuacion tenga todos
sus coeficients enteros.

La segunda manera de resolverlo aprovecha las formulas 1), 2), , n) y el teorema 4. Por
este teorema tenemos que el grado de P debe ser 4. Supongamos que

P (x) = a4 x4 + a3 x3 + a2 x2 + a1 x + a0
Entonces, aplicando las formulas anteriores para n = 4:

2 2 a3
1. + +ii=
3 3 a4
4 2 2 2 2 a2
2. + i i + i i i2 =
9 3 3 3 3 a4
313

4 4 2 2 a1
3. i i i2 i2 =
9 9 3 3 a4
4 a0
4. i2 =
9 a4

De donde resulta:
4
1) a3 = a4
3
 
4
2) a2 = + 1 a4
9
4
3) a1 = a4
3
4
4) a0 = a4
9
Al coeficiente a4 podemos asignarle cualquier valor. Tomando a4 = 9 nos resulta para
P (x) el polinomio 9x4 12x3 + 13x2 12x + 4.

Dado un polinomio P (x) intentar encontrar sus races resolviendo las relaciones (1), (2),
, (n) como un sistema de n ecuaciones con n incognitas no conduce a nada nuevo,
pues al ir elminando las incognitas (races) se vuelve a la ecuacion P (x) = 0. Pero este
metodo puede ser util si se conoce otra relacion entre las races, como veremos en el
siguiente ejemplo.

Ejemplo 11.4
Encuentre las races de x4 2x3 3x2 + 4x 1 sabiendo que el producto de dos de ellas es 1.

Sean a, b, c, d las races. Podemos suponer que ab = 1. Por la observacion anterior tenemos:

1. a + b + c + d = 2

2. ab + ac + ad + bc + bd + cd = 3

3. abc + abd + acd + bcd = 4

4. abcd = 1
Como ab = 1, tenemos que cd = 1 y reemplazando en 3) obtenemos:

5. c + d a b = 4
De 5) y 1) se concluye que a + b = 3 y c + d = 1 y combinandolas con ab = 1 y cd = 1
resulta finalmente:

3+ 5 3 5 1 + 5 1 5
a= ; b= ; c= ; d=
2 2 2 2
Las relaciones entre los coeficientes de un polinomio y sus races, permiten resolver una
gran variedad de problemas que ilustraremos en los siguientes ejemplos. De entre ellos
los mas importantes son los que se conocen como transformacion de ecuaciones.
314

Ejemplo 11.5
Si a, b, c son las races de x3 2x2 5x + 6 encuentre los valores de a2 + b2 + c2 y de a3 + b3 + c3 .

1. a + b + c = 2

2. ab + ac + bc = 5

3. abc = 6
Elevando al cuadrado 1) tenemos que a2 + b2 + c2 + 2(ab + ac + bc) = 4 y aprovechando
2) se obtiene que

a2 + b2 + c2 = 14

Elevando 1) al cubo se tiene que

a3 + b3 + c3 + 3a2 b + 3a2 c + 3b2 c + 3b2 c + 3ab2 + 3ac2 + 3bc2 + 6abc = 8

que es equivalente a

a3 + b3 + c3 + 3a(ab + ac + bc) + 3b(ab + ac + bc) + 3c(ab + ac + bc) 3abc = 8

de donde resulta que

a3 + b3 + c3 + 3(a + b + c)(ab + ac + bc) 3abc = 8

y finalmente

a3 + b3 + c3 = 20

Ejemplo 11.6
Resolver el sistema de ecuaciones

a + b + c = 1; a2 + b2 + c2 = 9; abc = 4
Es facil calcular que ab + bc + ca = 4, de donde resulta que a, b y c son las races de la
ecuacion x3 + x2 4x 4 = 0. Mas adelante daremos una formula para resolver las ecuaciones
de tercer grado. Por el momento, podemos encontrar por tanteo que sus races son 2, 2 y -1.

El siguiente ejemplo debe ser considerado un teorema.

Ejemplo 11.7 (Transformacion de Ecuaciones)

Sean r1 , r2 , , rn las races del polinomio


P (x) = a0 + a1 x + + an xn . Escribir un polinomio grado n, cuyas races sean:

a) r1 + h, r2 + h, , rn + h

b) r1 h, r2 h, , rn h
315

1 1 1
c) , , , ri 6= 0 i = 1, , n
r1 r2 rn
Las soluciones son las siguientes:

a) Sea Q(x) = P (x h) = a0 + a1 (x h) + + an (x h)n


Entonces Q(ri + h) = P (ri ) = 0 i = 1, , n
por lo tanto,Q es el polinomio pedido.
x x  x 2  x n
b) Sea Q(x) = P = a0 + a1 + a2 + + an
h h h h
Entonces Q(ri h) = P (ri ) = 0 i = 1, , n

c)
 
1 1 1 1
P = a0 + a1 + a2 + + an n
x x x2 x
 
1
= (a0 xn + a1 xn1 + a2 xn2 + + an )
xn
Sea Q(x) = a0 xn + a1 xn1 + + an .
Entonces
 
1 1 1
Q = a0 n + a1 n1 + + an
ri ri ri

1
= (a0 + a1 ri + + an rin )
rin

1
= P (ri ) = 0 i = 1, , n
rin

Ejemplo 11.8
Hallar el polinomio cuyas races son de signo contrario a las de P (x) = a0 + a1 x + a2 x2 + +
an xn .

En efecto, haciendo h = 1, en la letra b) del ejemplo 6, nos queda el polinomio

Q(x) = P (x) = a0 a 1x + a2 x2 + (1)n an xn


que sera el polinomio con la caracterstica pedida.

Ejemplo 11.9
Dado el polinomio P (x) = an xn + an1 xn1 + + a0 con races r1 , r2 , , rn , encontrar un
numero h tal que el polinomio cuyas races son r1 + h, r2 + h, , rn + h no tenga termino
en xn1 .

En el nuevo polinomio, el coeficiente de xn1 debera ser 0. Por las relaciones entre coeficientes
n n
X X an1 an1
y races tenemos que nh + ri = 0. Pero ri = de donde h = .
an nan
1 1
316

Luego, segun el ejemplo 6 caso a) tenemos que el polinomio pedido es


 
an1
P x .
nan
Apliquemos este analisis a un caso particular.
Ejemplo 11.10
Dada la ecuacion 2x3 6x2 + 10x 3 = 0, transformarla en otra sin termino de segundo grado.
6
En este caso h = = 1 de donde la ecuacion buscada es
32
2(x + 1)3 6(x + 1)2 + 10(x + 1) 3 = 0,
que al simplificarla se reduce a 2x3 + 4x + 3 = 0.
Ejemplo 11.11
Sean , , las races de a0 + a1 x + a2 x2 + a3 x3 . Encontrar el polinomio cuyas races son
2 , 2 , 2 .

Tenemos las siguientes relaciones:


a2
1. + + =
a3
a1
2. + + =
a3
a0
3. =
a3

Elevemos (1), (2) y (3) al cuadrado.

a22
1) 2 + 2 + 2 + 2( + + ) =
a23
a21
2) 2 2 + 2 2 + 2 2 + 2( + + ) =
a23
a20
3) 2 2 2 =
a23
De (1)y (2) tenemos

a22 2a1 a3
2 + 2 + 2 =
a23
De (2), (1) y (3) tenemos

a21 2a0 a2
2 2 + 2 2 + 2 2 =
a23
por lo tanto,el polinomio buscado es

a20 + (a21 2a0 a2 )x (a22 2a1 a3 )x2 + a23 x3


317

11.2. ECUACIONES DE GRADO INFERIOR A 5


Antes de seguir desarrollando aspectos generales del tema, estudiaremos algunas ecuacio-
nes en particular.

1. La ecuacion lineal o de primer grado.

Tiene la forma ax + b = 0 con a 6= 0.


Su unica solucion es b/a. Se llama lineal porque el grafico del polinomio P (x) = ax + b
es una recta.
2. La ecuacion cuadratica o de segundo grado.

Tiene la forma ax2 + bx + c = 0 con a 6= 0.


Estableceremos su solucion, por un metodo un tanto distinto del habitual.
Notemos primero que la ecuacion x2 + px + q = 0 con p = b/a y q = c/a tiene las
mismas soluciones que la original. Sustituyamos x por y + l, donde l es una constante
que elegiremos posteriormente en forma conveniente, para que la ecuacion resultante no
tenga termino en y. Nos queda y 2 + y(2l + p) + pl + q + l2 = 0.
p2
Tomando l = p/2 la ecuacion se reduce a y 2 + q = 0 de donde obtenemos
4
1 p
y= p2 4q.
2
1 p
Recordando que x = y + l nos queda que x = (p p2 4q) que expresada con los
2
coeficientes originales del polinomio

b b2 4ac
x=
2a
Observemos que en todo este argumento x representa una raz de la ecuacion y no la
variable de un polinomio. Lo mismo sucede con y. Analizando la formula final, vemos
que si b2 6= 4ac, la ecuacion tiene dos races; si b2 = 4ac tiene solo una. Si ademas
suponemos que a, b y c son numeros reales, entonces cuando b2 > 4ac las dos soluciones
son reales mientras que b2 < 4ac implica que ellas son complejas conjugadas.

Muchas cosas pueden decirse sobre la ecuacion de segundo grado, pero todas elementales
y bastante conocidas. Las propondremos de ejercicios.

En general, la ecuacion an xn + an1 xn1 + + a0 = 0 siempre se puede transformar en


una cuyo coeficiente principal sea 1, dividiendola por an y posteriormente hacer que el
an1
coeficiente de orden n 1 sea cero por medio de la transformacion x = y como
nan
vimos anteriormente.
3. La ecuacion cubica o de tercer grado.

Sin perdida de generalidad, la podemos describir como


318

1) x3 + px + q = 0.
Para resolverla seguiremos el metodo conocido como Formula de Cardano, pu-
blicada en 1545. Otras soluciones se propondran como ejercicios.
En 1) reemplacemos x por u + v donde u y v seran dos numeros que deberemos
determinar. Ella puede escribirse en la forma:
2) u3 + v 3 + (p + 3uv)(u + v) + q = 0.
Impongamosle a u y v la condicion
3) 3uv = p.
Entonces 2) queda reducida a
4) u3 + v 3 = q.
Elevando 3) al cubo
p3
5) u3 v 3 = .
27
Entonces 4) y 5) determinan un sistema de 2 ecuaciones con dos incognitas, u3 y
v 3 , de las cuales se conoce su suma y su producto. Este sistema fue resuelto en el
captulo sobre los complejos. En esta ocasion usaremos otro metodo. La ecuacion
(x u3 )(x v 3 ) = 0 tiene como soluciones u3 y v 3 . Efectuando el producto se tiene
que
6) x2 x(u3 + v 3 ) + u3 v 3 = 0
que por 4) y 5) se transforma en
p3
7) x2 + qx = 0, de donde obtenemos:
27
r
q q2 p3
u3 = + +
2 4 27
r
q q2 p3
v3 = +
2 4 27
r
q q 2 p 3
Sean A = + + y
r2 4 27
q q2 p3
B= +
2 4 27

Entonces
los posibles valores para u son 3 A, w 3 A y w2 3 A y para v, 3 B, w 3 B
y w2 3 B donde w es una raz cubica compleja de 1.

Finalmente, las soluciones de la ecuacion 2) son:



3

3

3

3
A+ B, w2 A + w B
Otras combinaciones no sirven porque u y v deben satisfacer 3) es decir 3uv = p.

3

3
En la practica se toma como A cualquiera de sus 3 valores y despues se elige B
p
de modo que 3 A 3 B = .
3
En el siguiente ejemplo veremos como conviene proceder cuando A y por lo tanto
tambien B no son reales.
319

Ejemplo 11.12
Resolver la ecuacion x3 6x + 2 = 0.

En este caso p = 6 y q = 2 de donde resulta que A = 1 + i 7 y B = 1 i 7.

3

3

3

3
Notemos que B = A y por lo tanto B = ( A). Como una solucion es A + B, esta
sera 2 Re 3 A.

Para calcular 3 A procederemos trigonometricamente.
r !
1 7
A = 1 + i 7 = 8 + i = 8(cos + i sen)
8 8
r
1 7
con cos = y sen = .
8 8
 
Luego, 3 A = 2 cos + i sen y la solucion anterior queda en 2 2 cos .
3 3 3

Determinemos las otras soluciones.


Sabemos que w2 = w; entonces

w 3 A + w2 3 B = w 3 A + (w 3 A) = 2 Re w 3 A
 
=2 2 cos + 120
3
Por ultimo

w2 3 A + w 3 B = w2 3 A + (w2 3 B) = 2 Re w2 3 A
 
= 2 2cos + 240
3
En resumen las 3 soluciones son

2 2 cos
3
   
2 2 cos + 120 = 2 2 cos 60
3 3
   
2 2 cos + 240 = 2 2cos 60 +
3 3
Calculemoslas numericamente usando calculadora, se obtiene aproximadamente

= 110, 7 y 2 2 cos = 2, 26
3
 
2 2 cos 60 = 2, 60
3
 
2 2 cos 60 + = 0, 34
3
320

4. La ecuacion bicuadrada o de cuarto grado.

Seguiremos muy de cerca la solucion de Ferrari (1522-1565) discpulo de Cardano pu-


blicada junto a la de la cubica en 1545.
La supondremos en la forma x4 + px2 + qx + r = 0 que sabemos, no implica perdida de
generalidad. Es equivalente a

1) x4 = px2 qx r.
y2
Sumemosle a ambos miembros yx2 + donde y sera un numero que determina-
4
remos de forma que el miembro de la derecha de 2) sea un cuadrado perfecto. 1)
queda en
 y 2 y2
2) x2 + = (p + y)x2 qx + r.
2 4
En segundo miembro es un cuadrado perfecto de la forma (ax + b)2 si y solo si
r
y2
3) 2 yp r = q
4
que establece la siguiente ecuacion para y
4) y 3 py 2 4ry + 4rp q 2 = 0
Se llama la resolvente. Encontrado un valor para y que la satisfaga, el segundo
miembro de 2) toma la forma (ax + b)2 y de ella obtenemos las dos ecuaciones
y y
5) x2 + = ax + b; x2 + = ax b
2 2
que dan las cuatro soluciones de la ecuacion bicuadratica.

Ejemplo 11.13
Resolver la ecuacion

x4 8x2 4x + 3 = 0.
En este caso p = 8, q = 4 y r = 3.
La resolvente es

y 3 + 8y 2 12y 112 = 0.
4 es una raz que pudo ser calculada aprovechando el material de la seccion anterior.
Reemplazando y por -4 en la ecuacion 2) queda

(x2 2)2 = 4x2 + 4x + 1 = (2x + 1)2


de donde logramos las dos ecuaciones

x2 2x 3 = 0 y

x2 + 2x 1 = 0

cuyas soluciones son -1 y 3 para la primera y -1 + 2 y 1 2 para la segunda. Estas
cuatro soluciones son las de la ecuacion original.
321

5. Ecuaciones de grados superiores.

Casi 300 anos despues de la publicacion de Cardano con las soluciones para las ecua-
ciones de tercer y cuarto grado, Niels Abel (1802-1829) en 1824 prueba que la ecuacion
general de quinto grado no puede ser resuelta por radicales. Por resolver una ecuacion
por radicales se entiende que, a partir de sus coeficientes y solo con las operaciones
de suma, multiplicacion, extraccion de raz y sus inversas, se obtengan las soluciones.
Posteriormente, Evaristo Galois (1811-1832) demuestra que en general las ecuaciones de
grado mayor que cuatro no pueden ser resueltas de ese modo.
Estos resultados negativos, que fueron establecidos al crearse nuevas teoras matematicas
que actualmente se conocen como teora de grupos y teora de campos, obligaron a buscar
nuevos caminos para resolver las ecuaciones.
Algo de esto veremos en la proxima seccion.

11.2.1. EJERCICIOS
1. Pruebe que las races de x2 + x + 1 satisfacen la ecuacion

x6 + 4x5 + 3x4 + 2x3 + x + 1 = 0.

2. Resolver la ecuacion
20x3 30x2 + 12x 1 = 0

siendo 1/2 una raz.

3. Sea la ecuacion a0 + a1 x+ a2 x2 + + an xn = 0 donde los coeficientes


son enteros.
Demostrar que si (c + d 2), con c y d enteros, es raz entonces c d 2 es tambien
raz de dicha ecuacion. ( 2 se puede reemplazar por cualquier numero irracional con el
mismo resultado).

4. Resolver la ecuacion 2x4 + x3 + 17x2 + 15x + 9 = 0 sabiendo que 1 + 2 es una raz.

5. Resolver x3 2(1 + i)x2 (1 2i)x + 2(1 + 2i) = 0 sabiendo que 1 + 2i es una raz.

6. Resolver las ecuaciones cubicas siguientes donde , y son las races:

i) x3 + 2x2 + 3x + 2 = 0 si = +
ii) 2x3 x2 18x + 9 = 0 si + = 0
iii) 2x3 x2 5x 2 = 0 si = 1
iv) x3 7x2 42x + 216 = 0 si 2 =

7. Pruebe que si las races de x3 + px2 + qx + r = 0, estan en progresion geometrica,


entonces rq 3 p3 = 0.

8. Cual es la relacion entre p y q si la ecuacion x3 + px + q = 0 tiene una raz multiple?

9. Resolver las ecuaciones cuarto grado de races , , , :

i) x4 2x3 + 2x2 x 2 = 0 si + = 1
322

ii) 2x4 + 13x3 + 25x2 + 15x + 9 = 0 si =

10. Encontrar la suma de los cuadrados de las races de las ecuaciones:

i) 2x4 6x3 + 5x2 7x + 1 = 0


ii) 3x5 3x3 + 2x2 + x 1 = 0

11. Resolver:

a) (ax2 + bx + c)2 (dx2 + ex + f )2 = 0


b) ax8 + bx6 + cx4 + bx2 + a = 0
c) ax5 + bx4 + cx3 + cx2 + bx + a = 0

12. Resolver:

n
X
xk = 0
k=0

para los siguientes valores de n: 1,2,3,4,5.

13. Resolver:

n
X
(1)k xk = 0
k=0

para los siguientes valores de n: 1,2,3,4,5.

14. Determine k y resuelva la ecuacion 2x4 15x3 + kx2 30x + 8 = 0 si sus races estan
en P.A.

15. Resuelva las siguientes ecuaciones sabiendo que sus races estan en P.H.

i) 30x3 47x2 + 24x 4 = 0


ii) 15x3 23x2 + 9x 1 = 0

16. Si la suma de dos de las races de la ecuacion x3 + px2 + p2 x + r = 0 es 1, pruebe que


r = (p + 1)(p2 + p + 1).

17. Si i con i = 1, 2, 3, 4 son las races de ax4 + bx3 + cx2 + dx + l = 0 encuentre el valor
de:

4 4
X 1 X 3 i
i) ii) donde p = 1 2 3 4
i p
i=1 i=1

18. Si , y son las races de x3 + qx + r = 0 forma las ecuaciones cuyas races son:

a) 2 + 2 , 2 + 2 , 2 + 2
323

19. Resuelva las siguientes ecuaciones:

i) x3 + 4x + 1 = 0
ii) x3 + 3x 14 = 0
iii) x3 27x 54 = 0
iv) x3 27x + 54 = 0
v) x3 6x 4 = 0

Dada la ecuacion x3 +3Hx+G = 0 con G2 +4H 6= 0 pruebe que sus races son las mismas
de la ecuacion a(x + b)3 b(x + a)3 = 0 donde a y b son las races de Hx2 Gx H 2 = 0.
(Otro metodo para resolver una ecuacion cubica).

20. Resuelva las siguientes ecuaciones:

i) x4 + 32x 60 = 0
ii) x4 + 4x3 3x2 + 16x 3 = 0
iii) x4 11x2 6x + 10 = 0

11.3. METODOS NUMERICOS ELEMENTALES


En lo que sigue veremos teoremas que nos daran mas informacion sobre las races de un
polinomio.

Teorema 11.5
Sea P (x) = a0 +a1 x+ +an xn un polinomio con todos sus coeficientes reales y sea z = a+bi
una raz de P (x) con b 6= 0. Entonces z = a bi tambien es raz de P (x).

Demostracion
Por propiedades de conjugacion de los numeros complejos tenemos las siguientes igualdades:

P (z) = a0 + a1 z + a2 z 2 + + an z n

= a0 + a1 z + a2 z 2 + + an z n

= a0 + a1 z + a2 z 2 + + an z n

= a0 + a1 z + a2 z 2 + + an z n

= P (z) = 0 = 0

Corolario
Todo polinomio con coeficientes reales y de grado impar tiene por lo menos una raz real.

Demostracion
El polinomio tiene un numero impar de races y por el teorema anterior el numero de races
complejas es cero o par. Luego, tiene por lo menos una raz real.
324

Corolario
Si a + bi es raz de un polinomio con coeficientes reales entonces el polinomio es divisible por
(x a)2 + b2 .

Demostracion
x (a + bi) divide a P (x)
x (a bi) tambien divide a P (x)

Luego,:

[x (a + bi)][x (a bi)] = [(x a) bi][(x a) + bi]

= (x a)2 + b2

divide a P (x).

Este corolario prueba que todo polinomio real puede ser factorizado en polinomios reales de
grado uno o dos.

El siguiente teorema da un metodo para encontrar las races racionales de una ecuacion
con coeficientes enteros.

Teorema 11.6
p
Sea P (x) = a0 + a1 x + + an xn un polinomio con coeficientes enteros y sea una raz
q
racional de P (x) con p y q numeros enteros sin factores comunes. Entonces p es un divisor de
a0 y q un divisor de an .

Demostracion
pn1 pn
 
p p
P = 0 = a0 + a1 + + an1 n1 + an n .
q q q q
De donde, multiplicando por q n

0 = a0 q n + a1 pq n1 + + an1 pn1 q + an pn (1)

Luego,

a0 q n = p(a1 q n1 + + an1 pn2 q + an pn1 )

Como p es factor del segundo miembro, debe ser factor del primer miembro. Luego, p divide
a a0 porque (p, q) = 1.

Tambien se tiene, de (1), que:

an pn = q(a0 q n1 + a1 pq n2 + + an1 pn1 )

y con un argumento similar, que q divide a an .


325

Ejemplo 11.14
Encontremos las races racionales de la ecuacion

12x3 + 8x2 17x + 5 = 0.


p
Las posibles races racionales son de la forma donde p divide 5 y q divide 12. Luego,:
q
p 1 1 1 1 1 5 5 5 5 5
, 1; 5; ; ; ; ; ; ; ; ; ; .
q 2 3 4 6 12 2 3 4 6 12
Averiguemos si 1 es raz:

12 8 17 5

1 12 20 3
12 20 3 8 = resto
1 no es raz.

Veamos si 5 es raz:

12 8 17 5

5 60 340 1615
12 68 323 1620 = R 6= 0
Ahora probemos 1/2

12 8 17 5

1/2 6 7 5
12 14 10 0 = R
Luego, 1/2 es raz.

Podramos continuar tratando de encontrar otras races racionales, pero ello no es necesario
en este caso, pues la ecuacion que resulto como cuociente es de segundo grado y se puede
resolver facilmente.

En efecto, el cuociente es:

12x2 + 14x 10 = 0

6x2 + 7x 5 = 0

7 49 + 120
x =
12

7 13 1/2
x = =
12 5/3
Luego, las races son: 1/2, 1/2 y 5/3
326

Nota
En realidad, en el ejemplo 11.12 se uso este metodo para obtener una raz de la resolvente.

A continuacion estudiaremos procesos generales de resolucion de ecuaciones con coeficien-


tes reales.

Nos limitaremos a las races reales. Tres son los aspectos que se deben considerar: la acota-
cion de las races, su separacion y el calculo de estas. Ellos corresponden a cursos de calculo
numerico. Nos limitaremos solo a algunas referencias.

Empecemos con la acotacion de las races.

El problema es el siguiente: Dado un polinomio P (x) encontrar dos numeros m y n tales que
si r es raz de P (x) entonces n < r < m.

Hay varios procedimientos para conseguir esta meta. Nosotros explicaremos uno, en que se
resuelve el problema por medio de un tanteo sistematico. Enunciemos previamente el siguiente
teorema.

Teorema 11.7
Sea P (x) = a0 + a1 x + + an xn un polinomio real no nulo tal que ai 0, i = 1, , n.
Sea a R+ . Entonces a no es raz de P (x).

Demostracion
Tenemos P (a) = a0 + a1 a + + an an .

Por hipotesis a0 0, a1 a 0, , an an 0.

Como el polinomio no es el nulo y a R+ , debe existir un termino de P (a) distinto de cero.


Luego, P (a) > 0 y por lo tanto a no es raz de P (x).

Corolario
Sea P (x) un polinomio con coeficientes reales de grado mayor o igual que 1 y sea m R+ .
Tenemos por teorema del resto que P (x) = (x m)Q(x) + P (m). Si ningun coeficiente de
Q(x) es negativo, P (m) > 0 y a > m, entonces P (a) > 0.

Demostracion
En efecto, como a > m, tenemos que a m > 0. Ademas, por el teorema 7, Q(a) > 0 y por
hipotesis P (m) > 0. Luego, P (a) = (a m)Q(a) + P (m) es mayor que cero.

Este corolario nos dice que si efectuamos la division de P (x) por (x m) con m R+ , re-
sultando un cuociente con todos sus coeficientes no negativos y ademas el resto es positivo,
entonces toda raz real de P (x) es menor que m.
327

Con estas observaciones podemos indicar en que consiste el metodo. Se tantean diferentes
valores para m hasta encontrar un Q(x) en que ninguno de sus coeficientes sea negativo y
para el cual tambien P (m) sea positivo. Entonces toda raz de P (x) es menor que m. En caso
que el coeficiente principal de P sea negativo, se aplica el metodo al polinomio P (x).

Para acotar inferiormente las races de P (x) se acotan superiormente las races de P (x). Si
n es una cota superior de las races reales de P (x), entonces n es una cota inferior de las
races reales de P (x).

Ejemplo 11.15
Acotemos las races de la ecuacion

6x5 + 27x4 100x3 200x 50 = 0.


Tomemos m = 1

6 27 100 0 200 50

1 6 33
6 33 67
Nos detenemos porque salio un coeficiente negativo. Tomemos m = 2

6 27 100 0 200 50

2 12 78
6 39 22
Tomemos m = 3

627 100 0 200 5

3 18 135 105 315 345


6 45 105 115 340
Luego, 3 es cota superior de las races. Se puede intentar mejorar esta cota tanteando numeros
comprendidos entre 2 y 3.

Ahora acotamos superiormente las races P (x)

P (x) = 6x5 + 27x4 + 100x3 + 200x 50


En este caso debemos acotar las races de P (x).

Mirando los coeficientes de la ecuacion notamos que la cota no puede ser 1 o 2 o 3 o 5.


Intentemos 8.

6 27 100 0 200 50

8 48 168 544 4352 33216


6 21 68 544 4152 33166
328

El numero 8 sirve. Intentemos mejorar la cota. Ensayemos el 7.

6 27 100 0 200 50

7 42 105 35 245 315


6 15 5 35 45 365
El numero 7 sirve. Haciendo los calculo vemos que el 6 no servira.

En resumen tenemos que las races reales de P (x) se encuentran entre -7 y 3.

El siguiente ejemplo muestra un uso de la acotacion.

Ejemplo 11.16
Encontrar las races racionales de

P (x) = x6 + 3x5 36x4 45x3 + 93x2 + 132x + 140


Por el metodo anterior encontramos que las races de P (x) estan entre -8 y 6.

En este caso a0 = 140 y a6 = 1, entonces los posibles valores para p son

1, 2, 4, 5, 7, 10, 14, 20, 28, 35, 70, 140


y los posibles para q son 1.

Entonces los posibles valores para son:

1, 2, 4, 5, y 7.
Los demas no se consideran porque sabemos que las races estan entre -8 y 6.

Ahora hay que probarlos uno tras otro:

1 3 36 45 93 132 140

1 1 4 32 77 16 148
1 4 32 77 16 148 288 = P (1)
1 no es raz. En forma analoga se calcula P (1). Resulta 108 y por lo tanto -1 no es raz.

1 3 36 45 93 132 140

2 2 10 52 194 202 140


1 5 26 97 101 70 0 = P (2)
2 es raz.

Ahora tenemos que P (x) = (x 2)(x5 + 5x4 26x3 97x2 101x 70) y las races restantes
lo seran de

Q(x) = x5 + 5x4 26x3 97x2 101x 70.


329

Debemos ver si 2 es raz doble.

1 5 26 97 101 70

2 2 14 24 242 686
1 7 12 121 343 756 = Q(2)

2 no es raz doble.

Sigamos ensayando, pero antes notemos lo siguiente:

4+1=5y5 no divide a P (1) = 108.

Luego, 4 no es raz.

Tambien -4 -1 = -5 y -5 no divide a P (1) = 288.

Luego, -4 tampoco es raz. (Encuentre una demostracion de estas afirmaciones).

1 5 26 97 101 70

2 2 6 64 66 70
1 3 32 33 35 0 = Q(2)

2 2 2 68 70
1 1 34 35 105

Luego, -2 es raz simple y el cuociente que queda es

Q1 (x) = x4 + 3x3 32x2 33x 35 y

P (x) = (x 2)(x + 2)Q1 (x)

Siguiendo el mismo procedimiento se llega finalmente que 5 y -7 son races simples y que
P (x) = (x 2)(x + 2)(x 5)(x + 7)(x2 + x + 1) y luego sus dos ultimas races son:

1 3 1 3
+i y i .
2 2 2 2

Antes de estudiar el problema de la separacion de las races, que lejos es el mas importante,
necesitamos material adicional.

En algunos de los siguientes teoremas se ocuparan nociones de Calculo Elemental. Fundamen-


talmente se usara que un polinomio es una funcion derivable y por lo tanto continua; que su
derivada tambien es un polinomio y las reglas de derivacion.

Nota
Anotaremos por P 0 (x) el polinomio derivado de P (x).
330

Teorema 11.8
Si es una raz de P (x) de multiplicidad m, entonces es raz de P 0 (x) de multiplicidad m 1.

Demostracion
Sabemos que P (x) = (x )m Q(x) con Q() 6= 0. Entonces

P 0 (x) = m(x )m1 Q(x) + (x )m Q0 (x)

= (x )m1 (mQ(x) + (x )Q0 (x))


Luego, es raz de P 0 (x) de multiplicidad, por lo menos, m 1.

Sea T (x) = mQ(x) + (x )Q0 (x).

Para que fuese de multiplicidad mayor tendra que ser raz de T (x). Pero T () = mQ() 6= 0.

Nota
De este teorema se desprende que, al ser una raz multiple de P (x), (x ) debe dividir
al maximo comun divisor de P y P 0 . Luego, las races multiples de P son las races de ese
maximo comun divisor.

Ejemplo 11.17
Resolver la ecuacion

6x5 + 45x4 + 40x3 450x2 1350x 1107 = 0


sabiendo que tiene races multiples.

Sea P (x) = 6x5 + 45x4 + 40x3 450x2 1350x 1107. Entonces P 0 (x) = 30x4 + 180x3 +
120x2 900x 1350.

Segun el procedimiento estudiado enel captulo anterior, obtenemos como maximo comun di-
visor entre P y P 0 al polinomio (x + 3)2 . Esto nos muestra que -3 es una raz triple de P (x).
Dividiendo obtenemos que P (x) = (x + 3)3 (6x2 9x 41).

Las otras races son las de 6x2 9x 41 = 0, vale decir



(9 + 1065)/12 y (9 1065)/12.

Nota
El teorema 8 nos permite, cuando sea necesario, considerar a las ecuaciones como si no tuvie-
ran races multiples, porque simplificando P por el maximo comun denominador entre este y
P 0 las podemos eliminar.

Si D es este maximo comun divisor, las races multiples de P , aun lo seran de D, pero con
multiplicidad una unidad menor.

El siguiente teorema se usara posteriormente.


331

Teorema 11.9
Sea una raz real de P (x) de multiplicidad m. Entonces

P 0 (x) P 0 (x)
lm = + y lm =
x+ P (x) x P (x)

Demostracion
P (x) = (x )m Q(x), con Q() 6= 0, de donde

P 0 (x) = m(x )m1 Q(x) + (x )m Q0 (x).


P 0 (x) m Q0 (x)
Luego, = +
P (x) x Q(x)
Q0 (x) Q0 ()
Por la continuidad de los polinomios se desprende que cuando x , , el
Q(x) Q()
m
cual es un numero real puesto que Q() 6= 0. Pero, cuando x + se tiene que
x
m
y cuando x , . Como estos lmites no cambian al sumarseles la constante
x
0
Q ()/Q(), el teorema queda establecido.

Corresponde, ahora, preocuparnos del problema de la separacion de las races. Este consiste
en dado P (x), determinar intervalos de modo que en cada uno de ellos se encuentre solo una
raz del polinomio.

El teorema sobre funciones continuas, que expondremos, es uno de los mas utiles en el
problema de la separacion.

Teorema 11.10
Sea F (x) una funcion continua en el intervalo [a, b]. Si F (a) y F (b) son de signos distintos,
entonces en [a, b] hay por lo menos un numero tal que F () = 0

Demostracion
Es una consecuencia inmediata de la continuidad. No daremos detalles.

El siguiente corolario se establece sin dificultad.

Corolario
Si el polinomio P no tiene races en el intervalo [a, b], entonces P (x) mantiene su signo para
todo x [a, b].

El siguiente teorema mejora la informacion dada en el teorema 11.10.

Teorema 11.11
Si P (a) y P (b) son de signos distintos, entonces el numero de races de P en el intervlo [a, b]
contadas segun su multiplicidad es impar; si son de signos iguales, es cero o par.

Demostracion
332

Sean 1 , 2 , , l las distintas races de P que estan en el intervalo [a, b] y sean m1 , m2 , , ml


sus multiplicidades. Entonces

P (x) = (x 1 )m1 (x 2 )m2 (x l )ml Q(x)


donde Q no tiene races en (a, b). Calculando P (a)/P (b) obtenemos que

a 1 m1 a l ml Q(a)
   
P (a)
=
P (b) b 1 b l Q(b)
Por el corolario anterior Q(a)/Q(b) es positivo, mientras que todos los cuocientes
(a : i)/(bi ) son negativos. Luego, el signo de P (a)/P (b) es el mismo de (1)m1 +m2 ++ml
de donde concluimos que si P (a) y P (b) son de signos distintos m1 + m2 + + ml es impar,
mientras que si son de igual signo, esa suma es cero o par.

En ocasiones este teorema se puede aplicar directamente como veremos en el siguiente


ejemplo.

Ejemplo 11.18
Pruebe que para todo valor real de l el polinomio

P (x) = (x 2)(x 5)(x 7)(x 9) + l(x 3)(x 6)(x 8)(x 10)


tiene todas sus races reales y distintas y separelas.

Observemos que P (3) < 0, P (6) > 0, P (8) < 0 y P (10) > 0, luego el polinomio tiene por lo
menos una raz en cada uno de los siguientes intervalos (3, 6), (6, 8) y (8, 10). Por otra parte,
el coeficiente de x4 es 1 + l. Si l = 1, el polinomio es de grado 3 y en esos intervalos estaran
sus 3 races. Si 1 + l > 0, sabemos por el teorema 10.1 que lm P (x) = . Luego, habra otra
x
raz en el intervalo (10, ).

Por ultimo si 1 + l < 0, por un corolario del teorema citado, lm P (x) = y en este caso
x
tendra una raz enel intervalo (, 3).

Se puede intentar separar las races de un polinomio tanteando valores para x hasta encontrar
cambios de signos. Estos intentos, sin ninguna sistematizacion, en la practica tienen exito solo
en algunas oportunidades. Ayudara bastante saber cuantas de las races de un polinomio
dado son positivas, cuantas negativas y cuantas complejas. Algunas de esta informacion nos
dara la llamada Regla de los Signos de Descartes.

Para su demostracion necesitamos un teorema previo, conocido como Teorema de de


Gua.

Teorema 11.12
Sea P un polinomio de grado n y sean r1 < r2 < rs sus races positivas.

Definamos Q(x) = xP 0 (x) + aP (x) con a un real distinto de cero. Entonces Q(x) tiene
por lo menos una raz en cada uno de los intervalos (r1 , r2 ), (r2 , r3 ) (rs1 , rs ). Ademas,
si m1 , m2 , , ms son las multiplicidades de r1 , r2 , , rs respectivamente y p = m1 + m2 +
333

+ ms , entonces Q tiene por lo menos p 1 races en que cada una se cuenta tantas veces
como sea su multiplicidad.

Demostracion
Q(x) P 0 (x) P 0 (x) P 0 (x)
=x + a. Por el teorema 9 lm = y lm = .
P (x) P (x) xri+ P (x)
xri+1 P (x)
Como x es positivo en el intervalo (ri , ri+1 ) se tiene que

Q(x) Q(x)
lm = y lm =
xri+ P (x) xri+1 P (x)

Entonces, para algun  > 0 se tiene que

Q(ri + ) Q(ri+1 
>0 < 0.
P (ri + ) P (ri+1 )
Como la funcion Q(x)/P (x) es continua existe (ri +, ri+1 ) y por lo tanto (ri , ri+1 ),
tal que Q()/P () = 0.

Pero esto fuerza que Q() = 0 y la primera afirmacion del teorema queda establecida.

Para la segunda notemos que ri es raz de Q(x) de multiplicidad mi 1. por lo tanto,Q(x)


tiene por lo menos m1 1 + m2 1 + + ms 1 + s 1 races; es decir p 1 races.

A continuacion formularemos y probaremos la Regla de los Signos de Descartes.

Definicion 11.3
Sea an , as , , aq la sucesion de los coeficientes no nulos de un polinomio P (x), escrita en
orden decreciente de sus ndices. Si dos coeficientes sucesivos tienen el mismo signo diremos
que hay una permanencia y si son de signos distintos que hay una variacion. Al numero total
de variaciones lo designaremos por v.

Por ejemplo, el polinomio 3x7 x5 + 2x4 + 2x3 x 4 tiene la sucesion 3, 1, 2, 2, 1, 4 que


posee 2 permanencias y 3 variaciones.

Teorema 11.13
Sea v el numero de variaciones y p el de races positivas no necesariamente distintas de un
polinomio P (x), cuya sucesion de coeficientes no nulos es an , as , , aq . Entonces v = p + 2h
con h un entero no negativo.

Demostracion
Siempre podemos suponer que el coeficiente principal de P (x) es positivo ya que este y
(1)P (x) tienen las mismas races. Primero probaremos que p v.

La demostracion es por induccion sobre v. Si v = 0, todos los coeficientes no nulos de P son


positivos y por lo estudiado en acotacion no tiene races positivas. Luego, en este caso p = 0.
334

Supongamos el teorema valido para los polinomios que presentan v variaciones y sea P un
polinomio con v + 1 variacion. Sean ai y aj con i > j, dos coeficientes sucesivos que presentan
cambio de signo. Notemos que no podemos asegurar que i = j + 1 pues entre ai y aj pueden
haber coeficientes nulos.

Sea v1 el numero de variaciones que hay en an , , ai , v2 las variaciones desde aj , , aq .


Entonces v = v1 + v2 + 1.

Definamos Q(x) = xP 0 (x) lP (x) donde l es una constante que determinaremos posterior-
mente. La sucesion de coeficientes de Q es:

(n l)an , (s l)as , , (i l)ai , (j l)aj , , (0 l)a0 .


Tomemos l de modo que i > l > j. Entonces los coeficientes (n l)an , , (i l)ai tienen
los mismos signos que los coeficientes an , , ai respectivamente y por tanto presentan v1
variacion. Los coeficientes j l)aj , , (0 l)a0 tienen los signos cambiados con respecto
a la sucesion aj , , a0 . Luego, tiene v2 variaciones. Pero (i l)ai y (j l)aj son de igual
signo. Luego, Q presenta v variaciones. Por el teorema anterior sabemos que Q tiene por lo
menos p 1 races positivas donde p es el numero de races positivas de P , contadas segun
sus multiplicidades.

Luego, por la hipotesis inductiva

p 1 v, es decir p v + 1.
Debemos probar ahora, que v p es par o cero. Recordemos que

P (x) = an xn + as xs + + aq xq con an > 0.


Notemos que si v es par an y aq tienen el mismo signo, mientras que son de signos distintos
cuando es impar. Sabemos, del captulo anterior que P (x) tiene el signo de an para x mayores
que un cierto m positivo, mientras que tiene el signo de aq para los x entre 0 y un cierto  > 0.

Sea a un real positivo menor que  y que toda raz positiva de P y b mayor que cualquiera de
esas races y tambien mayor que m. Entonces en [a, b] estan todas las races de P (x). Ademas,
v impar implica que P (a) y P (b) son de signos distintos y por el teorema 11 se tiene que p
impar. De igual forma, v par implica P (a) y P (b) son del mismo signo y por ese teorema, p
par. Luego, en ambos casos v p es par o cero.

Este teorema tambien se puede aplicar a las races negativas de P (x) ya que estas son las
races positivas de P (x).

En los siguientes ejemplos veremos aplicaciones de la regla de los signos.

Ejemplo 11.19
Cuantas races reales tiene el polinomio
P (x) = x6 + x4 x3 2x 1?
Tiene solo una variacion. No puede tener ninguna raz positiva porque la diferencia debe ser
par. Luego, tiene una raz positiva. Sea .
335

P (x) = x6 + x4 + x3 + 2x 1
presenta tambien una unica variacion. Luego, P (x) tiene una raz negativa. Sea .

Como P (1) = 4, P (0) = 1; P (1) = 2, P (2) = 67 tenemos que (1, 0) y (1, 2).

Vamos aprovechar la ocasion, para mostrar un metodo bastante ingenuo pero posible de usar
para calcular aproximadamente la raz .

Como (1, 2), debe ser de la forma 1.

Si pudieramos encerrarla enun intervalo 10 veces mas pequeno tendramos su primer decimal.
Al continuar con el ejemplo se vera como se disminuye el intervalo. Calculemos P (1, 4). Usemos
division sintetica, aproximando los calculos a dos decimales.

1 0 1 1 0 2 1

1,40 1,40 1,96 4,14 4,40 6,16 5,82


1 1,40 2,96 3,14 4,40 4,16 4,82
Tenemos que P (1,4) es positivo mientras que P (1) es negativo. Luego, (1, 1,4).

Calculemos P (1,2)

1 0 1 1 0 2 1

1,20 1,20 1,44 2,93 2,31 2,78 0,93


1 1,2062,44 1,93 2,31 0,78 0,07
Como resulto negativo tenemos que (1,2, 1,4).

Continuando con P (1,3) que resulta positivo, concluimos que 1,2 < < 1,3, es decir, es
de la forma 1,2

Metodos mas efectivos de aproximacion a las races se ven en los cursos de Calculo Numeri-
co.
Ejemplo 11.20
Cuantas races reales tiene P (x) = 5x4 4x3 + 3x2 2x + 1?

P (x) presenta 4 variaciones del signo. por lo tanto,puede tener 0, 2 o 4 races positivas.

Calculemos (1 + x)2 P (x).

(1 + x)2 P (x) = 5x6 + 6x5 + 1 que al no tener cambios de signos, no tiene races positivas. Pero
toda raz de P (x) lo es tambien de (1 + x)2 P (x). Luego, P (x) no tiene races reales.

El metodo de las variaciones de los signos solo da respuestas exactas cuando estas son cero o
uno. Con algunas triquinuelas algebraicas algo mas se puede conseguir, como veremos en el
siguiente ejemplo.
336

Ejemplo 11.21
Cuantas races mayores que dos podra tener la ecuacion x5 6x4 5x3 + x2 + 33 = 0?

Designaremos por P (x) el polinomio. Como tiene dos cambios de signo puede tener dos o cero
races positivas.

Formemos el polinomio cuyas races son las de P (x) disminuidas en 2; es decir, si Q(x) es
dicho polinomio y es raz de P (x) entonces 2 es raz de Q(x).

Q se puede construir por medio de las relaciones entre coeficientes y races, o tambien expre-
sando P (x) en potencias de (x 2). Usaremos este metodo que fue estudiado en el captulo
anterior.

La distribucion de los calculos mas conveniente es:

1 6 5 1 0 33

2 2 8 26 50 100
1 4 13 25 50 67

2 2 4 34 118
1 2 17 59 168

2 2 0 34
1 0 17 93

2 2 4
1 2 13

2 2
1 4

Luego,

P (x) = (x 2)5 + 4(x 2)4 13(x 2)3 93(x 2)2 168(x 2) 67


Entonces

Q(x) = x5 + 4x4 13x3 93x2 168x 67


Notemos que Q( 2) = P (), por lo tanto P tendra tantas races mayores que 2 como races
positivas tenga Q. En este caso, exactamente una.

11.3.1. EJERCICIOS
1. Hallar solamente las races racionales de las siguientes ecuaciones:

i) 3x3 26x2 + 34x 12 = 0


ii) 6x3 x2 + x 2 = 0
337

iii) 2x3 x2 + 1 = 0
iv) 6x4 11x3 x2 4 = 0
v) 3x4 40x3 + 130x2 120x + 27 = 0
vi) x6 8x5 + 7x4 + 68x3 100x2 160x + 192 = 0

2. Acote las races reales de las siguientes ecuaciones:


Encuentre sus races racionales.

i) x3 2x2 25x + 50 = 0
ii) x6 7x5 + 11x4 7x3 + 14x2 28x + 40 = 0
iii) 3x3 26x2 + 34x 12 = 0
iv) 2x4 4x3 + 3x2 5x 2 = 0
v) 6x5 + 11x4 3x3 + 5x 6 = 0
vi) 3x5 x4 + 6x3 2x2 + 3x 1 = 0

3. Resolver la ecuacion 2x5 3x4 2x3 + 3x2 4x + 6 = 0 sabiendo que i es una raz.
4. Resolverla ecuacion x3 9x2 + 33x 65 = 0 sabiendo que tiene una raz compleja de
modulo 13.
5. Si en x = a los polinomios P (x), P 0 (x), , P (n) (x) son todos positivos, entonces a es
una cota superior de las races de P (x).

(Sugerencia:

P 0 (a) P (n) (a)


P (x) = P (a) + (x a) + + (x a)n ).
1! n!

6. Sea P (x) = xn + an1 xn1 + + a0 . Sea M = max nk ak .


Sea una raz de P (x) entonces || 2M .

Sugerencia: Use el hecho que

n = an1 n1 + an2 n2 + + a0
y aplique la desigualdad triangular.
7. Cuantas races reales tienen las siguientes ecuaciones?

a) x4 x2 + x 2 = 0
b) x5 + x3 2x2 + x 2 = 0
c) 1 2x + 3x2 + (2n + 1)x2n = 0

8. Separe las races de:

a) 8x3 4x2 18x + 9 = 0


b) x4 3x2 + 10x 6 = 0
338

11.4. SOLUCIONES O SUGERENCIAS A LOS EJERCICIOS


DEL CAPITULO
Pagina 145


3. Es facil ver que si (c + d p)l = A + B p entonces (c d p)l = A B p. Por lo tanto,


P (c + d p) = A0 + B0 p + A1 + B1 p + + An + Bn p

= (A0 + + An ) + p(B0 + B1 + + Bn )

y vale cero ssi A0 + + An = 0 y B0 + + Bn = 0

Entonces


P (c d p) = A0 B0 p + A1 B1 p + + An Bn p

= (A0 + + An ) p(B0 + + Bn ) = 0

6. iv)

++ =7

+ + = 42

= 216

Luego, 3 = 216, es decir = 6.


7. Sean , y a las races
a
 
1
1. +1+a = p
a
 
1
2. 2 +1+a = q
a

3. 3 = r

q
De 1 y 2 = .
p
p
Luego, 3
r = , es decir, rq 3 p3 = 0.
q
339

11. c) Tiene la raz -1.


Se factoriza en (x + 1)(ax4 + (b a)x3 + (a + c b)x2 + (b a)x + a) = 0.

Simplificando el segundo factor por x2 , queda la ecuacion recproca


   
1 2 1
a x + 2 + (b a) x + +a+cb=0
x x

1 1
Haciendo a = x+ se tiene x2 + 2 = u2 2 que conduce a una ecuacion de grado 2 en u.
x x

16. Sean , y las races con + = 1. Tenemos que:

A. + + = p

B. + + = p2

C. = r

De A tenemos que = p1. Amplificando B por y usando C queda que r+ 2 = p2


de donde se obtiene el resultado.

18. a) Debemos expresar:


     
1 1 1 1 1 1
A = + + + + +

        
1 1 1 1 1 1 1 1 1 1 1 1
B = + + + + + + + +

   
1 1 1 1 1 1
C = + + +

en funcion de q y r sabiendo que

i) + + = 0
ii) + + = 0
iii) = r

Es inmediato que:

+ + q
A= 2 = 2
r
2 2 + 2 2 + 2 2 q2
 
++
B= +3 =
()2 r2
340

3 + 3 + 3 + 2
C=
()2
Es facil ver que 3 + 3 + 3 = 3 = 3r al elevar i) al cubo y usar ii).

5
Luego, C = , con eso tenemos que la ecuacion es:
r
2q 2 q 2 5
r3 + x + 2x = 0
r r r

Pagina 161

4. Sean , y r las races. Tenemos que: r = 65. Luego, r = 5.

6.
||n |an1 | + + |a0 | M |a|n1 + + M n

||n1
  
= Mn + + 1
M
 n  n
|| ||
M 1 M
Mn ||
M n ||
M 1 M 1

Luego,
 n
 n X ||
|| M
M ||
M 1

||
Pero esto es verdadero ssi 1 < < 2.
M

Es decir,

M < || < 2M
Captulo 12

LOS NUMEROS REALES


2a. PARTE

12.1. EL AXIOMA DEL SUPREMO


En este captulo finalizaremos la construccion de los numeros reales iniciada en el Captu-
lo 6 estudiando el axioma del supremo y alguna de sus consecuencias. Este axioma, tambien
llamado de completitud, tiene muchas maneras aparentemente distintas, pero formalmen-
te equivalentes de ser enunciado. Elegiremos una de las mas tradicionales. Otro enunciado
sera establecido posteriormente.

Necesitamos algunos conceptos previos.

Definicion 12.1
Sea S R.

a) Diremos que S es acotado superiormente si y solo si existe m R tal que x(x S


x m).
b) Diremos que S es acotado inferiormente si y solo si existe l R tal que x(x S l
x).
c) Diremos que S es acotado si y solo si S es acotado inferior y superiormente.

d) A cada numero real m que tenga la propiedad enunciada en el punto a) se le llama una
cota superior de S y a los que tengan la propiedad enunciada en b) una cota inferior de
S.

Ejemplo12.1 
1
Sea S = ; 3; 12, 5; 0, 6; 0
2
S es acotada tanto inferior como superiormente. Algunas cotas inferiores de S son los numeros
1
100, 1, , 0, 8. Algunas cotas superiores son 24, ....,12, 5.
2
1
Notemos que todo numero menor o igual que es cota inferior y todo numero mayor o igual
2
que 12, 5 es cota superior.

341
342

Ejemplo 12.2
R+ es acotado inferiormente (0 es una cota inferior y todo numero menor que 0 tambien lo
es), pero no es acotado superiormente, como sera probado mas adelante.

Ejemplo 12.3
S = {x R : x 0 x2 < 2} es acotado superior e inferiormente. 0 es cota inferior y 3 es
cota superior, pues (x > 0 x2 < 2) (0 < x2 < 9) (0 < x < 3).

Ejemplo 12.4
Probaremos posteriormente que ni Z ni Q son acotados superior o inferiormente.

Definicion 12.2
Sea S R acotado superiormente y R. Diremos que es supremo de S y escribiremos
= Sup S si y solo si

1. es cota superior de S.

2. Si b es cota superior de S entonces b.

Definicion 12.3
Sea S acotado inferiormente y R. Diremos que es el nfimo de S escribiremos = Inf S
si y solo si

1. es cota inferior de S.

2. Si c es cota inferior de S entonces c.

Nota
De las definiciones es inmediato que Sup S es la menor de las cotas superiores de S e Inf S
es la mayor de sus cotas inferiores.

Ejemplo 12.5

a) Remitiendonos al ejemplo 12.1 anterior, tenemos que

1
Sup S = 12, 5 e Inf S =
2
1
Notemos que y 12, 5 son elementos de S.
2
b) Con referencia al ejemplo 12.2, se observa que 0 = Inf R+ aunque 0 6 R+ .

c) Con respecto al ejemplo 12.3, tenemos que 0 = Inf S pero no es claro si S tiene o no
supremo. En realidad solo con los axiomas de campo y orden no es posible probar o
rechazar esta afirmacion.

Ejemplo 12.6
Los intervalos (, b), (, b], (a, b) y [a, b] tienen por supremo el numero b.
343

Ejemplo12.7   
1 1 1 1
Sea S = : n N , es decir, S = 1, , , , tenemos que Sup S = 1. Mas adelante
n 2 3 4
podremos probar que Inf S = 0.

Teorema 12.1

1. El supremo de un conjunto es unico, es decir, si y son supremos de S entonces


= .

2. Sea el supremo de S. Entonces x si y solo si x es cota superior de S.

Las demostraciones, muy elementales, las dejamos al lector.

Teorema 12.2
Sea S R acotado superiormente y una cota superior de S. Entonces = Sup S si y solo
si para todo  R+ se tiene que  no es cota superior de S.

Demostracion
Sea = Sup S y  R+ . Por la definicion de supremo, es la menor cota superior de S y
como  < , se tiene que  no puede ser cota superior de S.

Supongamos ahora, que para cualquier  R+ se tiene que  no es cota superior de S.


Probaremos que ningun numero menor que puede ser cota superior de S.

b
Supongamos que b < . Sea 0 = . Como 0 = ( + b)/2, se desprende que
2
b = 20 < , luego b < 0 < . Como 0 no es cota superior de S se tiene
que existe un elemento s en el, tal que 0 < s. Pero entonces b < s lo que prueba que b no
puede ser cota superior de S. Luego, es la menor de sus cotas superior, es decir, = Sup S.

Ahora podemos enunciar el axioma que falta.

12.2. AXIOMA DEL SUPREMO


Todo subconjunto de R no vaco y acotado superiormente, tiene supremo.

Las consecuencias de este axioma son muchas y de muy variadas ndoles. Las mas inmediatas
son las siguientes.

Teorema 12.3
Todo subconjunto de R no vaco y acotado inferiormente tiene nfimo.

Demostracion
Sea S R no vaco y acotado inferiormente. Entonces T = {x : x S} es no vaco y acotado
superiormente. (Si l es cota inferior de S, entonces l es cota superior de T ). Por el axioma,
tiene supremo. Sea = Sup T . Sea x S. Entonces x T y x . De donde x a, lo
que muestra que es cota inferior de S. Ademas, si l es una cota inferior de S se tiene que
l es cota superior de T ; de donde l ; es decir, l . Luego, es el nfimo de S.
344

Teorema 12.4 (Propiedad Arquimidiana)

Sean a y b R+ . Entonces existe n N tal que an > b.

Demostracion
Si as no fuera, el conjunto S = {a, 2a, 3a, , }. Sera acotado superiormente por b; luego,
tendra un supremo. Llamemosle r. Por el teorema 12.2, r a no sera cota superior de S.
Entonces debe haber un ma S tal que r a < ma. De donde r < (m + 1)a. Pero como
m N se tiene que (m + 1)a S, lo que contradice el hecho que r debe ser cota superior de
S.

Como corolarios de este teorema se obtienen los siguientes resultados cuyas demostracio-
nes las dejamos al lector.

Corolario 1.

N no es acotado superiormente.

De este corolario resultan de inmediato las afirmaciones de los ejemplos 12.2 y 12.4.

Corolario 10.2.

1
Sea  R+ . Entonces existe n N tal que < .
n

Corolario 10.3.

Sea x R. Entonces existe un unico n Z tal que n x < n + 1.

La importancia del siguiente concepto se vera posteriormente.

Definicion 12.4
Sea D R. Diremos que D es denso en R si y solo si para cada par de elementos a y b de R
con a < b, existe un d D tal que a < d < b.

Nota
Es inmediato que si D es denso en R, entonces entre a y b existen infinitos elementos de D.

Teorema 12.5
Q es denso en R.

Demostracion
1
Sean a y b elementos de R con a < b. Por el corolario 2, existe un natural q tal que < b a.
q
345

Por el corolario 3 existe un entero p tal que p aq + 1 < p + 1. De aqu, dividiendo por q,
sale que

p 1 p 1
a+ < + .
q q q q
p p 1 1
La segunda desigualdad muestra que a < . De a + y aprovechandoque < b a, sale
q q q q
p
que < b.
q
Este teorema nos asegura que dado un numero real r, siempre podemos encontrar un racional
q entre r y r +  por muy pequeno que sea ; en otras palabras, podemos aproximar r por
numeros racionales tanto cuanto queramos.

Los siguientes teoremas prueban el hecho fundamental que Q es un subconjunto propio


de R; es decir, existen numeros reales que no son racionales. A estos numeros los llamaremos
irracionales.

Teorema 12.6
Existe un unico numero en R+ cuyo cuadrado es 2. Lo designaremos 2.

Demostracion
Sea S = {x R : x 0 x2 2}.

En el ejemplo 3 vimos que S es acotado. Sea su supremo. Probaremos que 2 = 2. Como


1 S se tiene que 1.

2 2
Supongamos que 2 < 2. Definamos s = . Se tiene que 0 < s < 1.
( + 1)2
Es facil probar y lo dejamos de ejercicio que ( + s)2 < 2 + s( + 1)2 . Reemplazando en el
segundo miembro de esta desigualdad s por (2 2 )/( + 1)2 obtenemos que ( + s)2 < 2 y
por lo tanto a + s S. Pero + s > contradiciendo el hecho que es el supremo de S. Esta
contradiccion prueba que 2 2. Supongamos ahora, que 2 > 2. En este caso definamos
2 2
s= . Es facil ver que 0 < s < 1 y por lo tanto, s > 0.
( + 1)2
Se prueba de inmediato que ( s)2 > 2 s( + 1)2 de donde, reemplazando en el segundo
miembro s por (2 2)/( + 1)2 obtenemos que ( s)2 > 2. Esta desigualdad muestra
que s es una cota superior de S, contradiciendo el hecho que es la menor de sus cotas
superiores. Luego, la unica alternativa valida es que 2 = 2.

Debemos probar a continuacion que no hay otro real positivo tal que su cuadrado sea 2.

Sea R+ tal que 2 = 2.

Entonces 2 2 = 0 = ( )( + )
Como y son positivos tambien lo es +. Luego, = 0, lo que concluye la demostracion
del teorema.
346

Teorema
12.7
2 no es racional.

Demostracion
Por contradiccion.
p
Supongamos que 2= con p y q enteros y primos relativos. En estas condiciones tenemos, al
q
elevar al cuadrado, que p2 = 2q 2 . Esta igualdad muestra que p es par, puesto que el cuadrado
de un impar sigue siendo impar. Sea p = 2m, entonces 2q 2 = 4m2 de donde obtenemos que
q tambien es par. Pero esta conclusion contradice el hecho que p y q son primos relativos, lo
que prueba el teorema.

Teorema 12.8
R Q no es vaco.

Demostracion

Por los dos teoremas anteriores tenemos que 2 existe y no es elemento de Q. Luego, 2
R Q.

Definicion 12.5
Al conjunto RQ lo designaremos por I y a sus elementos los llamaremos numeros irracionales.

Teorema 12.9
Sea q un racional distinto de 0 y x un irracional. Entonces los siguientes numeros son irracio-
nales: x + q, x q, xq, x/q y q/x.

Demostracion
Son todas muy sencillas. Haremos dos a modo de ejemplo.

1. Si x + q fuese un racional p, se tendra x = p q. Pero p q Q contradiciendo el hecho


que x no es racional.
q q
2. Si fuese un racional p, se tendra que x = . Pero q/p Q en contradiccion con que
x p
x es irracional.

Teorema 12.10
El conjunto de los numeros irracionales 1 es denso en R.

Demostracion

Sean a yb R con a < b. Como
2 es positivo
se tiene que a 2 < b 2. Por ser Q denso en
R existe un q Q tal que a 2 < q < b 2.
q
De estas desigualdades obtenemos a < < b que prueba el teorema ya que q/ 2 es
2
irracional.
347

12.3. EXPONENTES RACIONALES


En esta seccion probaremos la existencia de races de orden n de los numeros reales po-
sitivos. Este hecho permitira generalizar la operacion de elevar a potencia, pudiendose usar
como exponentes numeros racionales.

Teorema 12.11
Sea a R+ y n un numero natural. Entonces existe un unico real positivo b tal que bn = a.

Este numero se acostumbra designarlo por n a.

Demostracion
Sea B = {x : x R+ xn < a}.

Demostraremos, primero, que B es no vaco y acotado superiormente.


a
Sea t = . Es inmediato que 0 < t < 1 y que t < a. Luego, tn < t < a, lo que demuestra
1+a
que t B.

Sea c = a + 1. Entonces c > 1 y c > a.

Luego, cn > c > a. Por lo tanto, si x B se tiene que xn < cn , lo que implica que x < c.
Luego, B es no vaco y acotado superiormente. Por el axioma, tiene supremo. Sea b = Sup B.

Probaremos, a continuacion, que bn = a. Lo haremos por contradiccion. Supongamos que


bn < a. Sea h un numero entre 0 y 1 tal que

a bn
h<
(b + 1)n bn
La razon de elegir este numero se vera en la argumentacion que sigue. Por el teorema del
binomio se tiene que
n
X n
X
n
(b + h) = Cin bni hi n
=b +h Cin bn1 hi1 .
i=0 i=1

Usando el hecho que 0 < h < 1, obtenemos la desigualdad


n
X
(b + h)n < bn + h Cin bn1 = bn + h[(b + 1)n bn ].
i=1

De la desigualdad inicial para h se tiene que

a bn
(b + h)n < bn + [(b + 1)n bn ] = a.
(b + 1)n bn
Luego, b + h B en contradiccion con el hecho que b = sup B.

Supongamos, ahora, que bn > a.


348

Elijamos un numero k entre 0 y 1 tal que cumpla con las dos condiciones k < b y k <
bn a
.
(b + 1)n bn
Como b k no es cota superior de B, existe algun elemento m en B tal que m > b k.

Entonces

mn > (b k)n = bn k(C1n bn1 + C2n bn2 k + + (1)n Cnn k n1 )

bn k(C1n bn1 + C2n bn2 k + + Cnn k n1 )

bn k(C1n bn1 + C2n bn2 + + Cnn )

= bn k[(b + 1)n bn ]

bn a
> bn [(b + 1)n bn ] a
(b + 1)n bn

Hemos probado la existencia de un elemento m B tal que mn > a. Pero esta desigualdad
contradice la definicion de B. Luego, la unica alternativa que queda es bn = a.

Demostraremos la unicidad de b. Si d es un real positivo tal que dn = a, entonces dn = bn lo


que implica que d = b.

Definicion 12.6
Sea a un real positivo y p/q un racional cn q N. Definimos

ap/q = q
ap

Teorema 12.12
La definicion anterior es independiente de la forma del racional. Es decir, si p/q = m/n,
entonces ap/q = am/n .

Demostracion

(ap/q )n q = (( q ap )q )n = apn

(am/n )nq = (( n am )n )q = amq
p m
Pero = si y solo si pn = mq.
q n
Luego, (ap/q )nq = (am/n )nq . Pero esto implica que ap/q = am/n .

Teorema 12.13
Sean a y b reales positivos y n un natural. Entonces

i) n ab = n a n b
r
n
a a
ii) n = n
b b
349


iii) ( m a)n = m an
iv) (an/m )m = an

Demostracion

i) De a = ( n a)n y b = ( n b)n , obtenemos que ab = ( n a)n ( n b)n = ( n a n b)n . Extrayendo
raz de orden n:

n
n
n
ab = a b.
a
r r
a n
ii) n
a= n
b= n b. Dividiendo por n b se obtiene el resultado.
b b

iii) ( m a)mn = (( m a)m )n = an . Tambien ( m an )m = an . Luego, ( m a)n )m = ( m an )m .

Por lo tanto, extrayendo raz de orden m se tiene:


( m a)n = m an .

iv) Es inmediata de la definicion.

Teorema 12.14
Sean a y b reales positivos y r y s racionales. Entonces

i) ar as = ar+s
ii) ar /as = ars
iii) (ar )s = ars
iv) (ab)r = ar br
v) (a/b)r = ar /br

Demostracion
Podemos suponer que r = m/q y s = n/q con m, n y q enteros, este ultimo positivo.

q
i) ar as = q am q an = q am an = am+n = a(m+n)/q = ar+s
ii) ars as = ars+s = ar
Dividiendo por as se obtiene la igualdad.
m m
nq
iii) amn = a q = (a q )nq .
Extrayendo raz de orden q 2 , nos queda que:

q m
2 2
nq
q 2 amn = q (a q )nq y por la definicion amn/q = (am/q )q 2 , es decir, ars = (ar )s .

iv) y v) son inmediatos del teorema anterior.


350

12.4. LAS FUNCIONES ax Y lga x


Antes de definir estas funciones necesitamos generalizar la elevacion a potencia admitien-
do como exponentes a cualquier numero real. Para esto debemos establecer previamente un
teorema y un corolario.

Teorema 12.15
Sea a un numero real mayor que 1 y r y s racionales. Entonces r < s implica que ar < as .

Demostracion
m n
Podemos suponer que r = y s = . Entonces r < s si y solo si m < n. Pero m < n implica
q q
que am < a . Por resultados ya conocidos se tiene que am/q < an/q , es decir, ar < as .
n

Corolario
Si a es un real entre 0 y 1 y r y s son racionales con r < s, entonces ar > as .

Demostracion
 r  s
1 1 1
Por la hipotesis se tiene que > 1. Luego, < , de donde se obtiene la afirmacion
a a a
Definicion 12.7
Sean a y x numeros reales con a > 0. Definimos

sup{ar : r x r Q} si a > 1






x
a = 1 si a = 1




nf{ar : r x r Q} si a < 1

Notemos que esta definicion se basa en el hecho que {ar : r x r Q} es acotado su-
periormente cuando a > 1, como se desprende del teorema 15 y que {ar : r x r Q}
es acotado inferiormente si 0 < a < 1. Esta ultima afirmacion es consecuencia inmediata del
corolario anterior.

Esta definicion de potencias coincide con la que tenamos de ax cuando x es un racional.


Esto es as puesto que por el teorema 15, ap/q con a > 1, es el mayor elemento del conjunto
p
{ar : r r Q} y por lo tanto, es su supremo.
q
En el caso que 0 < a < 1 se tiene que, como consecuencia del corolario ap/q = nf {ar : r
p/q r Q}.

Los siguientes dos teoremas generalizan las propiedades de las potencias. Sus demostraciones
son largas y tediosas. No las incluiremos.

Teorema 12.16
Sean a y b reales positivos. Entonces

1. (ab)x = ax bx
351

 a x ax
2. =
b bx

Teorema 12.17
Sea a un real positivo. Entonces

1. ax ay = ax+y

2. ax /ay = axy

3. (ax )y = axy
El siguiente teorema nos ayuda a establecer el grafico de la funcion ax .

Teorema 12.18
Sea a > 1. Entonces la funcion ax no es acotada superiormente.

Demostracion
Sea b un numero real. Encotraremos un x tal que ax > b.

b1
Sea n un natural mayor que .
a1
Entonces an = (1 + a 1)n 1 + n(a 1) > 1 + (b 1) = b.

Dejamos al lector probar la validez de este teorema en el caso en que 0 < a < 1.

Resumiendo las propiedades de ax , se obtiene que:

1. Es una funcion de R en R+ estrictamente creciente o decreciente, dependiendo si a > 1


o 0 < a < 1.

2. No es acotada superiormente.

3. En 0 toma el valor 1 y en 1 el valor de a.

Se puede demostrar, aunque no lo haremos en esta ocasion, que es sobre R+ . Por lo tanto, se
trata de una biyeccion de R en R+ .

Grafico de ax con a > 1 Grafico de ax con 0 < a < 1


Figura 13.1
352

Estudiaremos ahora la relacion inversa. Sabemos que la inversa de una funcion biyectiva tam-
bien es una biyeccion.

Se denota loga x y se lee logaritmo en base a de x. Su grafico se obtiene por la reflexion en


la recta de pendiente de 45 y que pasa por el origen del grafico de la funcion ax .

log(x) en base e

log(x) en base 1/e

Figura 13.2

Como la composicion de una biyeccion con su inversa es la identidad se tiene que

xa
1. alg =x

2. lga ax = x

Del grafico se tiene que loga x es estrictamente creciente cuando a > 1 y estrictamente decre-
ciente si 0 < a < 1.

En el siguiente teorema se resumen las propiedades algebraicas de esta funcion.

Teorema 12.19

1. lga (xy) = lga x + lga y


 
x
2. lga = lga x lga y
y

3. lga xy = ylga x

4. lga x = (lga b)(lgb x)

Demostracion
Son inmediatas a partir de los teoremas 12.16 y 12.17
353

12.4.1. EJERCICIOS
1. Sean A, B R c R. Definamos:

A + B = {x + y : x A y B}

A B = {x y : x A y B}

c + A = {c + x : x A}

c A = {c x : x A}

Sean A y B acotados superiormente. Pruebe que:

a) sup(a + B) = sup A + sup B


b) sup(c + A) = c + sup A
c) sup(c A) = c sup A si c > 0
d ) Si los elementos de A y B son todos positivos, entonces sup(A B) = sup A sup B.

2. Pruebe que todas las cotas superiores de un conjunto acotado forman un intervalo del
tipo (0, ).
 
p
3. Sea D = : p Z q N , con (p, q) = 1. Pruebe que D es denso en R.
q2

4. Sea M = {r + 3 2 : r s Q}. Pruebe que M es un campo ordenado.

5. Pruebe que n
m con n, m N es irracional o es natural.

6. Demuestre el Teorema 1.

7. Demuestre los Corolarios 1, 2 y 3 del Teorema 4.

8. Complete la demostracion del Teorema 9.

9. Sea S R y f y g funciones de S en R. Se dice que f es acotada superiormente (infe-


riormente) si y solo si {f (x) : x S} es acotado superiormente (inferiormente). En este
caso se define el sup f (nf f ) como el supremo (nfimo) de {f (x) : x S}. Sean f y g
acotadas superiormente.

Pruebe que:

a) sup(f + g) sup f + sup g


b) Si f (x) y g(x) son no negativos para todo x S entonces sup(f g) sup f sup g.

10. Pruebe que si n es un natural impar entonces la funcion xn es estrictamente creciene en


R.
354

11. Sea c un real y r un racional ambos mayores que uno. Pruebe que cr > 1 + (c 1)r.

m
Sugerencia: Sea r = + 1. Aplique A > G a los m + n numeros de los cuales m de ellos
n
son 1 y los restantes son 1 + (m + n)(c 1)/n.

12. Sea 0 < a < 1. Pruebe que la funcion ax no es acotada superiormente.

13. Demuestre el teorema 19.

12.5. SOLUCIONES O SUGERENCIAS A LOS EJERCICIOS


DEL CAPITULO
Pagina 176

1. d) Sea = sup A, = sup B y = sup A B. Sea m A B. Entonces m = xy con


x A y B. Luego, m .

Por lo tanto, . Supongamos que < . Sea  = . Entonces x
+
A y B tales que x >  y > beta . Luego, xy > ( )( ) = 2 + > .
Pero xy A B contradiciendo el hecho que es su supremo. Luego, = .

3. Sean a y b reales con a < b. Como los numeros primos no son acotados, existe q primo
p
tal que q 2 (b a) > 3. Sea p el entero siguiente al numero q 2 a. Si (p, q) = 1 entonces 2
q
p+1
es el racional pedido; sino satisface el enunciado.
q2
p pn
5. Si n
M = con (p, q) = 1 entonces n = M , es decir, q n |pn ; pero, sumado al hecho
q q
que (p, q) = 1, es fuerza que q = 1.

10. Sea x < y. Entonces

y n xn = (y x)(y n1 + y n2 x + + xn1 )

a) Si 0 < x < y. Entonces es inmediato que y n xn > 0.


b) Si x < y < 0. Entonces, por ser n impar, se tiene que y n1 +y n2 x+ +xn1 > 0.
Luego, y n > xn .
c) Si x < 0 < y entonces xn < 0, y n > 0.
Captulo 13

PROBLEMAS
COMPLEMENTARIOS

13.1. CAPITULO 1
1. Determine cuales de las siguientes relaciones de R en R son funciones.

a) {(x, y) : 0 < x < 1 x + y = 1}


b) {(x, y) : 0 < x < 1}
c) {(x, y) : x2 + y 2 = 4}
d ) {(x, y) : x + 2 = y}
e) {(x, y) : y = 0}
x2 y 2
f ) {(x, y) : + < 1}
4 9

2. Probar que las tres relaciones siguientes son biyecciones y calcular hf g y h1 f 1 g 1 .

x1 2x 5
f (x) = , g(x) = 2x + 1, h(x) =
3 4
3. Sean f, g y h funciones de R en R. Demostrar o rechazar cada una de las siguientes
proposiciones.

a) f (g + h) = f g + f h
b) f (g h) = (f g) (f h)
1 1
c) = g
f g f
1 1 1
d) = .
f g f g

4. Sea R una relacion de equivalencia definida en el conjunto {1, 2, 3, 4, 5, 6}. Sabiendo que
los pares (1, 5), (6, 2), (5, 4), (2, 4) y (3, 1) pertenecen a R pruebe que (6, 3) tambien
esta en la relacion.

355
356

5. En Z (Z {0}) se define la siguiente relacion R : (a, b)R(c, d) si y solo si ad = bc.


Probar que R es de equivalencia. Determinar la clase del elemento (3, 7).

6. En R R se define la siguiente relacion S : (a, b) S(c, d) si y solo si d b = 2(c a).


Probar que S es de equivalencia. Determinar la clase del elemento (1, 2).

7. En N se define la relacion D por aDb si solo si existe n N tal que b = na. Probar que
D es un orden parcial.

8. Encontrar los dominios de las siguientes funciones numericas de variable x:



a) a2 x2
p
b) 1 1 x2
1
c)
x2 4

d) 1 x + x 2

9. Sea f : R R definida por f (x) = ax + b. Encontrar una funcion g tal que se cumpla
que (f g f )(x) = cx + d.

10. Expresar analticamente las relaciones cuyos graficos son los siguientes:

Figura 13.1 Figura 13.2 Figura 13.3

11. Sea O un punto del plano. Diremos que los puntos P y Q estan relacionados si y solo si
O, P y Q son distintos y colineales o P = Q. Pruebe que es una relacion de equivalencia.
Determine la clase de equivalencia de P . Cual es la clase de equivalencia de O?

12. Se dice que una funcion numerica es par si y solo si f (x) = f (x) e impar si y solo si
f (x) = f (x) para todo x de su dominio. Pruebe que:

a) Si f y g son pares, entonces f + g y f g son pares.


b) Si f es par y g impar, entonces f g es impar.
c) La suma de dos impares es impar, mientras que su producto es par.
d ) Si Dom h = R entonces existen 2 funciones f y g tales que h = f + g con f par y
g impar.
e) Que caractersticas tienen los graficos de funciones pares y de las impares?
357

13. Dado que el grafico de f (x) es:

1 x
Dibuje los graficos de: f (x + 1); f (x); ; f ( ). Es f 1 (x) funcion?
f (x) 2
14. Pruebe que ninguna funcion de A en SA es sobre. (Sugerencia: Suponga que exista una
funcion f sobre de A en SA y sea X = {x : x
/ f (x)})

13.2. CAPITULO 2
1. Si uk+1 = 2uk + 1, probar que un + 1 = 2n1 (u1 + 1)

3 3 3 3n+1 3
2. Si 4 = = u1 + = u2 + = , probar que un = n+1
u1 u2 u3 3 1
3. Sea a1 = a2 = 1 y an+1 = 3an + an1 . Pruebe que an con an+1 son primos relativos.

4. Encuentre enteros p y q tales que:

a) 547p + 632q = 1
b) 398p + 600q = 2

5. Encuentre dos soluciones enteras y diferentes de la ecuacion 299x + 2474 = 13.

6. Demuestre por induccion que:


n
X k 2k 2n+1
a) =1
(k + 2)! (n + 2)!
k=1
n
X (1)k1 4(k + 1) 1 1
b) = + (1)n1
(2k + 1)(2k + 3) 3 2n + 3
k=1
c) (15 + 25 + + n5 ) + (17 + 27 + + n7 ) = 2(1 + 2 + + n)4
358

7. Calcule la suma de los primeros n terminos de:

a) 12 3 + 22 4 + 32 5 +
b) 12 22 + 32 42 +

8. a) A partir de f (r) = 3r4 (r 1)4 4r3 (r 1)3 , calcule


n
X
r7
r=1

n
X
b) A partir de f (r) = r3 (r 1)3 (2r 1) r2 (r 1)2 (2r 1), calcule r6 .
r=1

9. Pruebe que:

n
X 1 1 1 1
2 2
=
r + 1 r (r + 2) 16 4(n + 1)2 (n + 2)2
r=1

10. Calcule:
n
X 1
4k 2 1
k=1

11. Calcule:
n
X 1
. (Sugerencia: Fracciones parciales).
(1 + ka)(1 + (k + 1)a)
k=1

12. Calcule
n
X k
(k + 1)!
k=1

13. Calcule
n
X k (k 1)2
k!
k=1

14. Encuentre la suma de todos los numeros de las siguientes figuras:

1 1
1 2 1 3
a) 1 2 3 b) 1 3 5
.. ..
. .
1 2 3 n 1 3 5 7 2n 1

15. Demostrar que la suma de un numero impar de terminosDefinicion una P.A. es igual
al termino central multiplicado por el numero de terminos.

16. Use integracion para calcular la suma de n terminos de 1 + 2x + 3x2 + 4x3 , . . ..


359

13.3. CAPITULO 3
1. Demuestre que:
 
n
nr n
a) r+1=
r+1 r
r m r
   rk 
b) m k = k mk
c) n6 > n5
 
si n > 11

2. Simplifique:
n+1

3
a) n
2
n+1

r+1
b) n

r
n n n
  
c) k + 2 k1 + k2

3. Encuentre el 4 termino en el desarrollo de:

a) (2a 5b)7
1 n
 
b) x +
x

4. Encuentre el coeficiente de x4 en el desarrollo de:

(1 + 2x + 3x2 )5

5. Encuentre el valor de a para que los coeficientes x7 y x6 en el desarrollo de


(x + a)5 (x 2a)3 sean iguales.

6. Si (2 + 3x)7 = a0 + a1 x + a2 x2 + + a7 x7 . Entonces pruebe que:

ak+1 21 3k
=
ak 2k + 2

y de aqu deduzca que:

a0 < a1 < a2 < a3 < a4 ; a4 > a5 > a6 > a7

7. Encuentre el coeficiente de xn en:


 2n
1
a) 1+ x
4
1 2n
 
2
b) x +
x
360

8. Demuestre que:

n n n
  
1 2 2 n n 1
n + n + + n = n(n + 1)
0 1 n1
2

9. Simplifique

n  
X n nk
x (x 1)nk (x + 1)k
k
k=0

10. Demostrar que:

n    
X r+k r+n+1
= (r N
k n
k=0

11. Demostrar por induccion:

n    
X r r1
(1)k = (1)n (r N
k n
k=0

12. Encuentre relaciones igualando los coeficientes de xi en cada una de las siguientes iden-
tidades:

a) (1 x)n+1 = (1 + x)(1 + x)n


b) (1 + x)n+2 = (1 + 2x + x2 )(1 + x)n
c) (1 + x)n+3 = (1 + 3x + 3x2 + x2 )(1 + x)n

13. Pruebe que:

n
X k 2k 2n+1
=1
(k + 2)! (n + 2)!
k=1

14. Use los ejercicios 3.a) y 16, del captulo 3 para calcular:

m+n
X  
m n
k.
k k
k=1

13.4. CAPITULO 4
1. Determinar cuantos numeros enteros y positivos menores de 5000 pueden formarse con
los 8 dgitos, del 0 al 7 inclusive, si no se permite la repeticion.

2. Una persona tiene 7 libros de matematicas, y otra, 9. De cuantos modos pueden cambiar
un libro de uno por un libro de otro?
361

3. Calcular el numero de diagonales de un polgono convexo de n lados.

4. Hallar el numero de palabrasque conteniendo dos consonantes y dos vocales pueden


formarse con 5 consonantes y 3 vocales.

5. En cuantas formas puede escogerse un comite de 6 personas entre 12 personas si dos


personas determinadas no pueden aparecer en el mismo comite?

6. De cuantas maneras pueden colocarse p fichas rojas y q blancas en una fila con p > q
si se debe empezar y teminar con roja y no debe haber dos blancas vecinas?

7. De cuantas maneras pueden seleccionarse 10 fichas de un numero ilimitado de rojas,


blancas, azules y verdes? Cuantas de estas selecciones contienen los cuatro colores?

8. n cosas estan ordenadas en una fila. De cuantas maneras se pueden seleccionar tres de
modo que ningun par de ellas sean vecinas?
Lo mismo pero con un orden circular.

9. De cuantas maneras pueden dividirse cinco ninas y tres ninos en dos equipos de cuatro
personas si cada equipo debe tener por lo menos un varon?

10. Cuantas permutaciones hay de n elementos de modo que ningun par de los elementos
a, b, c sean vecinos?

11. Sean p y q numeros primos. Cuantos suman todos los divisores del numero n = pq q p ?

12. De cuantas formas 3 personas pueden repartirse entre si 6 manzanas, 1 naranja, 1


membrillo, 1 limon, 1 pera, 1 melon y 1 pomelo de modo que cada una obtenga 4
frutas?

13. De una gran cantidad de fichas blancas, rojas, y azules se eligen 20. De cuantas maneras
puede hacerse?

14. De cuantas formas pueden dividirse 3n objetos distintos en 3 personas si cada una debe
recibir n objetos?

15. De cuantas formas se pueden sentar 7 personas en torno a una mesa circular si:
a) No hay restricciones.
b) Dos particulares no pueden estar sentadas juntas.

16. En un plano hay n puntos de los cuales p son colineales. Determine cuantas rectas se
pueden trazar uniendo estos puntos y sabiendo que excluidos los p puntos mencionados
no hay conjuntos de mas de dos puntos colineales.

17. Si n puntos de un plano se unen de a dos por rectas determinan el numero de inter-
secciones entre ellas suponiendo que no hay tres puntos colineales y que ningun par de
rectas resultan paralelas.
362

13.5. CAPITULO 5
1. Defina un espacio muestral para los siguientes experimentos:

a) Se lanzan tres dados.


b) Se lanza un dado tres veces.
c) Se lanza una moneda repetidamente hasta obtener dos caras o dos sellos en forma
consecutiva.

2. Se sacan dos bolas de una caja que contiene 10 bolas blancas, 8 negras y 3 rojas. Cual
es la probabilidad que:

a) ambas sean blancas?


b) ambas sean del mismo color?
c) por lo menos una sea blanca?

3. Una caja contiene 5 bolas rojas, 6 blancas y 7 negras. Otra contiene 7 rojas, 8 blancas
y 9 negras. Se saca una bola de cada caja. Cual es la probabilidad que:

a) ambas sean blancas?


b) ambas sean del mismo color?
c) por lo menos una sea blanca?

4. Un grupo de 10 personas se sientan en un banco largo. Cual es la probabilidad que las


personas A y B queden vecinas?

5. El 5 % de los hombres y el 0,25 % de las mujeres padecen de daltonismo. Se elige una


persona al azar, que resulta ser daltonica. Cual es la probabilidad que sea mujer, bajo
el supuesto que el numero de hombres es igual al de mujeres?

6. Cual es la probabilidad de obtener k numeros 3 en n lanzamientos sucesivos de un


dado?

7. Se sacan 6 cartas de un naipe. Cual es la probabilidad que salgan 3 rojas y 3 negras?

8. 20 bolas se distribuyen al azar en 3 cajas. Cual es la probabilidad que la primera caja


tenga por lo menos 4?

9. Se sabe que P (A) = 52 y P (B) = 31 , P B = 14 . Determinar P (A|B), P (B|A), P (A B),


P (A0 |B 0 ) y P (A0 B 0 ).

10. Dos dados son tirados sucesivamente hasta que la suma 6 o 7 aparezcan. Cual es la
probabilidad que 6 ocurra primero?

11. Dos dados se tiran n veces en sucesion. Cual es la probabilidad que haya aparecido un
seis y un cinco?

12. El diagrama representa un circuito electrico con 5 rele.


Sea pi (i = 1, 2, 3, 4, 5) la probabilida que el rele i este cerrado. Cual es la probabilidad
que circule corriente entre L y R?
363

13. Una persona tiene n llaves distintas de las cuales solo una abre una puerta. Ella elige las
llaves al azar eliminando las que fallan. Cual es la probabilidad que se abra la puerta
en el k-esimo intento? Cual es si no se eliminan las que fallan?

14. Se tienen n cajas, cada una conteniendo fichas blancas y negras del modo siguiente: la
i-esima caja tiene i blancas y 4i negras. Se elige una caja al azar y de ella se extrae una
ficha. Cual es la probabilidad que sea blanca?

15. Se extraen cartas, una tras otra, de un naipe. Cual es la probabilidad que entre las
primeras 10 no halla ningun as?

16. Una persona A acierta en el centro de una diana con una probabilidad de 1/2 y otra
persona, B, con una probabilidad 2/5.
a) Cual es la probabilidad que, por lo menos, halla un acierto en la diana si disparan
simultaneamente?
b) Si hay un acierto, cual es la probabilidad que corresponda a B?

17. Una caja tiene 20 tarjetas numeradas del 1 al 20. Se saca una tarjeta, se anota su
numero y se devuelve a la caja. Esta operacion se realiza ocho vece sucesivas. Cual es
la probabilidad que los 8 numeros anotados sean distintos?

18. Se tienen en una fila 8 cartas de un naipe de las cuales 4 son rojas y 4 negras. Una persona
intenta adivinar los colores de ellas mirandolas por el reverso. Cual es la probabilidad
que acierte con el color en 4 cartas si la persona sabe cuantas cartas de cada color hay?

19. Considere una lotera de 25 numeros con 5 premios. Si alguien tiene 5 numeros, cual
es la probabilidad de ganar por lo menos un premio?

20. Una caja tiene n bolas blancas, n negras, n rojas y n azules. Se sacan 4. Cual es la
probabilidad que la muestra tenga bolas de por lo menos 3 colores?

21. Se tiene un grupo de personas de las cuales el 60 % son varones. De entre ellas el 40 %
fuma, mientras que el 60 % de las mujeres son fumadoras. Cual es la probabilidad que
una persona que fuma sea varon.
364

22. De dos eventos A y B se sabe que P (A) = 1/4, P (B | A) = 1/2, P (A | B) = 1/4. De


las siguientes afirmaciones cuales son verdaderas?

i) A B =
ii) A B
iii) P (A0 | B 0 ) = 3/4
iv) P (A | B) + P (A | B 0 ) = 1

23. Se lanzan 3 dados. Que es mas probable, obtener suma 9 o suma 10?

24. Un naipe ingles le falta una carta roja. De el se extraen 13 resultando todas del mismo
color. Cual es la probabilidad que todas sean negras?

25. Considere una muestra de 3 bolas extradas de una caja de la siguiente manera. Inicial-
mente la caja contiene 5 bolas blancas y 7 rojas. En cada etapa una bola es sacada, se
anota su color y se regresa a la casa acompanada de otra bola del mismo color. Esta ope-
racion se repite 3 veces. Calcule las probabilidades que la muestra contenga: a) Ninguna
bola blanca; b) Una bola blanca; c) Dos bolas blancas; d) Las tres sean blancas.

26. Sea A y B dos eventos independientes y tales que P (A B) = 1/6 y P (A B 0 ) = 1/3.


Puede calcularse P (A) y(B)?

27. La persona A acierta en el centro de una diana con probabilidad 1/2 y la persona B,
con una probabilidad 2/5. Cual es el la probabilidad que por lo menos uno acierte si
disparan simultaneamente? Si hay un acierto, cual es la probabilidad que corresponda
a B?

28. En una caja hay 12 bolas de las cuales 4 son blancas. Tres jugadores sacan sucesivamente
una bola de la caja. Gana el primero que saque blanca. Calcular la probabilidad de ganar
de cada uno de ellos.

29. Se tienen tres cajas como la del problema anterior y cada jugador saca las bolas de
su propia caja. Primero saca la persona A, despues B y por ultimo C. Calcular las
probabilidades de ganar de cada uno de ellos.

30. Se lanzan dos dados repetidamente. Cual es la probabilidad que aparezca la suma 6
antes que la suma 7?

31. En el poker se reparten 5 cartas a cada jugador. Cual es la probabilidad que las 5 cartas
sean del mismo color?

32. En el poker, cual es la probabilidad que entre las 5 cartas halla exactamente un par?
(La mano es el tipo a,a,b,c,d)

33. Generalice el problema 31 para una suma k (k 6= 7) cuya probabilidad es n/36.

34. Una caja tiene 3 bolas blancas y 7 negras. A y B juegan del modo siguiente: sacan
alternativamente una bola y gana el primero que saque blanca. Si A empieza, Cual es
su probabilidad de ganar?
365

35. El juego de craps se realiza del modo siguiente: se tiran dos dados. Se pierde de inmediato
si la suma que aparece es 2, 3 o 12 y se gana si la suma es 7 u 11. Si es otra (4,5,6,8,9,10),
se siguen tirando los dados hasta que se repita la primera la suma o se obtenga 7. Si la
suma 7 aparece antes, se pierde; pero si aparece primero la suma inicial, se gana Cual
es la probabilidad que el lanzador de los dados gane ?

36. De un grupo de 13 cartas, extraer 2 en forma aleatoria de una baraja inglesa. Se sabe
que hay, por lo menos, un as. Cual es la probabilidad que en ese grupo halla por lo
menos otro as?

37. En una fabrica de televisores las maquinas I, II y III producen el 28 %, el 32 % y el 40 %


del total, respectivamente. En la produccion de cada maquina el 3 %, 4 % y el 5 % son
televisores defectuosos, Se elige al azar un televisor de la produccion total, que resulta
ser defectuoso. Cual es la probabilidad que haya sido producido por la maquina I?

13.6. CAPITULO 6
1. Pruebe que la suma 1 + 12 + 13 + + n1 , con n > 1, nunca es un natural. Sugerencia:
Considere el m.c.m de los denominadores.

13.7. CAPITULO 7
1. El ano solar oficial tiene 365,2425 das. Explique: por que el ano 2000 fue bisiesto, pero
el 2100 no lo sera?

2. Sea N = 44444444 . Sea A la suma de los dgitos de N , escritos en forma decimal. Sea
B la suma de los dgitos de A, escritos en forma decimal. Pruebe que la suma de los
dgitos de B no es mayor que 13. Sugerencia: Acote N por una potencia de 10.

3. Cuantos numeros de 10 cifras hay en el sistema binario? Y en el sistema duodecimal?

4. Pruebe que cualquier natural puede escribirse como una suma de numeros de la forma
2n , (n = 0, 1, 2, ), donde cada sumando aparece a lo mas una vez.

5. Construya las tablas de la suma y del producto para los dgitos de 6 dedos.

6. Usando estas tablas, calcule 4 354 + 2 053 y 4 354 23.

7. En Farey(100), entre que racionales se encuentra el numero 5/82? Y el numero 4/63?

13.8. CAPITULO 8
1 xx+1 1 xn
1. a) Si 0 < x < 1, entonces <
n+1 n
1 1
b) Si 0 < x, entonces x + x3 + 3
x x
2. Si a, b, c, d, e son reales positivos y no todos iguales, pruebe que:

a) (a + b)n < 2n1 (an + bn )


366

b) (a2 + b2 + c2 )2 < (a + b + c)(a3 + b3 + c3 )


c) (a + b + c)2 > 3(bc + ca + ab)
d ) (a2 + b2 + c2 + d2 )(a3 + b3 + c3 + d3 ) < 4(a5 + b5 + c5 + d5 )
e) cd(a + b)2 < (ad + bc)(ac + bd)
f ) 25abcde < (a2 + b2 + c2 + d2 + e2 )(a3 + b3 + c3 + d3 + e3 )
a b c d
g) + + + > 4
b c d a
1
3. Pruebe que si 0 < x < , entonces
2
1x
(1 + x)(1 + x2 ) (1 + xn1 ) <
1 2x + xn
 8
1
4. Pruebe que (x + a)5 (a x)3 a 55 33 para todo x comprendido entre a y a.
4
x+a
(Sugerencia: Considere A G para 5 numeros iguales a y 3 numeros iguales a
5
ax
).
3

1
5. Use la sucesion f (k) = para probar que (n!)2 > (n + 1)n1 . (Sugerencia:
k(k + 1)
Aplique A > G a f (k)).

6. Pruebe que
 
1 1 1 2 n
a) < + + + <1
2 n 2 3 n+1

b) 2 > n n + 1 > 1
    
3n 1 1 2 n1 2n
c) < 1+ 2 1 + 2 + + 1 + 2
<
2n n n n n+1

7. Si a1 > a2 > > an+1 > 0 y pruebe que (a1 an+1 )n > nn (a1 a2 )(a2 a3 ) (an
an+1 ) a menos que esten en P.A.

8. Pruebe que el cubo es el paraleleppedo rectangular (P.R.)de mayor volumen para un


area dada.

9. Pruebe que:

1 1 1 n1
n( n n + 1 1) < + + + < n
1 2 n n n
1 2 n1
(Sugerencia: Aplique que A > G a las sucesiones , , , y
2 3 n
2 3 n+1
, , , ).
1 2 n
10. Resolver las siguientes inecuaciones:
367

(a + b)x ax + b
i) con a, b y x positivos.
ax + b a+b

ii) 2x + 1 + x 1 < 3x

11. Sean a, b, c positivos. Pruebe que

i) a + b > 2c an + bn > 2cn


ii) an = bn + cn implica am > bm + cm si m > n.

2n+3 1
12. Sea f (n) = . Pruebe que si n > 3, entonces 7 < f (n) < 8.
2n + 1

13.9. CAPITULO 9
1. Calcule:

a) (a + bw + cw2 )(a + bw2 + cw)


b) (a + b)(a + bw)(a + bw2 )
c) (a + bw + cw2 )3 + (a + bw2 + cw)3
d ) (aw2 + bw)(bw2 + aw)

donde w es una raz cubica compleja de 1.

2. Resuelva las siguientes ecuaciones:

a) z = z 3
b) |z| + z = 2 + i

3. Sean a y b dos numeros complejos, con b 6= 0 y sea t un real. Compruebe, por medio de
un dibujo, que z = a + bt es la ecuacion, de parametro t, de la recta que pasa por a y
es paralela al vector b.
z3 z 1
Pruebe que z1 , z2 y z3 son colineales si y solo si es un real.
z1 z 2
4. Describa los conjuntos de puntos z que satisfacen las siguientes condiciones:

a) |z z1 | = |z z2 |
b) |z 2| + |z + 2| = 5
c) |z 2| |z + 2| > 3
d ) Re z + Im z < 1

5. Si n N, demostrar que

n
(1 + i)n + (1 i)n = 2( 2 +1) cos n .
4
368

6. Demostrar que si 7 = 1 y 6= 1

2 3
+ + = 2
1 + 2 1 + 4 1 + 6
7. Dada las rectas z = a + bt y z = c + dt, indique condiciones que deben cumplir a, b, c y
d para que sean paralelas.

8. Si z = cos + i sen probar que:


1 1
a) z n + n
= cos(n)
z 2
1
b) z n n = 2i sen(n)
z
9. Resolver la ecuacion

a) z 6 + 7z 3 8 = 0

4 1 3
b) z = + i
2 2
10. Sea 6= 1 y 5 = 1. Sean x1 = 4 y x2 = 2 3 . Calcular:

x21 + x22 y x21 x22

11. Si n es multiplo de 4, pruebe que:

1
1 + 2i + 3i2 + + (n + 1)in = (n + 2 ni)
2
12. Simplifique

cis(2)(cis)2 cis 2(cis())2


+
cis( + ) cis ( + )

13. Calcule ii .

13.10. CAPITULO 10
1. Los restos al dividir un polinomio P por x a y x b son r y s, respectivamente.
Determinar el resto de la division de P por el producto (x a)(x b).

2. Pruebe que si P y Q son primos entre s entonces el m.c.d. entre P y T es igual al m.c.d.
entre P y QT .

3. Exprese x3 + 2x2 + x + 80 como un polinomio en:

i) x + 5
ii) x + 1
1
iii) x +
3
369

4. a) Si n es impar, pruebe que xm + 1 es un factor de xmn + 1.


b) Exprese x15 + 1 como un producto de cuatro factores.

5. Si x3 + 3x2 9x + c es de la forma (x + a)2 (x + b) pruebe que c es 5 o -27.

6. Pruebe que si R y R0 son los respectivos restos cuando P y P 0 son divididos por D
entonces P P 0 y RR0 dejan el mismo resto al dividirse por D.

7. Pruebe que nxn+1 (n + 1)xn + 1 es divisible por (x 1)2 .

8. Si el polinomio x4 + px3 + qx2 18x 12 se divide por (x + 1)(x + 3) el resto es 2x + 3.


Determinar p y q.

9. El resto cuando x2 3x + 2 divide a

px4 + qx3 18x2 + 15x 5 es 4x 7.

Pruebe que p = 1 y q = 4.

10. Exprese como suma de fracciones parciales:


x(x2 + x + 4)
(x + 2)(x2 + 2)
11. Exprese como suma de fracciones parciales
bx3 3ax2 + 6ax b
(x 2)(x 1)2 (x2 + x + 1)
2 1
12. Exprese como suma de fracciones parciales y cambie x por .
(x 3)(x 4)(x 5) x
Calcule la nueva funcion racional y expresela como una suma de fracciones parciales.
Repita el proceso con la sustituci0n de x por 1 x.
Nota
Los siguientes ejercicios constituyen una prueba del Lema de Gauss. Su enunciado es: Si
un polinomio con coeficientes enteros puede factorizarse en polinomios con coeficientes
racionales, entonces puede factorizarse en polinomios con coeficientes enteros.
Antes de proponer los ejercicios necesitamos una definicion.
Sea Q(x) un polinomio distinto de 0, con coeficientes enteros. Se define su contenido,
anotado por cont(Q), como el m.c.d. de sus coeficientes. Si cont(Q) = 1, diremos que Q
es primitivo.

13. Si Q y T son primitivos, entonces QT tambien lo es.


Sugerencia: Pruebe que para cualquier primo p hay algun coeficiente de QT del cual no
es factor.

14. Sean QX y T X polinomios con coeficientes enteros. Entonces, el cont(Q T ) = cont(Q)


cont(T ) =. (El contenido de un producto es igual a producto de los contenidos).

15. Demuestre el Lema de Gauss.


Sugerencia: Note que basta probarlo para polinomios primitivos.
370

13.11. CAPITULO 11
1. Hallar la ecuacion monica que tenga las mismas races de x5 13x4 + 68x3 176x2 +
220x 100 = 0, pero cada una de ellas simple.

2. Resolver:

a) x4 2x3 + 6x2 + 22x + 13 = 0 sabiendo que 2 + 3i es raz.


b) x6 3x5 + 4x4 6x3 + 5x2 3x + 2 = 0 sabiendo que i es raz.
c) 2x4 + x3 2x 8 = 0 sabiendo que tiene 4 races distintas pero de iguales modulos.

3. Pruebe que, si a > 1 la ecuacion:

i) x5 5a4 x + 4 = 0 tiene 3 races reales.


ii) x6 6a5 x + 5 = 0 tiene 2 races reales.

4. Pruebe que x6 2x5 + x4 + x2 2x + 1 = 0 tiene por lo menos dos races complejas.

5. Pruebe que la ecuacion x4 + 4x + 4 = 0 no tiene races reales.

6. Pruebe que P (x) = 8x3 + 12x2 + 30x 3 tiene solamente una raiz real y calculela hasta
el quinto decimal.

7. Separe las races reales de 2x5 4x4 9x 2 = 0.


n
X 1
8. Pruebe que si a1 , a2 , , an son distintos entonces la ecuacion = 0 tiene n1
x ak
k=1
races.

9. Resuelva:
(x + a)5 x5 a5 = 0

(x + a)7 x7 a7 = 0

10. Resuelva (x + a + b)5 x5 a5 b5 = 0.

11. Pruebe que las raices del polinomio

P (x) = 1 x/1! + x(x 1)/2! x(x 1)(x 2)/3! + + (1)n x(x 1) (x n + 1)/n!

son 1, 2, 3,...,n.

13.12. CAPITULO 12
n
2x+1
1. Sea A = 3x+8 : x 0} . Pruebe que A es acotado y determine su supremo y su nfimo.
SOLUCIONES O SUGERENCIAS
A LOS PROBLEMAS
COMPLEMENTARIOS

CAPITULO 1
1. a) Funcion con dominio (0, 1)
b) No es funcion
c) Limitando la raz a valores no negativos es funcion con domino [2, 2]
d ) y e) Ambas son funciones
f) No es funcion

2. f 1 (x) = 3x + 1, g 1 (x) = x1
2 , h1 (x) = 4x+5
2 , etc.

3. Solo c) es valida. Como ejemplo demostraremos que d) no esvalida.  Sea f (x) = x + 1


2 1 1
y g(x) = x . Entonces f g (x) = f (g(x)) = f (x2 ) = x2 +1 . Pero f g (x) = f1 g1 (x) =
1 1 1 1

1 1 1  = 1 1 x2

f x2 = f1 +1
= 1+x 2
x2 x2

4. Por transitividad y simetra se cumple que:


(3, 1) (1, 5) (3, 5) R; (3, 5) (5, 4) R (3, 4) R
(3, 4) (4, 2) R (3, 2) R; (3, 2) (2, 6) R (3, 6) R

5. (a, b)R(c, d) si y solo si ad = bc equivale a ab = dc , pues b y d no son ceros. Por lo tanto,


tenemos que la clase de equivalencia (3,7) esta formada por todas las parejas (x, y), tales
que xy = 73 .

6. Similar a 5)

7. Es obvio que D es refleja y transitiva. (a, b) D y (b, c) D si y solo si existe m y n


naturales tales que a = mb y b = na. De donde obtenemos que m = n = 1.

8. a) a x a
b) 1 x 1
c) x > 2 o x < 2
d ) x 1 x 2. Dominio=

371
372

9. (f g f )(x) = f (g(ax + b)) = ag(ax + b) + b = cx + d, de donde g(ax + b) = cx+db


a .
yb 2
Sea y = ax + b,entonces x = a . Reemplazando, g(y) = (cy cb + ad ab)/a .

10. a) |x| + |y| = 1


b) 0 x 1; 2 y 0
p
c) (x 1/2)2 + y 2 1/2


11. La clase de P son los puntos de la recta OP con exclusion de O. La clase de O es {O}.

12. Los casos a), b) y c) son inmediatos.


d) f (x) = 12 (h(x) + h(x)) y g(x) = 12 (h(x) h(x)).
e) Pares: simetricas con respecto al eje y. Impares: simetricas con respecto a la rotacion
en 180 grados.

13. Nos limitaremos a algunos casos. Grafico de f (x + 1) es el mismo, corrido una unidad
hacia la izquierda. Grafico de f ( x2 ) duplica la escala de los ejes. f 1 (x) no es funcion.

14. Supongamos que existe f : A SA sobre. Sea X = {x A : x / f (x)}. Como


X A debe existir y tal que f {y} = X. Si y X, por su definicion se tiene que
y/ f {y} = X . Si y
/ X, se tiene que y f {y} = X
373

CAPITULO 2
1. Induccion. un+1 = 2un + 1 = 2n (u1 + 1) 1
3
2. Induccion. Considere la ecuacion 4 = un + .
un+1
3. Induccion. an1 = an+1 3an .

4. a) 1 = 547 171 632 148.


b) 2 = 600 67 398 101.

5. En general si x0 , y0 es una solucion de la ecuacion ax + by = q, entonces x0 bt, y0 + at


tambien es solucion para cualquier valor de t.
n+1
X k2k 2n+1 (n + 1)2n+1
6. a) =1 + .
(k + 1)! (n + 2)! (n + 3)!
k=1

b) Similar.

c)1 + 2 + + n = n(n + 1)/2


n
X
7. a) k 2 (k + 2)
k=1

b) Considere los casos con n par e impar.

8. f (r + 1) f (r) = 24r7 8r3

9. f (r + 1) f (r) = 14r6 2r2 )


n
10. .
2n + 1
 
1 1 1
11. Sol.:
a 1 + a 1 + (n + 1)a
k 1 1
12. = .
(k + 1)! k! (k + 1)!

k (k 1)2 k1 k2
13. =
k! k! (k 1)!
n
X
1
14. a) 2 k(k + 1)
k=1
n
X
b) k2
k=1

15. a + (a + d) + (a + d) + + (a + 2nd) = (2n + 1)(a + nd)


374

16. Sea f (x) = 1 + 2x + + nxn1


1 xn
Z
f (x) = x + x2 + + xn = x .
1x
Derivando se obtiene el resultado.

CAPITULO 3
nr n nr n! n! n
 
1. a) r+1 r = r+1 r!(nr)! = (r+1)!(nr1)! = r+1
r m r! m! r (rk)! r rk
    
b) m k = m!(rm)! k!(mk)! = k (rm)! = k mk

c)
n n n! n! 1 1
 
6 > 5 6!(n6)! > 5!(n5)! 6(n6)! > (n5)!
(n5)! (n5)
6(n6)! > 1 6 > 1 n > 11

n+1
2. a) 3
n+1
b) r+1
n+2

c) k

3. a) 70,000a4 b3
b) n3 xn6


4. 530.
1
5. 8

6. Desarrollando, obtenemos que:

ak+1 C 7 26k 3k+1 21 3k


= k+17 7k k =
ak Ck 2 3 2k + 2
ak+1
Entonces ak+1 > ak sii ak > 1 sii

21 3k
>1
2k + 2
de donde k < 4

7. Para 1 k n se tiene que

n

k k kn!(k 1)!(n + 1 k)!
n = =n+1k
k1
k!(n k)!n!

por lo tanto,se tiene

1
n + n 1 + + 1 = n(n + 1)
2
375

(n + 1 k)!
8. El termino general se simplifica a = n + 1 k, con k = 1, 2, , n
(n k)!
9. Es inmediato que corresponde al desarrollo de

(x(x 1) + (x + k))n = (x2 + 1)n

10. Es facil demostrar que la afirmacion es valida para n = 1. Supongamos su validez para
n.

n+1
X  n          
r+k X r+k r+n+1 r+n+1 r+n+1 r+n+2
= + = + =
k k n+1 n n+1 n+1
k=0 k=0

11.
n              
X
k r r r r r r k r
(1) = + + + (1)
k 0 1 2 3 4 n
x=0
             
r1 r1 r1 r1 r1 r1 r1
= + +
0 0 1 1 2 2 3
       
r1 r1 n r1 n r1
= + + (1) + (1)
3 k n1 n+1
 
r1n
= (1)
n

12. Solo desarrollaremos el caso b). Los casos a) y c) son similares.

Pn+2 Pn
i=0 Cin+2 xi = (1 + 2x + x2 ) n i
i=0 Ci x Igualando los coeficientes de xi

Cin+2 = Cin + 2Ci1


n + Cn
i2

13. Caso c). Ckn+3 = Ck3


n n
+ 3Ck2 n
+ 3Ck1 + Ckn

14. Induccion

15.

CAPITULO 4
1. 7 de una cifra ; de dos cifras 7 7; de tres cifras 7 7 6; de cuatro cifras de 4 7 6 5.
En total, 1.190.

2. 7 9.
n2 3n
3.
2
376

4. Por casos. Consonantes y vocales distintas: 5!C25 C23 . Consonantes iguales y vocales
4! 4!
distintas: C15 C23 . Consonantes distintas y vocales iguales: C25 C13 . Consonantes y
2! 2!
4! 5 3
vocales iguales: C C1 .
2!2! 1
5. Nuevamente por casos. No estan ni A ni B; C610 . Esta A, pero no B;
C510 . Esta B, pero no esta A.

6. Se ordenan las P fichas rojas en una fila. Se debe colocar una blanca entre dos rojas.
Hay (p 1) lugares y hay que elegir q entre ellos. Solucion: Cqp1

7. Es la cantidad de soluciones enteras y no negativas de la ecuacion:


b + r + a + v = 10.
Para la segunda pregunta, la cantidad de soluciones enteras y no negativas de:
b + r + a + v = 6.

8. Generalicemos el problema.
Sea F (n, k) el numero de maneras de elegir k objetos de una fuente de n objetos, de
modo que ningun par de ellos sea consecutivo. Es inmediato de F (n, 1) = n. Ademas,
es bastante facil percatarse que F (n, k) = F (n 1, k) + F (n 2, k 1). Entonces, para
calcular F (n, 3) necesitamos calcular previamente F (n, 2).
F (1, 2) = 0, F (3, 2) = 1, F (4, 2) = F (3, 2) + F (2, 1) = 1 + 2 = 3
F (4, 2) = F (3, 2) + F (2, 1) = 3 + 2 = 5

9. Notemos que al formar un equipo, automaticamente se forma el otro. Sin restriccion, hay
C48 equipos, pero debemos restarle los casos en que el equipo esta formado por cuatro
ninas. Respuesta: C48 C45 .

10. Respuesta: (n 3)!C3n2 3!.

11. Respuesta: 1 + p + q + pq.


12!
12. Es equivalente a poner esas doce frutas en lnea. Respuesta: 6!

13. C 2 22 0

14. Cn3n Cn2n

15. a) 6!
b) Hay 5! 2 maneras distintas de sentar a estas dos personas juntas. Por lo tanto, se
tiene 6! 5! 2 maneras en que queden separadas.

16. Hay n numeros de cada una de las formas 3p, 3p + 1, 3p 1. Para que la suma de tres
numeros sea multiplo de 3, debe suceder uno de los siguientes casos: los tres numeros de
la forma 3p o los tres numeros de la forma 3p 1 o los tres numeros de la forma 3p + 1
o los tres numeros, uno de cada forma. Luego, el numero total de sumas divisibles por
3 es 3C3n + n3 .

17. La cantidad de rectas es C2n . El numero de puntos en que se cortan n rectas es 21 n(n1).
Luego, la solucion es 21 C2n (C2n 1)
377

CAPITULO 5
1. a) Considerando los dados como distinguibles, entonces podemos definir
U = {(a, b, c)/a, b, c, {1, . . . , 6}}. Este espacio tiene 216 elementos que son equi-
probables.

Considerando los dados como indistinguibles, podemos tomar como


U = {(x1 , x2 , . . . , x6 )/x1 + x2 + + x6 = 3}, con xi enteros no negativos.

Se interpreta del modo siguiente: xi representa la cantidad de i que aparecieron en


el lanzamiento. Por ejemplo: la tupla (0, 0, 0, 2, 0, 1) nos dice que en el lanzamiento
salieron dos 4 y un 6. En este espacio muestral los elementos no son equiprobables.
b) Coincide con el caso inicial del tem anterior.
c) U = {(a, b) : a es el numero de lanzamientos hasta conseguir 2 caras o dos sellos
consecutivamente y b es cara o sello }.

Por ejemplo: (5, c) representa la secuencia (s, c, s, c, c); mientras que (6, c) repre-
senta la secuencia (c, s, c, s, c, c).

La probabilidad de (a, b) es igual a (1/2)a

2. a) 3/7
b) 121/210
c) 31/42

3. a) 1/9
b) 73/216
c) 5/9

4. (9! 2)/10! = 1/5. Explicacion. Se consideran A y B como una sola persona. Estos se
pueden ubicar de 9! maneras. El 2 corresponde a las posiciones entre A y B.

P (D|M )P (M )
P (M |D) =
5. P (D|M )P (M ) + P (D|H)P (H)
1
=
21
1 k
6. Ckn

6

7. a) 1/3
b) 4/9

8. C326 C326 /C652

9. 34/75

10. P (A|B) = 43 , P (B|A) = 58 , P (A B) = 29


60 , P (A0 |B 0 ) = 31
40 , P (A0 B 0 ) = 31
60
378

11. Suma 6: (5,1); (4,2); (3,3); (2,4); (1,5).


Suma 7: (6,1); (5,2); (4,3); (3,4); (2,5); (1,6).
Quedan 25 parejas, cuyos resultados no tienen importancia.
La probabilidad de sacar 6 en el primer tiro es 5/36. La de sacar 6 en el segundo tiro,
no habiendo obtenido ni 6 ni 7 en el tiro anterior, es (25/36) (5/36), etc. En resumen,
la probabilidad de sacar 6 antes de 7 es 5/36(1 + 25/36 + (25/36)2 + ) = 5/36.
12. (1 P1 P2 ) es la probabilidad que por lo menos uno este abierto entre los reles 1 y 2.
(1 P3 P4 ) lo mismo para los reles 3 y 4. Solucion: (1 (1 P1 P2 )(1 P3 P4 ))P5
1
13. Equivale a que aparezca la suma 11. P (11) = 18 . Entonces, la probabilidad que en n
n
tiradas aparezca es 1 17
18 .

14. (n 1)k1 /nk ; 1/n


15. Es equivalente a juntar todas las fichas y sacar una blanca. La probabilidad es 1/5.
16. C148 0 es el numero de maneras de elegir 10 cartas sin ases. Ellas se pueden ordenar de
10! maneras. Las restantes 42 se ordenan de 42! maneras. Por lo tanto, la probabilidad
48
10!42!C10
P es igual a 52! = 246
595 = 0, 4134 . . .

17. a) La probabilidad que ninguno acierte es 3/10. Por lo tanto, la probabilidad que halla,
por lo menos, un acierto es 7/10.
1 3 1 2 5 1
b) La probabilidad de exactamente un acierto es 2 5 + 2 5 = 10 = 2 Luego, la
probabilidad que B sea el que acierte es 2/5.
18. 8!C820 /208
C24 C24
19. La unica manera de acertar exactamente 4 cartas es 2 negras y 2 rojas. Solucion: C48
=
18
35 .

20. La probabilidad de no sacar ningun premio es C520 /C525 .


21. P (F |V ) = P (V |F ) P (F )/(P (V |F ) P (F ) + P (M |F ) P (F ))
22. P B = 1/8.
23. Suma 9: 25/216. Suma 10: 1/8.
26 /(C 26 = 2/3)
24. C13 13
3 15 15 5
25. a) 13 ; b) 39 ; c) 52 ; d) 52 .

26. P (A) = 1/2 y P (B) = 1/3


27. La probabilidad que ninguno acierte es 3/10. Luego, la probabilidad de por lo menos un
acierto es 7/10. La respuesta para la segunda pregunta es 2/5.
28. Calcularemos la probabilidad para el primer jugador. Designemoslo por A. Sean B y C
los otros jugadores. Entonces la probabilidad de ganar A es:
Gane al primer intento + Pierda en el primer intento. Pierda B en su intento. Pierda C
en su intento. Gane A en su segundo intento + Etc.
P(A)=0.4667
379

29. Similar al problema anterior.

30. La probabilidad de suma 6 es 5/36 y la de 7 es 6/36. La probabilidad de sacar 6 antes que


7 es: 5/36+(25/36)(5/36)+(25/36)2 5/36+ = 5/36(1+25/36+(25/36)2 + ) = 5/11

31. 4 C513 /C552 = 0, 00198

Explicacion. 4 es el numero de pintas o colores; C513 , formas de seleccionar 5 cartas de


alguna pinta; C552 , formas de elegir 5 cartas del total.

32. 13 C24 C312 43 /C552 = 0, 4226

Explicacion. 13: seleccionar un numero. C24 : seleccionar dos cartas de ese numero. C312 :
seleccionar 3 numeros de los restantes. 43 : el color de las 3 cartas.

33. n/(n + 6)
3 763 76543 7654321
34. + + +
10 10 9 8 10 9 8 7 6 10 9 8 7 6 5 4

Explicacion. Gana en el primer saque o pierde A, pierde B y gana A o pierde A, pierde


B, pierde A, pierde B y gana A o etc.

35. Probabilidad de ganar en el primer tiro = 8/36


o probabibilidad de sacar suma 4 y ganar con esta suma= 3/36 3/9
o probabilidad de sacar 5 y ganar con esta suma = 4/36 4/10
o probabilidad de sacar 6 y ganar con esa suma = 5/36 5/11.
Las probablidades de ganar con las sumas 8, 9 y 10 son iguales a la de ganar con las sumas
6, 5, 4. Luego, la probablidad de ganr es 8/36 + 2(3/36 3/9 + 4/36 4/10 + 5/36 5/11) =
244
495 = 0, 4929

36. Sea A el evento. El grupo tiene, por lo menos, un as. Sea B el evento el grupo tiene,
P (B)
por lo menos, 2 ases . Se debe calcular P (B|A) = P P(BA)
(A) = P (A) , pues B A, etc.
5359
Respuesta: 14,498 0, 3696.

P (D|I)P (I)
P (I|D) =
P (D|I)P (I) + P (D|II)P (II) + P (D|III)P (III)
0, 03 0, 28
37. =
0, 03 0, 28 + 0, 04 0, 32 + 0, 05 0, 4
21
=
103

CAPITULO 6
1. Sea C el mnimo comun multiplo de los denominadores y supongamos que p = 1 + 1/2 +
1/3 + + 1/n. Entonces, Cp = C + C2 + C3 + + Cn . Notemos que C es par. Pero en la
suma C + C2 + C3 + + Cn existe solo un numero impar, a saber el que tenga el mayor
exponente de 2. Luego, la suma no es impar.
380

CAPITULO 7
1. 400 es el factor mas pequeno que convierte a 365, 2425 en un entero. Este es 146 097,
que corresponde al numero de das solares oficiales en 400 anos; pero equivale a 146 000
das calendario. Se hace necesario agregar 97 das a los 400 anos calendario para evitar el
desfase con el ano solar. Por esto, se opto agregar 1 da cada cuatro ano, incluyendo este
nuevo da en los anos cuyo numero fuera multiplo de 4. Pero este criterio, agrega 3 das
mas de los necesarios; para corregirlo se declaro como no bisiesto los anos terminados en
2 ceros. Pero esta solucion quito un da de mas, por lo tanto, se corrigio a que tambien
seran bisiestos los divisibles por 400.

2. N < (104 )4444 = 1017726 , que es un numero de 17.727 cifras. Luego, A < 17,727 9 =
1 + 9 + 43. Entonces, A tiene a lo mas 6 cifras. Por lo tanto, B 6= 54. Pero 13 es la suma
mayor de las dos cifras de los numeros no mayor a 54. Por lo tanto, la suma de las cifras
de B 6= 13.

3. 29 ; 11 129 .

4. Equivale a escribir el numero en base 2.


+ 0 1 2 3 4 5 * 0 1 2 3 4 5
0 0 1 2 3 4 5 0 0 0 0 0 0 0
1 1 2 3 4 5 10 1 0 1 2 3 4 5
5. 2 2 3 4 5 10 11 2 0 2 4 10 12 14
3 3 4 5 10 11 12 3 0 3 10 13 20 23
4 4 5 10 11 12 13 4 0 4 12 20 24 32
5 5 10 11 12 13 14 5 0 5 14 23 32 41
6. 10 451; 153 510.

7. 2/33 y 3/49; 5/79 y 3/47.

CAPITULO 8
1. a) La desigualdad es equivalente con

(1 + x + x2 + + xn1 )(n + 1) > (1 + x + x2 + + xn )n

que se simplifica: (1 + x + x2 + + xn1 ) > nxn . Pero, como 1 > x, se cumple que
xp > xn si p < n.
b) Equivale a x2 1 x3 (x2 1).
Si x > 1, obtenemos que 1 x3 .
Si x = 1, obtenemos que 0 = 0.
Si x < 1, multiplicado por 1, obtenemos que 1 x3 . En los 3 casos es verdadero para
x > 0.

2. a) Por Tchebychev
a+b 2 a+b a+b a2 + b2
 
= ,
2 2 2 2
381

a+b 2 a+b 2 a+b


  2
a + b2 a2 + b2
      
a+b
= , etc.
2 2 2 2 2 2
b) Usando Cauchy - Schway, acomodado del modo siguiente
(a1/2 a3/2 + b1/2 b3/2 + c1/2 c3/2 ) (a + b + c)(a3 + b3 + c3 )
c) A partir de 0 (a b)2 + (a c)2 + (b c)2 .
d) Inmediato a partir de Tchebychev.
e) Desarrollando se llega a que equivale a ab(c d)2 0.
f) A partir de A > G y multiplicar.
g) A > G
3. Como 0 < x < 21 , se tiene que x1 + x2 + + xn1 < 1. Por lo tanto, podemos aplicar
Weierstrass.
4. Usar la sugerencia.
q
1 1
5. A = n+1 y G = n (n!)2 (n+1) . De donde obtenemos (n + 1)n1 (n!)2 .

6. a) G < A. A < 1 por induccion.


b) Inmediato.
c) Use Weiertresse.
7. Use A > G, para los numeros (ak ak1 )
8. Sean a, b y c las dimensiones del P.R. Se tiene que V = abc y A = 2(ab + bc + ca). Pero
 3
2 ab + bc + ca
V = ab bc ca = (A/6)3
3
De donde 6V 2/3 A. Relacion valida para todos los P.R. Pero, en el caso del cubo se
tiene que 6V 2/3 = 6a2 = A.
9. Aplique la sugerencia.
10. i) x2 + 2x + 3 > 0 x
ii) y 2 2x + 3 > 0 x

iii) 3 2<x<3+ 2
iv) Por casos, x + 1 0 x + 1 < 0
v) Por casos
vi) |(1 x)(x2 + 1)| = (1 x)(x2 + 1). Por casos. 1 x > 0 1 x < 0
ax + b (a + b)x (x 1)(a2 x b2 )
11. i) = ssi (x 1)(a2 x b2 0), etc.
a+b ax + b (a + b)(ax + b)
ii) Se debe limitar a x 1. Ademas, considerar las races como positivas.
a+b a+b a+b a2 +b2
12. a) Por Tchebyshev. c < 2 ; por lo tanto, c2 < 2 2 2 . Etc.
2n+3 118
13. Es inmediato que f (n) < n
= 8. Ademas, f (4) = = 7, 53 . . .. Por ultimo,
2n 17
92
f (n + 1) f (n) = n > 0. Luego, es creciente.
(2 + 1)(2n+1 + 1)
382

CAPITULO 9
1. Use 3 = 1 y 1 + + 2 = 0. Los resultados son:
a) a2 + b2 + c2 ab ac bc
b) (a + b)(a b + 2c)(2a + b + c)
c) (a 2b + c)(a b + 2c)(2a + b + c)
d) a2 ab + b2
2. a) z = 1, 1, 0
b) a = 3/4, b = 1
3. La recta z = z1 + (z1 z2 )t pasa por z1 y z2 . (t = 0 y t = 1). z3 pertenece a dicha
z3 z1
recta, si y solo si z3 = z1 + (z1 z2 )t para algun t, que equivale a = t.
z1 z2
4. a) Simetral del trazo z1 z2 .
b) Elipse: x2 /(5/2)2 + y 2 /(3/2)2 =1.
c) El interior de las dos regiones determinada por la la hiperbola 36y 2 28x2 + 63 = 0.
d) El semiplano abierto que contiene al origen, determinado por la recta x+y=1.
5. 1 + i = 21/2 (cos(/4) + isen(/4). Luego, (1 + i)n = 2n/2 (cos(n/4) + isen(n/4)
En forma similar obtenemos que (1 i)n = 2n/2 (cos(n/4) isen(n/4). Sumando se
establece laidentidad.
6. es una raz compleja de orden 7 de la unidad. Por lo tanto: 1 + + + 6 = 0. El
resto es rutina.
7. a 6= c y d multiplo real de b.
8. Inmediato a partir de z n = cos(n) + ysen(n); z n = cos(n) ysen(n).

9. a) 1; -2; 1 i 3; 12 (1 i 3)

b) 21 ( 3 + i); 12 (1 i 3).
10. Use el hecho que 1 + + 2 + 3 + 4 = 0
11. Por induccion, sustituyendo n por 4p. Otra manera consiste en sustituir i por x, integrar,
sumar y despues derivar.
12. (ii = ((ei/2 ))i = e/2 = 0, 20787958 . . .

CAPITULO 10
1. El resto es de la forma mx + n. Sabemos que r = P (a) y s = P (b).
P (x) = Q(x a)(x b) + mx + n.
P (a) = r = ma + a; P (b) = s = mb + n.
o
El sistema ma+n=r
mb+n=s
rs
Soluciones m = y n = as bra b.
ab
383

2. Sea d = (P ,T ). Entonces d|P y d|QT .


Si X|P y X|QT se tiene que X|T , pues (P ,Q) = 1. por lo tanto,X|d.

3. a) (x + 5)3 13(x + 5)2 + 56(x + 5)


b) (x + 1)3 (x + 1) + 80
c) (x + 1/3)3 + (x + 1/3)2 + 2156/27

4. a) Sea y = xm . Entonces, (xmn + 1) = (y n + 1) = (y + 1)(y n1 y n2 + + 1).


b) (x + 1)(x2 x + 1)(x4 x3 + x2 x + 1)(x8 + x7 x5 x4 x3 + x + 1)

5. Igualando se llega al sistema:


2a + b = 3; a2 + 2ab = 9; a2 b = c, cuya solucion es a = 1 o 3, b = 5 o -3, c = 5 o -27.

6. Sean P = QM + R; P 0 = QM 0 + R0 y P P 0 = QT + S. Entonces P P 0 = QT + S =
QX +RR0 , donde X abrevia la suma de polinomios que aparecen al efectuar el producto.
Despejando: S = Q(X T ) + RR0 Pero gradoS <gradoQ.Luego, X T = 0, lo que
concluye la demostracion.

7. Por induccion. Es inmediato para n = 1.


(n + 1)xn+2 (n + 2)xn+1 + 1 = x(nxn+1 (n + 1)xn + 1) + xn+1 (x 1) (x 1)
= x(nxn+1 (n + 1)xn + 1) + (x 1)(xn+1 1)
= xQ(x)(x 1)2 + (x 1)2 S(x)

8. P (x) = (x + 1)(x + 3)Q(x) + 2x + 3. Dandole a x los valores 1 y 3 se obtiene


p = 4, q = 2.

9. Similar al problema anterior.


2 x
10. 1 x+2 + x2 +2

a b a b
11. + +
x2 + x + 1 x 2 (x 1)2 x1
1 2 1
12. i) x3 x4 + x5 .
1 1 1 1
ii) 5(5x1) + 2(4x1) 3(3x1) 30 .
1 2 1
iii) x+4 + x+3 x+2 .

13. Sea Q(x) = a0 + a1 x + + an xn y T (x) = b0 + b1 x + + bn xm , con los coeficientes


numeros enteros y p un numero primo. Debe existir algun coeficiente de Q y alguno de
T para los cuales p no es un factor. Sea ak el coeficiente de menor ndice, tal que p ak
y sea bl el de menor ndice para el cual p bl . Entonces, p divide a a0 , a1 , . . . , ak1 y
b0 , b1 , . . . , bl1 . El termino (ak bl + ak1 bl+1 + + a0 bl+k xl+k ) pertenece a Q(x) T (x)
pero no es divisible por p porque todos los sumandos lo son, excepto el primero. Luego,
el m.c.d de los coeficientes Q(x) T (x) es 1.

14. Es obvio que Q = cont(Q) Q y T = cont(T ) T con Q y T primitivos. Entonces,


QT = cont(Q) cont(T ) Q T . Luego, cont(QT ) = cont(Q) cont(T ) cont(Q T ). Pero
cont(Q T ) = 1.
384

15. Basta probarlo para polinomios primitivos. Sea P un polinomio con esas caractersticas
que se factoriza en QT , donde estos polinomios tienen coeficientes racionales. Sea b y
d los m.c.m de los denominadores de Q y T , respectivamente. Entonces, bQ y dT son
polinomios con coeficientes enteros. Por lo tanto, bdP = bQ dT .
bQ = cont(bQ)bQ y dT = cont(dT )dT , donde bQ y dT son primitivos.
bdp = cont(bQ)bQ cont(dT )dT = cont(bQ) cont(dT ) bQ dT .
Pero P , bQ y dT son primitivos. Por lo tanto, P = bQ dT .

CAPITULO 11
1. Sus distintas races son 1, 3 i, 3 + i, que originan la ecuacion (x 1)(x2 6x + 10) =
x3 7x2 + 16x 10.

2. a) (x (2 + 3i))(x (2 3i)) = x2 4x + 13, es un factor de la ecuacion. Luego, la


ecuacion se factoriza en (x + 1)2 (x2 4x + 13).
b) Tiene el factor (x2 + 1)2 . Se llega facilmente a la factorizacion (x 1)(x 2)(x2 + 1)2 .

c) Sea p el modulo comun. Como p4 = 4, se tiene que p = 2. Ademas, como tiene
exactamente una raz positiva (regla de los signos), esta debe ser 2, etc.

3. Haga los graficos.

4. Por la regla de los signos tiene 0, o 2 o 4 raices positivas y no tiene raices negativas.
Como es de grado 6, debe tener, por lo menos, 2 complejas.

5. La funcion tiene un mnimo en 1. Su valor es 1.

6. Use la regla de los signos. El valor aproximado es 0,09607

7. Por la regla de los signos hay solo una raiz positiva y 0 o 2 negativas. Es facil concluir
que la positiva esta en (1, 2). Como P (1) > 0 se concluye que hay 2 negativas. Se
encuentran en (1, 0) y (2, 1).

8. Amplificando por (x a1 ) . . . (x an ) resulta una ecuacion de grado n 1.

9. Es inmediato que a es una raz, en ambos casos.

10. Es inmediato que a y b son races.

11. Note que para cada natural q n, se cumple que P (q) = (1 1)q

CAPITULO 12
1. 0 y 1 son cotas inferior y superior de A, respectivamente. Su nfimo es 1/8 y su supremo
es 2/3.
385

13.13. BIBLIOGRAFIA
Solo citaremos los libros mas usados.

1. Durrel, Clement V. Advanced Algebra.

2. Knuth, Donald. Fundamental Algorithms.

3. Larson, Loren. Problem-Solving Through Problems.

4. Olmsted, John. The Real Number System.

5. Rademacher, Hans. Lectures on Elementary Number Theory.

6. Ross, Sheldon. A First Course in Probability.

7. Uspensky, J. V. Theory of Equations.

8. Vilenkin, N. De Cuantas Formas?

También podría gustarte